Bs Book
Bs Book
Bs Book
CHALLENGING
MATHEMATICAL
PROBLEMS
By Tanujit Chakraborty
1
By Tanujit Chakraborty
Challenging Mathematical Problems
2
Challenging Mathematical Problems
PREFACE
“This book is useful for the BSTAT and BMATH Entrances conducted by Indian Statistical
Institute and Chennai Mathematical Institute. Readers are also suggested to go through
www.ctanujit.in and www.ctanujit.org for other online resources. Since 2013, I am involved with
teaching for different entrance exams at ISI. I tried my best to cover several previous year’s
papers questions asked in ISI CMI BSTAT / BMATH Entrance exams. If you have any queries
you can mail me at tanujitisi@gmail.com.”
- Tanujit Chakraborty,
Research Scholar,
My Blog: www.ctanujit.org
3
Challenging Mathematical Problems
1. 10 Objective Problem Sets with Solutions (Total Solved Problems : 300) for all ISI BSTAT
& BMATH Entrances.
2. 10 Subjective Problem Sets with Solutions (Total Solved Problems : 100) for all ISI BSTAT
& BMATH Entrances.
3. 175 Subjective Problems with Solutions (Total Solved Problems : 175) for ISI & CMI
Examination.
4. Topic-wise Objective & Subjective Solved Problems (Total Solved Problems: 425) for ISI &
CMI Entrances.
4
Challenging Mathematical Problems
5
Challenging Mathematical Problems
numbers between 1 and 999, inclusive, (ii) Exactly one digit in each
999 number is even
divisible by 3 is ( ) = 333
3
The sum of all number in S is
Also, ‘0’ is divisible by 3. Hence, the
number of elements in s is = 333 + 1= 334. (a) 96100 (b) 133200 (c) 66600
(d) 99800
6
Challenging Mathematical Problems
7
Challenging Mathematical Problems
8
Challenging Mathematical Problems
𝑥 𝑥 𝑥 4 524
(−𝑥)(𝑒 − ⁄2 ) (−𝑥)(𝑒 − ⁄2 ) .𝑒 𝑥 𝑥𝑒 ⁄2 ∴ a= −8 − =− .
f(-x)= = = - 1+ 𝑒 𝑥 = 65 65
1+ 𝑒 −𝑥 1+ 𝑒 𝑥
-f (x) 18. In a special version of chess, a rook
moves either horizontally or vertically
∴ f is odd.
on the chess board. The number of
f (x) = x- [x] is not odd. ways to place 8 rooks of different
colors on a 8×8 chess board such that
Counter example:- no rook lies on the path of the other
f (-2.3) = -2.3 – [-2.3] =-2.3 – (-3) = 3- 2.3 = rook at the start of the game is
0.7 (a) 8× ⎿𝟖 (b)⎿8 × ⎿𝟖 (c)
64
𝟐𝟖 × ⎿𝟖 (d) 𝟐𝟖 × ( 8 )
f (2.3) = 2.3 – [2.3] = 2.3 -2 =0.3
Ans:- The first rook can be placed in any
∴ f(2.3) ≠ f(-2.3) row in 8 ways & in any column in 8 ways.
Thus f is not odd So, it has 82 ways to be disposed off. Since
no other rook can be placed in the path of
f (x) = 𝑥 2 sin 𝑥 + 𝑥 3 cos 𝑥 the first rook, a second rook can be placed in
72 ways for there now remains only 7 rows
f(-x) = -𝑥 2 sin 𝑥 − 𝑥 3 cos 𝑥 = -f(x)
and 7 columns. Counting in this manner, the
∴ f is odd here. number of ways = 82 . 72 . 62 … 12 = (8!)2
17. Consider the polynomial 𝒙𝟓 + 𝒂𝒙𝟒 + 19. The differential equation of all the
𝒃𝒙𝟑 + 𝒄𝒙𝟐 + 𝒅𝒙 + 𝟒. If (1+2i) and (3- ellipses centered at the origin is
2i) are two roots of this polynomial (a) 𝒚𝟐 + 𝒙(𝒚′)𝟐 − 𝒚𝒚′ = 𝟎
then the value of a is (b) x y y″+ 𝒙(𝒚′)𝟐 − 𝒚𝒚′ = 𝟎
(a) -524/65 (b) 524/65
(c) y y″+ 𝒙(𝒚′)𝟐 − 𝒙𝒚′ = 𝟎
(c) -1/65 (d) 1/65
(d) none
Ans:- (a) The polynomial has 5 roots. Since 𝑥2 𝑦2
complex root occur in pairs, so there is one Ans:- (d) 2
+ = 1, after differentiating
𝑎 𝑏2
real root taking it as m. w.r.t x, we get
So, m, 1+2i, 1-2i, 3+2i, 3-2i are the five 2𝑥 2𝑦𝑦′ 𝑦𝑦′ 𝑥
⤇ 𝑎2 + =0⤇ = − 𝑎2
𝑏2 𝑏2
roots.
(𝑦′)2 𝑦(𝑦″) 1
Sum of the roots= − 1 = 8 + 𝑚.
𝑎 ⤇ + = − 𝑎2
𝑏2 𝑏2
𝑏2
Product of the roots= (1+4)(9+4)m= 65 ⤇(𝑦′)2 + 𝑦(𝑦″)2 = − 𝑎2 .
4
m=65
4
∴ m = 65.
9
Challenging Mathematical Problems
Ans:- (c) Let R = (𝛼, 0). PR+RQ is least (a) (x-a)m= (y-b)n
⟹ PQR should be the path of light (b) (x-m) a= (y-n) b
10
Challenging Mathematical Problems
3 3.5 3.5.7
Similarly, = 1+4 + 4.8 + 4.8.12 + ⋯
11
Challenging Mathematical Problems
f(1)= 1 ……………..(2)
12
Challenging Mathematical Problems
𝜆+1 𝜆+2 2 2
∴ 𝐸2 = 1 + 𝜆2
=( ) ⇒ E = 1+ 𝜆,
ISI B.STAT/B.MATH 4
𝜆
13
Challenging Mathematical Problems
14
Challenging Mathematical Problems
Which has a real root 𝑥0 = -1 divisibility by 8, the last three digits must be
divisible by 8.
1 1 1 1 1
But, f (-1)= 1-1 + (2 − 3)+ (4 − 5) +6 > 0
i.e. 800+ 90+b |8.
The f (x)> 0 and hence f has no real zeros.
so (b+2)| 8 ; ∴ b = 6
𝑥 𝑥2 𝑥3 𝑥7
Now let, g (x)= 1+ 1 + + + ⋯+ For divisibility by 9, the sum of the digits A
2 3 7
+ 7 +3 +8+ 9 +b should be divisible by 9.
An odd degree polynomial has at least one
real root. i.e. a+ 7+ 3+ 8+ 9+ 6≡ 0 (mod 9)
⇒ 1+ 𝑥3 + 𝑥3 2 + ⋯ + 𝑥3 6 = 0 ∴ a= 3 only. Hence a+ b= 9
But this has no real zeros. Hence the given 9. 3 balls are distributed to 3 boxes at
polynomial has exactly one real zero. random. Number of way in which we
set at most 1 box empty is (a) 20
7. Number of roots between –𝜋 and 𝜋 of (b) 6 (c) 24 (d) none
𝟐
the equation 𝐬𝐢𝐧 𝒙. 𝒙= 1 is
𝟑 Ans:- (c) zero box empty + 1 box empty
(a) 1 (b) 2
(c) 3 (d) 4 = 3 balls in 3 boxes + {3C1 × 3 balls in 2
boxes}
3
Ans:- (d) sin 𝑥 = 2𝑥
= 3! + 3× (3)2 = 24.
3
Now, draw the curve of y= sin 𝑥 and y = 2𝑥
or xy = 3⁄2
10. The value of 𝐌𝐚𝐱 𝑰 (𝒂), 𝒘𝒉𝒆𝒓𝒆 𝑰(𝒂) =
𝒂
∴ there are 4 real roots. (Draw the graph 𝒂+𝟏
∫𝒂−𝟏 𝒆−|𝒙| 𝒅𝒙 is
yourself)
(a) ea (b) 2- 𝒆𝒂−𝟏 − 𝒆𝒂+𝟏
8. The number 𝒂𝟕𝟑𝟖𝟗𝒃 , a, b are digits, is (c) 𝒆𝒂−𝟏 − 𝒆𝒂+𝟏 (d) none
divisible by 72 , Then a+ b equals 𝑎
Ans:- (b) 𝐼 (𝑎) = ∫𝑎−1 𝑒 −|𝑥| 𝑑𝑥 +
(a) 10 (b) 9
𝑎+1 −|𝑥|
(c) 11 (d) 12 ∫𝑎 𝑒 𝑑𝑥
15
Challenging Mathematical Problems
= 1- 𝑒 𝑎−1 − (𝑒 −𝑎 − 1) ∴ 𝜃= 0, 𝜋, 2𝜋
𝑎+1 −|𝑥| Hence 3 roots are three between [0, 2𝜋]
And, ∫𝑎 𝑒 𝑑𝑥 = 𝑒 −𝑎 − 𝑒 −𝑎−1
𝟏 𝟏 𝟏
∴ I (a) = 2 - (𝑒 𝑎−1 + 𝑒 −𝑎−1 ) 13. If 𝒖𝒏 = + 𝟐(𝒏−𝟏) + 𝟑 (𝒏−𝟐) +
𝟏.𝒏
𝟏
𝑑 … . . + (𝒏−𝟏) ; Then 𝐥𝐢𝐦 𝒖𝒏 equals
∴ 𝑑𝑥 𝐼 (𝑎) = 0 𝒏⟶∞
(a) 0 (b) 1
𝑎−1 −𝑎+1
⇒𝑒 =𝑒 (c) ∞ (d) π
⇒ a= 0 1 1
Ans:- (a) 𝑢𝑛 = [(1 + 𝑛) +
(𝑛+1)
𝑎 1 1 1 1 1
Also, -1 < a < 0 ; ∫𝑎−1 𝑒 𝑥 𝑑𝑥 𝑎
= 𝑒 −𝑒 𝑎−1
(2 + 𝑛−1) + (3 + 𝑛−2) + ⋯ + (𝑛 + 1)]
𝑎+1 −|𝑥| 0 𝑎+1 −𝑥 1 1 1
And ∫𝑎 𝑒 𝑑𝑥 = ∫𝑎 𝑒 𝑥 𝑑𝑥 + ∫0 𝑒 𝑑𝑥 = (𝑛+1) 2 (1 + + ⋯ + 𝑛)
2
= (1- 𝑒 𝑎 ) – (𝑒 𝑎+1 − 1) 1 1
1+ + +⋯+
1
2 3 𝑛 𝑛
∴ lim 𝑢𝑛 = 2 lim . lim =
𝑎 𝑎+1 𝑛⟶∞ 𝑛⟶∞ 𝑛 𝑛⟶∞ 𝑛+1
= 2 -𝑒 − 𝑒
2. 0.1= 0.
𝑎−1 𝑎+1
∴ I (a) = 2- 𝑒 −𝑒 𝟏 𝟏
14. If x+ (𝒙) = -1, The value of 𝒙𝟗𝟗 + (𝒙𝟗𝟗 )
∴ I(a) is maximum at a = 0. is
𝟏 (a) 1 (b) 2
11. The value of ∏∞
𝒏=𝟐(𝟏 − 𝒏𝟐 ) is
(c) 0 (d) none
(a) 1 ( b) 0 (c) ½
1
(d) none Ans:- (b) If 𝑎𝑛 = 𝑥 𝑛 + 𝑥 𝑛
1
Ans:- (c) ∏∞ ∞
𝑛=2 (1 − 𝑛2 ) =∏𝑛=2 (1 + Then, 𝑎𝑛+1 = 𝑎𝑛 . 𝑎1 − 𝑎𝑛−1 𝑓𝑜𝑟 𝑛 ≥ 1
1 1 𝑛+1 𝑛−1
) (1 − 𝑛) = ∏∞
𝑛=2 . 𝑎0 = 2 , 𝑎2 = −𝑎1 − 𝑎0 = −1, 𝑎3 = 2, 𝑎4
𝑛 𝑛 𝑛
= −1, 𝑎5 = 2, 𝑎1 = −1 ; 𝑎𝑛
2+1 2−1 3+1 3−1
=( . )( . )……… = = −1, 𝑎𝑛−1 = −1.
2 2 3 3
3 1 4 2 1
. . . ……. = 2 .
2 2 3 3 𝑠𝑜 , 𝑎99 = 2
12. 𝒄𝒐𝒔𝟖 𝜽 − 𝒔𝒊𝒏𝟖 𝜽= 1. Number of roots 1 1
Or , ∴𝑎𝑛+1 = 𝑥 99 + 𝑥 99 = (𝑥 98 + 𝑥 98 ) (𝑥 +
are there in between [0, 2𝜋] is
1 1
(a) 1 (b) 2 (c) ) − (𝑥 97 + 𝑥 97 )
𝑥
3 (d) 0
= - 𝑎𝑛 − 𝑎𝑛−1 = +1 + 1 = 2
Ans:- (c) Note that , 𝑐𝑜𝑠 8 𝜃 = 1 + 𝑠𝑖𝑛8 𝜃 is
possible only if, 15. Consider the equation of the form𝒙𝟐 +
𝒃𝒙 + 𝒄 = 𝟎. The number of such,
𝑐𝑜𝑠 8 𝜃 = 1 𝑎𝑛𝑑 𝑠𝑖𝑛8 𝜃 =0 equations that have real roots and
16
Challenging Mathematical Problems
have coefficients b and c in the set {1, Ans:- (c) 𝑓 ″ (𝑥) = 4 = 𝑐𝑜𝑛𝑠𝑡𝑎𝑛𝑡
2, 3, 4, 5,6} , (b may be equal to c) is
(a) 16 (b) 19 ⇒ f(x)= 2𝑥 2 + 𝑎𝑥 + 𝑏
(c) 21 (d) none f(0)= 1 ⇒ b= 1
Ans:- (b) Let 𝑥 2 + 𝑏𝑥 + 𝑐 = 0 has real f(1)= 3+ a
roots, then 𝑏 2 − 4𝑐 ≥ 0, and also , s= {1, 2,
3, 4, 5, 6}. f(x) ≥f(1) ⇒ f′(1)=0
17
Challenging Mathematical Problems
𝜋𝑥 𝜋𝑥
⤇√5𝑥 − 𝑥 2 − 6 + > Taking dot product with c⃗ on both sides, we
2 2
get
⤇ √5𝑥 − 𝑥 2 − 6 > 0
a⃗. c⃗ = b⃗ .c⃗
2
⤇𝑥 − 5𝑥 + 6 < 0,
22. Let f(x)=
⤇ (x-2)(x-3)< 0, i.e. , x ∊ (2, 3). 𝒙|𝒙|; 𝒙 ≤ −𝟏
{[𝒙 + 𝟏] + [𝟏 − 𝒙]; −𝟏 < 𝑥 < 1
20. If f(x)= (𝟒 + 𝒙)𝒏 , n ∊ N and 𝒇𝒓 (𝟎) −𝒙 |𝒙|; 𝒙 ≥𝟏
represents the 𝒓𝒕𝒉 derivative of f(x) at 𝟐
𝒇𝒓 (𝟎) Then the value of ∫−𝟐 𝒇(𝒙)𝒅𝒙 is
x= o, then the value of ∑∞𝒓=𝟎 𝒓! =
𝒏 𝒏
(a) 𝟐 (b) 𝒆 (c) (a) - 𝟑
𝟖 𝟕
(b) - 𝟑
𝟓𝒏 (d) none 𝟕
(c) (d) none
𝟑
𝑛−1
Ans:- (c) f′(x)= n (4 + 𝑥)
−𝑥 2 , 𝑥 ≤ −1
𝑛−2
f″(x)= n (n-1) (4 + 𝑥) 1 , −1<𝑥 <0
Ans:- (a) f(x) = 2 , 𝑥=0
𝑓 𝑟 (𝑥)= n (n-1)….. (n- r+1) . (4 + 𝑥)𝑛−𝑟 , r 1 , 0<𝑥<1
≤𝑛 { 𝑥2 , 𝑥 ≥1
𝑛! 2
𝑓 𝑟 (0)= (𝑛−𝑟)! . 4𝑛−𝑟 , r ≤ 𝑛 ∴ f(x) is an even function, i.e. ∫−2 𝑓(𝑥)𝑑𝑥 =
2
2 ∫0 𝑓(𝑥)𝑑𝑥
= 0, r >𝑛
1 2
𝑓 𝑟 (0) = 2 {∫0 𝑓(𝑥)𝑑𝑥 + ∫1 𝑓(𝑥)𝑑𝑥 }
∴∑∞
𝑟=0 = ∑𝑛𝑟=0(𝑛𝑟). 4𝑛−𝑟 = (1 + 4)𝑛 =
𝑟!
5𝑛 . 𝑥3 2 8
= 2 (1 − )1 = − 3 .
3
21. The two lines r⃗ = a⃗ + 𝜆(b⃗ +c⃗) and r⃗
= b⃗ + 𝜇(c⃗ +a⃗) intersects at a point, 23. Area bounded by y = g(x), x-axis and
the lines x= - 2,
where 𝜆 and 𝜇 are scalars, then
Where g (x)=
(a) a⃗, b⃗ and c⃗ are non-coplanar
𝐦𝐚𝐱{ 𝒇(𝒕): −𝟐 ≤ 𝒕 ≤ 𝒙}, 𝒘𝒉𝒆𝒓𝒆 − 𝟐 ≤ 𝒙 < 0;
(b) |a⃗| = |b⃗| = |c⃗| {
𝐦𝐢𝐧{ 𝒇(𝒕): 𝟎 ≤ 𝒕 ≤ 𝒙}, 𝒘𝒉𝒆𝒓𝒆 𝟎 ≤ 𝒙 ≤ 𝟑
(c) a⃗.c⃗ = b⃗.c⃗
And f(x)= 𝒙𝟐 − |𝒙|, is equal to
(d) 𝜆(b⃗×c⃗) + 𝜇(c⃗×a⃗)=c⃗
𝟏𝟏𝟑 𝟏𝟏𝟏
Ans. (c) (a) (b)
𝟐𝟒 𝟐𝟒
𝟏𝟏𝟕
(c) (d) none
The two lines intersect 𝟐𝟒
18
Challenging Mathematical Problems
Ans:- (a) g(x)= Ans:- (b) Let (h, h) be the centre of the
2 ; −2 ≤ 𝑥 < 0 required circle.
2 1
{𝑥 − 𝑥 ; 0≤𝑥≤2 𝜋
1 1 ∴∠COD= ∠CBE= 4 , CB= h+ 2 AND BD=
−4 ; <𝑥≤3
2 h- 2.
1
0 h− 2 𝜋 1
∴ Required area = ∫−2 2𝑑𝑥 + ∫02(𝑥 − ∴ h+ 2 = 𝑐𝑜𝑠 4 = ,
√2
3 1 113
𝑥 2 )𝑑𝑥 + ∫1 (4) 𝑑𝑥 = 𝑢𝑛𝑖𝑡 2
2 24 2(√2+1)
⤇ h= = 2(3 + 2√2) .
(√2−1)
1 7
Then midpoints of BD and AC will be
⤇ -4 ≤ 𝑛 ≤ 4. coincident,
19
Challenging Mathematical Problems
1
∴ lim 𝑠𝑛 = lim (1 − 𝑛) = 1. ISI B.STAT/B.MATH
𝑛→∞
𝑛→∞ OBJECTIVE QUESTIONS &
SOLUTIONS
𝟑𝒙−𝟏
28. The limit 𝐥𝐢𝐦 ( 𝟑𝒙+𝟏)𝟒𝒙 equaqls
𝒙→∞
(a) 1 (b) 0 (c) SET – 3
𝟖 𝟒
−
𝒆 𝟑 (d) 𝒆 𝟗
1. Number of solutions are possible in
1
1−
1 −
𝑒 3 8 0≤ 𝒙 ≤ 𝟗𝟗 for the equation
Ans:- (c) lim {( 3𝑥
1 ) 𝑥 }4 = ( 1 ) 4 = 𝑒 −3
𝑥→∞ 1+ 𝑒3
3𝑥 𝟑−𝒙
|𝟑 − 𝟑𝒙 | + |𝟏 − 𝟑𝒙 | = 𝟏 − 𝟑𝒙 − is
𝟒
𝑘 𝑥
[since lim (1 + ) = 𝑒 𝑘 ].
𝑥→∞ 𝑥
(a) 1 (b) 0 (c) 2
(d) none
𝑥 𝑥
1 1 1 1 𝑥 − 𝑥 −
3 2 3 2
Ans:- (c) lim 𝑛 ( 1 + 2 + ⋯+ 𝑛 ) =1- {(3 +
2 ) - 2. 3 .
2 }
𝑥→∞ 1+ 1+ 1+ 2 2
𝑛 𝑛 𝑛
𝑥
1 1 𝑥 −
= ∫0 𝑑𝑥 = [log(1 + 𝑥)] 10 = log 𝑒 2. = 2 - (32 +
3 2 2
) <2
1+𝑥 2
i.e. |𝑥 2 + 𝑥 − 6| < 6
⇒ -6 < 𝑥 2 + 𝑥 − 6 < 6
20
Challenging Mathematical Problems
21
Challenging Mathematical Problems
22
Challenging Mathematical Problems
10. The remainder when 𝟏𝟔𝟗𝟎𝟐𝟔𝟎𝟖 + them are equal and not equal to other
𝟐𝟔𝟎𝟖𝟏𝟔𝟗𝟎 is divided by 7 is variable (say, i= j≠ 𝑘).
(a) 1 (b) 2 (c)
Case –I:- when i= j= k
3 (d) none
1
Ans:- (a) 1690= 7× 241 + 3; In this case ∑∞ ∞ ∞
𝑖=0 ∑𝑗=0 ∑𝑘=0 3𝑖 3𝑗 3𝑘 =
1 27
2608= 7 ×372 +4 ∑∞
𝑖=0 = 16
3𝑖
∞ 𝟏
11. The value of ∑∞ ∞
𝒊=𝟎 ∑𝒋=𝟎 ∑𝒌=𝟎 𝟑𝒊 𝟑𝒋 𝟑𝒌
is i.e. d (f(x), y) = d (x)
(𝒊 ≠𝒋≠𝒌)
Integrating we get, y. f(x) = x+ c
(a) 80/207 (b) 81/208
(c) 1/208 (d) none 𝑥+𝑐
or, f(x) = 𝑦
Ans:- (b) Let us first of all find the sum
𝒙
without any restriction i, j, k. 13. If ∫𝟎 𝒇(𝒙)𝒔𝒊𝒏𝒕𝒅𝒕 = constant, 0 < x <
2𝜋 and f(𝜋)= 2 Then find the value of
1 1 3 27
∑∞ ∞ ∞
𝑖=0 ∑𝑗=0 ∑𝑘=0 = (∑∞
𝑖=0 3𝑖 ) =
𝝅
f( 𝟐 )
3𝑖 3𝑗 3𝑘 8
23
Challenging Mathematical Problems
24
Challenging Mathematical Problems
𝜋
From (i) & (ii) , we get as the largest 20. Let a, b, c be any real numbers such
4
value. Hence k= 4. that 𝒂𝟐 + 𝒃𝟐 + 𝒄𝟐 = 𝟏 then the
quantity
18. The number of solution(s) of the ab +bc+ ca satisfies the conditions
𝟔𝟒
equation 𝒛𝟐 − 𝒛 − |𝒛|𝟐 − = 𝟎 is / (a) ab+ bc+ ac = constant
|𝒛|𝟓
(b) - ½ ≤ 𝒂𝒃 + 𝒃𝒄 + 𝒄𝒂 ≤ 𝟏
are
(c) - ¼ ≤ 𝒂𝒃 + 𝒃𝒄 + 𝒄𝒂 ≤ 𝟏
(a) 0 (b) 1 (c) 2 𝟏
(d) 3 (d) -1 ≤ 𝒂𝒃 + 𝒃𝒄 + 𝒄𝒂 ≤ 𝟐
𝜋
(a) > 8 (b) < 8
n (3𝜋)= r𝜋+ 2 (∵𝜆= 3𝜋) (c) 8 (d) none
25
Challenging Mathematical Problems
𝑦+𝑧 𝑧+𝑥 𝑦+𝑥 8 25. If 2x+ 4y= 1, then prove that the
=( )( )( ) . 𝑥𝑦𝑧 >
2 2 2
8 quantity 𝒙𝟐 + 𝒚𝟐 is always greater
√𝑦𝑧 √𝑧𝑥 √𝑧𝑦 . 𝑥𝑦𝑧 (By AM> GM than equal to
inequality) (a) 1/20 (b) 5/64
(c) 1 (d) none
𝑎 𝑏 𝑐
⇒ 1−𝑎 . 1−𝑏 . 1−𝑐 > 8
Ans:- (a) Maximize 𝑥 2 + 𝑦 2 subject to 2x+
23. Let a+ b +c = 1 then the value of the 4y -1=0 by
quantity is always √𝟒𝒂 + 𝟏 +
Method of Lagrange multiplier⟶
√𝟒𝒃 + 𝟏 + √𝟒𝒄 + 𝟏
(a) equals 21 (b) ≤ 21 F= 𝑥 2 + 𝑦 2 + 𝜆(2𝑥 + 4𝑦 − 1)
(c) > 21 (d) none
𝜕𝐹 𝜕𝐹
= 2𝑥 + 2𝜆 = 0 ; 𝜕𝑦 = 2𝑦 + 2𝜆 = 0
𝜕𝑥
Ans:- (b) 4a+ 4b+ 4c = 4
∴ x= -𝜆 ∴ y= -2𝜆
⇒(4a+ 1)+ (4b+ 1)+ (4c +1) = 7
1 1
Applying c-s inequality:- 𝑎1 = √4𝑎 + 1, 2x+ 4y = 1 𝑥𝑚𝑎𝑥 = + 10 , 𝑦𝑚𝑎𝑥 = 5
𝑎2 = √4𝑏 + 1, 𝑎3 = √4𝑐 + 1 & 𝑏𝑖 =1 −1
⇒ 𝜆= 10 ;
2
∴ (∑3𝑖=1 𝑎𝑖 .1) ≤ (∑3𝑖=1 𝑎𝑖 2 )(∑3𝑖=1 1) ; 1 4 5 1
where 𝑎𝑖 =𝑎1 , 𝑎2 , 𝑎3 ∴ 𝑥2 + 𝑦2 ≥ + 100 = = 20.
100 100
26
Challenging Mathematical Problems
1 2 1996
So, f(1997) + 𝑓 (1997) + ⋯ + 𝑓(1997)
1. If 𝑺𝒏 denotes the sum of first n terms of
= (1
⏟ + 1 + ⋯ + 1) an A.P. whose
27
Challenging Mathematical Problems
1 1 1
(a) A.P. and H.P. ⟹𝑥 , 𝑦 , 𝑧 are in A.P.
(b) A.P. and G.P. but not in H.P.
⟹ x, y, z are in H.P.
(c) G.P. and H.P.
(d) A.P., G.P. and H.P. 4. If a, b, c be the 𝒑𝒕𝒉 ,𝒒𝒕𝒉 and 𝒓𝒕𝒉 terms
respectively of an A.P. and G.P. both,
Ans. (b)
then the product of the roots of equation
𝑎𝑛+1 − 𝑎𝑛 𝒂𝒃 𝒃𝒄 𝒄𝒂 𝒙𝟐 – 𝒂𝒃𝒄𝒙 + 𝒂𝒄 𝒃𝒂 𝒄𝒃 =0 is equal to
𝜋
sin(2𝑛 + 1)𝑥 − sin(2𝑛 − 1)𝑥
=∫ 𝑑𝑥 (a) -1 (b) 1 (c) 2
0 sin 𝑥
𝜋 (d) (b-c)(c-a)(a-b)
2 cos 2𝑛𝑥. 𝑠𝑖𝑛𝑥 2 sin 2𝑛𝑥 𝜋
= ∫ 𝑑𝑥 = [ ]
0 𝑠𝑖𝑛𝑥 2𝑛 0 Ans. (b)
=0
a= x+ (p- 1)d, b= x+ (q-1)d, c= x+ (r-1)d
∴𝑎𝑛+1 = 𝑎𝑛 ⟹ 𝑎1 = 𝑎2 = 𝑎3 = ⋯
a=𝑚𝑛𝑝−1 , 𝑏 = 𝑚𝑛𝑞−1 , 𝑐 = 𝑚𝑛𝑟−1
Also 𝑎1 = 𝜋 ≠ 0
∴ Product of roots =
Hence 𝑎1 , 𝑎2 , … 𝑎𝑛 are in A.P. and G.P. but
not in H.P. (Equal numbers cannot be in (𝑚𝑛𝑝−1 )(𝑟−𝑞)𝑑 , (𝑚𝑛𝑞−1 )(𝑝−𝑟)𝑑 , (𝑚𝑛𝑟−1 )(𝑞−𝑝)𝑑 =
H.P) 𝑚0 . 𝑛0 = 1.
28
Challenging Mathematical Problems
Ans. (c)
4 8 1 1 1 1 1
Now 𝑎2 + 𝑏= 9 (10𝑛 − 1)2 + 9 (10𝑛 − = = 𝑟 − 𝑟+1 ∴ ∑𝑛𝑟=1 𝑡 = 1 −
𝑡𝑟 𝑟(𝑟+1) 𝑟
4 1 𝑛
1) = (10𝑛 − 1) 2 (10𝑛
− 1 + 2) = = 𝑛+1
9 𝑛+1
4 2𝑛
(10 − 1) = 𝑐 𝟏 𝟏
9
9. If a=∑∞ ∞
𝒓=𝟏 𝒓𝟒 , then ∑𝒓=𝟏 (𝟐𝒓−𝟏)𝟒 =
𝟏𝟔 𝒂
(a) 𝟏𝟓a (b) 𝟐
7. Let a=1 1 1.....1(55 digits),
𝟏𝟓 𝟏𝟒
b=1+10+𝟏𝟎𝟐 +…𝟏𝟎𝟒 , (c) 𝟏𝟔a (d) 𝟏𝟓a
c=1+𝟏𝟎𝟓 +𝟏𝟎𝟏𝟎 +𝟏𝟎𝟏𝟓 +…+𝟏𝟎𝟓𝟎 , then
Ans. (c)
(a) a=b+c (b) a=bc 1 1 1 1
(c) b=ac (d) c=ab ∑∞
𝑟=1 = 14 + 34 + 54 + ⋯ + 𝑡𝑜 ∞
(2𝑟−1)4
Ans. (b) 1 1
= (14 + 24 + 34 + ⋯ + 𝑡𝑜 ∞) −
1
1055 1 1 1
a= 1+ 10+ 102 + ⋯ + 1054 = = (24 + 44 + 64 + ⋯ + 𝑡𝑜 ∞)
10−1
1055 −1 105 −1
. = 𝑏𝑐
105 −1 10−1
29
Challenging Mathematical Problems
1 1 1 1 ∴(−∞, 3)
= a-24 (14 + 24 + 34 + ⋯ + 𝑡𝑜 ∞) = 𝑎 −
𝑎 15
= 16 𝑎 12. Let 𝛼, 𝛽 are the roots of the equation
16
𝒙𝟐 +ax +b=0, then maximum value of the
10. If 𝒂𝟏, 𝒂𝟐, 𝒂𝟑 ,…are in G.P. having 𝜶−𝜷 𝟐
expression - (𝒙𝟐 +ax +b) - ( ) will be
𝟐
common ratio r such that
∑𝒏𝒌=𝟏 𝒂𝟐𝒌−𝟏 =∑𝒏𝒌=𝟏 𝒂𝟐𝒌+𝟐 ≠ 𝟎, then number 𝟏
(a) 𝟒 (𝒂𝟐 − 𝟒𝒃) (b) 0
of possible values of r is
(c) 1 (d) none
(a) 1 (b) 2
Ans. (b) let z= - (𝑥 2 -ax +b)
(c) 3 (d) none of these
𝐷 𝑎2 −4𝑏 4𝑏−𝑎2
Ans. (c) Now, 𝑧𝑚𝑎𝑥. = − 4𝑎 = − = =
4 4
𝛼−𝛽 2
Given 𝑎1 + 𝑎3 + 𝑎5 + ⋯ + 𝑎2𝑛−1 = 𝑎4 + +( 2
)
𝑎6 + 𝑎8 + ⋯ + 𝑎2𝑛+2
∴Thus the maximum value of the given
= 𝑟 3 (𝑎1 + 𝑎3 + 𝑎5 + ⋯ + 𝑎2𝑛−1 ) equation is 0.
31
Challenging Mathematical Problems
19. Let 𝒙𝟐 + 𝒚𝟐 = 𝒄𝟐 , then the least value Putting the all I value and then multiplies the
of 𝒙−𝟐 + 𝒚−𝟐 is in equations,
𝟏 𝟏
𝑐2 4
∴ 𝑧𝑚𝑖𝑛. = 𝑐4
= 𝑐 2. (a) 𝒏(𝟐𝒄)𝒏 (b) (𝒏 + 𝟏)𝒄𝒏
4 𝟏
(c) 𝟐𝒏𝒄𝒏 (d) done
𝒏+𝟏
20. 𝒏𝒏 ( 𝟐 )𝟐𝒏 >
Ans. (a) AM ≥ GM
(a) n! (b) (𝒏!)𝟐 1 1
So, LHS ≥ 𝑛(𝑎1 … 2𝑎𝑛 )𝑛 = 𝑛(2𝑐)𝑛
(c) (𝒏!)𝟑 (d) none.
1
13 +23 +⋯+𝑛3
Ans. (c) > (13 . 23 . … . 𝑛3 )𝑛
𝑛
𝒙 𝒆𝐜𝐨𝐬 𝒕
[∵AM> GM] 23. If f(x) = ∫𝟎 𝒅𝒕, then 2f(𝜋) =
𝒆𝐜𝐨𝐬 𝒕 +𝒆−𝐜𝐨𝐬 𝒕
1
𝑛(𝑛+1)2 (a) 0 (b) 𝜋 (c) –𝜋
⇒ > {(𝑛!)3 }𝑛
4
(d) none of these
𝑛+1 2𝑛
⇒ 𝑛𝑛 ( ) > {(𝑛!)3 }.
2 Ans. (b)
𝜋 𝑒 cos 𝑡
𝑓(𝜋) = ∫0 𝑑𝑡……….(1)
𝑒 cos 𝑡 +𝑒 −cos 𝑡
21. If 𝒂𝟏 , 𝒂𝟐 , 𝒂𝟑 , … . , 𝒂𝒏 are non- negative
𝜋 𝑒 −cos 𝑡
and 𝒂𝟏 , 𝒂𝟐 , 𝒂𝟑 , … . , 𝒂𝒏 = 𝟏, 𝑓(𝜋) = ∫0 𝑑𝑡 …………(2)
𝑒 −cos 𝑡 +𝑒 cos 𝑡
32
Challenging Mathematical Problems
𝜋 Ans. (b)
[∵ 0 < x <4 ∴ [𝑥] = 0]
Let P= (x, y)
𝒙 𝟏
25. Let g(x) = ∫𝟎 𝒇(𝒕)𝒅𝒕 , 𝒘𝒉𝒆𝒓𝒆 𝟐 ≤
Equation of tangent to the curve at P(x, y) is
𝒇(𝒕) ≤ 𝟏, 𝒕 ∈ [𝟎, 𝟏]𝒂𝒏𝒅𝟎 ≤ 𝒇(𝒕) ≤
𝟏 𝟑 𝑑𝑦
𝒇𝒐𝒓 𝒕 ∈ (𝟏, 𝟐]. Then (a) − 𝟐 ≤ 𝒈(𝟐) < Y- y = 𝑑𝑥 (𝑋 − 𝑥)
𝟐
𝟏 𝟑
(b) 𝟎 ≤ 𝒈(𝟐) < 2 (c) 𝟐 < 𝑔(𝟐) ≤ 𝑑𝑦
𝟐
𝟓
When X= 0, Y= y – x 𝑑𝑥
(d) 2 < g(2 )< 4
𝟐
𝑑𝑦
2 1 ∴B≡ (0, 𝑦 − 𝑥 )
𝑑𝑥
Ans. (b) 𝑔(2) = ∫0 𝑓(𝑡) 𝑑𝑡 = ∫0 𝑓(𝑡) 𝑑𝑡 +
2 1
∫1 𝑓(𝑡) 𝑑𝑡 𝑎𝑠 2 ≤ 𝑓(𝑡) ≤ 1 𝑓𝑜𝑟 0 ≤ 𝑡 ≤ 1, Area of 𝛥AOB ant=k
1 𝑑𝑦 𝑥 2 𝑑𝑦
1 1 1 ∴2 𝑥 (𝑦 − 𝑥 𝑑𝑥 ) ⟹ 𝑥𝑦 − = ±2𝑘 ⟹
1 𝑑𝑥
∴ ∫ 𝑑𝑡 ≤ ∫ 𝑓(𝑡)𝑑𝑡 ≤ ∫ 1 𝑑𝑡
𝑥 2 𝑑𝑦
0 2
𝑑𝑦 1
0 0 − 𝑥𝑦 = ±2𝑘 = 𝑐 ⟹ 𝑑𝑥 + 𝑦 (− 𝑥) =
𝑑𝑥
1 2
𝑜𝑟, 2 ≤ 𝑥2
1
∫0 𝑓(𝑡)𝑑𝑡 ≤ 1 ………. (1) 1
𝐼. 𝐹. = 𝑒 − log 𝑥 =
1 𝑥
𝑎𝑠 0 ≤ 𝑓(𝑡) ≤ 𝑓𝑜𝑟 1 < 𝑡 ≤ 2,
2 1 𝐶
𝑠𝑜𝑙𝑢𝑡𝑖𝑜𝑛 𝑖𝑠 𝑔𝑖𝑣𝑒𝑛 𝑏𝑦 𝑦, = ∫ 3 𝑑𝑥 + 𝑎
2 2 2 𝑥 𝑥
1
∴ ∫ 0 𝑑𝑡 ≤ ∫ 𝑓(𝑡) 𝑑𝑡 ≤ ∫ 𝑑𝑡
1 1 1 2 𝐶
𝑜𝑟 𝑦 = 𝑥 (− ) + 𝑎𝑥
2𝑥 2
𝑜𝑟, 0 ≤
2 1
≤ 2 ……………. (2) 𝑜𝑟 2𝑥𝑦 = −𝐶 + 2𝑎𝑥 2
∫1 𝑓(𝑡) 𝑑𝑡
33
Challenging Mathematical Problems
𝜋 𝜋
⟹ x= tan−1 𝑓(𝑥) ∴ − 2 < 𝑥 < 2
𝑜𝑟 2𝑎𝑥 2 − 2𝑥𝑦 − 𝐶 = 0 ………….(1)
𝟐+𝐬𝐢𝐧 𝒙 𝒅𝒚
29. If y= (x) and (𝒅𝒙) =
Here h= -1, a= a, b= 0 𝒚+𝟏
𝝅
− 𝐜𝐨𝐬 𝒙, 𝒚(𝟎) = 𝟏, 𝒚 ( 𝟐 ) equals:
∴ ℎ2 > ab. Hence curve (1) is a
hyperbola (a) 1/3 (b) 2/3 (c) -
𝒅 𝒌 𝒅𝒙 1/3 (d) 1
27. The function f(k) = 𝒅𝒌 ∫𝟎 𝟏−𝐜𝐨𝐬 𝒌.𝐜𝐨𝐬 𝒌
2+sin 𝑥 𝑑𝑦
satisfies the differentiable equation Ans. (a) Given, = − cos 𝑥
𝑦+1 𝑑𝑥
𝒅𝒇 ………..(1) & 𝑦(0) =
(a) 𝒅𝒌 + 𝟐𝒇(𝒌). 𝐜𝐨𝐭 𝒌 = 𝟎
1 ………….(2)
𝒅𝒇
(b) 𝒅𝒌 + 𝟐𝒇(𝒙). 𝐜𝐨𝐬 𝒌 = 𝟎
𝑑𝑦 cos 𝑥
𝒅𝒇
(1) ⟹ ∫ = −∫ 𝑑𝑥
(c) 𝒅𝒌 − 𝟐𝒇(𝒌). 𝒄𝒐𝒔𝟐 𝒌 = 0 𝑦+1 2 + sin 𝑥
(d) none of these ⟹ log(𝑦 + 1) = − log(2 + sin 𝑥) + log 𝑐
𝑐
Ans. (a) 𝑓(𝑥) = ⟹𝑦+1=
2 + sin 𝑥
1
= 𝑐𝑜𝑠𝑒𝑐 2 𝑘 𝑐
1−cos 𝑘 cot 𝑘
⟹𝑦 = 2+sin 𝑥 −
𝑑𝑓 1 …………..(3)
= 2 𝑐𝑜𝑠𝑒𝑐 𝑘 (– 𝑐𝑜𝑠𝑒𝑐 𝑘 cot 𝑘)
𝑑𝑘
= −2𝑓(𝑘) cot 𝑘 𝑐
𝐴𝑙𝑠𝑜 𝑔𝑖𝑣𝑒𝑛 𝑦(0) = 1 ⟹ 1 = −1⟹𝑐
2
𝑑𝑓 =4
𝑜𝑟 + 2𝑓(𝑘)𝑐𝑜𝑡𝑘 = 0
𝑑𝑘
2 − sin 𝑥
∴ 𝑓𝑟𝑜𝑚 (3), 𝑦(𝑥) =
28. The largest value of ‘c’ such that there 2 + sin 𝑥
exists a differentiable function f(x) for –c<
𝜋 2−1 1
x < c that satisfies the equation 𝒚𝟏 = 𝟏 + ∴𝑦=( )= =
2 2+1 3
𝒚𝟐 with f(0)= 0 is
(a) 1 (b) 𝜋
𝝅 𝝅
(c) (d) 30. If 𝒍𝟏 𝒂𝒏𝒅 𝒍𝟐 are the side length of two
𝟑 𝟐
variables squares 𝒔𝟏 𝒂𝒏𝒅 𝒔𝟐 , respectively.
𝑑𝑦
Ans. (d) = 1 + 𝑦 2 ⟹ tan−1 𝑦 = 𝑥 + If 𝒍𝟏 = 𝒍𝟐 + 𝒍𝟐 𝟑 + 𝟔, then the rate of
𝑑𝑥
𝑘 change of the area of 𝒔𝟐 with respect to
rate of change of the area of 𝒔𝟏 when
∵ f(x) satisfies the equation 𝒍𝟐 = 𝟏 is
∴tan−1 𝑓(𝑥) = 𝑥 + 𝑘 (a) 3/2 (b) 2/3
(c) 4/3 (d) none
Now, f(0)= 0= k= 0
34
Challenging Mathematical Problems
𝑑∆2 1
When𝑙2 = 1, 𝑙1 = 8 , 𝑡ℎ𝑒𝑛 = 32. 1 1
(1− 100 )
𝑑∆1
= 2(2 100
−1+ 2 2
1 + 3 ∑100
𝑟=1 𝑡𝑟 =
1−
2
1
2101 − 2 + 1 − 2100 + 3 ∑100
𝑟=1 𝑡𝑟
𝝅 Ans. (c)
𝒔𝒊𝒏𝟐 𝒏𝒙
1. If 𝒂𝒏 =∫ 𝒅𝒙, then
𝟐
𝟎 𝒔𝒊𝒏𝟐 𝒙
𝒂𝟏 𝒂𝟓𝟏 𝒂𝟏𝟎𝟏 𝑡𝑟 = r. r != (r+1 -1)r!= (r+1)! –r!
[𝒂𝟐 𝒂𝟓𝟐 𝒂𝟏𝟎𝟐 ] = ∴∑𝑛𝑟=1 𝑡𝑟 = (𝑛 + 1)! − 1! = (𝑛 + 1)! − 1
𝒂𝟑 𝒂𝟓𝟑 𝒂𝟏𝟎𝟑
(a) 1 (b) 0
(c) -1 (d) none of these 4. If m = ∑∝𝒓=𝟎 𝒂𝒓 , n=∑∝𝒓=𝟎 𝒃𝒓 where
0<a<1,0<b<1,then the quadratic equation
Ans. (b) 𝑎𝑛+2 + 𝑎𝑛 − 2𝑎𝑛+1 = 0
whose
⟹ 𝑎1 , 𝑎2 , 𝑎3 , … 𝑎𝑟𝑒 𝑖𝑛 𝐴. 𝑃.
roots are a and b is
∴ 𝑎1 + 𝑎101 = 2𝑎1 + 2𝑎1 + 100𝑑 =
(a) mn𝒙𝟐 +(m+n-2mn)x+mn-m-n+1=0
2(𝑎1 + 50𝑑) = 2𝑎51
(b) mn𝒙𝟐 +(2mn-m-n)x+mn-m-n+1=0
𝑎2 + 𝑎102 = 2𝑎52 , 𝑎3 + 𝑎103 = 2𝑎53
(c) mn𝒙𝟐 +(2mn+m+n)x+mn+m+n+1=0
2. If 𝒕𝒓 =𝟐𝒓⁄𝟑 +𝟐−𝒓⁄𝟑 , then (d) mn𝒙𝟐 -(2mn+m+n)x+mn+m+n+1=0
∑𝟏𝟎𝟎 𝟑 𝟏𝟎𝟎
𝒓=𝟏 𝒕𝒓 − 𝟑 ∑𝒓=𝟏 𝒕𝒓 +1 =
Ans. (a)
1 𝑚−1 𝑛−1
m = 1−𝑎 ⟹ 𝑎 = , 𝑠𝑖𝑚𝑖𝑙𝑎𝑟𝑙𝑦 𝑏 = .
𝑚 𝑛
35
Challenging Mathematical Problems
or, 𝑥 2 − (
𝑚−1
+
𝑛−1
) 𝑥+
(𝑚−1)(𝑛−1)
=0 Then f(−∞) = ∞ > 0, 𝑓(0) = 2𝑏101 −
𝑚 𝑛 𝑚𝑛
𝑎101 − 𝑐 101 < 0, 𝑓(∞) = ∞ > 0.
or, mn𝑥 2 − (2𝑚𝑛 − 𝑚 − 𝑛)𝑥 + 𝑚𝑛 − 𝑚 −
Hence equation f(x)= 0 has one root in (-
𝑛+1=0
∞, 0)and other in(0, ∞).
5. If ∑𝒏𝒓=𝟏 𝒓𝟒 = 𝒂𝒏 , then ∑𝒏𝒓=𝟏 𝒓𝟒 (𝟐𝒓 −
7. If the sum of the series ∑∝𝒏=𝟎 𝒓𝒏 , |r|<1, is
𝟏)𝟒 =
s, then sum of the series ∑∝𝒏=𝟎 𝒓𝟐𝒏 is
(a) 𝒂𝟐𝒏 +𝒂𝒏 (b) 𝒂𝟐𝒏 -𝒂𝒏 𝟐𝒔
(c) 𝒂𝟐𝒏 -16𝒂𝒏 (d)𝒂𝟐𝒏 +16𝒃𝒏 (a) 𝒔𝟐 (b) 𝒔𝟐−𝟏
𝒔𝟐 𝒔𝟐
Ans. (c) ∑𝑛𝑟=1(2𝑟 4 4
− 1) = 1 + 3 + 5 + 4 4 (c)𝟐𝒔+𝟏 (d)𝟐𝒔−𝟏
(2𝑛 4
⋯+ − 1)
Ans. (d)
= [14
+2 +3 +⋯+ 4 4 (2𝑛)4 ] 4
− [2 +
s= ∑∞ 𝑛 2 3
𝑛=0 𝑟 = 1 + 𝑟 + 𝑟 + 𝑟 + ⋯ 𝑡𝑜 ∞ =
4 + 64 + ⋯ + (2𝑛)4 ]
4
1
1−𝑟
= 𝑎2𝑛 − 24 (14 + 24 + 34 + ⋯ + 𝑛4 ) =
1 𝑠−1
𝑎2𝑛 − 16𝑎𝑛 . ∴ r=1- =
𝑠 𝑠
𝟎 (𝒌 ∊ 𝑹) has
𝟐 𝟒 𝟐𝒏
8. The limit of the product √𝟓,√𝟓,…. √𝟓
(a) both roots positive as n→∞ is
(b) both roots negative
𝟏
(a) 𝟓 (b) 𝐥𝐨𝐠 𝟏𝟎 𝟓
(c) one positive and one negative root
(d) both roots imaginary. (c) 1 (d) 5
36
Challenging Mathematical Problems
(a) A.P. (b) G.P. 11. The set of all real number x such that
(c) H.P. (d)none of ||3-x|-|x+2||=5 is
these
(a) [3, ∞) (b) (-∞, −𝟐]
𝑚7𝑞 𝑚7𝑟 (c) (-∞, −𝟐] ∪[3, ∞) (d) (-
Ans. (b) 𝑚7𝑝 = 𝑚7(𝑞−𝑝) , 𝑚7𝑞 = 𝑚7(𝑟−𝑞)
∞, −𝟑] ∪[2, ∞)
∴ q- p= r- q
Ans. (c) (||3 − x| − |x + 2||)2 = 25
∴ 𝑚7𝑝 , 𝑚7𝑞 , 𝑚7𝑟 are in G.P.
⤇(3 − x)2 + (x + 2)2 − 2|3-x||x+2|= 25
10. Let n be a positive integer and
⤇ x 2 − 𝑥 − |−x 2 + 𝑥 + 6| = 6
(𝟏 + 𝒙 + 𝒙𝟐 )𝒏 = 𝒂𝟎 + 𝒂𝟏 𝒙 + ⋯ + 𝒂𝟐𝒏 𝒙𝟐𝒏 ,
then the value of 𝒂𝟎 𝟐 − 𝒂𝟏 𝟐 + So, it is clear that −x 2 + 𝑥 + 6 < 0 ,
𝒂𝟐 𝟐 … . . +𝒂𝟐𝒏 𝟐 is
i.e. −x 2 + 𝑥 + 6 ≥ 0
(a) 0 (b) 𝒂𝟎
(c) 𝒂𝒏 (d) 𝒂𝟐𝒏 (x-3)(x+2)≥ 0. So, x ≤ −2 & 𝑥 ≥ 3
37
Challenging Mathematical Problems
2 2
𝒅𝟐 𝒚 𝒅𝒚 z𝑒 𝑥 = ∫ 𝑥 3 𝑒 𝑥 𝑑𝑥 (put 𝑥 2 = 𝑢)
+ 𝟐𝒄 𝒅𝒙 + 𝒌𝒚 = 𝟎, 𝒘𝒉𝒆𝒓𝒆 𝒄 < 0, 𝑘 >
𝒅𝒙𝟐
0, 𝒄𝟐 − 𝒌. Then 1 2
= 2 (𝑥 2 − 1)𝑒 𝑥 + 𝑐
(a) |y(x)|⟶∞ 𝒂𝒔 𝒙 → ∞
(b) |y(x)|⟶𝟎 𝒂𝒔 𝒙 → ∞ ∴ General solution is given by:- √𝑦 =
1 2
(𝑥 2 − 1) + 𝑐𝑒 −𝑥
2
(c) 𝐥𝐢𝐦 |𝐲(𝐱)| exists & is finite
𝒙→±∞
1
(d) none Since y(0)=0, so, c= 2
38
Challenging Mathematical Problems
(a) 3 (b) 6 1
When x∊(−10, − )
(c) 9 (d) none 3
39
Challenging Mathematical Problems
0 −1/3 0 𝑥2 0 1
∫ 𝑓(𝑥) 𝑑𝑥 = ∫ 𝑑𝑥 + ∫ (−1) 𝑑𝑥 = [2𝑥 − ] + 2[𝑥]
−10 −10 −1/3 2 −1 0
1 1
−3 0 = 0 − (−2 − ) + 2(1 − 0)
= [𝑥] − (𝑥) 1 2
−10 −3 9
=
1 1 2
= (− + 10) − (0 + )
3 3
2 23. Let f(x) be a continuous function such
= 10 − that f(a-x)+f(x)=0 for all x ∊[0, a].
3
𝝅 𝒂 𝒅𝒙
𝒃 𝐬𝐢𝐧 𝒙 Then ∫𝟎 equals
21. The equation ∫𝟒𝝅 (𝒂|𝐬𝐢𝐧 𝒙| + 𝟏+𝐜𝐨𝐬 𝒙 + 𝟏+𝒆𝒇(𝒙)
−
𝟒
(a) a (b) a/2 (c) ½
𝒄) 𝒅𝒙= 0 gives a relation between
f(a) (d) none of these
(a) a, b and c (b) a and b Ans. (b) Given, f(a- x)= - f(x)
(c) b and c (d) a and c
𝑎 𝑑𝑥 𝑎 𝑑𝑥
𝜋 Now 2I= ∫0 +∫ =
1+𝑒 𝑓(𝑥) 0 1+𝑒 𝑓(𝑎−𝑥)
Ans. (d) I = 2a∫0 |sin 𝑥| 𝑑𝑥 + 0 +
4
𝑎 𝑑𝑥 𝑎 𝑑𝑥 𝑎
𝜋 ∫0 + ∫0 1+𝑒 −𝑓(𝑥) = ∫0 𝑑𝑥 = 𝑎
𝜋
𝜋 1+𝑒 𝑓(𝑥)
4
∫ 𝑐 𝑑𝑥 = 2𝑎 ∫0 𝑠𝑖𝑛𝑥 𝑑𝑥 + 𝑐. 2
−
𝜋
4
4 𝑎
∴𝐼=
𝜋 2
𝜋
= −2𝑎[𝑐𝑜𝑠𝑥] 4 + 𝑐 24. Let f(x) be an integrable odd function
0 2
1 𝜋 in [-5, 5] such that f(10+ x)= f(x),
= −2𝑎 ( − 1) + 𝑐
√2 2 𝟏𝟎+𝐱
then ∫𝐱 𝐟(𝐭)𝐝𝐭 equals
22. Let f(x) = max. {2- x, 2, 1+ x} then
𝟏 𝟓
∫−𝟏 𝒇(𝒙) 𝒅𝒙= (a) 0 (b) 2∫𝐱 𝐟(𝐱)𝐝𝐱
(c) > 0 (d) none of these
(a) 0 (b) 2 (c)
𝑥+10
9/2 (d) none of these Ans. (a) Let y= ∫𝑥 𝑓(𝑡) 𝑑𝑡…… (1)
= 1+ x, x ≥1 ∴ y is independent of x.
1 0 Putting x= -5 in (1), we get
I= ∫−1 𝑓(𝑥) 𝑑𝑥 = ∫−1 𝑓(𝑥)𝑑𝑥 +
1 0 1
∫0 𝑓(𝑥)𝑑𝑥 = ∫−1(2 − 𝑥)𝑑𝑥 + ∫0 2 𝑑𝑥 5
y= ∫−5 𝑓(𝑡) 𝑑𝑡 = 0 …………. (2)
40
Challenging Mathematical Problems
𝟐 𝟐𝟗
(d) none of these
(c) (𝒉, 𝟑) , 𝒘𝒉𝒆𝒓𝒆 𝒉 > 𝟏𝟖
Ans. (d) Slope of tangent at p=
(d) none of these 𝑑𝑦 𝑦 −𝑦
𝑎𝑡 (𝑥3 , 𝑦3 ) = 2𝑎𝑥3 + 𝑏 = 𝑥2−𝑥1
𝑑𝑥 2 1
Ans. (c)
[given]……….(A)
2 2 10
Given parabola is (𝑦 − ) = 2 (𝑥 − ) ∵ A and B lie on the parabola,
3 9
Let X= 𝑥 −
10
,𝑌 = 𝑦 − 3
2 ∴𝑦1 = 𝑎𝑥1 2 + 𝑏𝑥1 + 𝑐 ………………(1)
9
And 𝑦2 = 𝑎𝑥2 2 + 𝑏𝑥2 + 𝑐 ……………..(2)
∴𝑌 2 = 2𝑥 becomes the equation of parabola
with reference to the new origin. ∴ 𝑦1 − 𝑦2 = [𝑎(𝑥1 + 𝑥2 )(𝑥1 − 𝑥2 ) +
𝑏](𝑥1 − 𝑥2 )
Hence equation of normal will be
𝑦 −𝑦
∴ 𝑥2−𝑥1 = 𝑎 (𝑥1 + 𝑥2 ) + 𝑏
2 1
41
Challenging Mathematical Problems
∴ From (A), a(𝑥1 + 𝑥2 ) + 𝑏 = 2𝑎𝑥3 + 𝑏 (a) (a, 2a) (b) (4a, 4a)
𝑥1 +𝑥2
(c) (a- 2a) (d) none of
⟹ = 𝑥3 these
2
∴ 0≤ 2t ≤ 2 ⟹ 0≤ t ≤ 1
𝑡(𝑡 2 +1)
Since f(x)= 0, for all x ≤ 0
= 2
∴ f(x) is a many –one function
Which is an increasing function hence its
𝑒 𝑥 −𝑒 −𝑥
maximum value occurs at t= 1 Let y= ,𝑥 ≥ 0
𝑒 𝑥 +𝑒 −𝑥
𝑒𝑥 1+𝑦
⟹𝑒 −𝑥 = 1−𝑦
29. The point A on the parabola 𝒚𝟐 = 𝟒𝒙
for which |AC-AB | is maximum, where 1+𝑦 1 1+𝑦
⟹𝑒 2𝑥 = 1−𝑦 ⟹ 𝑥 = 2 log 1−𝑦
B≡ (𝟎, 𝒂) 𝒂𝒏𝒅 𝑪 ≡ (−𝒂, 𝟎) is
42
Challenging Mathematical Problems
⟹(∑3𝑖=1 𝑎𝑖 𝑎𝑖 + 1)2 −
(∑3𝑖=1 𝑎𝑖 2 )(∑4𝑖=1 𝑎𝑖 2 ) ≤ 0
ISI B.STAT/B.MATH
OBJECTIVE QUESTIONS & ⟹(𝑎1 𝑎2 + 𝑎2 𝑎3 + 𝑎3 𝑎4 )2 − (𝑎1 2 + 𝑎2 2 +
𝑎3 2 )(𝑎2 2 + 𝑎3 2 + 𝑎4 2 ) ≤ 0
SOLUTIONS
⟹(𝑎2 2 − 𝑎1 𝑎3 )2 + (𝑎3 2 − 𝑎2 𝑎4 )2 +
SET – 6 (𝑎2 𝑎3 − 𝑎1 𝑎4 )2 = 0
⟹(𝑎2 2 − 𝑎1 𝑎3 )2 = 0, (𝑎3 2 − 𝑎2 𝑎4 )2 =
1. If x ∊={1, 2, 3,……, 9}and 0, (𝑎2 𝑎3 − 𝑎1 𝑎4 )2 = 0
fn(x)=xxx……x (n digits),then 𝑎 𝑎 𝑎
⟹𝑎2 = 𝑎3 = 𝑎4
𝒇𝟐𝒏 (3)+𝒇𝒏 (2) = 1 2 3
𝟏 𝒓 𝟏
(a) 2𝒇𝟐𝒏 (1) 3. Let a = 𝒏! + ∑𝒏−𝟏
𝒓=𝟏 (𝒓+𝟏)!, b = 𝒎! +
(b) 𝒇𝟐𝒏 (1) ∑𝒎−𝟏
𝒓
then a+b equaals
𝒓=𝟏 (𝒓+𝟏)!
(c) 𝒇𝟐𝒏 (1)
(d)−𝒇𝟐𝒏 (4) (a) 0 (b) 1 (c)2
(d) none of these
Ans. (c)
Ans. (c)
2 3
𝑓𝑛 (𝑥) = 𝑥. 1 + 𝑥. 10 + 𝑥. 10 + 𝑟 𝑟+1−1 1 1
𝑛−1 (10𝑛 −1) 𝑥 𝑛 = = − (𝑟+1)!
⋯ 𝑥. 10 =𝑥 = 9 (10 − 1) (𝑟+1)! (𝑟+1)! 𝑟!
10−1
𝑟 1
2 3 2 ∴ ∑𝑛−1
𝑟=1 (𝑟+1)! = 1 − 𝑛! ⟹ 𝑎 = 1
∴𝑓𝑛 (3) + 𝑓𝑛 (2) = [9 (10𝑛 − 1)] +
2 1 1 𝑟
(10𝑛 − 1) = 9 (10𝑛 − 1)(10𝑛 − 1 + 2) = Similarly, 𝑚! + ∑𝑚
𝑟=1 (𝑟+1)! = 1
9
10𝑛 −1
= 𝑓2𝑛 (1)
9 ∴ a= 1, b=1 ⟹a+ b= 2
43
Challenging Mathematical Problems
𝟏 𝒏
4. If ∑𝒌𝒏=𝟏[𝟑 + 𝟗𝟎] =21, where [x] denotes triangle and [r] denotes the integral part
of x, then [r] + [-r]=
the integral part of x, then k=
(a) 0 (b) 1
(a) 84 (b) 80
(c) -1 (d) none
(c) 85 (d) none of these
Ans. (b)
Ans. (b)
1 𝑛 Since root of equation
21= ∑𝑘𝑛=1 [3 + 90] , 𝑤ℎ𝑒𝑟𝑒 𝑚 = 𝑘!
F(x)= 𝑥 2 + 2(𝑎 − 3)𝑥 + 9 = 0 lie between-
1 1 1 2 1 59
= [3 + 90] + [3 + 90] + ⋯ + [3 + 90] + 6 and 1
1 60 1 61 1 𝑘
[3 + 90] + [3 + 90] + ⋯ + [3 + 90] ∴(i) D≥ 0 (ii) f(-6)> 0 (iii) f(1) > 0 (iv) -
𝛼+𝛽 𝛼+𝛽
6< (v) 1 >
= (0 + 0 + ⋯ 𝑡𝑜 59 𝑡𝑒𝑟𝑚𝑠). +(1 + 1 + 2 2
⋯ 𝑡𝑜 (𝑘 − 59)𝑡𝑒𝑟𝑚𝑠) 27
Hence 6 ≤ 𝛼 < 4
∴ 21= k- 59⟹ k= 80.
∴|a| =6
5. Let f: R→R such that f(x) is 6−2 4 18
continuous and attains only rational value 𝑎3 = 2 + 3𝑑 = 2 + 3. =2+7=
21 7
at all real x and f(3)=4. If 1 1
1 1 − 1 2 3
𝒂𝟏 ,𝒂𝟐 ,𝒂𝟑 ,𝒂𝟒 ,𝒂𝟓 are in H.P., then 6 2
= 2 + 18. ( 21 ) = 2 − 7 = 14
ℎ18
∑𝟒𝒓=𝟏 𝒂𝒓 𝒂𝒓+𝟏 =
18 14
(a) f(5).𝒂𝟏 𝒂𝟓 (b) f(3).𝒂𝟒 𝒂𝟓 ∴ 𝑎3 ℎ18 = . = 12
7 3
(c) f(3).𝒂𝟏 𝒂𝟐 (d) f(2).𝒂𝟏 𝒂𝟑
7. If 𝒙𝟏 , 𝒙𝟐 , 𝒙𝟑 , 𝒙𝟒 , 𝒙𝟓 are in H.P. then
Ans. (a) 𝟏
(∑𝟒𝒌=𝟏 𝒙𝒌 𝒙𝒌+𝟏 ) is a root of equation
𝒙 𝟏 𝒙𝟓
Since f(x) is continuous and attains only
rational values (a) 𝒙𝟐 − 𝟑𝒙 + 𝟐 = 𝟎
(b) 𝒙𝟐 − 𝟓𝒙 − 𝟒 = 𝟎
∴ f(x)= constant= 4
(c) 𝒙𝟐 − 𝟗𝒙 + 𝟐𝟎 = 𝟎
∴ f(2) = f(3) = f(5) = 4 (d) 𝒙𝟐 − 𝟔𝒙 − 𝟖 = 𝟎
Since 𝑎1 , 𝑎2 , 𝑎3 , 𝑎4 , 𝑎5 are in H.P. Ans.(c)
∴ 𝑎1 𝑎2 + 𝑎2 𝑎3 + 𝑎3 𝑎4 + 𝑎4 𝑎5 = 𝑥1 , 𝑥2 , 𝑥3 , 𝑥4 , 𝑥5 are in H.P.
4𝑎1 𝑎5 = 𝑓(5). 𝑎1 𝑎5
∴∑4𝑘=1 𝑥𝑘 𝑥𝑘+1 = 𝑥1 𝑥2 + 𝑥2 𝑥3 + 𝑥3 𝑥4 +
6. If three successive terms of a G.P. with 𝑥4 𝑥5 = 4𝑥1 𝑥5
common ratio r >1 from the sides of a
44
Challenging Mathematical Problems
1 𝑧 2 −1
∴𝑥 ∑4𝑘=1 𝑥𝑘 𝑥𝑘+1 = 4 => 𝑧 2 + 𝑥 2 − 2𝑧𝑥 = 1 + 𝑥 2 => 𝑥 =
1 𝑥5 2𝑧
𝐹(𝑥 2 ) = 𝑥 2 (1 + 𝑥) ………..(2) 1 2𝑛 𝑛
= (− 2
)= 2
2 1+𝑛 𝑛 −1
From (1), F’(x)= f(x)
10. If f(x)= ae2x+ bex +cx satisfies the
∴ f(4)= F’(4)…………..(3) conditions f(0)= -1, f’(log 2)= 28,
From (1), 𝐥𝐨𝐠 𝟒
[𝒇(𝒙) − 𝒄𝒙]𝒅𝒙 =
𝟑𝟗
∫𝟎 𝟐
, then
F’(𝑥 2 ).2x= 2x+ 3𝑥 2
(a) a= 5, b=6, c= 3 (b) a= 5, b= - 6,
2+3𝑥 c= 0 (c) a= -5, b=6, c= 3 (d)
⟹ F’(𝑥 2 ) = [∵ 0 < 𝑥 < ∞ ∴ 𝑥 ≠ 0]
2
none
2+6
⟹ 𝐹 ′ (4) = = 4 [Putting x= 2] Ans. (b)
2
f’(x)= 2 𝑎𝑒 2𝑥 + 𝑏𝑒 𝑥 + 𝑐𝑥
𝒏 𝒏𝟐 −𝟏
(a) 𝒏𝟐 −𝟏 (b) 𝒏
𝒏 𝟏−𝒏𝟐
∴f’ (log 2)= 𝑎𝑒 log𝑒 4 + 𝑏𝑒 log𝑒 2 + 𝑐
(c) − 𝒏𝟐 −𝟏 (d) 𝒏
Given 8a+ 2b+c= 28……….(3)
Ans. (a)
log 4 39
Given, ∫0 (𝑎𝑒 2𝑥 + 𝑏𝑒 𝑥 ) 𝑑𝑥 =
2
Put z= x+√1 + 𝑥 2
𝑎 log 4 39
∴ z- x= √1 + 𝑥 2 ⟹ [ 𝑒 2𝑥 + 𝑏𝑒 𝑥 ] =
2 0 2
45
Challenging Mathematical Problems
𝑎 𝑎 39 2
⟹ 2 𝑒 log 16 + 𝑏𝑒 log 4 − (2 + 𝑏) = ∫ [𝑎2 + (4 − 4𝑎)𝑥 + 4𝑥 3 ] 𝑑𝑥
2
1
⟹ 15a + 6b= 39……….(4) 2 2
= 𝑎2 [𝑥] + (2 − 2𝑎)[𝑥 2 ]
1 1
Thus a= 5, b= -6, c= 0 2
+ [𝑥 4 ]
𝒅 𝒆𝐬𝐢𝐧 𝒙
1
11. Let 𝒇(𝒙) = ,𝒙 >
𝒅𝒙 𝒙
𝟐 = 𝑎2 + (2 − 2𝑎)3 + 15 , 𝐺𝑖𝑣𝑒𝑛, 𝑎2 − 6𝑎
𝟒 𝟐𝒆𝐬𝐢𝐧 𝒙
0. 𝐼𝑓 ∫𝟏 𝒙 𝒅𝒙 = 𝒇(𝒌) − 𝒇(𝟏), + 21 ≤ 12
√𝒏
15. 𝐋𝐭 ∑𝒏𝒓=𝟏 ( =
𝒏→∞ √𝒓 (𝟑√𝒓+𝟒√𝒏)𝟐
46
Challenging Mathematical Problems
=2 𝑑𝑦
For curve (1), 𝑑𝑥 = 𝑒 𝑥 (1 + 𝑥) > 0
[∵𝑒 cos 𝑥 . sin 𝑥 𝑖𝑠 𝑎𝑛 𝑜𝑑𝑑 𝑓𝑢𝑛𝑐𝑡𝑖𝑜𝑛 = 2]
∴ y is increasing.
47
Challenging Mathematical Problems
1 r = 3, 103 − 93 = 271
Required area = 2∫0 𝑥𝑒 𝑥 𝑥 𝑑𝑥 =
2[𝑥𝑒 𝑥 − 𝑒 𝑥 ] 10 = 2[(𝑒 − 𝑒) − (0 − 𝑒 0 )] = 2 ∴ r= 3.
20. A bag contains unlimited number of 22. N men and n women sit along a line
white, red, black, and blue balls. The alternatively in x ways and along a circle
number of ways of selecting 10 balls so in y ways such that x= 10y, then the
that there is at least one ball of each color number of ways in which n men can sit at
is a round table so that all shall not have
same neighbors is
(a) 180 (b0 270
(c) 192 (d) none (a) 6 (b) 12
(c) 36 (d)
Ans. (d) Number of ways= coefficient of none
𝑋10 𝑖𝑛 (𝑋 + 𝑋 2 + 𝑋 3 + ⋯ )4
𝑥 2.⎿𝑛⎿𝑛
Ans. (b) = = 2𝑛
= coefficient of 𝑋10 𝑖𝑛 𝑋 4 (1 − 𝑋)−4 𝑦 ⎿𝑛−1 ⎿𝑛
(a) ∑𝟏𝟎
𝒓=𝟔 𝟏𝟎𝒄𝒓 . 𝟑
𝒓
(b) (a) 512 (b) 640
∑𝟓𝒓=𝟏 𝟏𝟎𝒄𝒓 . 𝟐𝒓 (c) (c) 672 (d) none
∑𝟏𝟎
𝒓=𝟔 𝟏𝟎𝒄𝒓 (d) none Ans. (c) We have to put 2 twice in each
numbers, so any 2 out of the 7 places can be
Ans. (d) Since total number of ways
chosen in 7𝑐2 ways. The remaining 5 places
predicting the results of one match is 3 , so
can be filled with the other two numbers in
results of 10 match is 310 , now number of
25 ways.
ways that the result of one match is correct
is 1 and also number of ways to predict The required numbers of numbers are 7𝑐2 ×
wrongly of one match is 2 . 25 = 672.
No. of ways to predict wrongly exactly r
matches =10𝑐𝑟 . 2𝑟 110−𝑟
𝒌 𝑴−𝒌 𝑴−𝒌
26. The value of {∑𝟏𝟎𝟎
𝒊=𝟎 ( 𝒊 )(𝟏𝟎𝟎−𝒊)(𝟏𝟎𝟎−𝒊)} /
10
∴ The required number is 3 − 𝑴
(𝟏𝟎𝟎 ), where M - k > 100, k >100, is
∑4𝑟=1 10𝑐𝑟 . 2𝑟
𝒌 𝑴
(a) (b)
𝑴 𝒌
𝒌
24. Let 1 to 20 are placed in any around a (c) 𝑴𝟐 (d) none
circle. Then the sum of some 3 𝑘 𝑀−𝑘 𝑀−𝑘
𝑀
consecutive numbers must be at least Ans. (a) {∑100
𝑖=0 ( 𝑖 )(100−𝑖)(100−𝑖 )}/ (100)
𝑘 𝑀
Ans. (c) Suppose 𝑥1 , 𝑥2 , … . , 𝑥20 be the 𝑘
𝑀
(100) 𝑀
.100 (100 ) 𝑘
= 𝑀−100 . 𝑀 − 𝑀 = 𝑀.
numbers placed around the circle. Now the (100) (𝑀−100) (100)
Thus from Pigon hole principle that at least (a) Odd (b) even
one of the sums must be ≥ 32. (c) prime (d) none
25. The number of different seven–digit Ans. (b) since n is odd, let n= 2m+ 1, where
numbers can be written using only there m is a non-negative integer.
digits 1, 2, 3 under the condition that the
digit 2 occurs twice in each number is
49
Challenging Mathematical Problems
𝝅 𝝅
OBJECTIVE QUESTIONS &
(a) 𝟒 (b) 𝟐
SOLUTIONS
𝝅 𝝅
(c) - 𝟐 (d) 𝟑
SET – 7
√𝑛−√𝑛−1
Ans. (b) 𝑡𝑛 = sin−1 ( )
√𝑛(𝑛+1)
Ans. (a) 2𝑦 𝑧
⇒ x. + 3. 3 = 3
2
𝑟 = 𝑟2 + 𝑟3 − 𝑟1
Applying AM ≥ GM,
Δ Δ Δ Δ
= s−b + s−c − s−a 𝑥+2.
2𝑦
+3.
𝑧
𝑦 𝑧
s
So, 2 3
≥ 6√𝑥( 2)2 (3)3
1+2+3
1 1 1 1
⟹ + = + x𝑦 2 𝑧 3
s s−a s−b s−c 3
⇒ (6)6 ≥ 22 .33
2s − a s(s − a)
⟹ = ⇒ x𝑦 2 𝑧 3 ≤ 16.
27
2s − b − c (s − b)(s − c)
2s − a A s 𝒅𝒚
⟹ = cot 2 ⟹ 5. If y(t) is a solution of (1+t) 𝒅𝒕 − 𝒕𝒚 = 𝟏
a 2 a
1 A s and y(t) then y(1) equals
= (cot 2 + 1) ⟹
2 2 a (a) ½ (b) e + ½ (c) e+ ½
1
∊ ( , 2) (d)- ½
2
𝑑𝑦 𝑡 1
Ans:- (d) 𝑑𝑡 − 1+𝑡 𝑦 = 1+𝑡
𝑡
3. If 𝒂𝟏 , 𝒂𝟐 , … . , 𝒂𝒏 are positive real nos,
𝒂 𝒂 𝒂 𝒂
∴I.F. = 𝑒 − ∫1+𝑡𝑑𝑡 = 𝑒 −(𝑡−log(1+𝑡)) =
then 𝒂𝟏 + 𝒂𝟐 + ⋯ + 𝒂𝒏−𝟏 + 𝒂𝒏 is always 𝑒 −𝑡 . (1 + 𝑡)
𝟐 𝟑 𝒏 𝟏
𝑎1 𝑎𝑛−1 𝑎𝑛
∴y𝑒 −𝑡 . (1 + 𝑡) = −𝑒 −𝑡
∴ 𝑎 +⋯+ +𝑎 ≥𝑛
2 𝑎𝑛 1 1
∴y= − 1+𝑡 ∴y(1)= - ½.
𝟐 𝟑
4. The maximum possible value of x𝒚 𝒛
subject to the condition xyz ≥ 𝟎 and
51
Challenging Mathematical Problems
d) Neither a) nor b) Ans:- (a) if x < 0, LHS = -ve but RHS= +ve
= (𝛼 2 + 1)( 𝛽 2 + 1) 𝑥. 2
1⁄
𝑥
1 1
+ 𝑥 . 2𝑥 ≥ 2√2 ⁄𝑥+𝑥 ≥ 2. √22 =4
1⁄ 1
⇒ 𝑥. 2 𝑥 = . 2𝑥 ; so, x= 1.
𝑥
𝟏 𝟏
7. Let u = (√𝟓 − 𝟐) ⁄𝟑 − (√𝟓 + 𝟐) ⁄𝟑 and
𝟏 𝟏
v= (√𝟏𝟖𝟗 − 𝟖) ⁄𝟑 − (√𝟏𝟖𝟗 + 𝟖) ⁄𝟑,
9. Let f (x) and g (x) be functions, which
Then for each positive integer n, 𝒖𝒏 +
take integers as arguments. Let
𝒗𝒏+𝟏 = ? f (x+ y) = f (x) + f (y) + 8 for all integers x
(a) -1 (b) 0 and y. Let f (x) = x for all negative
(c) 1 (d) 2 numbers x and let g(8)=17, then f (0)=?
⇒ (u-1)( 𝑢2 − 𝑢 + 4)= 0
𝟐𝟎𝟎𝟕.𝟐𝟎𝟎𝟔.𝟐𝟎𝟎𝟒.𝟐𝟎𝟎𝟑
𝑢2 − 𝑢 + 4 is always +ve. So, u= 1 10. Let x = [ 𝟏 ], where [x]
× (𝟐𝟎𝟎𝟓)𝟒
𝟑
Similarly 𝑣 3 + 15𝑣 + 16 = 0 denotes the greatest integer integer less
((𝒙+𝟏).𝒙𝟐 )+𝟏
⇒ (v +1)( 𝑣 2 − 𝑣 + 16)= 0 than or equal to x. then (𝒙𝟐 +𝟏)
is
2 1 1 2
8. The number of real values of x = [3(1+2005)(1+2005 )(1-2005)(1-2005)]
satisfying the equation
4 1
𝟏 𝟏 = [3(1-(2005)2)( 1-(2005)2 )]
𝒙. 𝟐 ⁄𝒙 + 𝒙 . 𝟐𝒙 = 4 is / are
⇒ x=2.
(a) 1 (b) 2
(c) 3 (d)4
52
Challenging Mathematical Problems
53
Challenging Mathematical Problems
𝟏
(a) k < - 𝟐 (b) k > 4
𝟏 |𝒙|
(c) − 𝟐 ≤ 𝒌 ≤ 𝟒 (d) 19. Total number of solutions of sinx = 𝟏𝟎
𝟏
≤𝒌≤𝟓 is
𝟐
The remainder is same as the remainder 21. A particle has an angular speed of 3
obtained by dividing 33 with 15, i.e., 3. rad /sec and the axis of rotation passes
through the point (1, 2, 2) and (1, 2, -2),
then the velocity of the particle at the
𝒆𝒕𝒂𝒏𝒙 −𝒆𝒙
point P(3, 6, 4) is
18. The value of 𝐋𝐭 is
𝒙→𝟎 𝒕𝒂𝒏𝒙−𝒙 𝟑 𝟑
(a) (22, 8, -2) (b) (𝟐𝟐, 𝟎, 𝟐)
√𝟏𝟕 √𝟏𝟕
(a) 0 (b) 1 𝟑
(c) (𝟐𝟐, −𝟖, −𝟐) (d)
(c) e (d) none √𝟏𝟕
𝟑
(𝟐𝟐, −𝟖, 𝟐)
𝑒 tan 𝑥 −𝑒 𝑥 𝑥 𝑒 tan 𝑥 −1 √𝟏𝟕
Ans. (b) Lt = Lt 𝑒 ( tan 𝑥− 𝑥 ) =
𝑥→0 tan 𝑥− 𝑥 𝑥→0
𝑒 0 . log 𝑒 𝑒 = 1 Ans:- (c)
54
Challenging Mathematical Problems
55
Challenging Mathematical Problems
56
Challenging Mathematical Problems
∴ 0≤ 𝑐𝑜𝑠 2 𝑛! 𝑥𝜋 < 1 ∴ k= 2
𝟏 𝟑
Thus f(x) = 0, when x is rational (a) 𝟖 (b) 𝟖
𝟓
= 1, when x is irrational (c) 𝟖 (d) none
28. The normal at any point P (𝒕𝟐 , 𝟐𝒕) on 3200 9100 (1 + 8)100
the parabola 𝒚𝟐 = 𝟒𝒙 meets the curve = =
8 8 8
again at Q, the area of 𝛥POQ, O being the 1 + 100𝐶1 8 + 100𝐶1 82 + ⋯ + 8100
𝒌 =
origin is |𝒕| (𝟏 + 𝒕𝟐 )(𝟐 + 𝒕𝟐 ) then 8
1
= + 𝑎𝑛 𝑖𝑛𝑡𝑒𝑔𝑒𝑟
(a) k > 2 (b) k=2 8
(c) k < 2 (d) k= 1 3200 1
∴{ }=8
8
Ans. (b) Given P ≡ (𝑡 2 , 2𝑡)
30. Which of the following function does
Given parabola is 𝑦 2 = 4𝑥 ……….. (1) not obey mean value theorem in [0, 1]
Here a= 1. 𝟏 𝟏 𝟐 𝟏
(a) f(x)= 𝟐 - x, x < ½; f(x)=(𝟐 − 𝒙) , 𝒙 ≥ 𝟐
Let Q=( 𝑡1 2 , 2𝑡1) 𝐬𝐢𝐧 𝒙
(b) f(x) = , 𝒙 ≠ 𝟎; 𝒇(𝒙) = 𝟏, 𝒙 = 𝟎
𝒙
Since normal at P meet the curve again at Q.
(c) f(x)= x |x|
2 𝑡 2 +2 (d) f(x)= |x|
∴ 𝑡1 = −𝑡 − 𝑡 = ………… (2)
𝑡
Ans. (a) Let f(x) = 𝑥 3 − 3𝑥 + 𝑘
Now O≡ (0, 0), P≡ (𝑡 2 , 2𝑡), Q=( 𝑡1 2 , 2𝑡1 )
Then f’(x) = 3(𝑥 2 − 1) < 0 𝑖𝑛 (0, 1)
Given,
𝑘
⟹ f’(x) has no root in (0, 1)
|𝑡|
(1 + 𝑡 2 )(2 + 𝑡 2 ) = 𝑎𝑟𝑒𝑎 𝑜𝑓 𝛥𝑃𝑂𝑄
But f(x) = 0 has two distinct roots 𝛼and 𝛽 in
1 (0, 1)
= 2 |𝑡 2 . 2𝑡1 − 2𝑡. 𝑡1 2 | = |𝑡 2 𝑡1 − 𝑡𝑡1 2 | =
2
𝑡 2 +2 (𝑡 2 +2) ⟹ f’(x)= 0 has at least one root in (𝛼, 𝛽).
|−𝑡 2 ( )−𝑡 |
2 𝑡2
(𝑡 2 +2)
= (𝑡 2 + 2) |𝑡 + | = (𝑡 2 + 2) |𝑡 +
𝑡
(𝑡 2 +2) (1+𝑡)2
| = (𝑡 2 + 2)2
𝑡 |𝑡|
57
Challenging Mathematical Problems
ISI B.STAT/B.MATH 𝑥
𝑡𝑟+1 =
(𝑟𝑥 + 1){(𝑟 + 1)𝑥 + 1}
OBJECTIVE QUESTIONS &
(𝑟 + 1)𝑥 + 1 − (𝑟𝑥 + 1)
SOLUTIONS =
(𝑟𝑥 + 1)[(𝑟 + 1)𝑥 + 1]
SET – 8 1 1
= −
(𝑟𝑥 + 1) (𝑟 + 1)𝑥 + 1
𝑛−1
1
1. If [x] denotes the integral part of x, ∴ 𝑆𝑛 = ∑ 𝑡𝑟+1 = 1, 𝑥 ≠ 0 = 0, 𝑥
𝐬𝐢𝐧[𝒄𝒐𝒔𝒙] 𝑛𝑥 + 1
then 𝐋𝐭 = 𝑟=0
𝒙→𝟎 𝟏+[𝒄𝒐𝒔𝒙] =0
𝐬𝐢𝐧 𝟏
(a) 0 (b) 1 (c) 1
𝟐 ∴ Lt 𝑆𝑛 = Lt (1 − 𝑛𝑥+1)
𝑛→∞ 𝑛→∞
(d) does not exist
1, 𝑥 ≠ 0
Ans. (a) Lt [cos 𝑥] = 0 Thus, f(x)= {
𝑥→0−0 0, 𝑥 = 0
Ans. (c)
58
Challenging Mathematical Problems
59
Challenging Mathematical Problems
60
Challenging Mathematical Problems
𝒙 𝟏 3(sin 𝑥−𝑠𝑖𝑛4 𝑥) 𝜋
9. Let f(x)= ∫𝟎 𝒕 𝒔𝒊𝒏 𝒕 𝒅𝒕, then the number ∴ f(x)=[ ] = 3, 𝑥 ≠ = 3, 𝑥 =
sin 𝑥−𝑠𝑖𝑛𝑛 𝑥 2
𝜋
of points of discontinuity of f(x) in (0, 𝜋) is
2
61
Challenging Mathematical Problems
⟹ |𝑓 ′ (𝑥)| ≤ 0
Number of such values of x= p- 1+p- 1= 2p
-2 ⟹|𝑓 ′ (𝑥)| ≤ 0 for all real x
(a) |b|=|a| (b) |b|≤ |𝒂| (c) has a value 𝟗𝒆𝟐 − 𝟒𝒆 when x=e
(c) |b|≥ |𝒂| (d) none of these (d) has a differential coefficient 27e -8
when x= e
Ans. (b)
𝑑 𝑥3
𝑠𝑒𝑐 4 𝜃 𝑡𝑎𝑛4 𝜃 1 𝑠𝑒𝑐 2 𝜃 − 𝑡𝑎𝑛2 𝜃 Ans. (c) 𝑑𝑥 ∫𝑥 2 𝑓(𝑡) 𝑑𝑡 = 𝑓(𝑥 3 ). 3𝑥 2 −
+ = =
𝑎 𝑏 𝑎+𝑏 𝑎+𝑏 𝑓(𝑥 2 ). 2𝑥
𝑠𝑒𝑐 2 𝜃
⟹ 𝑎(𝑎+𝑏) [(𝑎 + 𝑏)𝑠𝑒𝑐 2 𝜃 − 𝑎] + = 𝑙𝑜𝑔𝑥 3 . 3𝑥 2 − log 𝑥 2 . 2𝑥
𝑡𝑎𝑛2 𝜃
= 9𝑥 2 𝑙𝑜𝑔𝑥 − 4𝑥 𝑙𝑜𝑔𝑥
[(𝑎 + 𝑏)𝑡𝑎𝑛2 𝜃 + 𝑏] = 0 = 𝑥 𝑙𝑜𝑔𝑥(9𝑥 − 4)
(𝑎+𝑏)𝑏
⟹ a𝑡𝑎𝑛2 𝜃 + 𝑏𝑠𝑒𝑐 2 𝜃 = 0
62
Challenging Mathematical Problems
The middle terms of the A.P., G.P. and H.P. (a) 3 (b) 6 (c) 2
are 𝛼𝑛 , 𝛽𝑛 𝑎𝑛𝑑 𝛾𝑛 respectively (d) none of these.
63
Challenging Mathematical Problems
100×101
Ans. (b) 𝑥 𝑛 = 𝑥1+2+3+⋯+100 = 𝑥 2 = Thus a= b= c.
𝑥 5050 21. If p, q be non zero real numbers and
𝟏
⟹ n= 5050 f(x)≠ 0 in [0, 2] and ∫𝟎 𝒇(𝒙). (𝒙𝟐 + 𝒑𝒙 +
𝟐
𝟐 𝒒)𝒅𝒙 = ∫𝟎 𝒇(𝒙). (𝒙𝟐 + 𝒑𝒙 + 𝒒)𝒅𝒙 = 𝟎
19. If p, q, r, s ∊ R, then equation (𝒙 +
𝒑𝒙 + 𝟑𝒒)(- 𝒙𝟐 + 𝒓𝒙 + 𝒒)(- 𝒙𝟐 + 𝒔𝒙 − 𝟐𝒒)= then equation 𝒙𝟐 + 𝒑𝒙 + 𝒒= 0 has
0 has (a) two imaginary roots
(a) 6 real roots (b) no root in (0, 2)
(b) at least two real roots (c) one root in (0,1) and other in (1,2)
(c) 2 real and 4 imaginary roots (d) one root in (-∞, 𝟎) and other in (2,∞)
(d) 4 real and 2 imaginary roots. Ans. (c)
Ans. (b) Let F(x) = ∫ 𝑓(𝑥)(𝑥 2 + 𝑝𝑥 + 𝑞)𝑑𝑥,
𝐷1 + 𝐷2 + 𝐷3 = 𝑝2 − 12𝑞 + 𝑟 2 + 4𝑞 + Then according to question
𝑠 2 + 8𝑞 = 𝑝2 + 𝑟 2 + 𝑠 2 ≥ 0
F(1) –F(0) = 0, F(2) – F(1) = 0
⟹ at least one of 𝐷1 , 𝐷2 , 𝐷3 ≥ 0
∴ F(0) = F(1) and F(1)= F(2)
20. If a, b, c, d, are four non-zero real
numbers such that (𝒅 + 𝒂 − 𝒃)𝟐 + Hence, equation F’(x)= 0 i.e. equation
(𝒅 + 𝒃 − 𝒄)𝟐 = 0 and roots of the equation f(x). (𝑥 2 + 𝑝𝑥 + 𝑞) = 0
a(b-c) 𝒙𝟐 + b(c-a)x + c(a-b)= 0 and real
i.e., equation 𝑥 2 + 𝑝𝑥 + 𝑞 = 0 has at least
and equal, then a, b, c
one root (here exactly one root) in (0, 1) and
(a) are equal (b) are not equal (c) are exactly one root in (1, 2).
zero (d) none of the above
22. If a, b, c, ∊ R, a ≠ 0 and (𝒃 − 𝟏)𝟐 <
Ans. (a) Equation 𝑎(𝑏 − 𝑐)𝑥 2 + 4𝒂𝒄, then the number of roots of the
𝑏(𝑐 − 𝑎)𝑥 + 𝑐(𝑎 − 𝑏)= 0 has equal roots system of equation (in three unknowns
𝒙 𝟏 , 𝒙𝟐 , 𝒙𝟑 )
2𝑎𝑐
⟹ b= 𝑎+𝑐 …….(1)
𝒂𝒙𝟏 𝟐 + 𝒃𝒙𝟏 + 𝒄 = 𝒙𝟐
(𝑑 + 𝑎 − 𝑏)2 + (𝑑 + 𝑏 − 𝑐)2 =0
𝒂𝒙𝟐 𝟐 + 𝒃𝒙𝟐 + 𝒄 = 𝒙𝟑
⟹ a-b = b- c = -d ⟹ 2b= a+ c
𝒂𝒙𝟑 𝟐 + 𝒃𝒙𝟑 + 𝒄 = 𝟏 is
……(2)
4𝑎𝑐 (a) 0 (b) 1 (c)
⟹ 𝑎+𝑐 = a +c ⟹ (𝑎 − 𝑐)2 = 0⟹ a= c
2 (d) 3
From (2), b= a Ans. (a) Let f(x) = 𝑎𝑥 2 + (𝑏 − 1)𝑥+c
64
Challenging Mathematical Problems
As (𝑏 − 1)2 − 4𝑎𝑐 < 0. From (A) and (B) all positive value of x are
given by x≤ 0 𝑜𝑟 𝑥 ≥ 4
∴ 𝑎𝑓(𝑥1 ), 𝑎𝑓(𝑥2 ), 𝑎𝑓(𝑥3 ) > 0.
23. If α, 𝛽 are the roots of the equation (a) only for x ≥0 (b) only for x≤ 0
𝒙𝟐 -ax +b= 0 and 𝑨𝒏 = 𝜶𝒏 + 𝜷𝒏 then (c) for all real x (d) only for x ≠0
which of the following is true?
Ans. (c)
(a) 𝑨𝒏+𝟏 = 𝒂𝑨𝒏 + 𝒃𝑨𝒏−𝟏
(b) 𝑨𝒏+𝟏 = 𝒃𝑨𝒏 + 𝒂𝑨𝒏−𝟏 Since A.M. ≥ G.M
𝜋
Given, 𝐴𝑛 = 𝛼 𝑛 + 𝛽 𝑛 [∵ least value of sin (𝑥 + 4 ) = −1]
Now, 𝐴𝑛+1 = 𝛼 𝑛+1 + 𝛽 𝑛+1 26. How many different nine digit
= (𝛼 𝑛 + 𝛽 𝑛 )( 𝛼 +𝛽)-𝛼𝛽(𝛼 𝑛−1 + 𝛽 𝑛−1) numbers can be formed from the number
223355888 by rearranging its digits so
= a𝐴𝑛 + 𝑏𝐴𝑛−1 that the odd digits occupy even positions?
65
Challenging Mathematical Problems
Number of odd digits = 4, number of even 29. Range of f(x) = 𝒔𝒊𝒏𝟐𝟎 𝒙 + 𝒄𝒐𝒔𝟒𝟖 𝒙 is
digits= 5
(a) [0, 1] (b) (0, 1)
Number of even places= 4 (c) (0, ∞) (d) none of
these
Odd digits can be arranged in even paces in
|4̲
ways. Even digits can be arranged in Ans. (b)
|2̲|2̲
|5̲
remaining 5 places in |2̲|3̲ ways 0 ≤ 𝑠𝑖𝑛2 𝑥 ≤ 1 ⟹ 𝑠𝑖𝑛20 𝑥 ≤ 𝑠𝑖𝑛2 𝑥
[∵
27. For 2≤ 𝒓 ≤ 𝒏, (𝒏𝒓) + 𝟐(𝒓−𝟏
𝒏 𝒏
) + (𝒓−𝟐)= 𝑠𝑖𝑛20 𝑥 𝑎𝑛𝑑 𝑐𝑜𝑠 48 𝑥 𝑐𝑎𝑛𝑛𝑜𝑡 𝑏𝑒 𝑧𝑒𝑟𝑜 𝑎𝑡 𝑎 𝑡𝑖𝑚𝑒
]
(a) (𝒏+𝟏
𝒓−𝟏
) (b) 2(𝒏+𝟏
𝒓+𝟏
)
⟹ 0 < f(x) ≤ 1. Hence range of f(x) = (0,
(c) 2(𝒏+𝟐
𝒓
) (d) (𝒏+𝟐
𝒓
)
1)
Ans. (d)
30. Let x, y, z = 105, where x, y, z ∊N.
(𝑛𝑟) 𝑠𝑡𝑎𝑛𝑑𝑠 𝑓𝑜𝑟 𝑛𝐶𝑟 Then number of ordered triplets (x, y, z)
satisfying the given equation is:
Now 𝑛𝐶𝑟 + 2𝑛𝐶𝑟−1 + 𝑛𝐶𝑟−2
(a) 15 (b) 27
= (𝑛𝐶𝑟 + 𝑛𝐶𝑟−1 ) + (𝑛𝐶𝑟−1 + 𝑛𝐶𝑟−2 ) (c) 6 (d) none of these
𝟐 105= 3× 5 × 7
28. If ∑𝟏𝟎
𝒊=𝟏 𝐬𝐢𝐧
−𝟏
𝑿𝒊 = 𝟓𝝅, 𝒕𝒉𝒆𝒏 ∑𝟏𝟎
𝒊=𝟏 𝑿𝒊 =
When no 1 is taken as a solution, number of
(a) 0 (b) 5
solutions= |3̲ = 6
(c) 10 (d) none of these
When only 1’ s taken, number of solutions=
Ans. (c)
3𝐶2 . |3̲ = 18
10
𝜋 When two 1’s are taken, number of solutions
∑ sin−1 𝑥𝑖 = 5𝜋 = 10.
2 |3
̲
𝑖=1 = 3𝐶1 . |2̲ = 3
𝜋
⟹ sin−1 𝑥𝑖 = 2 , ∀𝑖 ⟹ 𝑥𝑖 = 1 ∀ 𝑖 ⟹
∴ Reqd. number= 6+18+3 =27
∑10 2
𝑖=1 𝑥𝑖 = 1
66
Challenging Mathematical Problems
67
Challenging Mathematical Problems
𝑥 2 +1 𝒙𝟐 −𝟐
−𝑡 2 ′ (𝑥) 5. If ∫ =
𝑓(𝑥) = ∫ 𝑒 𝑑𝑡 𝑓 𝒙𝟐 +𝟐
𝒅𝒙
(𝒙𝟒 +𝟓𝒙𝟐 +𝟒) 𝐭𝐚𝐧−𝟏 ( )
𝑥2 𝒙
2 2 4
= 𝑒 −(𝑥 +1) . 2𝑥 − 𝑒 −𝑥 . 2𝑥 𝒍𝒐𝒈|𝒇(𝒛)| + 𝒄, then
2𝑥
= (𝑥 2 +1)2 [1 (a) f(z) = 𝐭𝐚𝐧−𝟏 𝒛 , 𝒘𝒉𝒆𝒓𝒆 𝒛 = √𝒙 + 𝟐
𝑒 𝟐
4 +(𝑥 2 +1)2 (b) f(z) = 𝐭𝐚𝐧−𝟏 𝒛 , 𝒘𝒉𝒆𝒓𝒆 𝒛 = 𝒙 + 𝒙
− 𝑒 −𝑥 ]
𝒙+𝟐
2𝑥 (c) f(z)= 𝐬𝐢𝐧−𝟏 𝒛 , 𝒘𝒉𝒆𝒓𝒆 𝒛 =
2𝑥 2 +1 𝒙
= 2 2 [1 − 𝑒 ]
𝑒 (𝑥 +1) (d) none of these
2 +1
2(𝑒 2𝑥 − 1)
= 2 2 (−𝑥) Ans. (b)
𝑒 (𝑥 +1)
𝑥 2 −2
But 𝑒 2𝑥 2 +1
>1 I= ∫ 𝑥2 +2
𝑑𝑥
(𝑥 4 +5𝑥 2 +4) tan−1 ( )
𝑥
68
Challenging Mathematical Problems
Ans. (d) 1 4
𝑁𝑜𝑤 𝐼 = ∫ 𝑧 −5/4 𝑑𝑧 = 𝑧 −1/4 + 𝐶
3 3
1
I= ∫ 𝑥 log (1 + 𝑥) 𝑑𝑥 = ∫ 𝑥 𝑙𝑜𝑔 (𝑥 + 4 𝑥 − 1 1/4
= .( ) +𝐶
1)𝑑𝑥 − ∫ 𝑥 𝑙𝑜𝑔 𝑥 𝑑𝑥 3 𝑥+2
𝑥2 1 𝑥2 𝑥2
= log(𝑥 + 1) − 2 ∫ 1+𝑥 𝑑𝑥 − 𝑙𝑜𝑔𝑥 +
2 2
1 𝑥2 𝑥2 𝟏+𝒏.𝒙𝒏−𝟏 −𝒙𝟐𝒏
∫ 𝑥 𝑑𝑥 = log(𝑥 + 1) − 𝑙𝑜𝑔𝑥 − 8. ∫ 𝒆𝒙 𝒅𝒙=
2 2 2 (𝟏−𝒙𝒏 )√𝟏−𝒙𝟐𝒏
1 1 1
2
∫ (𝑥 − 1 + 𝑥+1) 𝑑𝑥 + 2 ∫ 𝑥 𝑑𝑥
𝒆𝒙 √𝟏−𝒙𝟐𝒏 𝒆𝒙 √𝟏−𝒙𝒏
(a) +𝒄 (b) +𝒄
𝟏−𝒙𝟐𝒏 𝟏−𝒙𝒏
𝑥2 𝑥2 1
= log(𝑥 + 1) − log 𝑥 − log(𝑥 + 1) 𝒆𝒙 √𝟏−𝒙𝟐𝒏
2 2 2 (c) +𝒄 (d) none of these
𝟏−𝒙𝒏
𝑥
+ +𝑐
2 Ans. (c)
1 𝑛𝑥 𝑛−1 1−𝑥 𝑛
𝐿= . √
(1−𝑥 𝑛 )2 1+𝑥 𝑛
] 𝑑𝑥 = ∫ 𝑒 𝑥 {𝑓(𝑥) +
2
𝑓 ′ (𝑥)]𝑑𝑥,
𝒅𝒙
7. ∫ 𝟑 𝟓 =
(𝒙−𝟏)𝟒 (𝒙+𝟐)𝟒
1 − 𝑥 2𝑛
𝟏 𝑤ℎ𝑒𝑟𝑒 𝑓(𝑥) = √ = 𝑒 𝑥 𝑓(𝑥) + 𝐶
𝟒 𝒙−𝟏 𝟒 𝟒 𝒙−𝟏 1 − 𝑥𝑛
(a) 𝟑 (𝒙+𝟐) + 𝒄 (b) 𝟑 √𝒙+𝟐 + 𝒄
1 − 𝑥 2𝑛
𝟏 = 𝑒𝑥 +𝐶
𝒙+𝟐 𝟒 1 − 𝑥𝑛
(c) (𝒙−𝟏) + 𝒄 (d) none of these
(𝒙+𝟏)
9. ∫ 𝒙(𝟏+𝒙𝒆𝒙)𝟐 𝒅𝒙 = 𝒍𝒐𝒈|−𝒇(𝒙)| + 𝒇(𝒙) +
Ans. (a)
𝒄 𝒕𝒉𝒆𝒏 𝒇(𝒙) =
𝑑𝑥
𝐼= ∫ 5 𝟏 𝟏
(a) 𝒙+𝒆𝒙 (b) 𝒙+𝒙𝒆𝒙
𝑥+2 4
(𝑥 − 1)2 (
𝑥 − 1) 𝟏
(c) (𝟏+𝒙𝒆𝒙)𝟐 (d) none of
𝑥+2 these
𝑝𝑢𝑡 𝑧 = , 𝑡ℎ𝑒𝑛
𝑥−1
Ans. (b)
(𝑥 − 1). 1 − (𝑥 − 2). 1
𝑑𝑥 = 𝑑𝑥 Put z= x𝑒 𝑥 , then dz = (𝑒 𝑥 + 𝑥𝑒 𝑥 ) dx
(𝑥 − 1)2
3 𝑑𝑧 1 1 1
= − 𝑑𝑥 I= ∫ 𝑧(1+𝑧)2 = ∫ [𝑧 − 1+𝑧 − (1+𝑧)2 ] 𝑑𝑧 =
(𝑥 − 1)2
𝑧 1 𝑥𝑒 𝑥 1
log 1+𝑧 + 1+𝑧 + 𝐶 = 𝑙𝑜𝑔 |1+𝑥𝑒 𝑥 | + 1+𝑥𝑒 𝑥 + 𝑐
69
Challenging Mathematical Problems
(a) 1 (b) 0
𝝅
𝐬𝐢𝐧(𝟐𝒏−𝟏)𝒙 𝜷𝒏+𝟏 −𝜶𝒏+𝟏
10. If 𝑰𝒏 = ∫𝟎 𝟐 𝒅𝒙, 𝒂𝒏𝒅 𝒂𝒏 = (c) (d) none of
𝒔𝒊𝒏𝒙 𝒏+𝟏
𝝅
𝐬𝐢𝐧 𝒏𝜽 these
∫𝟎 ( 𝐬𝐢𝐧 𝜽 ) 𝟐 𝒅𝜽, 𝒕𝒉𝒆𝒏 𝒂𝒏+𝟏 − 𝒂𝒏 =
𝟐
Ans. (b)
(a) 𝑰𝒏 (b) 2𝑰𝒏
(c) 𝑰𝒏 + 𝟏 (d) 0 Put z = g(f(x)), then dz = g’(f(x)) f’(x) dx
70
Challenging Mathematical Problems
𝑟𝜋 Ans.(a)
x will be real only when sin =0
𝑛
Ans. (c) 𝑏2
For real roots ac ≤ 4
71
Challenging Mathematical Problems
(a) A.P. (b) G.P. (c) H.P. (d) none of (a) -1 (b) 0
these. (c) 1 (d) none of these
Ans. (a) (𝑥 − 𝑎1 + 𝑎2 )2 + (𝑥 − 𝑎2 + Ans. (c) Constant term
𝑎3 )2 + ⋯ + (𝑥 − 𝑎𝑛−1 + 𝑎𝑛 )2 = 0
1 1 1 1
c = 1.2 + 2.3 + ⋯ + 𝑛(𝑛+1) = 1 − 𝑛+1
⟹𝑎1 − 𝑎2 = 𝑎2 − 𝑎3 = ⋯ = 𝑎𝑛−1 −
𝑎𝑛 = 𝑥 1
lim 𝑐 = lim (1 − )=1
⟹ 𝑎1 , 𝑎2 , 𝑎3 , … . 𝑎𝑛 are in A.P. with 𝑛→∞ 𝑛→∞ 𝑛+1
common difference x. 𝝅
21. If 𝜽𝒊 ∊ [𝟎, 𝟔 ] , 𝒊 = 𝟏, 𝟐, 𝟑, 𝟒, 𝟓 and
19. Let f(x) = 𝒂𝒙𝟐 + 𝒃𝒙 + 𝒄 and g(x) = 𝒔𝒊𝒏𝜽𝟏 𝒛𝟒 + 𝒔𝒊𝒏𝜽𝟐 𝒛𝟑 + 𝒔𝒊𝒏𝜽𝟑 𝒛𝟐 +
af(x) + bf′(x) + cf″(x) If f(x) > 0 for all x , 𝒔𝒊𝒏𝜽𝟒 𝒛 + 𝒔𝒊𝒏𝜽𝟓 = 𝟐 then z satisfies
then the sufficient condition for g(x) to be
𝟑 𝟏 𝟏 𝟑
> 0 v x is (a) |z|> 𝟒 (b) |z|< 𝟐 (c) 𝟐
< |z|< 𝟒 (d)
none of these
72
Challenging Mathematical Problems
Ans.(a) At x = 2, 3𝑥 − 𝑥 + 2 = 9
𝜋
Since 0≤ 𝜃𝑖 , ≤ For x >2, 3𝑥 − 𝑥 + 2 > 9
6
When|𝑧| < 1, from (1), 23. The number of real roots of the
𝟏
1 1 equation (𝟗 + 𝒔𝒊𝒏𝒙)𝟏−𝒙 + (𝟏𝟎 +
2 < 2 . 1−|𝑧| 𝟏 𝟏
𝒔𝒊𝒏𝒙)𝟏−𝒙 = (𝟏𝟏 + 𝒔𝒊𝒏𝒙)𝟏−𝒙 for x ∊ (0, 1)
1 3 is
∴ 1 − |𝑧| < 4 ⟹ |𝑧| > 4
3
(a) exactly one (b) at least
When |z|> 1, clearly |𝑧| > 4 one (c) at most one (d)
none of these
3
Thus |𝑧| > 1, 𝑐𝑙𝑒𝑎𝑟𝑙𝑦 |𝑧| > 4
Ans. (a)
|𝒙|
22. Number of solutions of 𝟑 = |𝟐 −
Given eqn. is f(x) = 1,
|𝒙|| is
1
1 1−𝑥
(a) 0 (b) 2 where f(x) = (1 + ) −
10+sin 𝑥
(c) 4 (d) infinite 1
1 1−𝑥
(1 − 10+sin 𝑥)
Ans. (b)
= x – 2, 2≤ 𝑥 < ∞
73
Challenging Mathematical Problems
(a) m= n (b) m ≤ n 2𝑥 + 3𝑦 = 1 2𝑥 + 3𝑦 = 11
∴ }, }
(c) m ≥ n (d) m > n 𝑥 − 3𝑦 = 11 𝑥 − 3𝑦 = 1
Hence, eqn. (1) cannot have more than one Where 𝛼= {x}= x-[x]
root. ⟹𝑥 3 − 𝑥 = 3 − 𝛼. But 0≤ 𝛼 < 1
∴𝑚≤1 ∴ 2 < 𝑥3 − 𝑥 ≤ 3
Let f(x) = (𝑥 − 𝑎1 )101 + (𝑥 − 𝑎2 )101 + ⋯ + For x ≥ 2,
(𝑥 − 𝑎𝑛 )101
𝑥 3 − 𝑥 = 𝑥(𝑥 2 − 1) ≥ 2(22 − 1) = 6
∴ f’(x) > 0 ⟹ f(x) is an increasing function
For x≤ −1, 𝑥 3 − 𝑥 = 𝑥(𝑥 2 − 1) < 0
Also f(−∞) = −∞ < 0 𝑎𝑛𝑑 𝑓(∞) = ∞ >
0 For -1 < x< 0, 𝑥 3 − 𝑥 < 1 < 2
∴ f(x) = 0 has exactly one real root For 0 < x ≤ 1, 𝑥 3 − 𝑥 < 𝑥 3 < 1 < 2
∴ n= 1 For x= 0, 𝑥 3 − 𝑥 = 0 < 2
74
Challenging Mathematical Problems
Thus m= 2, n=1
26. If [x] denotes the integral part of x 27. If [x] denotes the integral part of x
𝟐 |𝒙|
−𝟏 𝟏+𝒕 and m= [𝟏+𝒙𝟐 ] , 𝒏 =
and k = 𝒔𝒊𝒏 > 0, then integral
𝟐𝒕
𝟏
value of 𝛼 for which the equation (x- 𝒊𝒏𝒕𝒆𝒈𝒓𝒂𝒍 𝒗𝒂𝒍𝒖𝒆𝒔 𝒐𝒇 , then
𝟐−𝒔𝒊𝒏𝟑𝒙
[k])(x+𝛼) - 1 = 0 has integral roots is
(a) m≠ n (b) m > n
(a) 1 (b) 2 (c) m + n = 0 (d) 𝒏𝒎 = 𝟎
(c) 4 (d) none of these
Ans. (a)
Ans. (d)
|𝑥|
2 1+𝑡 2
0 ≤ 1+𝑥 2 < 1 ;
−1 1+𝑡
For sin 𝑡𝑜 𝑏𝑒 𝑑𝑒𝑓𝑖𝑛𝑒𝑑, | |≤1
2𝑡 2𝑡
|𝑥|
∴ m= [1+𝑥2 ] = 0
1+𝑡 2
⟹ <1
2𝑡
Again 1≤ 2 − sin 3𝑥 ≤ 3
⟹ 1+ |𝑡|2 ≤ 2|𝑡|
1 1
2
∴3 ≤ 2−sin 3𝑥 ≤ 1
⟹ (1+ |𝑡|) ≤ 0
1
⟹ (1+ |𝑡|)2 = 0 ⟹ |𝑡| = 1 ∴ n= integral value of 2−sin 3𝑥= 1
75
Challenging Mathematical Problems
Ans. (b)
8 ISI B.STAT/B.MATH
Let f(x) = 𝜋 𝑥 + cos 𝑥 + cos 2𝑥 + cos 3𝑥
OBJECTIVE QUESTIONS &
8 SOLUTIONS
𝑡ℎ𝑒𝑛 𝑓 ′ (𝑥) = 𝑥 − 𝑠𝑖𝑛𝑥 − 2sin 2𝑥
𝜋
− 3 sin 3𝑥 SET – 10
f(x) is continuous and differentiable at every
point
1. Let
𝜋
Also f(0)= f(2 ) ∴ By Rolle’s theorem f(x) = cosx(sinx + √𝒔𝒊𝒏𝟐 𝒙 + 𝒔𝒊𝒏𝟐 𝜽),
𝜋
f’(c)= 0 for at least one c in (0, 2 ) where ‘𝜃’ is a given constant,
30. Let f(x) and g(x) be differentiable then maximum value of f(x) is
functions for 0≤ 𝒙 ≤ 𝟏 such that f(0) = 2,
(a) √𝟏 + 𝐜𝐨𝐬𝟐 𝜽 (b) √𝟏 + 𝐬𝐢𝐧𝟐 𝜽
g(0)= 0, f(1)= 6.Let there exist a real
number c in (0, 1) such that f’(c)= 2g’(c), (c) |cos𝜃| (d) none
then g(1)= Ans. (b) {𝑓(𝑥)𝑠𝑒𝑐𝑥 − 𝑠𝑖𝑛𝑥}2 = 𝑠𝑖𝑛2 𝑥 +
(a) 1 (b) 2 𝑠𝑖𝑛2 𝜃,
(c) -2 (d) -1 ⤇ 𝑓 2 (𝑥)(1 + 𝑡𝑎𝑛2 𝑥) − 2𝑓(𝑥𝑡𝑎𝑛𝑥) =
Ans. (b) 𝑠𝑖𝑛2 𝜃
76
Challenging Mathematical Problems
⤇𝑓 2 (𝑥) ≤ 1 + 𝑠𝑖𝑛2 𝜃
4. The polar equation r= acos𝜃 represents
i.e. |f(x)|≤ √1 + 𝑠𝑖𝑛2 𝜃.
(a) a spiral (b) a parabola
(c) a circle (d) none
𝟏+√𝟐+𝟑√𝟑+⋯+𝒏√𝒏
2. 𝐥𝐢𝐦 Ans. (c) 𝑟 2 = 𝑎𝑟𝑐𝑜𝑠𝜃
𝒏→∞ 𝒏
77
Challenging Mathematical Problems
𝝅 1 1 1
6. If f: (𝟎, 𝒏) → 𝑹, 𝒅𝒆𝒇𝒊𝒏𝒆𝒅 𝒃𝒚 𝒇(𝒙) = 𝑆𝑛 = (1 − 1+𝑥) + (1+𝑥 − 1+2𝑥) +
∑𝒏𝒌=𝟏[𝟏 + 𝐬𝐢𝐧 𝒌𝒙], where [x] denotes the 1 1
(1+2𝑥 − 1+3𝑥) + ⋯ + (1+(𝑛−1)𝑥 −
1
78
Challenging Mathematical Problems
∴ cos {(x+ 3)}- [x+ 3]] is a periodic function 12. If f is an increasing function and g is a
with period 1. decreasing function such that g(f(x))
exists, then
𝟑
(a) g(f(x)) is an increasing function
10. If f(x) = 𝟐𝒔𝒊𝒏 𝝅𝒙+𝒙−[𝒙] , where [x] (b) g(f(x)) is an decreasing
denotes the integral part of x is a periodic
function with period (c) nothing can be said
(d) g(f(x)) is a constant function
(a) 1 (b) 2 (c)
𝜋 (d) none of these Ans. (b)
2𝜋 2𝜋 −𝑥 2 , 𝑥 ≤ 0
Period of f(x) = L.C.M. of 𝑎𝑛𝑑 = 𝑓(𝑥) = {
1 |𝑎| 𝑥2, 𝑥 ≥ 0
𝐿.𝐶.𝑀.𝑜𝑓 2𝜋 𝑎𝑛𝑑 2𝜋
𝐻.𝐶.𝐹.𝑜𝑓 1 𝑎𝑛𝑑 |𝑎| −2𝑥 2 , 𝑥 ≤ 0
∴ 𝑓 ′ (𝑥 ) = {
2𝑥, 𝑥 ≥ 0
Since k= H.C.F. of 1 and |a|
∴ f’(x)=>0 ∴ f(x) is an increasing function
1 |𝑎|
∴ 𝑘= an integer= m (say) and = an integer and consequently it is a one –one function.
𝑘
= n (say)
Also f(−∞)= -∞, f(∞)=∞, Hence range f=
𝑛 𝑛 R
∴ |a|=𝑚 ⟹ 𝑎 = ± 𝑚 = a rational number.
79
Challenging Mathematical Problems
𝟏 𝟏 ∴ Period of f(x) is 4
14. Let f(r) = 1+ 𝟐 +𝟑 + ⋯ +
𝟏
, 𝒕𝒉𝒆𝒏 ∑𝒏𝒊=𝟏 𝒇(𝒊)= 16. If f(x) = (𝒂 − 𝒙𝒏 )𝟏/𝒏 , x > 0 and g(x)> x
𝒓
⋁x ∊R, then for all x > 0
(a) (n+1) f(n)-(n- 1) (b) (n+ 1) f(n)- n
(c) n f(n)- (n-1) (d) (n-1) f(n) (a) g(g(x))= f(f(x)) (b) g(g(x))>2
f(f(x)) (c) g(g(x))< f(f(x)) (d)
Ans. (b) g(g(x))> f(f(x))
1
f(1)+ f(2)+ …+f(n)= 1+ (1 + 2) + Ans. (d)
1 1 1 1 1
(1 + 2 + 3) + ⋯ (1 + 2 + 3 + ⋯ + 𝑛) f(x)= (𝑎 − 𝑥 𝑛 )1/𝑛 , 𝑥 > 0
1
(𝑛 − 1) (𝑛 − 2) ∴ f(f(x))= f(y), where y= f(x)= (1 − 𝑦 𝑛 )𝑛 =
=𝑛+ + +⋯
2 3 1
[𝑛 − (𝑛 − 1)] {1 − (𝑎 − 𝑥 𝑛 )} 𝑛 = 𝑥, 𝑥 > 0
+
𝑛 Given, g(x)-> 0 ∀ x 𝜖 R
1 1 1
= 𝑛 (1 + + +⋯+ ) ∴ g(g(x))-g(x)> 0 ∀ x 𝜖 R
2 3 𝑛
1 2 𝑛−1
−( + +⋯+ ) [Putting g(x) in place of x]
2 3 𝑛
Adding we get, g(g(x))-x> 0
1 1
= 𝑛𝑓(𝑛) − [(1 − ) + (1 − ) + ⋯
2 3 ⟹ g(g(x))> x ⟹g(g(x))> f(f(x)), x > 0
1
+ (1 − )]
𝑛 [∵ f(f(x))= x, x > 0]
80
Challenging Mathematical Problems
𝑠𝑔𝑛(1), 𝑥 > 0
𝑠𝑔𝑛 (𝑠𝑔𝑛(𝑥)) = { 𝑠𝑔𝑛(0), 𝑥 = 0
𝑠𝑔𝑛(−1), 𝑥 < 0 20. Let P is an odd prime and n= 1+p!,
1, 𝑥 > 0 then total number of prime in the list n+1,
= { 0, 𝑥 = 0 = 𝑠𝑔𝑛(𝑥) n+2, n+3, ……, n+p-1 is equal to
−1, 𝑥 < 0
(a) P- 3 (b) P- 5
𝑥, 𝑥 > 0 (c) 0 (d) none
|𝑥|𝑠𝑔𝑛(𝑥) = {0, 𝑥 = 0 = 𝑥, ∀ 𝑥 𝜖 𝑅
𝑥, 𝑥 < 0 Ans. (c) ∵ n= 1+p!
81
Challenging Mathematical Problems
𝝅 𝝅 𝝅
make angles 𝟔 , 𝟑 , 𝟒 respectively with the
𝑑𝑥 𝑑𝑦
positive direction of the x- axis. Then the ∴2𝑥 𝑑𝑡 + 2𝑦 𝑑𝑡 = 0
value of
𝑑𝑦 𝑥 𝑑𝑥 𝑥
𝜸 𝜸 = − = − .2
∫𝜷 𝒇′ (𝒙). 𝒇′′ (𝒙)𝒅𝒙 + ∫𝜶 𝒇′′ (𝒙)𝒅𝒙 is equal 𝑑𝑡 𝑦 𝑑𝑡 𝑦
to 𝑥
𝑤ℎ𝑒𝑛 𝜃 = 30°, = cot 30° = √3
𝟏 𝟏 𝑦
(a) − (b)
√𝟑 √𝟑
(c) 0 (d) none of 𝑑𝑦
∴ = −2√3𝑓𝑡/𝑠𝑒𝑐
these 𝑑𝑡
Ans. (a) 1 𝑑𝑥 2 1 𝑑𝑦 2
√
𝑛𝑜𝑤 𝑉 = ( ) +( )
1 2 𝑑𝑡 2 𝑑𝑡
Given, 𝑓 ′ (𝛼) = , 𝑓 ′ (𝛽) = √3, 𝑓 ′ (𝛾) = 1
√3 1
= √22 + 12 = 2 𝑓𝑡/𝑠𝑒𝑐
𝛾 𝛾 2
Now ∫0 𝑓 ′ (𝑥)𝑓 ′′ (𝑥) 𝑑𝑥 + ∫𝛼 𝑓 ′′ (𝑥) 𝑑𝑥 =
1 2 1 2
[2 (𝑓 ′ (𝑥)) ] 𝛽𝛾 + [𝑓 ′ (𝑥)] 𝛼𝛾 = 2 (𝑓 ′ (𝑦)) −
1 1
{𝑓 ′ (𝛽)}2 + 𝑓 ′ (𝛾) − 𝑓 ′ (𝛼) = (1 − 3) + 24. Two persons are moving on the curve
2 2
1 1 𝒙𝟑 + 𝒚𝟑 = 𝒂𝟑 . When the position of first
1− = −
√3 √3 and second persons are (𝛼, 𝛽) and (𝛾, 𝛿)
the second persons is in the direction of
the instantaneous motion, then
23. A rod of length 10ft sides with ends on 𝜸 𝜹 𝜶 𝜷
the co-ordinates axes. If the end on x-axis (a) 𝜶 + 𝜷 + 𝟏 = 𝟎 (b) 𝜸 + 𝜹 − 𝟏 =
moves with constant velocity of 2ft/ 𝟎 (c) 𝛼𝛾 +𝛽𝛿 = 1 (d) none
minute, then the magnitude of the velocity of these
of the middle point at the instant the rod
makes an angle of 30° with x-axis is Ans. (a)
82
Challenging Mathematical Problems
or (2+b)x- y= 1+ b
27. If ur denotes the number of one–one
1+𝑏 functions from
∴ OA= 2+𝑏 𝑎𝑛𝑑 𝑂𝐵 = −(1 + 𝑏)
{𝒙𝟏 , 𝒙𝟐 , … , 𝒙𝒓 } 𝒕𝒐 {𝒚𝟏 , 𝒚𝟐 , … , 𝒚𝒓 } such that
Since 𝛥 AOB lies in the first quadrant, f(𝒙𝒊 ) ≠ 𝒚𝒊 , for i= 1, 2, 3, …, r then 𝒖𝟒 =
83
Challenging Mathematical Problems
⟹ [r⃗ a⃗ b⃗] = 0
28. Number of positive unequal integral
solutions of equation x+ y+ z = 6 is
a⃗ .c⃗=|c⃗|……………(1)
29. The plain containing the two straight
lines r⃗= a⃗+ 𝜆b⃗ and r⃗= b⃗+ 𝜇a⃗ is |c⃗-a⃗|= 2√2 …………….(2)
𝜋
(a) [r⃗ a⃗ b⃗] = 0 Angle between (a⃗ ×b⃗ ) and c⃗= 6
(b) [r⃗ a⃗ a⃗ × 𝒃⃗] = 0
Now,
(c) [r⃗ b⃗ a⃗ × 𝒃⃗] = 0
𝜋
(d) none |(a⃗ ×b⃗ )× 𝑐⃗|= |a⃗ ×b⃗ ||c⃗|sin 6 =
3
Ans. (a) |𝑐⃗|……(3)
2
84
Challenging Mathematical Problems
From (3), |𝑐⃗ − 𝑎⃗|2 = 8 Q2. Find the number of positive integers
less than or equal to 6300 which are not
⟹(𝑐⃗ − 𝑎⃗).( 𝑐⃗ − 𝑎⃗)= 8 divisible by 3, 5 and 7.
⟹|𝑐⃗|2 + |𝑎⃗|2 − 2𝑎⃗. 𝑐⃗ = 8 Solution:- S= {1, 2, 3, …, 6300}
⟹ |𝑐⃗|2 + 9 − 2|𝑐⃗| = 8 (from (1)) Let A: Set of integers divisible by 3
∴ |𝑐⃗| = 1 B: Set of integers divisible by 5
3
From (3), |(a⃗ ×b⃗ )× 𝑐⃗ | = . C: Set of integers divisible by 7
2
𝒙𝟐 +𝟐𝒙+𝒄
We need to count the no. of solutions of 𝑥1 + y= , as x varies over real numbers,
𝒙𝟐 +𝟒𝒙+𝟑𝒄
𝑥2 + ⋯ + 𝑥8 = 7 range over all real numbers.
Which satisfies 0 ≤ 𝑥𝑖 ≤ 7, i= 1, 2, 3, …, 8 Solution:-
……………….(1)
𝑥 2 + 2𝑥 + 𝑐
The number of solution of (1) is= coefficient of 𝑦= ;
𝑥 2 + 4𝑥 + 3𝑐
𝑥 7 in (1 + 𝑥 + 𝑥 2 + ⋯ + 𝑥 7 )8
⟾ 𝑥 2 𝑦 + 4𝑥𝑦 + 3𝑐𝑦 = 𝑥 2 + 2𝑥 + 𝑐
= 𝑐𝑜𝑒𝑓𝑓𝑖𝑐𝑖𝑒𝑛𝑡 𝑜𝑓 𝑥 7 𝑖𝑛 (1 − 𝑥 8 )8 (1 − 𝑥)8
⟾ (𝑦 − 1)𝑥 2 + 2𝑥(2𝑦 − 1) + 𝑐(3𝑦 − 1)
7 8 )(1
= coefficient of 𝑥 in (1 − 8𝑥 + 8𝑐1 𝑥 + = 0 [∵ 𝑥 𝑖𝑠 𝑟𝑒𝑎𝑙]
4𝑐2 𝑥 2 + 10𝑐3 𝑘 + ⋯ )
∴ {2(2𝑦 − 1)}2 − 4(𝑦 − 1). 𝑐(3𝑦 − 1) ≥ 0
= 14𝑐7
(2𝑦 − 1)2
= 3432. (𝐴𝑛𝑠) ⟾𝑐 ≤ ∵ 0 < 𝑐 < 1,
(𝑦 − 1)(3𝑦 − 1)
1
𝑆𝑜, < 𝑦 < 1.
3
85
Challenging Mathematical Problems
𝑛
Q4. Let X = {0, 1, 2, 3, …, 99}. For a, b in X, 1 1 1
𝑑𝑥
we define a * b to be the remainder obtained = lt ∑ 𝑟 = ∫
𝑛 →∞ 𝑛 0 1+𝑥
by dividing the product ab by 100. For 𝑟=1 1 + 𝑛
86
Challenging Mathematical Problems
𝑙𝑎 2 𝑚𝑎 2 𝑂𝑄
⟾ ℎ2 + 𝑘 2 + 𝑛
ℎ + 𝑛
𝑘 = 0, i.e. the ∴ 𝑅1 = , 𝑛𝑜𝑤, 𝑂𝑃 = 𝑅2 (√2 + 1), 𝑅2
required focus of M. √2 + 1
𝑂𝑃 √2 − 1
2 2 = = 𝑅1
So, the equation of the circle is 𝑥 + 𝑦 + √2 √2 + 1
𝑙𝑎 2 𝑚𝑎 2
𝑛
𝑥 + 𝑛
𝑦 = 0. 2
√2−1 √2−1
∴ 𝑅3 = 𝑅2 = 𝑅1 ( )
√2+1 √2+1
Q7. Draw the graph (on plain paper) of f(x) =
min {|x| - 1, |x -1|- 1, |x- 2| - 1}. ∴ 𝐴𝑟𝑒𝑎 = 𝜋(𝑅1 2 + 𝑅2 2 + ⋯ +∝)
𝜋 18−16 𝜋 1
= 8
𝑅1 2 (3 ) = 4 . 100. 3 𝑠𝑞. 𝑐𝑚 [∵
√2−4 √2−4
𝑅1 = 10𝑐𝑚. ]
25𝜋
=3 𝑠𝑞. 𝑐𝑚 [𝑝𝑟𝑜𝑣𝑒𝑑]
√2−4
Solution:- 𝑂𝑂1 =
𝑅1 √2 ∴ 𝑂𝑃2 = 𝑅1 √2 − 𝑅1
Q9. Consider the system of equations x + y =
∴ 𝑂𝑄 = 𝑅1 √2 + 𝑅1 = 𝑅1 (√2 + 1) 2, ax + y = b. Find conditions on a and b
under which
87
Challenging Mathematical Problems
(i) the system has exactly one solution; From (1) And (2), we get 𝑘1 = 4, 𝑘2 = 0.
1 𝑛 1
(ii) the system has no solution; ∴𝑥𝑛 = 4 (2) = 2𝑛−2 .
1 𝑘1 1, 2, 3, …., n
Again, 𝑥1 = 2 = 𝑘1 (2) ≠ 𝑘2 (1)1 = 2
+
𝑘2 …………… (1) n+1, n+2 , n+3, ….., 2n
1 2 𝑘1 2n + 1, 2n +2, 2n +3,…….., 3n
And 𝑥2 = 1 = 𝑘2 (2) + 𝑘2 (1)1 = 4
+ 𝑘2
…………. (2) ⁞
88
Challenging Mathematical Problems
Since each row has n consecutive positive So, sum of roots = coefficient of x = 0
integers, the product of the numbers in each row
is divisible by n!. The product of all the numbers ∴ 𝛼 = 0.
(kn)! Is divisible by (𝑛!)𝑘 . So, roots are -3𝛽, -𝛽, 𝛽, 3𝛽.
Q3. All the permutations of the letters a, b, c,
∴ −3𝛽 2 + 3𝛽 2 − 9𝛽 2 − 𝛽 2 − 3𝛽 2 + 3𝛽 2 =
d, e are written down and arranged in
−(3𝑚 + 1)
alphabetical order as in a dictionary. Thus
the arrangement abcde is in the first position ⟾ −10𝛽2 = −(3𝑚 + 1)
and abced is in the second position. What is
the position of the arrangement debac? 3𝑚 + 1
∴ 𝛽2 = .
10
Solution:-
Also, 9𝛽 4 = 𝑚2
Words starting with ‘a’ ⟶ 4!
3𝑚 + 1 2
″ ″ ″ ‘b’ ⟶ 4! ∴ 9( ) = 𝑚2
10
″ ″ ″ ‘c’ ⟶ 4! 3
⟾ 9𝑚 + 3 = ±10𝑚 ⟾ 𝑚 = 3, − .
19
″ ″ ″ ‘d’ ⟶ 3! + 3! + 3! + 3!
(ii)𝑥 2 − 𝑎𝑥 − 𝑏 = 0
(i.e. da ⟶ 3!, db ⟶ 3!, dc ⟶ 3!, de ⟶ 3!)
Let roots be 𝛼, 𝛽, ∣𝛼∣ <1, ∣𝛽∣ <1,
Total no. of words before debac including it is =
3 × 4! + 3 × 3! + 3 = 93. ∴∣𝛼 +𝛽∣ ≤ ∣𝛼∣ + ∣𝛽∣ <2, as ∣𝛼∣ <2, or ∣𝛼∣ ∣𝛽∣ <1
(i) |b| < 1, (ii) a+ b <1 and (iii) b - a <1. Q5. Let a and b be real numbers such that the
equations 2x + 3y = 4 and ax – by = 7 have
Solution:- exactly one solution. Then, show that the
equations 12x – 8y = 9 and bx + ay = 0 also
(i) 𝑥 4 − (3𝑚 + 1)𝑥 2 + 𝑚2 = 0
have exactly one solution.
Let four roots
A B C D E F Solution:-
be 𝛼 - 3𝛽, 𝛼 –
A × 0 ② 0 ② ②
B 2 × 2 2 0 2 𝛽, 𝛼+ 𝛽, 𝛼+ 2𝑥 + 3𝑦 = 4
} ________________ (i)
C 0 0 × 2⃞ 2⃞ 0 3𝛽. 𝑎𝑥 − 𝑏𝑦 = 7
D 2 0 0 × 2 2
E 0 2 0 0 × 2
F 0 0 2 0 0 ×
89
Challenging Mathematical Problems
90
Challenging Mathematical Problems
⦟BDO =⦟CDO, BD = DC & OD is common. (b) Use the information obtained in (a) to
draw a rough sketch of the graph of f(x) on
∴ 𝛥BOD ≡ 𝛥COD. plain paper.
𝑥+1 1 1 4 1
Solution:- f(x) = (𝑥−1)(𝑥−7) = − . + . .
3 𝑥+1 3 𝑥−7
1 1 4 1
(a) f’(x) = 3 . (𝑥+1)2 − 3 . (𝑥−7)2
𝛥BOF, (b)
91
Challenging Mathematical Problems
Solution:-
(a) Case-I:-
If x = 2m, then
If x = 2m+ 1
Solution:- -1 ≤ 𝑐𝑜𝑠 2 (𝑥 3 + 𝑥) ≤ 1 implying 𝑝𝑥 = 𝑥 𝑛 + 𝑎𝑛−1 𝑥 𝑛−1 + ⋯ + 𝑎0 = 𝑥 𝑛 +
−2 ≤ 2𝑐𝑜𝑠 2 (𝑥 3 + 𝑥) ≤ 2 𝑎𝑛−1 (𝑒𝑣𝑒𝑛 + 1) + 𝑎𝑛−2 (𝑒𝑣𝑒𝑛 + 1) + ⋯ +
𝑎1 (𝑒𝑣𝑒𝑛 + 1)𝑎0 = 𝑥 𝑛 + 𝑎0 + (𝑎𝑛−1 𝑒𝑣𝑒𝑛 +
By AM ≥ GM inequality we have 2𝑥 + 2−𝑥 ≥ 2
𝑎𝑛−2 𝑒𝑣𝑒𝑛 + ⋯ + 𝑎1 𝑒𝑣𝑒𝑛) + (𝑎1 + 𝑏2 + ⋯ +
So 2𝑐𝑜𝑠 2 (𝑥 3 + 𝑥) = 2𝑥 + 2−𝑥 = 2 satisfies 𝑎𝑛−1 ) = 𝑥 𝑛 + (𝑎𝑛−1 𝑒𝑣𝑒𝑛 + 𝑎𝑛−2 𝑒𝑣𝑒𝑛 + ⋯ +
when x = 0. 𝑎1 𝑒𝑣𝑒𝑛) + (𝑎0 + 𝑎1 + ⋯ + 𝑎𝑛−1 )
So there is only one value of x = 0 which satisfy = odd ⟹ x = (2m +1) cannot be a root of the
the given equation. equation P(x) = 0 [proved.]
SOLUTIONS
(a) Write (√𝟐 − 𝒄𝒏−𝟏 ) 𝒊𝒏 𝒕𝒆𝒓𝒎𝒔 𝒐𝒇 (√𝟐 −
SET – 3 𝒄𝒏 ).
𝟏
(b) Show that |√𝟐 − 𝒄𝒏+𝟏 | < |√𝟐 − 𝒄𝒏 |.
𝟏+√𝟐
92
Challenging Mathematical Problems
93
Challenging Mathematical Problems
3 1
24 2
3 1
3
X g(x)
0 0
1 1
4 4
1 1 Q5. Show that the area of the bounded region
2 2
3 3 enclosed between the curves
4 4
1 0 𝟐
𝒚𝟑 = 𝒙𝟐 𝒂𝒏𝒅 𝒚 = 𝟐 − 𝒙𝟐 , 𝒊𝒔 𝟐 .
1 1 𝟏𝟓
14 4
1
1 ½ Solution:- 𝑦 3 = 𝑥 2 …………. (i)
2 ⁞
⁞ y = 2−𝑥 2 …………… (ii)
⟾ y = 2 - 𝑦3
⟾(y -1) (𝑦 2 + 𝑦 + 2) = 0
∴y = 1, y = ½ (-1±√3)
1
(iii) h(x) = [𝑥]
x h(x)
1 1
1 1
14
1 1
12 1
3 1
14
1 ∴ Area of shaded region = ∫−1(𝑦1 − 𝑦2 )𝑑𝑥 =
2 2
2 1 1 2
∫−1 [2 − 𝑥 2 − 𝑥 3 ] 𝑑𝑥 = 2 15 𝑠𝑞. 𝑢𝑛𝑖𝑡𝑠.
1 2
2
2
94
Challenging Mathematical Problems
Q6. We say that a sequence {𝒂𝒏 } has As roots are rational, hence D is a perfect
property P, if there exists a positive square.
integer m such that 𝒂𝒏 ≤1 for every n ≥
i.e. 𝑝2 − 4𝑞 = 𝑘 2 , where k ∊ I.
m. For each of the following sequences,
determine whether it has the property P or, 𝑝2 = 𝑘 2 + 4𝑞.
or not. [Do not use any result on limits.]
Now, when p = even, 𝑘 2 + 4𝑞 = even.
𝟐𝟎𝟎
𝟎. 𝟗 + 𝒊𝒇 𝒏 𝒊𝒔 𝒆𝒗𝒆𝒏 ⤇ 𝑘 2= even, so k = even.
(𝒊) 𝒂𝒏 = { 𝒏
𝟏 −𝑒𝑣𝑒𝑛 ±𝑒𝑣𝑒𝑛
𝒊𝒇 𝒏 𝒊𝒔 𝒐𝒅𝒅 ⤇x= 2
= integer …………………
𝒏
(i)
𝒏𝝅
𝐜𝐨𝐬
𝟐
𝟏+ 𝒊𝒇 𝒏 𝒊𝒔 𝒆𝒗𝒆𝒏 Again when p = odd, 𝑘 2 + 4𝑞 = odd
(𝒊𝒊) 𝒂𝒏 = { 𝒏
𝟏
𝒊𝒇 𝒏 𝒊𝒔 𝒐𝒅𝒅. ⤇ 𝑘 2= odd, so k= odd.
𝒏
Here, {𝑎𝑛 } is a decreasing sequence and Q8. Consider the set S of all integers between
200 and including 1000 and 99999. Call two
𝑎2000 = 0.9 + 2000 = 1.
integers x and y in S to be in the same
∴ 𝑎𝑛 satisfies property P. equivalence class if the digits appearing in x
and y are the same. For example, if x = 1010,
(ii) Let n = 4k (even) y= 1000 and z = 1201, then x and y are in the
same equivalence class, but y and z are not.
1 4𝑘𝜋
i.e., 𝑎4𝑘 = 1 + 4𝑘 cos ( )= 1+ Find the number of distinct equivalence
2
1 1 classes that can be formed out of S.
cos(2𝑘𝜋) = 1 + 4𝑘 > 1.
4𝑘
Solution:-
[∵ cos (2k𝜋) = 1] So, here 𝑎𝑛 does not
If ‘a’ is a member of some equivalence class
satisfy P.
then it’s distinct digit determine the equivalence
Q7. Suppose that the roots 𝒙𝟐 + 𝒑𝒙 + 𝒒 = 𝟎 class completing. Hence, no. of equivalence
are rational numbers and p, q are integers. classes are the number of ways in which ‘i’
Then show that the roots are integers. integers can be selected from {1, 2, 3, …, 9} for
2 ≤ i ≤ 5 and {1, 2, 3, …, 9} for i = 1.
Solution:-
Now, this can be done in 9 + ∑5𝑖=2(10
𝑖
)=
2
The roots of the equation 𝑥 + 𝑝𝑥 + 𝑞 = 0 are x
∑5𝑖=1(10
𝑖
) − 1.
−𝑝±√𝑝2 −4𝑞
= 2
.
95
Challenging Mathematical Problems
𝒏−𝟏
𝒙𝒏 −𝟏 ISI B.STAT & B.MATH
Q9. For x > 0, show that 𝒙−𝟏
≥ 𝒏𝒙 𝟐 ,
where n is a positive integer. SUBJECTIVE QUESTIONS &
SOLUTIONS
Solution:-
1 1 1 1 1 1 2𝑐𝑜𝑠 2 (𝑥 3 + 𝑥) = 2𝑥 + 2−𝑥
= ( − )+ ( − )
6 2 𝑛+1 6 3 𝑛+3 ∴ 𝑐𝑜𝑠 2 (𝑥 3 + 𝑥) = 1
2 1
+ (1 − )
3 𝑛+1 For x = 0, the equation is satisfied.
1 29 4 1
= [ − −
6 6 𝑛+1 𝑛+2
1
− ] (𝑝𝑟𝑜𝑣𝑒𝑑)
𝑛+3 Q3. There are 1000 doors 𝑫𝟏 , 𝑫𝟐 , … , 𝑫𝟏𝟎𝟎𝟎
and 1000 persons 𝑷𝟏 , 𝑷𝟐 , … , 𝑷𝟏𝟎𝟎𝟎 . Initially
96
Challenging Mathematical Problems
all the doors were closed. Person 𝑷𝟏 goes and ∴f(0) = 0 and f’(x) = 2x –sin x –x cos x
opens all the doors. Then person 𝑷𝟐 closes
doors 𝑫𝟐 , 𝑫𝟒 , … , 𝑫𝟏𝟎𝟎𝟎 and leaves the odd- = (x –sin x) + x (1 –cos x)
numbered doors open. Next, 𝑷𝟑 changes the 𝛱
In the interval [0, 2 ], 𝑥 − 𝑠𝑖𝑛𝑥 ≥ 0 𝑎𝑛𝑑 1 −
state of every third door, that is,
𝑫𝟑 , 𝑫𝟔 , … , 𝑫𝟗𝟗𝟗 . (For instance, 𝑷𝟑 closes the 𝑐𝑜𝑠𝑥 ≥ 0.
open door 𝑫𝟑 and opens the closed door 𝑫𝟔 ,
∴ f’(x)≥ 0 ∴ f(x) is an increasing function of x
and so on.) Similarly, 𝑷𝒎 changes the state of 𝛱
in[0, 2 ]
the doors 𝑫𝒎 , 𝑫𝟐𝒎 , 𝑫𝟑𝒎 , … , 𝑫𝒏𝒎 , … while
leaving the other doors untouched. Finally,
It’s min value will be f(0) = 0, maximum value
𝑷𝟏𝟎𝟎𝟎 opens 𝑫𝟏𝟎𝟎𝟎 if it were closed and closes 𝛱 𝛱2 𝛱
it if it were open. At the end, how many doors will be 𝑓 ( 2 ) = 4
− 2.
remain open?
Q5. Let A and B be two fixed points 3 cm
Solution:- By the problem, the persons 𝑃𝑚 will apart.
change the state of the door 𝐷𝑛 , where m│ n i.e.
m is one of the factors of n. (a) Let P be any point not collinear with A
and B, such that PA = 2PB. The tangent at p
At first, all the doors were closed and we are to to the circle passing through the points P, A
determine the no. of doors remaining open, i.e. and B meets the extended line AB at the point
we are to determine the no. of doors whose K. Find the lengths of the segments KB and
states are finally changed. KP.
Now, for the door 𝐷𝑛 , n will either have even or (b) Hence or otherwise, prove that the locus
odd no. of factors. of all points P in the plane such that PA =
2PB is a circle.
It is obvious, for even no. of factors, state of
doors remain same, so whenever no. of fact (n) Solution:-
odd, state of Or𝐷𝑛 changes.
(a) AB = 2a = 3 cm, let P(h, k) be the co-
⟹ (n) = 2k +1, k ∊ 𝐼 + ⟹ 𝜎(𝑛) = (2𝑝 + ordinate of P and PA = 2PB
1)(2𝑞 + 1) × … (2𝑟 + 1) … . ; ⟹ 𝑛 =
𝑎2𝑝 𝑏 2𝑞 𝑐 2𝑟 …. For prime a, b, c …..
∴(ℎ + 𝑎)2 + 𝑘 2 = 4{(ℎ − 𝑎)2 + 𝑘 2 }
⟹Square numbered door remains open.
⟾ 3ℎ2 + 3𝑘 2 − 10ℎ𝑎 + 3𝑎2 = 0
∴No. of doors remaining open = (√1000) = 31
97
Challenging Mathematical Problems
∴ KB = OK - OB=
5𝑎
−𝑎 =
2𝑎 2 3
= × = 1𝑐𝑚. (ii)│x│- │y│ ≥ 1.
3 3 3 2
There are four cases:- (I) x – y ≥ 1 when x > 1, y
>0
(II) x + y ≥ 1 when x
≥ 1, y < 0
(III) –x –y ≥ 1 when
x < 0, y > 0
(IV) –x + y ≥ 1
when x< 0, y <0
Graph of (i):-
[case -I]
[case -II] ⟾ 64 −𝑥 2 = 12𝑥 ⟾ (x - 4) (x + 16) = 0 ∴ x
𝜋 𝜋 = - 16, 4
X (4n +1)
2
N𝜋+(−1)𝑛 6
But x = -16 is not possible as radius of a circle is
y 1 ½
√64 units.
𝜋 𝜋
X (4n +1)2 N𝜋+(−1)𝑛 6
y -1 -½
98
Challenging Mathematical Problems
4 64𝜋 (2𝑚)! 1
= 2∫0 (𝑦1 − 𝑦2 )𝑑𝑥 + 2
, 𝑤ℎ𝑒𝑟𝑒 𝑦1 = Let P(m) is true, i.e., 22𝑚.(𝑚!)2 ≤ ,
√3𝑚+1
√64 − 𝑥 2 , 𝑦2 = 2√3𝑥
(2𝑚+2)!
4 P(m+ 1) = =
22𝑚+2 .[(𝑚+1)!]2
= 2 ∫ (√64 − 𝑥2 − 2√3𝑥) 𝑑𝑥 + 32𝜋 (2𝑚)!(2𝑚+1)(2𝑚+2)
0
4.22𝑚 (𝑚!)2 (𝑚+1)2
𝑥 − √64 − 𝑥 2
= 2[ (2𝑚)! (2𝑚+1)
2 = . ≤
4.22𝑚 (𝑚!)2 2(𝑚+1)
3 1 2𝑚+1 1
64 𝑥 𝑥2 4 . . ; [𝑠ℎ𝑜𝑤 𝑖𝑡]
√3𝑚+1 2(𝑚+1) √3𝑚+4
+ sin−1 ( ) − 2√3. ]
2 8 3 0
2 ∴ P(m+1) is true.
+ 32𝜋
1
So, by induction 𝑥𝑛 ≤ ∀ 𝑛 ∈ ℕ.
16 √3𝑛+1
= (8𝜋 − √3)𝑠𝑞. 𝑢𝑛𝑖𝑡𝑠.
3
Q9. Show that if n is any odd integer greater
than 1, then 𝒏𝟓 − 𝒏 is divisible by 80.
Solution:-
Take n = 2k + 1,
99
Challenging Mathematical Problems
𝑦 = 1. ∴𝑆1 + 𝑆2 ≥ + 8
1
2
𝑎+𝑏−𝑏𝑦
∴x= = 1. 1 25
𝑎 ∴S≥2+8+4 = 2
.
In the third line, (a+ b) x – ay = LHS = b = RHS.
Solution:-
100
Challenging Mathematical Problems
The polynomial Q(x) vanishes for k = 0, 1, …., ⟾cot 𝑥𝑛+1 = cot 𝑥𝑛+2 − (𝑛 − 1) [dividing
n bysin 𝑥𝑛+1 + sin 𝑥𝑛 ]
i.e. (x+ 1) P(x) –x = a(x) (x-1) (x- 2) … (x- n) ∴cot 𝑥𝑛 = cot 𝑥𝑛−1 + 2−𝑛 = cot 𝑥𝑛−2 +
2−(𝑛−1) + 2−𝑛 = ⋯ … … … … … … ….
Putting x = -1, 1 = a(1) (-2) (-3) …. (-1 -n)
= cot 𝑥1 + 2−2 + 2−3 + ⋯ + 2−𝑛
⟹1 = a (−1)𝑛+1 (𝑛 + 1)1 ⟹ 𝑎 =
1 1
.
(−1)𝑛+1 (𝑛+1)!
= 2−1 + 2−2 + 2−3 + ⋯ + 2−𝑛
𝑎𝑥(𝑥 − 1)(𝑥 − 2) … (𝑥 − 𝑛) + 𝑥 1 𝑛
𝑃(𝑥) = = 1 − ( ) [∵ tan−1 2 = 𝑥1 ]
(𝑥 + 1) 2
1 𝑛
(−1)𝑛+1 𝑥(𝑥 − 1)(𝑥 − 2) … (𝑥 − 𝑛) ∴cot 𝑥𝑛 = 1 − (2) .
= +𝑥
(𝑛 + 1)!
lt cot 𝑥𝑛 = 1. ⟾ cot ( lt 𝑥𝑛 ) = 1
{(𝑛+1)𝑛(𝑛−1)…3.2.1}+𝑥+1 𝑛 →∞ 𝑛 →∞
∴ P(n+ 1)= (−1)𝑛+1 (𝑛+1)! 𝛱
(𝑛+2) ⟾ lt 𝑥𝑛 = .
𝑛 →∞ 4
(𝑛 + 1)
+ (𝑛 + 1) Q4. Consider the circle of radius 1 with its
(𝑛 + 1)!
= (−1)𝑛+1 centre at the point (0, 1). From this initial
(𝑛 + 2)
(−1)𝑛+1 + 𝑛 + 1 position, the circle is rolled along the positive
= x-axis without slipping. Find the locus of the
(𝑛 + 2)
point P on the circumference of the circle
1 which is on the origin at the initial position of
𝑓𝑜𝑟 𝑛 = 𝑜𝑑𝑑
∴ P(n+ 1) = { 𝑛
the circle.
𝑛 + 2 𝑓𝑜𝑟 𝑛 = 𝑒𝑣𝑒𝑛
Solution:-
Let ‘P’ be the point on the circle NP, let the line
OMX on which the circle rolls is X-axis and the
Q3. Suppose 𝒙𝟏 = 𝐭𝐚𝐧−𝟏 𝟐 > 𝒙𝟐 > 𝒙𝟑 > ⋯ point ‘O’ is origin. Radius of circle is one unit,
are positive real numbers satisfying
𝐬𝐢𝐧(𝒙𝒏+𝟏 − 𝒙𝒏 ) + 𝟐−(𝒏+𝟏) 𝒔𝒊𝒏𝒙𝒏 𝒔𝒊𝒏𝒙𝒏+𝟏 =
𝟎 𝒇𝒐𝒓 𝒏 ≥ 𝟏. Find
𝝅
𝒄𝒐𝒕𝒙𝒏 . 𝑨𝒍𝒔𝒐, 𝒔𝒉𝒐𝒘 𝒕𝒉𝒂𝒕 𝐥𝐢𝐦 𝒙𝒏 = 𝟒 .
𝒏→∞
Solution:-
101
Challenging Mathematical Problems
Solution:-
𝑥 2 +1 (𝑥 2 −1)2 +2
y = 𝑥 2 −1 = 𝑥 2 −1
2 𝑑𝑦 4𝑥
∴ y = 1+ ∴ = −
𝑥 2 −1 𝑑𝑥 (𝑥 2 −1)2
=
1
│ ∫0 (−√𝑥 − 𝑥 2 )𝑑𝑥 │ = 1 𝑠𝑞. 𝑢𝑛𝑖𝑡𝑠.
Solution:-
𝑥 𝑛 −1
Q6. Find the area of the region in the xy- P= 1 + 𝑥 + 𝑥 2 + ⋯ + 𝑥 𝑛−1 = 𝑥−1
plane, bounded by the graphs of
If P is prime, then x – 1 = 1 ⤇ x = 2.
𝟐
y = 𝒙 , 𝒙 + 𝒚 = 𝟐 𝒂𝒏𝒅 𝒚 = −√𝒙.
2𝑛 −1
∴ P= = 2𝑛 − 1 is a prime.
Solution:- 2−1
102
Challenging Mathematical Problems
But we know 2𝑛 − 1 is prime, so, by Perimeter of the marked region of the circle =
contradiction n is also prime. 2𝜋a –a (2𝜋 -𝜃) = a𝜃
Q8. Show that for every positive integer n, 7 Perimeter of the base of the cone = 2𝜋r
divides 𝟑𝟐𝒏+𝟏 + 𝟐𝒏+𝟐 . 𝑎𝜃
∴ 2𝜋r = a𝜃 ⟹r = 2𝜋
Solution:- 32𝑛+1 + 2𝑛+2 = 3. 32𝑛 + 4. 2𝑛 =
1
3(2 + 7)𝑛 + 4. 2𝑛 Volume of the cone = V = 𝜋𝑟 2 ℎ
3
Solution:- 𝜃3
∴ 2𝜃 = √4𝜋 2 − 𝜃 2 − =0
√4𝜋2 −𝜃2
(𝑏 + 𝑐)2
𝑏2 + 𝑐 2 ≥ ⟹8𝜋 2 − 2𝜃 2 − 𝜃 2 = 0
2
𝑏2 + 𝑐 2 𝑏 + 𝑐 (∵ 𝜃 ≠ 0)
≥
𝑏+𝑐 2
2
2 2 2 2
∴ 𝜃 = 2𝜋 √3
𝑐 +𝑎 𝑐+𝑎 𝑎 +𝑏
𝑠𝑖𝑚𝑖𝑙𝑎𝑟𝑙𝑦, ≥ &
𝑐+𝑎 2 𝑎+𝑏
𝑎+𝑏 𝑑2 𝑣 𝑎3
≥ . [ 2] = [2√4𝜋 2 − 𝜃 2
2 𝑑𝜃 𝜃=2π √2 24𝜋 2
3
𝑏 2 + 𝑐 2 𝑐 2 + 𝑎2 𝑎2 + 𝑏 2 1 4𝜃 3𝜃 2
𝐴𝑑𝑑𝑖𝑛𝑔, 𝑤𝑒 𝑔𝑒𝑡 + + − −
𝑏+𝑐 𝑐+𝑎 𝑎+𝑏 2 √4𝜋 2 − 𝜃 2 √4𝜋 2 − 𝜃 2
≥𝑎+𝑏+𝑐
2𝜃 4
+ 3 ]
(4𝜋 2 − 𝜃 2 )2 𝜃=2π √
2
3
Q10. Out of a circular sheet of paper of
radius a, a sector with central angle 𝜃 is cut ∴ The volume of the funnel is max. when 𝜃
out and folded into the shape of a conical 2
funnel. Show that the volume of the funnel is equals 2𝜋 √3 [proved]
𝟐
maximum when 𝜃 equals 2𝜋√ .
𝟑
Solution:-
103
Challenging Mathematical Problems
Solution:-
Q3. A pair of complex numbers 𝒛𝟏 , 𝒛𝟐 is said
2 (n- 2) > n -2 ⤇ 2n – 4+ 2 > n ⤇ 2(n- 1) > n to have property P if for every complex
………….. (i) number z, we can find real numbers r and s
such that z = r 𝒛𝟏 + 𝒔𝒛𝟐. Show that a pair
3 (n- 3) > n -3 ⤇ 3n – 9 > n- 3 ⤇ 3 (n- 2) > n 𝒛𝟏 , 𝒛𝟐 has property P if and only if the points
…………… (ii) 𝒛𝟏 , 𝒛𝟐 and 0 on the complex plane are not
collinear.
Similarly, we have 4 (n- 3) > n
……………… (iv) Solution:-
5 (n- 4) > n …………….. Now, 𝑧1 = 𝑎 + 𝑖𝑏, 𝑧2 = 𝑥 + 𝑖𝑦, 0 = 0 + 𝑖. 0
(v)
And let 𝑧1 , 𝑧2 , 0 be collinear then
⁞ 0 0 1
𝑎𝑥
│𝑎 𝑏 1 │ = 0 ⟾ = 𝑘(𝑠𝑎𝑦)
Multiplying all these up to (n- 2) terms, we get 𝑏𝑦
𝑥 𝑦 1
[1.2.3. … . . (𝑛 − 1)]2 > 𝑛𝑛−2 ∴ 𝑧1 = 𝑏𝑘 + 𝑖𝑏 = 𝑏(𝑘 + 𝑖), 𝑧2 = 𝑦(𝑘 + 𝑖)
𝑛𝑛 So, ∃ some real ‘r’ or‘s’ such that z = r𝑧1 +
⤇ [(𝑛 − 1)!]2 >
𝑛2 𝑠𝑧2 = 𝑟𝑏(𝑘 + 𝑖) + 𝑠𝑦(𝑘 + 𝑖) = (𝑟𝑏 + 𝑠𝑦)(𝑘 +
⤇ [𝑛!]2 > 𝑛𝑛 . 𝑖),
Q2. Show that for all real x, the expression Which does not hold good.
𝒂𝒙𝟐 + 𝒃𝒙 + 𝒄 (where a, b, c are real constants So, 𝑧1 , 𝑧2 and 0 should not be collinear.
(𝟒𝒂𝒄−𝒃𝟐 )
with a > 0), has the minimum value 𝟒𝒂
.
Q4. In a club of 80 members, 10 members
Also find the value of x for which this
play none of the games Tennis,
minimum value is attained.
Badminton and Cricket. 30 members play
Solution;- exactly one of these three games and 30
members play exactly two of these games.
Let P = 𝑎𝑥 2 + 𝑏𝑥 + 𝑐
45 members play at least one of the games
1 among Tennis and Badminton. Determine
= [4𝑎2 𝑥 2 + 4𝑎𝑏𝑥 + 4𝑎𝑐]
4𝑎 the number of Cricket playing members.
104
Challenging Mathematical Problems
Solution:- Solution:-
1 1 3
log 𝑃 = lt {𝑙𝑜𝑔 (1 + ) + log (1 + )
𝑛 →∞ 2𝑛 2𝑛 2𝑛
Q5. Each pair in a group of 20 persons is 2𝑛 − 1
+ ⋯ + log (1 + )}
classified by the existence of kinship relation 2𝑛
and friendship relation between them. The 𝑛
1 1 2𝑟 − 1
following table of data is obtained from such = lt ∑ log (1 + )
a classification. 2 𝑛 →∞ 𝑛 2𝑟
𝑟=1
4𝑥
∴ 𝑓 ′ (𝑥) = ,
(1 − 𝑥 2 )2
x -3 3 -2 2 0 1
-2 1
2
f(x) 2
3
2
3
2
1
2
1 5 17 17
4 4 3 3
3 3
For simplicity, centre of 𝐶1 ≡ (0, 0); 𝐶2 ≡
(3, 0); 𝐶3 ≡ (0, 4)
106
Challenging Mathematical Problems
CD = BC Sin 60° = 1.
√3
=
√3 ⁞ ⁞
2 2
1 3 𝑛8 4 ≡ 1 (𝑚𝑜𝑑 16)
∴AD = AB+ BD = 1+2 = 2
________________________
1 3
∴ Required volume = 𝜋. 4
3 𝑛1 4 + 𝑛2 4 + ⋯ + 𝑛8 4 ≡ 8 (𝑚𝑜𝑑 16)
3 1 1 3𝜋 𝜋
∴ (2 − 2) 𝑠𝑞. 𝑢𝑛𝑖𝑡 = 3 . 4
𝑠𝑞. 𝑢𝑛𝑖𝑡 = 4 𝑠𝑞. 𝑢𝑛𝑖𝑡. But 1993 ≡ 9 (mod 16)
(b) Hence or otherwise show that we cannot Q1. Let x be a positive number, A sequence
find integers 𝒏𝟏 , 𝒏𝟐 , … , 𝒏𝟖 such that𝒏𝟏 𝟒 + {𝒙𝒏 } of real numbers is defined as follows:
𝒏𝟐 𝟒 + ⋯ + 𝒏𝟖 𝟒 = 𝟏𝟗𝟗𝟑.
𝟏 𝟓 𝟏 𝟓
𝒙𝟏 = 𝟐 (𝒙 + 𝒙) , 𝒙𝟐 = 𝟐 (𝒙𝟏 + 𝒙 ) , …, and in
Solution 𝟏
general,
(a) Let n = 2a + 1
𝟏 𝟓
𝒙𝒏+𝟏 = 𝟐 (𝒙𝒏 + 𝒙 ) 𝒇𝒐𝒓 𝒂𝒍𝒍 𝒏 ≥ 𝟏.
𝑛4 = (2𝑎 + 1)4 𝒏
𝒙𝒏−√𝟓 𝟐𝒏
= (4𝑎2 + 4𝑎 + 1)2 (a) Show that, for all n ≥ 1,
𝒙−√𝟓
= ( ) .
𝒙𝒏 +√𝟓 𝒙+√𝟓
= [4𝑎(𝑎 + 1) + 1]2
(b) Hence find 𝐥𝐢𝐦 𝒙𝒏 .
𝒏→∞
2 (𝑎 2
= 16𝑎 + 1) + 8𝑎(𝑎 + 1) + 1
Solution:-
2
= 16[𝑎(𝑎 + 1)] + 8𝑎(𝑎 + 1) + 1
107
Challenging Mathematical Problems
1 5 √5 𝑥𝑛−1 √5
(a) 𝑥𝑛 = 2 (𝑥𝑛−1 + 𝑥 )= 2
( +𝑥 )
𝑛−1 √5 𝑛−1
𝑥𝑛 1 𝑥𝑛−1 √5
⟾ = ( + ).
√5 2 √5 𝑥𝑛−1
By componendo-dividend, we get-
1 𝑥𝑛−1 √5
𝑥𝑛 − √5 2 ( √5 + 𝑥𝑛−1 ) − 1
=
𝑥𝑛 + √5 1 𝑥𝑛−1 √5
2 ( √5 + 𝑥𝑛−1 ) + 1
Q3. Sketch on plain paper, the graph of the
𝑥𝑛−1 2 + √5 − 2√5 𝑥𝑛−1
= function y = sin(𝒙𝟐 ), in the range 0 ≤ x ≤
𝑥𝑛−1 2 + √5 + 2√5 𝑥𝑛−1
√𝟒𝝅.
2 22
𝑥𝑛−1 − √5 𝑥𝑛−2 − √5 Solution:-
=( ) = ( ) =⋯
𝑥𝑛−1 + √5 𝑥𝑛−2 + √5 Taking different values of x, we get different y.
2𝑛−1
𝑥1 − √5
=( ) x 0 √𝜋/4 √𝜋/2 √𝜋 √3𝜋/2 √2𝜋
𝑥1 + √5
√5𝜋/2 √3𝜋 √7𝜋/2 √4𝜋
2𝑛
𝑥 − √5 y 0 1/√2 1 0 -1 0 1
=( ) (𝑝𝑟𝑜𝑣𝑒𝑑) 0 -1 0
𝑥 + √5
𝑥−√5
⟾ 𝑥+√5 < 1
2𝑛
𝑥−√5
∴(𝑥+√5) ⟶ 0 𝑎𝑠 𝑛 → ∞.
𝑥−√5
⟾ 𝑥+√5 ⟶ 0 ⟾ 𝑥 − √5 ⟶ 0
Q4. If n is a positive integer greater than 1
∴ lt 𝑥𝑛 = √5. such that 3n + 1 is perfect square, then show
𝑛 →∞
that n + 1 is the sum of three perfect squares.
Q2. Draw the region of points (x, y) in the
plane, which satisfy |y| ≤|x| ≤ 1. Solution:-
3n + 1 = 𝑎2
⤇ a is not a multiple of 3.
108
Challenging Mathematical Problems
Or, n = 3𝑘 2 + 2𝑘 ⁞
Or, n + 1 = 3𝑘 2 + 2𝑘 + 1 = 𝑘 2 + 𝑘 2 + 𝑤𝑘 > 𝑧𝑘
(𝑘 + 1)2
But 𝑤𝑘+1 = 𝑧𝑘+1
i.e. , sum of three perfect squares.
So, 𝑢𝑘 + 𝑤𝑘 = 𝑣𝑘 + 𝑧𝑘 ⤇ (u+ w)k = (v+
2 2
Taking a = 3k + 2, 3n + 1= (3k + 2) = 9𝑘 + z)k
6𝑘 + 4;
But 𝑢𝑘+1 + 𝑤𝑘+1 > 𝑣𝑘+1 + 𝑧𝑘+1 .
Or, n =3𝑘 2 + 4𝑘 + 1
So, u + w > v + z.
Or, n + 1= 3𝑘 2 + 4𝑘 + 2 = 𝑘 2 + (𝑘 + 1)2 +
(𝑘 + 1)2 Q6. Consider the set of points S = {(x, y): x, y
are non-negative integers ≤ n}.
i.e. sum of 3 perfect squares. [Proved]
Find the number of squares that can be
formed with vertices belonging to S and sides
parallel to the axes.
Q5. Let x = (𝒙𝟏 , 𝒙𝟐 , … , 𝒙𝒏 ), y =
(𝒚𝟏 , 𝒚𝟐 , … , 𝒚𝒏 ) , where 𝒙𝟏 , 𝒙𝟐 , … , 𝒙𝒏 Solution:-
, 𝒚𝟏 , 𝒚𝟐 , … , 𝒚𝒏 are real numbers. We write
The number of squares with sides of unit length
x > y, if for some k, 1 ≤ k ≤ (n- 1), 𝒙𝟏 =
= 𝑛 × 𝑛 = 𝑛2 .
𝒚𝟏 , 𝒙𝟐 = 𝒚𝟐 , … , 𝒙𝒌 = 𝒚𝒌 , but 𝒙𝒌+𝟏 >
𝒚𝒌+𝟏 . Show that for u = (𝒖𝟏 , … , 𝒖𝒏 ), The number of squares with sides of length 2
v=(𝒗𝟏 , … , 𝒗𝒏 ), w = (𝒘𝟏 , … , 𝒘𝒏 ) and z = units is = (𝑛 − 1) × (𝑛 − 1) = (𝑛 − 1)2 and so
(𝒛𝟏 , … , 𝒛𝒏 ), if u > v and w > z, then (u +w) on.
> (v +z).
∴ Total number of squares = 𝑛2 + (𝑛 − 1)2 +
𝑛(𝑛+1)(2𝑛+1)
Solution:- u > v ⋯ + 22 + 12 = 6
.
⁞
Solution:-
𝑢𝑘 = 𝑣𝑘 𝑠𝑖𝑛4 𝑥 𝑐𝑜𝑠 4 𝑥 1
+ = ,
But 𝑢𝑘+1 > 𝑣𝑘+1 𝑎 𝑏 𝑎+𝑏
109
Challenging Mathematical Problems
110
Challenging Mathematical Problems
P(x) = 𝑥 4 + 𝑎𝑥 3 + 𝑏𝑥 2 + 𝑐𝑥 + 𝑑, now sum of Q1. Sketch the set A ∩ B in the Argand Plane,
the roots= -a 𝒛+𝟏
where A = {z: |𝒛−𝟏| ≤ 1} and B= {z: |z| - Re z ≤
⟹ 𝛼+ 𝛽+ 𝛾+ 𝛿= -a 1}.
1
𝛿=7 ∴𝑦 2 = 2 (𝑥 + 2) is a parabola, having its vertex
1
We know, 𝛼𝛽 +𝛽𝛾 + 𝛾𝛿 +𝛿𝛼 +𝛼𝛾 +𝛽𝛿 = b at (− 2 , 0) and axis on X-axis.
𝛼𝛽 𝛾𝛿 = d
∴ a, b, c, d any integers.
111
Challenging Mathematical Problems
112
Challenging Mathematical Problems
Solution:-
Let y = 1- √𝑥 …………… (i)
3 Let us try to write,
And 𝑦0 = √𝑥 ……………. (ii)
𝑥 3 + 𝑥 + 1 = (𝑎𝑥 + 𝑏)(𝑐𝑥 2 + 𝑑𝑥 + 𝑒)
For function (i) x = 0, y= 1; x = 1, y =0; x=
1 1 = 𝑎𝑐𝑥 3 + (𝑏𝑐 + 𝑎𝑑)𝑥 2
,𝑦 = ; + (𝑏𝑑 + 𝑎𝑒)𝑥 + 𝑏𝑒;
4 2
113
Challenging Mathematical Problems
𝜋 sin ℎ𝑥
∫0 │ 𝑥
│𝑑𝑥, as x ranges from [0, 𝜋], so
𝜋 𝑠𝑖𝑛𝑧 𝜋 𝑠𝑖𝑛𝑧
Now, ∫0 │ 𝑧
│ 𝑑𝑧 = ∫0 𝑧 𝑑𝑧 ≥
𝜋 𝑠𝑖𝑛𝑧 1 𝜋
∫0 𝜋 𝑑𝑧 = 𝜋 ∫0 𝑠𝑖𝑛𝑧 𝑑𝑧
2𝜋 𝑠𝑖𝑛𝑧 𝜋 −𝑠𝑖𝑛𝑦
Now, ∫𝜋 │ │ 𝑑𝑧 = ∫0 │ │𝑑𝑦 , where
𝑧 𝜋+𝑦
𝜋 + y = z,
(ii) PD⏊ BC, PE ⏊ AC, PF ⏊AB.
𝜋 𝑠𝑖𝑛𝑦 1 𝜋 2
= ∫0 𝜋+𝑦 𝑑𝑦 ≥ ∫ 𝑠𝑖𝑛𝑦
2𝜋 0
𝑑𝑦 = 2𝜋 So, from (i) ⦟PAF = ⦟PAE = 30°,
3𝜋 𝑠𝑖𝑛𝑧
Proceeding in this way, ∫2𝜋 │ 𝑧
│𝑑𝑧 ≥ ⦟PCE = ⦟PCD = 30°, ⦟PBD= ⦟PBF= 30°.
2 𝑛𝜋 𝑠𝑖𝑛𝑧 2
, … . , ∫(𝑛−1)𝜋 │ 𝑧 │ ≥ 𝑃𝐹 1 𝐴𝐹
3𝜋 𝑛𝜋 ∴In 𝛥PAF, 𝐴𝐹 = tan 30 ° = 𝑜𝑟, 𝑃𝐹 = .
√3 √3
2 2 2 2 2 1
∴𝐼 ≥ 𝜋 + 2𝜋 + 3𝜋 + ⋯ + 𝑛𝜋 = 𝜋 (1 + 2 + ⋯ + Similarly, for 𝛥PBD, we get PD =
1 1 1
). 𝐵𝐷, 𝑎𝑛𝑑 for 𝛥PCE, we get PE= 𝐶𝐸.
𝑛 √3 √3
1
𝑃𝐷+𝑃𝐸+𝑃𝐹 (𝐵𝐷+𝐶𝐸+𝐴𝐹) 1
∴ 𝐵𝐷+𝐶𝐸+𝐴𝐹 = √3𝐵𝐷+𝐶𝐸+𝐴𝐹 = .
√3
Q9. Inside an equilateral triangle ABC, an
arbitrary point P is taken from which the As each of the PD, PE, PF can be represented
perpendiculars PD, PE and PF are dropped w.r.t. BD, CE, AF respectively, so the specified
onto the sides BC, CA and AB, respectively. ratio does not depend upon the choice of the
𝑷𝑫+𝑷𝑬+𝑷𝑭 point P.
Show that the ratio 𝑩𝑫+𝑪𝑬+𝑨𝑭 does not depend
upon the chice of the point P and find its Q10. AB is a chord of a circle C.
value.
(a) Find a point P on the circumference of C
Solution:- We have (i) AB = BC = CA & ⦟ B such that PA. PB is the maximum.
=⦟C =⦟A = 60°
(b) Find a point P on the circumference of C
[∵ 𝛥ABC is equilateral] which maximizes PA+ PB.
𝑥+𝑦 2 𝑥−𝑦 2
∴ PA.PB = xy = ( 2
) −(2
)
114
Challenging Mathematical Problems
Position of P:-
𝐴𝑃 𝐴𝐵
AP = sin 𝑃 × sin 𝐵 ; 𝐵𝑃 = sin 𝑃 × sin 𝐴 ;
𝐴𝐵
∴ AP + BP = sin 𝑃 × (sin 𝐴 + sin 𝐵) =
𝐴𝐵 𝐴+𝐵 𝐴−𝐵
sin 𝑃
× 2 sin 2 cos 2 =
𝑥 𝑦
𝐴𝐵 𝑃 𝐴−𝐵 Equation of PQ: + = 1.
sin 𝑃
. 2 cos 2 cos 2 . 𝑎 𝑏
ISI B.STAT & B.MATH Both the lines PQ & RS meet at point S,
whose locus we are to find, the variables
SUBJECTIVE QUESTIONS &
being a, b, which are connected by 𝑎2 +
SOLUTIONS 𝑏 2 = 𝐿2 .
115
Challenging Mathematical Problems
1 1
⟾ 𝑎 = (𝐿2 𝑥)3 & 𝑏 = (𝐿2 𝑦)3 ; (ii) Again, y = f(x) = [x] + √{𝑥}
(i) We know 0 ≤ {x} < 1 Now, distance travelled by troop, when the
soldier reached that point = x m.
⟾√{𝑥} ≥ {𝑥}
𝑥 𝑑𝑖𝑠𝑡𝑎𝑛𝑐𝑒
∴ Time taken = 𝑉 = .
⟾[x] + √{𝑥}≥ [x] + {x} 𝑡 𝑣𝑒𝑙𝑜𝑐𝑖𝑡𝑦
𝑥 𝑥+5 𝑉 𝑥+5
Given that 𝑉 = ⤇ 𝑉𝑠 =
𝑡 𝑉𝑠 𝑡 𝑥
…………….(i)
116
Challenging Mathematical Problems
Solution:- ⟾ (2 − 𝑚)2 − 𝑚2 ≥ 0
When ‘n’ is perfect square, ⟾ 2 − 2𝑚 ≥ 0 ⟾ 𝑚 ≤ 1.
Then √𝑛 is an integer.
117
Challenging Mathematical Problems
⟾ a (a -x) = 𝑥 2 ⟾𝑎2 − 𝑎𝑥 − 𝑥 2 = 0,
𝛱
= sin 𝐴 + sin 𝐵 + sin (𝐶 − )
2 𝑥 2 𝑥
⟾ ( ) + − 1 = 0,
𝑎 𝑎
𝛱
= sin 𝐴 + sin 𝐵 + sin 𝐷 , 𝐷=𝐶− .
2 𝑥 −1+√5
∴𝑎 = 2
.
𝛱
𝑁𝑜𝑤, 𝐴 + 𝐵 + 𝐶 = 𝛱; ∴ 𝐴 + 𝐵 + 𝐶 − 𝜃 𝜃 𝑥 −1+√5
2 Now, x = BC = 2a sin ∴ sin = = =
𝛱 2 2 2𝑎 4
=𝐴+𝐵+𝐷 = . sin 18°
2
𝜃
Let, f(x)= sin x, we plot its graph such that ⟾ = 18° ⟾ 𝜃 = 36°
2
𝛱
taking the abscissa A, B, D as 𝐴 + 𝐵 + 𝐷 = 2 ,
𝛱
⟾⦟BAC= 36°.
or, plotting in the interval [0, 2 ]
∴ Centroid of 𝛥PQR≡G≡
𝐴+𝐵+𝐷 sin 𝐴+sin 𝐵+sin 𝐷 Q10. Let a, b, c, d be positive real numbers
( , )
3 3 such that abcd = 1. Show that (1+ a)(1+ b)(1+
c)(1+ d)≥ 16.
We take a point f(x) = sin x, such that it is of the
same abscissa that of G, but of greater ordinate. 1+𝑎 1+𝑏 1+𝑐
Solution:- 2
≥ √𝑎, 2
≥ √𝑏, 2
≥ √𝑐,
𝐴+𝐵+𝐷 𝐴+𝐵+𝐷 1+𝑑
∴M≡( , 𝑠𝑖𝑛 3 ) 2
≥ √𝑑.
3
𝛱
When A, B, D are not distinct, i.e., A= B=D= 6 ,
equality holds. ⟾ sin 𝐴 + sin 𝐵 + sin 𝐷 ≤
3 3
2
, sin 𝐴 + sin 𝐵 − cos 𝐶 ≤ 2.
118
Challenging Mathematical Problems
1 1 𝑑 𝑛+1
Q1. For a real number x, let [x] denote the 𝑁𝑜𝑤, 𝑃𝑛+1 ( ) = 𝑒 𝑡 . 𝑛+1 . 𝑓(𝑡)
𝑡 𝑑𝑡
largest integer less than or equal to x and <x>
denote x – [x]. Find all the solutions of the
1 𝑑 1 −
1
−
1 1 1
= 𝑒𝑡 [ {𝑃𝑛 ( )} . 𝑒 𝑡 + 𝑒 𝑡 ( 2 ) 𝑃𝑛 ( )]
equations 13[x] +25 <x> = 271. 𝑑𝑡 𝑡 𝑡 𝑡
𝑑 1 1 1
Solution:- = [𝑃𝑛 ( )] + 2 𝑃𝑛 ( ).
𝑑𝑡 𝑡 𝑡 𝑡
13 [x] + 25 〈x〉 = 271 1 𝑑 𝑑𝑡
Let us put 𝑡 = x, 𝑃𝑛+1 (𝑥) = 𝑑𝑥 {𝑃𝑛 (𝑥)} / 𝑑𝑥 +
271−13[𝑥]
or, 〈x〉 = 25
, 𝑥2.
𝑑
We know 0 ≤ 〈x〉 <1, ∴ 𝑃𝑛 (𝑥) = 𝑥 2 𝑃𝑛 (𝑥) − 𝑥 2 𝑑𝑥 {𝑃𝑛 (𝑥)} =
𝑑
⟾0≤
271−13[𝑥]
<1 𝑥 2 (𝑃𝑛 (𝑥) − 𝑑𝑥 𝑃𝑛 (𝑥)).
25
⟾ 13[x] - 271 > -25 Q3. Study the derivative of the function f(x) =
𝒙𝟑 − 𝟑𝒙𝟐 + 𝟒, and roughly sketch the graph
⟾ [x] > 18. 9 of f(x), on plain paper.
The nearest integers in this interval are 19 and Solution:- 𝑓(𝑥) = 𝑥 3 − 3𝑥 2 + 4
20.
𝑓 ′ (𝑥) = 3𝑥 2 − 6𝑥 = 3𝑥(𝑥 − 2)
271−13×19
Putting [x] = 19, 〈x〉 = 25
= 0.96
f’(x) > 0 for -∞ ,x < 0
271−13×20
Putting [x] =n 20, 〈x〉 = 25
= 0.44 f’(x) < 0 for 0< x < 2
𝒅
𝑷𝒏+𝟏 (𝒙) = 𝒙𝟐 (𝑷𝒏 (𝒙) − 𝑷 (𝒙)).
𝒅𝒙 𝒏
119
Challenging Mathematical Problems
Solution:-
Q4. Study the derivative of the function So, total number of all possible ordered k –
tuples of such kinds are = (𝑘−1+100 ).
f(x) = 𝐥𝐨𝐠 𝒆 𝒙 − (𝒙 − 𝟏), 𝒇𝒐𝒓 𝒙 > 0, and 100
√25−𝑥 2 −4
Now, lim 𝑀(𝑥) = lim
𝑥→3 𝑥→3 𝑥−3
3
= − 4 [Do yourself applying L’Hospital Rule]
𝟐𝟓 𝟔
Q7. Solve 𝟔𝒙𝟐 − 𝟐𝟓𝒙 + 𝟏𝟐 + + 𝟐 = 𝟎.
𝒙 𝒙
Solution:-
25 6
6𝑥 2 − 25𝑥 + 12 + 𝑥
+ 𝑥 2 = 0;
6 25
⟾ 6𝑥 2 + 12 + 2
− 25𝑥 + =0
𝑥 𝑥
120
Challenging Mathematical Problems
6 25 (1 + 𝑥)𝑛+1 𝑛𝑐1 𝑥 2
⟾ 6𝑥 2 − 12 + 2
− 25𝑥 + + 24 = 0 = 𝑛𝑐0 𝑥 + +⋯
𝑥 𝑥 𝑛+1 2
1 2 1 𝑛𝑐𝑛 𝑥 𝑛+1
⟾ 6 (𝑥 − ) − 25 (𝑥 − ) + 24 = 0 + + 𝑐𝑜𝑛𝑠𝑡𝑎𝑛𝑡
𝑥 𝑥 𝑛+1
1 1
Let, 𝑥 − 𝑥 = 𝑦; so, 6𝑦 2 − 25𝑦 + 24 = 0 Putting x = 0, then constant = 𝑛+1.
3 1 (1 + 𝑥)𝑛+2
For y = 2 , 𝑥 = 2 𝑜𝑟 − 2.
(𝑛 + 1)(𝑛 + 2)
𝑛𝑐0 𝑥 2 𝑛𝑐1 𝑥 3
= + + …
1.2 2.3
Q8. (i) In the identity 𝑛𝑐𝑛 𝑥 𝑛+2 𝑥
+ +
𝒏! (𝑛 + 1)(𝑛 + 2) 𝑛 + 1
𝒙(𝒙 + 𝟏)(𝒙 + 𝟐) … (𝒙 + 𝒏) + 𝑐𝑜𝑛𝑠𝑡𝑎𝑛𝑡 .
𝒏
𝑨𝒌 1
= ∑ , Putting x= 0, then constant = (𝑛+1)(𝑛+2).
𝒙+𝒌
𝒌=𝟎
(1+𝑥)𝑛+2 𝑛𝑐0 𝑥 2 𝑛𝑐1 𝑥 3
Prove that 𝑨𝒌 = (−𝟏)𝒌 (𝒏𝒌) . So, (𝑛+1)(𝑛+2) = + + …+
1.2 2.3
𝑛𝑐𝑛 𝑥 𝑛+2 𝑥 1
𝟏 𝟏 + 𝑛+1 + (𝑛+1) (𝑛+2).
(ii) Deduce that:(𝒏𝟎) 𝟏.𝟐 − (𝒏𝟏) 𝟐.𝟑 + (𝑛+1)(𝑛+2)
𝟏 𝟏 𝟏
(𝒏𝟐) 𝟑.𝟒 − … + (−𝟏)𝒏 (𝒏𝒏) (𝒏+𝟏)(𝒏+𝟐) = 𝒏+𝟐 Putting x= -1, we get
𝑛𝑐0 𝑛𝑐1 𝑛𝑐2 𝑛𝑐
Solution:- 0= − + … + (−1)𝑛 (𝑛+1)(𝑛+2)
𝑛
−
1.2 2.3 3.4
1 1
(i) From n! = ∑𝑛𝑘=0 𝐴𝑘 (𝑥 + 1)(𝑥 + + (𝑛+1)(𝑛+2)
.
(𝑛+1)
2) … (𝑥 + 𝑘 − 1)(𝑥 + 𝑘 + 1) … (𝑥 + 𝑛) 1 1 1
So, (𝑛0) 1.2 − (𝑛1) 2.3 + (𝑛2) 3.4 + ⋯ +
Putting x = - k, 1 1
(−1)𝑛 (𝑛𝑛) (𝑛+1)(𝑛+2) = 𝑛+2. (Proved)
n! = ∑𝑛𝑘=0(−1)−𝑘 𝐴𝑘 . 𝑘! (𝑛 − 𝑘)!
⤇ 𝐴𝑘 = (−1)𝑘 (𝑛𝑘).
Q9. A regular five pointed star is inscribed in
𝑛 a circle of radius r. Show that the area of the
(ii)Considering (1 + 𝑥) = 𝑛𝑐0 + 𝑛𝑐1 𝑥 +
𝝅
𝟏𝟎𝒓𝟐 𝐭𝐚𝐧( )
𝑛𝑐2 𝑥 2 + ⋯ + 𝑛𝑐𝑛 𝑥 𝑛 . region inside the star is 𝟏𝟎
𝝅 .
𝟑−𝒕𝒂𝒏𝟐 ( )
𝟏𝟎
121
Challenging Mathematical Problems
Suppose, O be the centre of the circle which lies Q10. For the following function f study its
in the star and whereas ⦟MOL = 𝜃, so, ⦟LAM = derivatives and use them to sketch its graph
𝜃 on plain paper:
2
.
𝒙−𝟏 𝒙+𝟏
Here 5. R. 𝜃 = 2𝜋r f(x) = 𝒙+𝟏 + 𝒙−𝟏 for x ≠ -1, 1.
2𝜋 𝑥−1 𝑥+1
∴𝜃= Solution:- f(x) = 𝑥+1 + 𝑥−1 [𝑓𝑜𝑟 𝑥 ≠ 1, 𝑥 ≠
5
2(𝑥 2 +1) 4 −8𝑥
From 𝛥OAL, −1] = (𝑥 2 −1)
= 2 + 𝑥 2 −1. ∴ 𝑓 ′ (𝑥) = (𝑥 2 −1)2 .
𝜋
⦟AOL = 2 5 ⟹ 2 = 5 .
1 𝜋 For, −∞ < 𝑥 < −1, 𝑓 ′ (𝑥)𝑖𝑠 𝑝𝑜𝑠𝑖𝑡𝑖𝑣𝑒;
1 7𝜋 1
Thus area of 𝛥AOL = 2 𝑎𝑏 sin 10 = 2 𝑅 2
𝜋 𝜋
sin sin 10 𝜋
5 × sin 7
𝜋 10
𝑠𝑖𝑛2 7 10
1 𝜋 𝜋 𝜋
= 𝑅 2 2 sin cos sin
2 10 10 10
𝜋 𝜋
7𝜋 𝜋 𝑅 2 𝑠𝑖𝑛2 10 cos 10
[∵ sin = sin 3 ] =
10 10 𝜋 3𝜋
3 sin − 4 sin
10 10
2 𝜋
𝑅 tan 10
= 𝜋
3 − 𝑡𝑎𝑛2 10
𝜋 𝜋
[Dividing 𝑁 𝑟 & 𝐷 𝑟 by sin 10 𝑐𝑜𝑠 2 10]
𝜋
10𝑅2 tan
10
Hence, required area of the star = 𝜋
3−𝑡𝑎𝑛2
10
[proved]
122
Challenging Mathematical Problems
123
Challenging Mathematical Problems
Sol.: Let 𝑎1 < 𝑎2 < −< 𝑎𝑛 = 1996 and let q So p = 3, suppose p, q ≠3; then p│5𝑞 − 2𝑞
be the ratio of the geometric progression and q│5𝑝 − 2𝑝 . Without lose of generality
𝑎𝑖1 … … … 𝑎𝑖𝑛 ; clearly q≠ 0 ± 1. By reversing assume p >q, so that (p, q -1) = 1. Then if a is
the geometric progression if needed, we may an integer such that 2a ≡5 (mod q), then the
assume |q| > 1, and so |𝑎𝑖1 |< |𝑎𝑖2 | < −|𝑎𝑖𝑛 |. order of a mod q divides p as well as q -1, a
Note that either all of the terms are positive, or contradiction.
they alternate in sign; in the latter case, the
Hence one of p, q is equal to 3. If q ≠ 3, then
terms of either sign form a geometric
q│53 − 23 = 9.13. so q = 13, and similarly p
progression by themselves. ∊(3, 13).
There cannot be three positive terms, or else Thus the solutions are (p, q) = (3, 3), (3, 13),
we would have a three term geometric (13, 3).
progression a, b, c which is also an arithmetic
progression, violating the AM –GM inequality. 6. Find the side length of the smallest
equilateral triangle in which three dises of
Similarly, there cannot be three negative terms, radii 2, 3, 4 can be placed without overlap.
so there are at most two terms of each sign and
n ≤4. Sol.: A short computation shows that dises of
radii 3 and 4 can be fit into two corners of an
If n = 4, we have 𝑎1 < 𝑎2 < 0 < 𝑎3 < equilateral triangle of side 11√3 so as to just
𝑎4 𝑎𝑛𝑑 2𝑎2 = 𝑎2 + 𝑎3 , 2𝑎3 = 𝑎2 + 𝑎4 . In this touch, and that a disc of radius 2 easily fits into
case, q < -1 and the geometric progression is the third corner without overlap. On the other
either 𝑎3 , 𝑎2 , 𝑎4 , 𝑎1 𝑜𝑟 𝑎2 , 𝑎3 , 𝑎1 , 𝑎4 . Suppose hand, if the discs of radii 3 and 4 fit into an
the former occurs (the argument in similar in equilateral triangle without overlap, there exists
the latter case): then a line separating them (e.g. a tangent to one
2𝑎3 𝑞 = 𝑎3 𝑞 3 + 𝑎3 𝑎𝑛𝑑 2𝑎3 + 𝑎3 𝑞 3 + 𝑎3 𝑞 2, perpendicular to their line of centers) dividing
giving q =1, a contradiction. the triangle into a triangle and a (possibly
degenerate) convex quadrilateral. Within each
We deduce n = 3 and consider two possibilities. piece, the disc can be moved into one of the
If 𝑎1 < 𝑎2 < 0 < 𝑎3 = 1996, 𝑡ℎ𝑒𝑛 2𝑎2 = corners of the original triangle. Thus the two
𝑎2 𝑞 2 + 𝑎2 𝑞, so 𝑞 2 + 𝑞 − 2 = 0 𝑎𝑛𝑑 𝑞 = −2, discs fit into the corners without overlap, so the
yielding (𝑎1 , 𝑎2 , 𝑎3 ) = (−3992, −998, 1996). If side length of the triangle must be at least
𝑎1 < 0 < 𝑎2 < 𝑎3 = 1996, then 11√3.
124
Challenging Mathematical Problems
∠AOB, so O lies on the circumcircle of AHB, are disjoint and none contains the center
and similarly on the circumcircle of CHD. The square, so each contains one colored
radical axes of the circumcircles of AHB, CHD square. In particular, (2, 2) and (2, 4) are not
and ABCD concur; these lines are AB, CD and colored. Replacing (3, 3) with (2, 3) in the
HO, so E, H, O are collinear. Now note that list shows that (3, 2) and (3, 4) are not
𝜋
∠OHF = ∠OHC+∠CHF= ∠ODC+∠CBF = 2 − colored. Similar symmetric arguments now
𝜋
∠𝐶𝐴𝐷 + ∠𝐶𝐵𝐷. 𝑆𝑜 ∠𝐸𝐻𝐹 = ∠𝑂𝐻𝐹 = as show that no squares beside the center
2
desired. (Compare IMO 1985/5.) square can be covered, a contradiction.
Thus 7 squares are needed.
8. A 𝟕 × 𝟕 chessboard is given with its four
corners deleted. (a) Write -5 in the 7 squares listed above
(a) What is the smallest number of and 1 in the remaining squares. Then
squares which can be colored black so clearly each cross has a negative sum,
that an uncolored 5 squares (Greek) but the total of all of the numbers is 5
cross cannot be found? (−7) + (45 -7) = 3.
(b) Prove that an integer can be written in 9. If 𝛼, 𝛽, 𝛾 are the roots of 𝒙𝟑 − 𝒙 − 𝟏 = 𝟎,
𝟏−𝜶 𝟏−𝜷 𝟏−𝜸
each square such that the sum of the compute 𝟏+𝜶 + 𝟏+𝜷 + 𝟏+𝜸.
integers in each 5 squares cross is
negative while the sum of the Sol.: The given quantity equals
numbers in all squares of the board is
1 1 1
positive. 2( + + ) − 3.
𝛼+1 𝛽+1 𝛾+1
Sol.: The 7 squares
Since 𝑃(𝑥) = 𝑥 3 − 𝑥 − 1 has roots 𝛼, 𝛽, 𝛾, the
(2, 5), (3, 2), (3, 3), (4, 6), (5, 4), (6, 2), (6, 5) polynomial 𝑃(𝑥 − 1) = 𝑥 3 − 3𝑥 2 + 2𝑥 − 1
has roots 𝛼+1, 𝛽+1, 𝛾+1.
suffice, so we need only show that 6 or fewer
will not suffice. The crosses centered at By a standard formula, the sum of the
reciprocals of the roots of 𝑥 3 + 𝑐2 𝑥 2 + 𝑐1 𝑥 +
(2, 2), (2, 6), (3, 4), (5, 2), (5, 6), (6, 4) 𝑐0 𝑖𝑠 − 𝑐1 /𝑐0, so the given expression equals
2(2)-3= 1.
are disjoint, so one square must be colored in
each, hence 5 or fewer squares do not suffice. 10. Find all real solution to the following
Suppose exactly 6 squares are colored. Then system of equations:
none of the squares (1, 3), (1, 4), (7, 2) can be 𝟒𝒙𝟐
=𝒚
colored; by a series of similar arguments, no 𝟏 + 𝟒𝒙𝟐
𝟒𝒚𝟐
square on the perimeter can be colored. =𝒛
𝟏 + 𝟒𝒚𝟐
Similarly, (4, 3) and (4, 5) are not covered, and
𝟒𝒛𝟐
by a similar argument, neither is (3, 4) or (5, 4). = 𝒙.
𝟏 + 𝟒𝒛𝟐
Thus the center square (4, 4) must be covered.
4𝑥 2
Sol.: Define 𝑓(𝑥) = (1+4𝑥2 ) ; the range of f is [0,
Now the crosses centered at
1), so x, y, z must lie in that interval. If one of x,
(2, 6), (3, 3), (5, 2), (5, 6), (6, 4) y, z is zero, then all three are, so assume they
125
Challenging Mathematical Problems
11. Let f(n) be the number of permutations Using the sum-to-product formula, we rewrite
𝒂𝟏 , … , 𝒂𝒏 of the integers 1, …, n such that this as
(i) 𝒂𝟏 = 𝟏;
cos 𝛽 − cos 7𝛽 = cos 2𝛽 − cos 6𝛽 + cos 𝛽
(ii) │𝒂𝒊 − 𝒂𝒊+𝟏 │ ≤ 𝟐, 𝒊 = 𝟏, … , 𝒏 − 𝟏.
− cos 3𝛽.
Determine whether f(1996) is divisible by
Cancelling cos 𝛽, we have cos 3𝛽 − cos 7𝛽 =
3.
cos 2𝛽 − cos 6𝛽, which implies
Sol.: Let g(n) be the number of permutations of
sin 2𝛽 sin 5𝛽 = sin 2𝛽 sin 4𝛽.
the desired form with 𝑎𝑛 = 𝑛. Then either
𝜋
𝑎𝑛−1 = 𝑛 − 1 𝑜𝑟 𝑎𝑛−1 = 𝑛 − 2; in the latter Now sin 5𝛽 = sin 4𝛽 , 𝑠𝑜 9𝛽 = 𝜋 𝑎𝑛𝑑 𝛽 = 9 .
case we must have 𝑎𝑛−2 = 𝑛 − 1 𝑎𝑛𝑑 𝑎𝑛−3 =
𝑛 − 3. Hence g(n) = g(n-1) + g(n -3) for n ≥4. In 13. Let 𝒓𝟏 , 𝒓𝟐 , … , 𝒓𝒎 be a given set of positive
particular, the values of g(n) modulo 3 are g(1) = rational numbers whose sum is 1. Define
1, 1, 1, 2, 0, 1, 0, 0….. repeating with period 8. the function f by 𝒇(𝒏) = 𝒏 −
∑𝒎𝒌=𝟏 ⎿⌊𝒓𝒌 𝒏⌋⏌ for each positive integer n.
Now let h(n) = f(n) – g(n); h(n) counts Determine the minimum and
permutations of the desired from where n maximum values of f(n).
occurs in the middle, sandwiched between n-1
and n -2. Removing n leaves an acceptable Sol.: Of course ⎿⌊𝑟𝑘 𝑛⌋⏌ ≤ 𝑟𝑘 𝑛, 𝑠𝑜 𝑓(𝑛) ≥ 0,
permutation, and any acceptable permutation with equality for n = 0, so 0 is the minimum
on n -1 symbols can be so produced except value. On the other hand, we have 𝑟𝑘 𝑛 −
those ending in n -4, n -2, n -3, n -1. Hence h(n) ⎿⌊𝑟𝑘 𝑛⌋⏌ < 1, 𝑠𝑜 𝑓(𝑛) ≤ 𝑚 − 1.
= h(n -1)+ g(n -1) –g(n -4) = h(n -1)+ g(n -2); one
Here equality holds for n = t- 1 if t is the least
checks that h(n) modulo 3 repeats with period
common denominator of the 𝑟𝑘 .
24.
14. Let H be the orthocenter of acute triangle
Since 1996 ≡ 4 (mod 24), we have f(1996) ≡
ABC. The tangents from A to the circle with
f(4) = 4(mod 3), so f(1996) is not divisible by 3.
diameter BC touch the circle at P and Q.
12. Let ∆ABC be an isosceles triangles with AB Prove that P, Q, H are collinear.
= AC. Suppose that the angle bisector of
Sol.: The line PQ is the polar of A with respect to
∠B meets AC at D and that BC = BD + AD.
the circle, so it suffices to show that A lies on
Determine ∠A.
the pole of H.
(𝜋−𝛼)
Sol.: Let 𝛼 =∠A, 𝛽= and assume AB = 1.
4 Let D and E be the feet of the altitudes from A
and B, respectively; these also lie on the circle,
Then by the Law of Sines,
126
Challenging Mathematical Problems
𝑎1 + 𝑏1 = 4 (12, 4), (24, 8), (36, 12), (48, 16) {1, 2, 3, 4} {5, 6, 7, 8} {1, 2, 5, 6} {3, 4, 7, 8}
𝑎1 + 𝑏1 = 5 (20, 5), (40, 10), (15, 10), (30, 20), {3, 4, 5, 6} {1, 3, 5, 7} {2, 4, 6, 8} {1, 3, 6, 8}
(45, 30)
{2, 4, 5, 7} {1, 4, 5, 8} {2, 3, 6, 7} {1, 4, 6, 7}
𝑎1 + 𝑏1 = 6 (30, 6)
{1, 2, 7, 8} {2, 3, 5, 8}
𝑎1 + 𝑏1 = 7 (42, 7), (35, 14), (28, 21)
𝑎1 + 𝑏1 = 8 (40, 24)
127
Challenging Mathematical Problems
18. Prove that if a sequence {𝑮(𝒏)}∞ Sol.: We first note that if S is the circumcircle
𝒏=𝟎 of
of an acute triangle KLM, then for any point X
integers satisfies
≠S inside the triangle, we have
G(0) = 0,
min{𝑋𝐾, 𝑋𝐿, 𝑋𝑀} < 𝑆𝐾 = 𝑆𝐿 = 𝑆𝑀,
128
Challenging Mathematical Problems
Since the discs centered at K, L, M whose Sol.: Suppose K, L, M have already been
bounding circles pass through S cover the entire constructed. The triangle ALK and BYL are
triangle. similar because
20. An acute angle XCY and points A and B on = (𝑘 + 1)𝑓(𝑥) + 𝑓(𝑥) + 𝑓(𝑥 2 ) + 𝑘𝑓(𝑥)
the rays CX and CY, respectively, are given
such that │CX│< │CA│= │CB│< │CY│. = (2𝑘 + 2)𝑓(𝑥) + 𝑓(𝑥 2 ) = (3𝑘 + 4)𝑓(𝑥).
Show how to construct a line meeting the Setting x = 1995 so that f(x) ≠ 0, we deduce
ray CX and the segments AB, BC at the (𝑘 + 2)2 = 3𝑘 + 4, which has roots k = 0, -1.
points K, L, M, respectively, such that For k = 0, an example is given by
KA.YB = XA.MB= LA.LB ≠ 0
𝑓(𝑝1 𝑒1 … 𝑝𝑛 𝑒𝑛 ) = 𝑒1 𝑔(𝑝1 ) + ⋯ + 𝑒𝑛 𝑔(𝑝𝑛 ).
129
Challenging Mathematical Problems
Where g(5) = 1996 and g(p) = 0 for all 23. Let ABC be a triangle and construct
primes p ≠5 for k = 1, as example is given by squares ABED, BCGF, ACHI externally on
the sides of ABC. Show that the points D, E,
𝑓(𝑝1 𝑒1 … 𝑝𝑛 𝑒𝑛 ) = 𝑔(𝑝1 ) + ⋯ + 𝑔(𝑝𝑛 ) F, G, H, I are concyclic if and only if ABC is
22. A triangle ABC and points K, L, M on the equilateral or isosceles right.
sides AB, BC, CA respectively, are given Sol.: Suppose D, E, F, G, H, I are concyclic; the
such that perpendicular bisectors of DE, FG, HI coincide
𝑨𝑲 𝑩𝑳 𝑪𝑴 𝟏
= = = with those of AB, BC, CA respectively, so the
𝑨𝑩 𝑩𝑪 𝑪𝑨 𝟑 center of the circle must be the circumcenter
Show that if the circumcircles of the
O of ABC. By equating the distances OD and
triangles of the triangles AKM, BLK,
OF, we find
CML are congruent, then so are the in
circles of these triangles. (cos 𝐵 + 2 sin 𝐵)2 + 𝑠𝑖𝑛2 𝐵
= (cos 𝐶 + 2 sin 𝐶)2 = 𝑠𝑖𝑛2 𝐶
Sol.: We will show that ABC is equilateral, so
that AKM, BLK, CML are congruent and hence Expanding this end cancelling like terms, we
have the same in radius. determine 𝑠𝑖𝑛2 𝐵 + sin 𝐵 cos 𝐵 = 𝑠𝑖𝑛2 𝐶 +
sin 𝐶 cos 𝐶.
Let R be the common circumradius; then
Now note that
KL = 2R sin A, LM = 2R sin B, MK = 2R sin C,
2(𝑠𝑖𝑛2 𝜃 + sin 𝜃 cos 𝜃) = 1 − cos 2𝜃 + sin 𝜃
So the triangles KLM and ABC are similar.
Now we compare areas: 𝜋
= 1 + √2 sin (2𝜃 − ).
2
4
[AKM] = [BLK] = [CLM] = [ABC],
9 𝜋 𝜋
Thus we either have B = C or 2𝐵 − 4 + 2𝐶 − 4
1
So, [KLM] = 3[ABC] and the coefficient of
3𝜋
1 = 𝜋, 𝑜𝑟 𝐵 + 𝐶 = .
similarity between KLM and ABC must be √ . 4
3
By the law of cosines applied to ABC and In particular, two of the angles must be equal,
AKM. say A and B, and we either have A = B = C, so
the triangle is equilateral, or 𝐵 + (𝜋 − 2𝐵) =
𝑎2 = 𝑏 2 + 𝑐 2 − 2𝑏𝑐 cos 𝐴 3𝜋 𝜋
4
, in which case A = B = 4 and the triangle is
1 2 2𝑝 2 𝑐 2 2𝑏 𝑐 isosceles right.
𝑎 = ( ) + ( ) −2 cos 𝐴.
3 3 3 3 3
24. Let a, b be positive integers with a odd.
2 2 2 Define the sequence {𝒖𝒏 } as follows: 𝒖𝟎 =
From these we deduce 𝑎 = 2𝑏 − 𝑐 , and
similarly 𝑏 2 = 2𝑐 2 − 𝑎2 , 𝑎2 = 2𝑎2 − 𝑏 2 . 𝒃 & n ∊ ℕ.
Combining these gives 𝑎2 = 𝑏 2 = 𝑐 2 , so ABC 𝟏
𝒖 𝒊𝒇 𝒖𝒏 𝒊𝒔 𝒆𝒗𝒆𝒏
is equilateral, as desired. 𝒖𝒏+𝟏 = { 𝟐 𝒏
𝒖𝒏 + 𝒂 𝒐𝒕𝒉𝒆𝒓𝒘𝒊𝒔𝒆
(a) Show that 𝒖𝒏 ≤ 𝒂 for some n ∊ℕ.
(b) Show that the sequence {𝒖𝒏 } is
periodic from some point onwards.
130
Challenging Mathematical Problems
131
Challenging Mathematical Problems
Translate each point of AM by the vector MB. E lie on that circle, so ZN = ZE implies
No colored point can have a colored image, so that ZE ⊥MN.
all of the colored intervals of AB can be placed (b) As determined in (a), MN is the
in MB without overlap, and their total length perpendicular bisector of segment ZE.
1 The angle bisector AI of ∠EAZ passes
therefore does not exceed 10. Applying this
through the midpoint of the minor arc
reasoning to each of the 5 segments gives the
EZ, which clearly lies on MN;
desired result.
therefore this midpoint is k. By
28. Prove that every integer k > 1 has a similar reasoning, L is the midpoint of
multiple less than 𝒌𝟒 whose decimal the major are EZ. Thus KL is also a
expansion has at most four distinct digits. diameter of circle EAZ, so KL = MN.
30. Given 81 natural numbers whose prime
Sol.: Let n be the integer such that 2𝑛−1 ≤ 𝑘 ≤ divisors belong to the set {2, 3, 5}, prove
2𝑛 . For n ≤6 the result is immediate, so there exist 4 numbers whose product is
assume n > 6. the fourth power of an integer.
Let S be the set of nonnegative integers less Sol.: It suffices to take 25 such numbers. To
than 10𝑛 whose decimal digits are all 0s or 1s. each number, associate the triple (𝑥2 , 𝑥3 , 𝑥5 )
Since │S│ = 2𝑛 > 𝑘, we can find two recording the parity of the exponents of 2, 3
elements a < b of S which are congruent and 5 in its prime factorization. Two numbers
modulo, k, and b − a only has the digits 8, 9, 0, have the same triple if and only if their
1 in its decimal representation. On the other product is a perfect square. As long as there
hand, are 9 numbers left, we can select two whose
product is a square, in so doing, we obtain 9
𝑏 − 𝑎 ≤ 𝑏 ≤ 1 + 10 + ⋯ + 10𝑛−1 < 10𝑛
such pairs. Repeating the process with the
< 16𝑛−1 ≤ 𝑘 4 ,
square roots of the products of the pairs, we
Hence b – a is the desired multiple. obtain four numbers whose product is a
fourth power.
29. Let ABC be ab acute triangle, AD, BE, CZ its
altitudes and H its orthocenter. Let AI, A𝛩 31. Prove the following inequality for positive
be the internal and external bisectors of real numbers x, y, z:
angle A. Let M, N be the midpoints of BC, 𝟏 𝟏
(𝒙𝒚 + 𝒚𝒛 + 𝒛𝒙) ( 𝟐
+
AH, respectively. Prove that (𝒙 + 𝒚) (𝒚 + 𝒛)𝟐
(a) MN is perpendicular EZ; 𝟏 𝟗
+ ) ≥ .
(b) If MN cuts the segment AI, A𝛩 at (𝒛 + 𝒙)𝟐 𝟒
the points K, L, then KZ = AH.
Sol.: After clearing denominators, the given
Sol.: inequality becomes
132
Challenging Mathematical Problems
means the coefficient of 𝑥 3 𝑦 3 in the final We need only make sure that the sequence
expression is -6, and that 𝑥 2 𝑦 2 𝑧 2 is 6.) obtained is indeed decreasing, but this
follows because by assumption, 𝑚 <
Recall Schur’s inequality: (𝑎𝑘𝑚+1), 𝑎𝑛𝑑 𝑠𝑜 𝑚 − (𝑎𝑘𝑘 ) < (𝑘−1
𝑎𝑘
).
𝑥(𝑥 − 𝑦)(𝑥 − 𝑧) + 𝑦(𝑦 − 𝑧)(𝑦 − 𝑥)
33. Let P(x) be a polynomial with rational
+ 𝑧(𝑧 − 𝑥)(𝑧 − 𝑦) ≥ 0
coefficients such that 𝑷−𝟏 (𝑸) ⊆ 𝑸. Show
Multiplying by 2xyz and collecting symmetric that P is linear.
terms, we get
Sol: By a suitable variable substitution and
constant factor, we may assume P(x) is monic
∑ 𝑥 4 𝑦𝑧 − 2𝑥 3 𝑦 2 𝑧 + 𝑥 2 𝑦 2 𝑧 2 ≥ 0
and has integer coefficients; let P(0)= 𝑐0 . If p
𝑠𝑦𝑚
is a sufficiently large prime, the equation
On the other hand, P(x)= p +𝑐0 has a single real root, which by
assumption is rational and which we may also
∑(𝑥 5 𝑦 − 𝑥 4 𝑦 2 ) + 3(𝑥 5 − 𝑥 3 𝑦 3 ) ≥ 0 assume is positive (since P has positive
𝑠𝑦𝑚 leading coefficient). However, by the rational
root theorem, the only rational roots of P(x) –
By two applications of AM-GM; combining the
p - 𝑐0 can be ±1 𝑎𝑛𝑑 ± 𝑝. Since the root must
last two displayed inequalities gives the
be positive and cannot be 1 for large p, we
desired result.
have P(p) –p -𝑐0 = 0 for infinitely many p, so
32. Prove that for every pair m, k of natural P(x) = x +𝑐0 is linear.
numbers, m has a unique representation in
34. For each positive integer n, find the
the from
𝒂𝒌 𝒂𝒌−𝟏 𝒂𝒕 greatest common divisor of n! +1 and
𝒎 = ( )+( ) + ⋯+ ( ) (n+1)!.
𝒌 𝒌−𝟏 𝒕
Sol: If n + 1 is composite, then each prime
where 𝒂𝒌 > 𝒂𝒌−𝟏 > ⋯ > 𝒂𝒕 ≥ 𝒕 ≥ 𝟏.
divisor of (n+ 1)! is a prime less than n,
Sol.: We first show uniqueness. Suppose m is which also divides n! and so does not divide
represented by two sequences 𝑎𝑘 , … , 𝑎𝑡 and n! +1. Hence f(n) = 1. If n +1 is prime, the
𝑏𝑘 , … . , 𝑏𝑡 . Find the first position in which they same argument shows that f(n) is a power of
differ, without loss of generally, assume this n +1, and in fact n +1 │n! +1 by Wilson’s
position is k and that 𝑎𝑘 > 𝑏𝑘 . Then theorem. However, (𝑛 + 1)2 does not divide
(n +1)!, and thus f(n) = n +1.
𝑏 𝑏 𝑏 −𝑘+1
𝑚 ≤ ( 𝑘 ) + ( 𝑘−1 ) + ⋯ + ( 𝑘 )<
𝑘 𝑘−1 1
𝑏 +1
( 𝑘 ) ≤ 𝑚, a contradiction.
1 35. For each positive integer n, let S(n) be the
sum of the digits in the decimal expansion
To show existence, apply the greedy
of n. Prove that for all n,
algorithm: find the largest 𝑎𝑘 such that
𝑎 𝑺(𝟐𝒏) ≤ 𝟐𝑺(𝒏) ≤ 𝟏𝟎𝑺(𝟐𝒏) & show
( 𝑘 ) ≤ 𝑚, and apply the same algorithm with that there exists n such that S(n) =
𝑚
m and k replaced by 𝑚 − (𝑎𝑘𝑘 ) 𝑎𝑛𝑑 𝑘 − 1. 1996S(3n).
133
Challenging Mathematical Problems
Solution: It is clear that S(a +b) ≤ S(a) + 38. Let 𝑭𝒏 denote the Fibonacci sequence, so
S(b), with equality if and only if there are no that 𝑭𝟎 = 𝑭𝟏 = 𝟏 and 𝑭𝒏+𝟐 = 𝑭𝒏+𝟏 + 𝑭𝒏
carries in the addition of a and b. Therefore for n ≥0. Prove that
S(2n) ≤ 2S(n). Similarly S(2n) ≤ 5S(10n) = (i) The statement “𝑭𝒏+𝒌 − 𝑭𝒏 is
5S(n). An example with S(n) = 1996S(3n) is divisible by 10 for all positive
133 … 35 (with 5968 threes). integers n” is true if k = 60 and
false or any positive integer k
36. Let F be +the midpoint of side BC of < 60;
triangle ABC. Construct isosceles right (ii) The statement “𝑭𝒏+𝒕 − 𝑭𝒏 is
triangles ABD and ACE externally on sides divisible by 100 for all positive
AB and AC with the right angles at D and E, integers n” is true if t = 300
respectively. Show that DEF is an isosceles and false or any positive
right triangle. integer t<300.
Solution: Identifying A, B, C with numbers on Solution: A direct computation shows that the
(𝐵+𝐶)
the complex plane, we have F = 2
,𝐷 = Fibonacci sequence has period 3 modulo 2
𝐵 + (𝐴 − 𝐵)𝑟, 𝐸 = 𝐴 + (𝐶 − 𝐴)𝑟, 𝑤ℎ𝑒𝑟𝑒 𝑟 = and 20 modulo 5(compute terms until the
(1+𝑖) 𝐴(1−𝐼) 𝐵 𝐶𝑖
. 𝑇ℎ𝑒𝑛 𝐸 − 𝐹 = 2 − 2 + 2 𝑎𝑛𝑑 𝐷− initial terms 0, 1 repeat, at which time the
2
𝐴(1+𝑖) 𝐵𝑖 𝐶
entire sequence repeats), yielding (a). As for
𝐹 = 2 − 2 − 2 ; in particular, 𝐷 − 𝐹 = (b), one computes that the period mod 4 is 6.
𝑖(𝐸 − 𝐹) and so DEF is an isosceles right The period mod 25 turns out to be 100, which
triangle. is awfully many terms to compute by hand,
but knowing that the period must be a
multiple of 20 helps, and verifying the
37. Show, with proof, how to dissect a square recurrence 𝐹𝑛+8 = 𝑡𝐹𝑛+4 + 𝐹𝑛 , where t is an
into at most five pieces in such a way that integer congruent to 2 modulo 5, shows that
the pieces can be reassembled to from the period divides 100, finally, an explicit
three squares no two of which have the computation shows that the period is not 20.
same area. 39. Prove that for all positive integers n,
𝒏
Solution: We dissect a 7 × 7 square into a 2 × 𝟐𝟏/𝟐 . 𝟒𝟏/𝟒 … . (𝟐𝒏 )𝟏/𝟐 < 4.
2 square A, a 3 × 3 square B, and three pieces
Solution: It is sufficient to show
C, D, E which from a 6 × 6 square, as shown
below. 𝑥
𝑛
∑ = 2;
𝐶𝐶𝐶𝐶𝐶𝐴𝐴 2𝑛
𝑛=1
𝐶𝐶𝐶𝐶𝐶𝐴𝐴
𝐶𝐶𝐶𝐶𝐶𝐷𝐷 𝑥 𝑥 𝑥 𝑥
𝑛 1 1
𝐶𝐶𝐶𝐶𝐶𝐷𝐷 ∑ 𝑛 = ∑ ∑ 𝑘 = ∑ 𝑛−1 = 2.
𝐶𝐶𝐶𝐶𝐵𝐵𝐵 2 2 2
𝑛=1 𝑛=1 𝑛=1 𝑛=1
𝐶𝐶𝐶𝐶𝐵𝐵𝐵
𝐸𝐸𝐸𝐸𝐵𝐵𝐵 40. Let p be a prime number and a, n positive
integers.
Prove that if 𝟐𝒑 + 𝟑𝒑 = 𝒂𝒏 , then n =
1.
134
Challenging Mathematical Problems
135
Challenging Mathematical Problems
45. Let A and B be opposite vertices of a cube Solution: It suffices to draw 2n -1 squares:
of edge length 1. Find the radius of the in terms of coordinates, we draw a square
sphere with center interior to the cube, with opposite corners (0, 0) and (i, i) for
tangent to the three faces meeting at A and 1 ≤ i ≤ n and a square with opposite
tangent to the three edges meeting at B. corners (i, i) and (n, n) for 1 ≤ i≤ n -1.
Solution: Introduce coordinates so that A = To show this many squares are necessary,
(0, 0, 0), B = (1, 1, 1) and the edges are note that the segments from (0, i) to (1, i)
parallel to the coordinate axes. If r is the and from (n -1, i) to (n, i) for 0 < i < n all
radius of the sphere, then (r, r, r) is its center, must lie on different squares, so surely 2n
and (r, 1, 1) is the point of tangency of one of -2 squares are needed. If it were possible
the edges at B. Therefore 𝑟 2 = 2(1 − to obtain the complete grid with 2n -2
𝑟)2 , 𝑔𝑖𝑣𝑖𝑛𝑔 𝑟 2 − 4𝑟 + 2 = 0 and so r = 2 − √2 squares, each of these segments would lie
(the other root puts the center outside of the on one of the squares, and the same
cube). would hold for the segments from (i, 0) to
(i, 1) and from (i, n-1) to (i, n) for 0 < I <
n. Each of the aforementioned horizontal
46. Given an alphabet with three letters a, b, c segments shares a square with only two
find the number of words of n letters of the vertical segments, so the only
which contain an even number of a’s. possible arrangements are the one we
gave above without the square with
Solution: If there are 2k occurrences of a, corners (0, 0) and (n, n), and the 90°
𝑛 rotation of this arrangement, both of
these can occur in (2𝑘 ) places, and the
remaining positions can be filled in 2𝑛−2𝑘 which are insufficient. Hence 2n -1
ways. So the answer is squares are necessary.
136
Challenging Mathematical Problems
that no two triangles have overlapping Solution: Assume, on the contrary, that there
interiors, and no vertex lies in the interior exist p > 0 such that 𝑎𝑝+𝑛 = 𝑎𝑛 for every n.
of an edge of another triangle. Let A, B, C Since ⌊𝑥𝑛 ⌋ ⟶ ∞ 𝑎𝑠 𝑛 → ∞,we have
be three vertices of the triangulation and ⎿⌊𝑥 𝑛+𝑝 ⌋⏌ − ⎿⌊𝑥 𝑛 ⌋⏌ > 0 for some n; then
let 𝜽 be the smallest angle of the triangle setting 𝑎𝑛+𝑝 = 𝑎𝑛 and solving for x, we get
∆ABC. Suppose no vertices of the
triangulation lie inside the circumcircle of ⌊𝑥 𝑛+𝑝+1 ⌋ − ⌊𝑥 𝑛+1 ⌋
𝑥=
∆ABC. Prove there is a triangle 𝜎 in the ⌊𝑥 𝑛+𝑝 ⌋ − ⌊𝑥 𝑛 ⌋
triangulation such that 𝜎 ∩ ∆ABC ≠ 𝜃 and
And so x is rational.
every angle of 𝜎 is greater than 𝜃.
Put y = 𝑥 𝑝 and
Sol.: We may assume 𝜃 = ∠A. The case where
ABC belongs to the triangulation is easy, so 𝑝−1
assume this is not the case. If BC is an edge of 𝑏𝑚 = ∑ 𝑥 𝑝−𝑘−1 𝑎𝑚𝑝+𝑘
the triangulation, one of the two triangles 𝑘=0
bounded by BC has common interior points = ⎿⌊𝑥 𝑚+𝑝 ⌋ − 𝑥 𝑝 ⎿⌊𝑥 𝑚 𝑟⌋⏌
with ABC, and this triangle satisfies the = ⎿⌊𝑦 𝑚+1 ⌋ − 𝑦⎿⌊𝑦 𝑚 ⌋⏌.
desired condition. Otherwise, there is a
triangle BEF in the triangulation whose Since 𝑎𝑝+𝑛 = 𝑎𝑝 , we have 𝑏𝑚+1 = 𝑏𝑚 , and y
interior intersects BC. Since EF crosses BC at is also rational number which is not an
an interior point, ∠BEF < ∠BAF < ∠BAC, so integer. Now put 𝑐𝑚 = ⎿⌊𝑦 𝑚+1 −
triangle BEF satisfies the desired condition. 𝑦 𝑚 ⌋⏌; 𝑡ℎ𝑒𝑛 𝑐𝑚+1 = 𝑦𝑐𝑚 = 𝑦 𝑚 𝑐1 . This means
𝑐𝑚 cannot be an integer for large m, a
contradiction.
49. Let m and n be positive integers with 51. Let 𝜃 be the maximum of the six angles
gcd(m, n) = 1. Compute gcd(𝟓𝒎 + between the edges of a regular
𝟕𝒎 , 𝟓𝒏 + 𝟕𝒏). tetrahedron and a given plane. Find the
minimum value of 𝜃 over all positions of
Sol.: Let 𝑠𝑛 = 5𝑛 + 7𝑛 . 𝐼𝑓 𝑛 ≥ 2𝑚 , note that
the plane.
𝑠𝑛 = 𝑠𝑚 𝑠𝑛−𝑚 − 5𝑚 7𝑚 𝑠𝑛−2𝑚 ,
Sol.: Assume the edges of the tetrahedron 𝛤
So gcd(𝑠𝑚 , 𝑠𝑛 ) = gcd(𝑠𝑚 , 𝑠𝑛−2𝑚 ) … similarly, = ABCD have length l. If we place the
if m < n < 2m, we have gcd(𝑠𝑚 , 𝑠𝑛 )= tetrahedron so that AC and BC are parallel to
gcd(𝑠𝑚 , 𝑠𝑛−2𝑚 ). Thus by the Euclidean the horizontal plane H. We obtain 𝜃 = 45°,
algorithm, we conclude that if m + n is even, and we shall show this is the minimum angle.
then gcd(𝑠𝑚 , 𝑠𝑛 ) = gcd(𝑠1 , 𝑠2 ) = 12, and if m Let a, b, c, d be the projections of A, B, C, D to
+ n is odd, then gcd(𝑠𝑚 , 𝑠𝑛 ) = gcd(𝑠0 , 𝑠1 ) = 2. the horizontal plane H, and 𝑙1 , … , 𝑙6 the
projection of the edges 𝐿1 , … , 𝐿6 . Since the
50. Let x > 1 be a real number which is not an angle between 𝐿1 and H has cosine l, it
integer. For n = 1, 2, 3, …., let 𝒂𝒏 = suffices to consider the shortest 𝑙𝑖 .
⎿⌊𝒙𝒏+𝟏 ⌋⏌ − 𝒙⎿⌊𝒙𝒏 ⌋. Prove that the
If a, b, c, d from a convex quadrilateral with
sequence {𝒂𝒏 } is not periodic.
largest angle at a, then one of ab or ad is at
137
Challenging Mathematical Problems
1
most since bd ≤ 1. Otherwise, it is easily n even, 𝑙 = 4𝑟 𝑞 2 𝑝2𝑎𝑛−2 𝑞2 𝑝𝑟 . 𝑞 2 𝑝2𝑎𝑛−1 +
√2
shown that one of the 𝑙1 originating from the 1
vertex inside the convex hull has length at n even, 𝑙 = 4𝑟 + 2𝑞 2 𝑝2𝑎𝑛−2 + 𝑞𝑞 2 𝑝𝑟 +
1
most . 𝑞𝑞 2 𝑝2𝑎𝑛−1 + 𝑞
√3
138
Challenging Mathematical Problems
Sol.: Assume AB ≠ CD. Draw the plane Solution: The statement holds for n = 9 by
through AC bisecting the dihedral angle choosing 1, 2, 22 , … , 28 , since in that case
formed by the planes ABC and ACD, then
draw a line l in that plane perpendicular to AC │𝜖1 + ⋯ + 𝜖𝑔28 │ ≤ 1 + 2 + ⋯ + 28 < 93 .
through the midpoint O to AC. Now let B’ and
However, if n = 10, then 210 > 103 , so by the
D’ be the images of B and D, respectively,
pigeonhole principle, there are two subsets A
under the half-turn around the line l; by
and B of {𝑥1 , … , 𝑥10 } whose sums are
assumption, B’ ≠ D and D’ ≠ B; since ∠BAC =
congruent modulo 103 . Let 𝜖1 = 1 if 𝑥𝑖 occurs
∠ACD, B’ lies on CD and D’ lies on AB. Now
in A but not in B, -1 if 𝑥𝑖 occurs in B but not in
note that the quadrilateral BB’D’D has total
A, and 0 otherwise; then ∑ 𝜖𝑖 𝑥𝑖 is divisible by
angular sum 2𝜋. However, a non-polar
𝑛3 .
quadrilateral always has total angular sum
less than 2𝜋 (divide it into two triangles, 58. Let x, y be real numbers. Show that if the
which each have angular sum 𝜋, and apply the set
spherical triangle inequality) ∠ABC + ∠CBD
{𝐜𝐨𝐬(𝒏𝝅𝒙) + 𝐜𝐨𝐬(𝒏𝝅𝒚) │𝒏 ∈ 𝑵}
> ∠ABD, so the lines AB and CD are coplanar.
Is finite, then x, y ∈ Q.
Contradicting the assumption that ABCD is a
tetrahedron. Sol.: Let 𝑎𝑛 = cos 𝑛𝜋𝑥 𝑎𝑛𝑑 𝑏𝑛 = sin 𝑛𝜋𝑥.
Then
56. For a natural number k, let p(k) denote the
smallest prime number which does not (𝑎𝑛 + 𝑏𝑛 )2 + (𝑎𝑛 − 𝑏𝑛 )2 = 2(𝑎𝑛 2 + 𝑏𝑛 2 )
divide k. If p(k) > 2, define q(k) to be the = 2 + (𝑎2𝑛 + 𝑏2𝑛 ).
product of all primes less than p(k),
otherwise let q(k)= 1. Consider the If {𝑎𝑛 + 𝑏𝑛 } is finite, it follows that {𝑎𝑛 − 𝑏𝑛 }
𝒙𝒏 𝒑(𝒙𝒏 ) is also a finite set, and hence that {𝑎𝑛 } is
sequence. 𝒙𝟎 = 𝟏, 𝒙𝒏+𝟏 = 𝒏=
𝒒(𝒙𝒏 )
finite, since
𝟎, 𝟏, 𝟐, …
Determine all natural numbers n such 1
𝑎𝑛 = [(𝑎𝑛 + 𝑏𝑛 )(𝑎𝑛 − 𝑏𝑛 )].
that 𝒙𝒏 = 𝟏𝟏𝟏𝟏𝟏𝟏. 2
Sol.: An easy induction shows that if And similarly {𝑏𝑛 } is finite. In particular,
𝑝0 , 𝑝1 , …, are the primes in increasing order 𝑎𝑚 = 𝑎𝑛 for some m < n, and so (n –m)𝜋x is
an n has base 2 representations 𝑐0 + 2𝑐1 + an integral multiple of 𝜋. We conclude x and y
4𝑐2 + ⋯, then 𝑥𝑛 = 𝑝0 𝑐0 𝑝1 𝑐1 … in particular, are both rational.
111111 = 3.7.11.13.37 =
59. Let ABCD be a cyclic quadratilateral and
𝑝1 𝑝3 𝑝4 𝑝5 𝑝10 , 𝑠𝑜 𝑥𝑛 = 111111 if and only if n
let M be the set of incenters and excenters
= 210 + 25 + 24 + 23 + 21 = 1082.
of the triangles BCD, CDA, DAB, ABC (for a
57. Find the greatest positive integer n for total of 16 points). Show that there exist
which there exist n nonnegative integers two sets of parallel lines K and L, each
𝒙𝟏 , 𝒙𝟐 , … , 𝒙𝒏, not all zero, such that for any
139
Challenging Mathematical Problems
consisting of four lines, such that any line Now consider the excenters of the from
of K ∪ L contains exactly four points M. 𝑋𝑌 where X and Y are opposite vertices in
ABCD. We shall prove the claim with K =
Solution: Let T be the midpoint of the arc {𝐵𝐶 𝐶𝐵 , 𝐼𝐶 𝐼𝐵 , 𝐼𝐷 𝐼𝐴 , 𝐴𝐷 𝐷𝐴 }, L =
AB of the circumcircle of ABC, I the {𝐴𝐵 𝐵𝐴 , 𝐼𝐴 𝐼𝐵 , 𝐼𝐶 𝐼𝐷 , 𝐶𝐷 𝐷𝐶 }.
incenter of ABC, and 𝐼𝐵 , 𝐼𝐶 the excenters of
ABC opposite B and C, respectively. We Consider the rectangle 𝐵𝐶 𝐼𝐷 𝐵𝐴 𝑃, where P
first show TI = TA = TB = T𝐼𝑐 . Note that is an unknown point. From the second
observation above, the midpoint K of
(∠𝐶+∠𝐴)
∠TAI = ∠TAB + ∠BAI = = diagonal 𝐵𝐴 𝐵𝐶 is the midpoint of arc CDA,
2
∠𝐼𝐶𝐴 + ∠𝐼𝐴𝐶 = ∠𝑇𝐴𝐼 so it lies on the internal bisector BK of
triangle ABC. Again by the first
So TI = TA, and similarly TI = TB. observation, we conclude M = 𝐷𝐴 ,
Moreover, in the right triangle so𝐷𝐴 lies on the lines 𝐵𝐶 𝐶𝐵 and 𝐵𝐴 𝐴𝐵 , and
𝜋 𝜋 so on, proving the claim.
𝐴𝐼𝐶 𝐼, ∠𝐴𝐼𝐶 𝑇 = − ∠𝐴𝐼𝑇 = − ∠𝑇𝐴𝐼
2 2
60. Let n ≥ 3 be an integer and 𝒙𝟏 , 𝒙𝟐 , … , 𝒙𝒏−𝟏
= ∠𝑇𝐴𝐼𝐶 , 𝑠𝑜 𝑇𝐴 = 𝑇𝐼𝐶 𝑎𝑙𝑠𝑜 nonnegative integers such that
𝒙𝟏 + 𝒙𝟐 + ⋯ + 𝒙𝒏−𝟏 = 𝒏
We next show that the midpoint U of 𝐼𝐵 𝐼𝐶 𝒙𝟏 + 𝟐𝒙𝟐 + ⋯ + (𝒏 − 𝟏)𝒙𝒏−𝟏
is also the midpoint of the arc BAC. Note = 𝟐𝒏 − 𝟐.
that the line 𝐼𝐵 𝐼𝐶 bisects the exterior Find the minimum of the sum
angles of ABC at A, so the line 𝐼𝐵 𝐼𝐶 passes 𝒏−𝟏
through the midpoint V of the arc BAC. 𝑭(𝒙𝟏 , … , 𝒙𝒏−𝟏 ) = ∑ 𝒌𝒙𝒌 (𝟐𝒏 − 𝒌).
Considering the right triangles 𝒌=𝟏
(𝐼 𝐼 )
𝐼𝐵 𝐵𝐼𝐶 𝑎𝑛𝑑 𝐼𝐵 𝐶𝐼𝐶 , we note BU = 𝐵2 𝐶 = Sol.: The desired sum can be written as
𝐶𝑈, so U lies on the perpendicular
𝑛−1
bisector of BC, which suffices to show U =
V. (Note that 𝐼𝐵 𝑎𝑛𝑑 𝐼𝐶 lie on the same 2𝑛(2𝑛 − 2) − ∑ 𝑘 2 𝑥𝑘 .
𝑘=1
side of BC as A, so the same is true of U).
Now note
Let E, F, G, H be the midpoints of the arcs
AB, BC, CD, DA. Let 𝐼𝐴 , 𝐼𝐵 , 𝐼𝐶 , 𝐼𝐷 be the 𝑛−1 𝑛−1
2
incenters of the triangles BCD, CDA, DAB, ∑ 𝑘 𝑥𝑘 = ∑ 𝑥𝑘 + (𝑘 − 1)(𝑘 + 1)𝑥𝑘
ABC, respectively. Let 𝐴𝐵 , 𝐴𝐶 , 𝐴𝐷 be the 𝑘=1 𝑘=1
140
Challenging Mathematical Problems
2𝑛(2𝑛 − 2) − (𝑛2 − 𝑛) = 3𝑛2 − 3𝑛, with 3, but 427 ≡ 1(mod 27), so this fails. Hence p
equality for 𝑥1 = 𝑛 − 1, 𝑥2 = ⋯ = 𝑥𝑛−2 = < q.
0, 𝑥𝑛−1 = 1.
Since p and q are odd primes, q ≥ p +2, so
61. Let n, r be positive integers and A a set of (3𝑝−1)
< 3. Since this quantity is an integer,
(𝑞−1)
lattice points in the plane, such that any
and it is clearly greater than 1, it must be 2.
open disc of radius r contains a point of A.
That is, 2q = 3p +1. On the other hand, p -1
Show that for any coloring of the points of (9𝑝+1)
A using n colors, there exist four points of divides 3q -1= 2
as well as (9p +1) –(9p
the same color which are the vertices of a -9) = 10. Hence p = 11, q = 17.
rectangle.
63. Let n ≥ 3 be an integer and p ≥ 2n -3 a
Sol.: Consider a square of side length L = prime. Let M be a set of n points in the
4n𝑟 2 with side parallel to the coordinate axes. plane, no three collinear, and let f: M ⟶{0,
One can draw (2𝑛𝑟 2 ) = 4𝑛2 𝑟 2 disjoint disks 1, …, p -1} be a function such that:
of radius r inside the square, hence such a (i) Only one point of M maps to 0,
square contains at least 4𝑛2 𝑟 2 points of A. and
(ii) If A, B, C are distinct points in
The lattice point in A lie on L -1 = 4n𝑟 2 − 1 M and k is the circumcircle of
vertical lines; by the pigeonhole principle, the triangle ABC, then
some vertical line contains n +1 points of A.
Again by the pigeonhole principle, two of ∑ 𝒇(𝑷) ≡ 𝟎(𝒎𝒐𝒅 𝑷).
these points are colored in the same color. 𝑷 ∈𝑴∩𝒌
Now consider an infinite horizontal strip Show that all of the points of M lie on a
made of ribbons of side length L; some two of circle.
them have two points in the same position in
the same color, and these four points from the Solution: Let X be the point mapping to 0. We
vertices of a rectangle. first show that if every circle through X and
two points of M contains a third point of M,
62. Find all prime numbers p, q for which the then all of the points of M lie on a circle.
congruence 𝜶𝟑𝒑𝒒 ≡ 𝜶(𝒎𝒐𝒅 𝟑𝒑𝒒) holds Indeed, consider an inversion with center at
for all integers 𝛼. X. Then the image of M – {X} has the property
that the line through any two of its points
Sol.: Without loss of generality assume p ≤ q; contains a third point; it is a standard result
the unique solution will be (11, 17), for which that this means the points are collinear.
one many check the congruence using the (Otherwise, find a triangle ABC minimizing
Chinese Remainder Theorem. the length of the altitude AH; there is another
point N on BC, but then either ABN OR CAN
We first have 23𝑝𝑞 ≡ 2(𝑚𝑜𝑑 3), which means
has a shorter altitude than AH, contradiction).
p and q are odd. In addition, if 𝛼 is a primitive
root mod p, then 𝛼 3𝑝𝑞−1 ≡ 1 (𝑚𝑜𝑑 𝑝) implies Now suppose the points of M do not lie on a
that p -1 divides 3pq -1 as well as 3pq -1- circle. By the above, there exists a circle
3q(p -1) = 3q -1, and conversely that q -1 passing through M and only two points A, B of
divides 3p -1. If p = q, we now deduce p = q = M. Let f(A) = i, so that by the hypothesis, f(B)
141
Challenging Mathematical Problems
142
Challenging Mathematical Problems
66. Let n ≥ 3 be an integer and X ⊆ {1, 2, …, Sol.: There are more numbers that not of this
𝒏𝟑 } a set of 3𝒏𝟐 elements. Prove that one form. Let n = 𝑘 2 + 𝑚3 , where k, m, n ∈ N and
can find the distinct numbers 𝒂𝟏 , … , 𝒂𝟗 in n ≤ 1000000. Clearly k ≤ 1000 and m ≤ 100.
X such that the system Therefore there cannot be more numbers in
𝒂𝟏 𝒙 + 𝒂𝟐 𝒚 + 𝒂𝟑 𝒛 = 𝟎 the desired from than the 1000000 pairs (k,
𝒂𝟒 𝒙 + 𝒂𝟓 𝒚 + 𝒂𝟔 𝒛 = 𝟎 m).
𝒂𝟕 𝒙 + 𝒂𝟖 𝒚 + 𝒂𝟗 𝒛 = 𝟎
Has a solution (𝒙𝟎 , 𝒚𝟎 , 𝒛𝟎 ) in nonzero 68. Let x, y, p, n, k be natural numbers such
integers. that
𝒙𝒏 + 𝒚𝒏 = 𝒑𝒌 .
Sol.: Label the elements of X in increasing
order 𝑥1 < ⋯ < 𝑎3𝑛2 , and put Prove that if n > 1 is odd, and p is an odd
prime, then n is a power of p.
𝑋1 = {𝑥1 , … , 𝑥𝑛2 }, 𝑋2 = {𝑥𝑛2 +1 , … . , 𝑥2𝑛2 }, 𝑋3
= {𝑥𝑛2 +1 , … , 𝑥3𝑛2 }, Sol.: Let m = gcd(x, y). Then x = m𝑥1 , 𝑦 =
𝑚𝑦1 and by virtue of the given equation,
Define the function f: 𝑋1 × 𝑋2 × 𝑋3 → 𝑋 × 𝑋 𝑚𝑛 (𝑥1 𝑛 + 𝑦1 𝑛 ) = 𝑝𝑘 , and so m = 𝑝𝛼 for some
as follows: f(a, b, c) = (b –a, c –b). nonnegative integer 𝛼. It follows that 𝑥1 𝑛 +
𝛼
𝑦1 𝑛 = 𝑝𝑘−𝑛 . (1)
The domain of f contains 𝑛6 elements. The
range of f, on the other hand, is contained in Since n is odd,
the subset of X × 𝑋 of pairs whose sum is at
most 𝑛3 , a set of cardinality. 𝑥1 𝑛 + 𝑦1 𝑛
= 𝑥1 𝑛−1 − 𝑥1 𝑛−2 𝑦1
𝑥1 + 𝑦1
𝑛3 −1 + 𝑥1 𝑛−3 𝑦1 2 − ⋯
𝑛3 (𝑛3 − 1) 𝑛6
∑ 𝑘= < . − 𝑥1 𝑦1 𝑛−2 + 𝑦1 𝑛−1 ,
2 2
𝑘=1
Let A denote the right side of the equation. By
By the pigeonhole principle, some three the condition p > 2, it follows that at least one
triples (𝑎𝑖 , 𝑏𝑖 , 𝑐𝑖 ) (i = 1, 2, 3) map to the same of 𝑥1 , 𝑦1 is greater than 1, so since n > 1. A >
pair, in which case x =𝑏1 − 𝑐1 , 𝑦 = 𝑐1 − 1.
𝑎1 , 𝑧 = 𝑎1 − 𝑏1 is a solution in nonzero
𝛼
integers. Note that 𝑎𝑖 , cannot equal 𝑏𝑗 since 𝑋1 From (1) it follows that A(𝑥1 + 𝑦1 ) = 𝑝𝑘−𝑛 ,
and 𝑋2 and so on, and that 𝑎1 = 𝑎2 implies so since 𝑥1 + 𝑦1 > 1, A >1, both of these
that the triple (𝑎1 , 𝑏1 , 𝑐1) and (𝑎2 , 𝑏2 , 𝑐2 ) are numbers are divisible by p, moreover, 𝑥1 +
identical, a contradiction. Hence the nine 𝑦1 = 𝑝𝛽 for some natural number 𝛽.
numbers chosen are indeed distinct.
Thus
67. Which are there more of among the
natural numbers from 1 to 1000000, 𝐴 = 𝑥1 𝑛−1 − 𝑥1 𝑛−2 (𝑝𝛽 − 𝑥1 ) + ⋯
inclusive: numbers that can be − 𝑥1 (𝑝𝛽 − 𝑥1 )𝑛−2
represented as the sum of a perfect square + (𝑝𝛽 − 𝑥1 )𝑛−1
and a (positive) perfect cube, = 𝑛𝑥1 𝑛−1 + 𝐵𝑝.
or numbers that cannot be?
143
Challenging Mathematical Problems
= 9⏟
1…9…⏟
10 … 01 ⏟
10 … 01 … ⏟
10 … 01
Sol.: Suppose any two committees have at
9 8 8 8
most three common members. Have two
deputies count the possible ways to choose a The second and third factors are composed of
chairman for each of three sessions of the 9 units, so the sum of their digits is divisible
Duma. The first deputy assumes that any by 9, that is, each is a multiple of 9. Hence
deputy can chair any session, and so gets 1081𝑛 − 1 is divisible by 93 = 729, as is
16003 possible choices. The second deputy 1081𝑛 − 1 for any n.
makes the additional restriction that all of the
71. Two piles of coins lie on a table. It is
chairmen belong to a single committee. Each
known that the sum of the weights of the
of the 16000 committees yields 803 choices,
coins in the two piles are equal, and for
but this is an over count; each of the 16000
(16000−1) any natural number k, not exceeding the
pairs of committees give at most 33
2 number of coins in either pile, the sum of
overlapping choices. Since the first deputy the weights of the k heaviest coins in the
counts no fewer possibilities than the second, first pile is not more than that of the
we have the inequality second pile. Show that for any natural
number x, if each coin (in either pile) of
16000.15999 3
16003 ≥ 16000. 803 − 3 . weight not less than x is replaced by a coin
2
of weight x, the first pile will not be lighter
However, than the second.
144
Challenging Mathematical Problems
𝑥𝑠+1 + ⋯ + 𝑥𝑛 ≥ 𝑥𝑡 + 𝑦𝑡+1 + ⋯ + 𝑦𝑚 . Since L’s. Number the rows 1,…, 5 and the columns
𝑥1 + ⋯ + 𝑥𝑛 = 𝑦1 + ⋯ + 𝑦𝑚 = 𝐴, this 1, …, 7, and consider the 12 squares lying at
inequality can be equivalently written 𝑥𝑠 + the intersections of odd numbered rows with
(𝐴 − 𝑥1 − ⋯ − 𝑥𝑚 ) ≥ 𝑥𝑡 + (𝐴 − 𝑦1 − ⋯ − 𝑦𝑡 ), odd numbered columns. Each of these cells is
which in turn can be rewritten covered by k L’s, so at least 12k L’s must be
used in total. But these cover 3. 12k > 35k
𝑥1 + ⋯ + 𝑥𝑠 + 𝑥(𝑡 − 𝑠) ≤ 𝑦1 + ⋯ + 𝑦𝑡 , cells in total, a contradiction.
This is what we will prove, 73. Points E and F are given are given on side
If t ≥ s, then BC of convex quadrilateral ABCD (with E
closer than F to B). It is known that ∠BAE
𝑥1 + ⋯ + 𝑥𝑠 + 𝑥(𝑡 − 𝑠) = ∠CDF and ∠EAF = ∠FDE. Prove that
= (𝑥1 + ⋯ + 𝑥𝑠 ) ∠FAC = ∠EDB.
+⏟ (𝑥 + ⋯ + 𝑥)
𝑡−𝑠 Solution: By the equality of angles EAF and
≤ (𝑦1 + ⋯ + 𝑦𝑠 ) FDE, the quadrilateral AEFD is cyclic.
+ (𝑦𝑠+1 + ⋯ + 𝑦𝑡 ), Therefore ∠AEF + ∠FDA = 180°. By the
equality of angles BAE and CDF we have
Since 𝑥1 + ⋯ + 𝑥𝑠 ≤ 𝑦1 + ⋯ + 𝑦𝑠 (from the
given condition) and 𝑦𝑠+1 ≥ ⋯ ≥ 𝑦𝑡 ≥ 𝑥. ∠ADC + ∠ABC = ∠FDA +∠CDF +∠AEF -∠BAE
= 180°
If t < s, then 𝑥1 + … + 𝑥𝑠 + 𝑥(𝑡 − 𝑠) ≤ 𝑦1 +
⋯ + 𝑦𝑡 is equivalent to Hence the quadrilateral ABCD is cyclic, so
∠BAC = ∠BDC. It follows that ∠FAC = ∠EBD.
𝑥1 + ⋯ + 𝑥𝑠 ≤ 𝑦1 + ⋯ + 𝑦𝑡 + (𝑥
⏟ + ⋯ + 𝑥)
𝑡−𝑠 74. Find all natural numbers n, such that there
exist relatively prime integers x and y and
The latter inequality follows from the fact
an integer k >1 satisfying the equation
that
𝟑𝒏 = 𝒙𝒌 + 𝒚𝒌 .
𝑥1 + ⋯ + 𝑥𝑠 ≤ 𝑦1 + ⋯ + 𝑦𝑠
= (𝑦1 + ⋯ + 𝑦𝑡 )
Sol.: The only solution is n = 2.
+ (𝑦𝑡+1 + ⋯ + 𝑦𝑠 )𝑎𝑛𝑑 𝑦𝑠 ≤ ⋯
≤ 𝑦𝑡+1 ≤ 𝑥. Let 3𝑛 = 𝑥 𝑘 + 𝑦 𝑘 , where x, y are relatively
prime integers with x > y, k > 1, and n a
natural number. Clearly neither x nor y is a
72. Can a 𝟓 × 𝟕 checkerboard be covered by multiple of 3. Therefore, if k is even𝑥 𝑘 𝑎𝑛𝑑 𝑦 𝑘
L’s (figures formed from a 𝟐 × 𝟐 square by are congruent to 1 mod 3, so their sum is
removing one of its four 𝟏 × 𝟏 corners), congruent to 2 mod 3, and so is not a power
not crossing its borders, in several layers of 3.
so that each square of the board is covered
by the same number of L’s? If k is odd and k > 1, then 3𝑛 = (𝑥 +
𝑦)(𝑥 𝑘−1 − ⋯ + 𝑦 𝑘−1 ). Thus x + y = 3𝑚 for
Sol.: No such covering exists. Suppose we are some m ≥ 1. We will show that n ≥ 2m. Since
given a covering of a 5 × 7 checkerboard with 3
𝑘
(see the solution to Russia 3), by putting
L’s such that every cell is covered by exactly k
145
Challenging Mathematical Problems
𝑥 2 − 𝑥𝑦 + 𝑦 2 − 𝑥 − 𝑦 = (𝑥 − 𝑦)2 +
(𝑥 − 1)(𝑦 − 1) − 1 ≥ 0, 76. At the vertices of a cube are written eight
pair wise district natural numbers, and on
Since (𝑥 − 𝑦)2 ≥ 1. each of its edges is written the greatest
common divisor of the numbers at the end
points of the edge. Can the sum of the
75. Show that if the integers 𝒂𝟏 , … , 𝒂𝒎 are numbers written at the vertices be the
nonzero and for each k = 0, 1, …, m(n < m same as the sum of the numbers written at
-1), the edges?
𝒂𝟏 + 𝒂𝟐 𝟐𝒌 + 𝒂𝟑 𝟑𝒌 + ⋯ + 𝒂𝒎 𝒎𝒌 = 𝟎,
Sol.: This is not possible. Note that if a and b
Then the sequences 𝒂𝟏 , … , 𝒂𝒎 contains at least are natural numbers with a > b, then gcd(a,
𝑎
n + 1 pairs of consecutive terms having b) ≤ b and gcd(a, b) ≤2 .It follows that if a ≠ b,
opposite sings. (𝑎+𝑏)
then gcd(a, b)≤ 3
. Adding 12 such
Solution: We many assume 𝑎𝑚 > 0, since inequalities, corresponding to the 12 edges,
otherwise we may multiply each of the we find that the desired condition is only
(𝑎+𝑏)
numbers by -1. Consider the sequence possible if gcd(a, b) = in each case. But
3
𝑏1 , … , 𝑏𝑚 , where 𝑏𝑖 = ∑𝑛𝑗=0 𝑐𝑗 𝑖 𝑗 for an in this case the larger of a and b is twice the
smaller; suppose a = 2b. Consider the
146
Challenging Mathematical Problems
numbers c and d assigned to the vertices of 78. Can the number obtained by written the
the other end points of the other two edges numbers from 1 to n in order (n >1) be
coming out of the vertex labeled a. Each of the same when read left-to-right and right-
these is either half of or twice a. If at least one to left?
is less a, it equals b; otherwise, both are equal.
Either option contradicts the assumption that Sol.: This is not possible. Suppose N = 123
the numbers are distinct. …321 is an m digit symmetric number,
formed by writing the numbers from 1 to n in
77. Three sergeants and several solders serve succession. Clearly m >18. Also let A and B be
in a platoon. The sergeants take turns on the numbers formed from the first and last k
duty. The commander has given the digits, respectively, of N, where k = ⌊𝑚/2⌋⎿ .
following orders: Then if 10𝑝 is the largest power of 10 dividing
(a) Each day, at least one task must be A, then n> 2. 10𝑝+1 , that is, n has at most p
issued to a soldier. +2 digits. Moreover, A and B must contain the
(b) No soldier may have more than fragments
two tasks or receive more than one
tasks in a single day. 99
⏟ …9⏟
100 … 01 𝑎𝑛𝑑 ⏟
100 … 0 ⏟
199 … 9
𝑝 𝑝 𝑝 𝑝
(c) The lists of soldiers receiving tasks
for two different days must not be Respectively, which is impossible.
the same.
(d) The first sergent violating any of 79. Do there exist three natural numbers
these orders will be jailed. greater than 1, such that the square of
each, minus one, is divisible by each of the
Can at least one of the sergeants, others?
without conspiring with the others,
Sol.: Such integers do not exist. Suppose a ≥ b
give tasks according to these rules and
≥ c satisfy the desired condition. Since 𝑎2 −
avoid being jailed?
1is divisible by b, the numbers a and b are
Sol.: The sergeants who goes third can avoid relatively prime. Hence the number 𝑐 2 − 1,
going to jail. We call a sequence of duties by which is divisible by a and b, must be a
the first, second and third sergeants in multiple of ab, so in particular 𝑐 2 − 1 ≥
succession a round. To avoid going to jail, the 𝑎𝑏. 𝐵𝑢𝑡 𝑎 ≥ 𝑐 𝑎𝑛𝑑 𝑏 ≥ 𝑐, 𝑠𝑜 𝑎𝑏 ≥ 𝑐 2 , a
third sergeant on the last day of each round contradiction.
gives tasks to precisely those soldiers who
80. In isosceles triangle ABC (AB = BC) one
received one task over the previous two days.
draws the angle bisector CD. The
(Such soldiers exist by the third condition).
perpendicular to CD through the center of
With this strategy, at the end of each cycle
the circumcircle of ABC intersects BC at E.
each soldier will have received either two
The parallel to CD through E meets AB at
tasks or none, and the number of the latter
F. Show that BE = FD.
will have decreased. It will end up, at some
point, that all of the soldiers have received Solution: We use directed angles modulo 𝜋.
two tasks, and the first sergeant will go to jail. Let O be the circumcircle of ABC, and K the
intersection of BO and CD. From the equality
of the acute angles BOE and DCA having
147
Challenging Mathematical Problems
2 2 2
𝑏𝑘−1 2 𝑏𝑘−2 3(𝑏 2 − 𝑐 2 )
𝑘𝑚 ≤ 𝑥1 + ⋯ + 𝑥𝑘 = −2 <
𝑏𝑘 2 𝑏𝑘
√2 (𝑏 − 𝑐)2 + 2𝑏 2 − 𝑐 2 + √2 (𝑏 − 𝑐)2 + 2𝑐 2 − 𝑏 2
𝑀2 𝑀
≤ 2+2 .
𝑚 𝑚 Since 𝑎2 > (𝑏 − 𝑐)2 by the triangle inequality.
However,
148
Challenging Mathematical Problems
2 (𝑏 − 𝑐)2 + 2𝑏 2 − 𝑐 2 = (2𝑏 − 𝑐)2 , so we more other agents, but no two agents are
have both watching each other. Moreover, any
10 agents can be ordered so that the first a
3(𝑏2 −𝑐 2 )
3(𝑏 − 𝑐) < . watching the second, the second is
2𝑏−𝑐+│2𝑐−𝑏│
watching the third, etc. , and the last is
If b ≤2c then the two sides are equal, a watching the first. Show that any 11 agents
contradiction. If b ≤ 2c we get 9(𝑏 − 𝑐)2 < can also be so ordered.
3(𝑏 2 − 𝑐 2 ); upon dividing off 3(b –c) and
Sol.: We say two agents are partners if
rearranging, we get 2b < 4c, again a
neither watches the other. First note that
contradiction. Thus we cannot have b > c or
each agent watches at least 7 others; if an
similarly b < c, so b = c.
agent were watching 6 or fewer others, we
84. Find all real solutions of the equation could take away 6 agents and leave a group of
10 which could not be arranged in a circle.
√𝒙𝟐 − 𝒑 + 𝟐√𝒙𝟐 − 𝟏 = 𝒙 Similarly, each agent is watched by at least 7
others. Hence each agent is allied with at
For each real value of p. most one other.
Sol.: Squaring both sides, we get Given a group of 11 agents, there must be one
agent x who is not allied with any of the
𝑥 2 = 5𝑥 2 − 4 − 𝑝 + 4√(𝑥 2 − 𝑝)(𝑥 2 − 1
others in the group (since allies come in
Isolating the radical and squaring again, we pairs). Remove that agent and arrange the
get other 10 in a circle. The Removed agent
watches at least one of the other 10 and is
16(𝑥 2 − 𝑝)(𝑥 2 − 1) = (4𝑥 2 − 𝑝 − 4)2 , watched by at least one. Thus there exists a
pair, u, v of agents with u watching v, u
Which reduces to (16 − 8𝑝)𝑥 2 = 𝑝2 − 8𝑝 + watching x and x watching v (move around
16. Since x ≥ 0(it is the sum of two square the circle until the direction of the arrow to x
│𝑝−4│
roots), we have x = changes); thus x can be spliced into the loop
√16−8𝑝
between u and v.
If a solution exists. We need only determine 𝒏
when this value actually satisfies. Certainly 86. Let ∏𝟏𝟗𝟗𝟔 𝟑 𝒌𝟏
𝒏=𝟏 (𝟏 + 𝒏𝒙 ) = 𝟏 + 𝒂𝟏 𝒙 +
149
Challenging Mathematical Problems
87. In a parallelogram ABCD with ∠A <𝟗𝟎°, triangles of total base less than 1 and height
the circle with diameter AC meets the lines at most h, as is the area contained in the
CB and CD again at E and F, respectively, triangle but not the rectangles. Hence the sum
and the tangent to this circle at A meets BD 1 ℎ
differs from 2 but at most 2, as desired.
at P. Show that P, F, E are collinear.
89. In a convex quadrilateral ABCD, triangles
Sol.: Without loss of generality, suppose B, D,
ABC and ADC have the same area. Let E be
P occur in that order along BD. Let G and H be
the intersection of AC and BD, and let the
the second intersection of AD and AB with the
parallels through E to the lines AD, DC, CB,
circle. By Menelaos’s theorem, it suffices to
BA meet AB, BC, CD, DA at K, L, M, N,
show that
respectively. Compute the ratio of the
𝐶𝐸. 𝐵𝑃. 𝐷𝐹 areas of the quadrilaterals KLMN and
=1 ABCD.
𝐸𝐵. 𝑃𝐷. 𝐹𝐶
Find note that Solution: The triangles EKL and DAC are
homothetic, so the ratio of their areas equals
𝐵𝑃 𝐴𝐷 sin ∠𝐵𝐴𝑃 sin ∠𝐴𝑃𝐷 sin ∠𝐵𝐴𝑃
= sin ∠𝐴𝑃𝐵 sin ∠𝐷𝐴𝑃 = sin ∠𝐷𝐴𝑃 𝐸𝐾 𝐸𝐿 𝐵𝐸 2 1
𝐴𝐵 𝐷𝑃 (𝐴𝐷) (𝐶𝐷) = (𝐵𝐷) = 4, since B and D are
Since AP is tangent to the circle, ∠BAP = equidistant from the line AC. Similarly the
1
∠HAP ratio of the areas of EMN and BCA is 4, so the
1
= 𝜋 - ∠HCA = 𝜋 -∠FCA; similarly, ∠DAP = union of the triangles EKL and EMN has area 4
∠GCA = ∠EAC. We conclude that of ABCD.
𝐵𝑃 𝐴𝐷
=
sin ∠𝐹𝐴𝐶
=
𝐹𝐶 As for triangle EKN, its base KN is parallel to
𝐴𝐵 𝐷𝑃 sin ∠𝐸𝐴𝐶 𝐸𝐶
BD and half as long, so its area is one-fourth
𝐷𝐹 𝐷𝐴 that of ABD. Similarly EML has area one-
Finally we note that = because the right
𝐵𝐸 𝐴𝐵 fourth that of BCD, and so the union of the
triangles AFD and AED have the same angles
two triangles EKN and EML has area one
at B and D and are thus similar. This prove
fourth that of ABCD, and so the quadrilateral
the claim.
KLMN has area one-half that of ABCD.
88. Given real number s 0 = 𝒙𝟏 < 𝒙𝟐 < ⋯ <
90. Find the maximum number of pair wise
𝒙𝟐𝒏 < 𝒙𝟐𝒏+𝟏 = 𝟏 with 𝒙𝒊+𝟏 − 𝒙𝒊 ≤ 𝒉 for 1
disjoint sets of the from 𝑺𝒂,𝒃 =
𝟏−𝒉
≤ i ≤ 2n, show that 𝟐
< {𝒏𝟐 + 𝒂𝒏 + 𝒃: 𝒏 ∈ 𝒁}𝒘𝒊𝒕𝒉 𝒂, 𝒃 ∈ 𝒁.
𝟏+𝒉
∑𝒏𝒊=𝟏 𝒙𝟐𝒊 (𝒙𝟐𝒊+𝟏 − 𝒙𝟐𝒊−𝟏 ) < .
𝟐 Solution: Only two such sets are possible, for
Sol.: The different between the middle example, with (a, b) = (0, 0) and (0, 2) (since
1 2 is not a difference of squares). There is no
quantity and is the difference between the
2 loss of generality in assuming a ∈ {0, 1} by a
sum of the areas of the rectangles bounded by suitable shift of n, and the sets generated by
the lines x = 𝑥2𝑖−1 , 𝑥 = 𝑥2𝑖+1 , 𝑦 = 0, 𝑦 = 𝑥2𝑖 (0, a) and (1, b) have the common value
and the triangle bounded by the lines y = 0, x (𝑎 − 𝑏)2 + 𝑎 = (𝑎 − 𝑏)2 + (𝑎 − 𝑏) + 𝑏.Thus
= 1, x = y. The area contained in the we have a = 0 or a = 1 universally.
rectangles but not the triangle is a union of
150
Challenging Mathematical Problems
151
Challenging Mathematical Problems
Sol.: We have q(n) > q(n+1) if and only if n If x = 0, we have m = 1, yielding (x, y, z) = (0,
+1 is a perfect square. Indeed, if n +1 = 𝑚2 , 1, 2). Otherwise, 3𝑚 − 1 is divisible by 4, so m
𝑥 𝑚 𝑚
then is even and 22+1 = (3 2 + 1) (3 2 − 1). The
𝑚2 − 1 𝑚2 two factors on the right are powers of 2
𝑞(𝑛) = ⎣ ⎦ = 𝑚 + 1, 𝑞(𝑛 + 1) = ⎣ ⎦ differing by 2, so they are 2 and 4, giving x =
𝑚−1 𝑚
=𝑚 4 and (x, y, z) = (4, 2, 5).
On the other hand, for n = 𝑚2 + 𝑑 with 0 ≤ 97. The sides a, b, c and u, v, w of two triangles
𝑚2 +𝑑 𝑑 ABC and UVW are related by the
d≤ 2m, 𝑞(𝑛) = ⎣ 𝑚 ⎦ =𝑚+ ⎣𝑚⎦
equations.
Which is non-decreasing.
𝒖(𝒗 + 𝒘 − 𝒖) = 𝒂𝟐 ,
95. Let a, b, c be positive real numbers.
𝒗(𝒘 + 𝒖 − 𝒗) = 𝒃𝟐 ,
(a) Prove that 𝟒(𝒂𝟑 + 𝒃𝟑 ) ≥ (𝒂 + 𝒃)𝟑
(b) Prove that 𝟗(𝒂𝟑 + 𝒃𝟑 + 𝒄𝟑 ) ≥ 𝒘(𝒖 + 𝒗 − 𝒘) = 𝒄𝟐 .
(𝒂 + 𝒃 + 𝒄)𝟑
Prove that ABC is acute, and express the angles
Sol.: Both parts follow from the Power Mean U, V, W in terms of A, B, C.
inequality: for r > 1 and 𝑥1 , … , 𝑥𝑛 positive,
Sol.: Note that 𝑎2 + 𝑏 2 − 𝑐 2 = 𝑤 2 − 𝑢2 −
𝑥1 𝑟 + ⋯ + 𝑥𝑛 𝑟
1/𝑟
𝑥1 + ⋯ + 𝑥𝑛 𝑣 2 + 2𝑢𝑣 = (𝑤 + 𝑢 − 𝑣)(𝑤 − 𝑢 + 𝑣) > 0 by
( ) ≥ , the triangle inequality, so cos 𝐶 > 0. By this
𝑛 𝑛
reasoning, all of the angles of triangle ABC are
Which in turn follows from Jensen’s acute. Moreover,
inequality applied to convex function 𝑥 𝑟 .
𝑎2 + 𝑏 2 − 𝑐 2
cos 𝐶 =
96. Find all Solutions in non-negative integers 2𝑎𝑏
x, y, z of the equation.
𝟐𝒙 + 𝟑𝒚 = 𝒛𝟐 (𝑤 + 𝑢 − 𝑣)(𝑤 − 𝑢 + 𝑣)
=√
4𝑢𝑣
Sol.: If y = 0, then 2𝑥 = 𝑧 2 − 1 = (𝑧 + 1)(𝑧 −
1), so z +1 and z -1 are powers of 2. The only
powers of 2 which differ by 2 are 4 and 2, so 𝑤 2 − 𝑢2 − 𝑣 2 + 2𝑢𝑣 1
=√ = √1 − cos 𝑈
(x, y, z) = (3, 0, 3). 4𝑢𝑣 √2
152
Challenging Mathematical Problems
Sol.: It suffices to show that ∠𝐵2 𝐵1 𝑂1 = 2 sin 2 sin 1 + 2(2 sin 4 . sin 1) + … + 89
𝜋
∠𝐵1 𝐵2 𝑂2 = 2 , where 𝑂1 𝑎𝑛𝑑 𝑂2 are the (2 sin 178 . sin 1) = 90 cos 1. (3)
centers of 𝑆1 𝑎𝑛𝑑 𝑆2 , respectively. By power –
of-a-point. P𝐴1 . 𝑃𝐵1 = 𝑃 𝐾 2 = 𝑃𝐴2 . 𝑃𝐵2 , so Using the identity 2 sin 𝑎 . sin 𝑏 = cos(𝑎 − 𝑏) −
triangles P𝐴1 𝐴2 𝑎𝑛𝑑 𝑃𝐵2 𝐵1 are similar. cos(𝑎 + 𝑏), we find
1
Therefore ∠P𝐵1 𝐵2 = ∠𝑃𝐴2 𝐴1 = 2 ∠𝑃𝑂𝐴1 , 2 sin 2 . sin 1 + 2(2 sin 4 . sin 1) + ⋯ + 89
where O is the center of S.
(2 sin 178 . sin 1)
Now note that the homothety at 𝐴1 carrying
𝑆1 to S takes 𝑂1 𝑡𝑜 𝑂 𝑎𝑛𝑑 𝐵1 𝑡𝑜 𝑃, so ∠𝑃𝑂𝐴1 = = (cos 1 − cos 3) + 2(cos 3 − cos 5) + ⋯ +
∠𝐵1 𝑂1 𝐴1. From this we deduce ∠P𝐵1 𝐵2 = 89(cos 177 − cos 179)
∠𝐵1 𝑂1 𝑁, where N is the midpoint of 𝐴1 𝐵1 .
= cos 1 + cos 3 + cos 5 + ⋯ + cos 175
Finally, ∠𝐵2 𝐵1 𝑂1 = 𝜋 − ∠P𝐵1 𝐵2 − ∠𝑂1 𝐵1 𝑁 =
𝜋
2
, as desired. cos 177 − 89 cos 179
99. Find all solutions in positive real numbers = cos 1 + (cos 3 + cos 177) + ⋯
a, b, c, d to the following system of + (cos 89 + cos 91)
equations: − 89 cos 179
a + b+ c + d = 12
= cos 1 + 89 cos 1 = 90 cos 1,
abcd = 27 +ab +ac +ad +bc +bd +cd.
So (1) is true.
Sol.: The first equation implies abcd = ≤ 81 by
the arithmetic geometric mean inequality, Note: An alternate solution involves complex
with equality holding for a = b = c = d = 3. numbers. One expresses sin n as
Again by AM-GM,
𝜋𝑖𝑛 −𝜋𝑖𝑛
1/2 (𝑒 180 −𝑒 180 )
abcd ≥ 27 +6 (𝑎𝑏𝑐𝑑) 𝑎𝑛𝑑 𝑢𝑠𝑒𝑠 𝑡ℎ𝑒 𝑓𝑎𝑐𝑡 𝑡ℎ𝑎𝑡
(2𝑖)
However, 𝑥 2 − 6𝑥 − 27 ≥ 0 𝑓𝑜𝑟 𝑥 ≤
𝑥 + 2𝑥 2 + ⋯ + 𝑛𝑥 𝑛 = (𝑥 + ⋯ + 𝑥 𝑛 ) +
−3 𝑜𝑟 𝑥 ≥ 9, so (𝑎𝑏𝑐𝑑)1/2 ≥ 9, hence abcd ≥
(𝑥 2 + ⋯ + 𝑥 𝑛 ) + ⋯ + 𝑥 𝑛
81. We conclude abcd = 81, and hence a = b=
c= d= 3. 1
= 𝑥−1 [(𝑥 𝑛+1 − 𝑥) + (𝑥 𝑛+1 − 𝑥 2 ) + ⋯ +
100. Prove that the average of numbers n (𝑥 𝑛−1 − 𝑥 𝑛 )]
𝐬𝐢𝐧 𝒏° (𝒏 = 𝟐, 𝟒, 𝟔, … , 𝟏𝟖𝟎)𝒊𝒔 𝐜𝐨𝐭 𝟏°.
𝑛𝑥 𝑛+1 𝑥 𝑛+1 −𝑥
= − .
𝑥−1 (𝑥−1)2
Solution: All arguments of trigonometric
functions will be in degrees. We need to prove 101. For any nonempty set S of real
numbers, let 𝜎(S) denote the sum of the
153
Challenging Mathematical Problems
elements of S. Given a set A of n positive triangle ABC and the interior of its
integers, consider the collection of all 𝟐
reflection A’B’C’ in l has area more than
𝟑
distinct sums 𝜎(S) as S ranges over the
the area of triangle ABC.
nonempty subsets of A. Prove that this
collection of sums can be partitioned into n Solution :
classes so that in each classes, the ratio of
the largest sum of the smallest sum does In all of the solutions, a, b, c denote the lengths
not exceed 2. of the sides BC, CA, AB, respectively, and we
assume without loss of generality that a ≤ b ≤
Sol.: Let A = {𝑎1 , 𝑎2 , … , 𝑎𝑛 } where 𝑎1 < 𝑎2 < c.
⋯ < 𝑎𝑛 . For i = 1, 2, …, n let 𝑠𝑖 = 𝑎1 + 𝑎2 +
⋯ + 𝑎𝑖 𝑎𝑛𝑑 𝑡𝑎𝑘𝑒 𝑠0 = 0. All the sums is Choose l to be the angle bisector of ∠A. Let P be
question are less than or equal to 𝑠𝑛 , and if 𝜎 is the intersection of l with BC. Since AC ≤ AB, the
one of them, we have intersection of triangles ABC and A’B’C’ is the
disjoint union of two congruent triangles. APC
𝑠𝑖−1 < 𝜎 < 𝑠𝑖 (1) and APC’. Considering BC as a base, triangles
APC and ABC have equal altitudes, so their
For an appropriate i. Divide the sums into n
areas are in the same are in the same ratio as
classes by letting 𝐶𝑖 denote the class of sums
their bases:
satisfying (1). We claim that these classes have
the desired property. To establish this, it 𝐴𝑟𝑒𝑎 (𝐴𝑃𝐶) 𝑃𝐶
= .
suffices to show that (1) implies. 𝐴𝑟𝑒𝑎 (𝐴𝐵𝐶) 𝐵𝐶
154
Challenging Mathematical Problems
that contain no four consecutive terms All angles will be in degrees. Let x = ∠PCB. Then
equal to 0, 0, 1, 1 or 1, 1, 0, 0 in that order. ∠PBC = 80 –x. By the Law of Sines,
Prove that 𝒃𝒏+𝟏 = 𝟐𝒂𝒏 for all positive
𝑃𝐴 𝑃𝐵 𝑃𝐶 sin ∠𝑃𝐵𝐴 sin ∠𝑃𝐶𝐵 sin ∠𝑃𝐴𝐶
integers n. 1 = 𝑃𝐵 𝑃𝐶 𝑃𝐴 = sin ∠𝑃𝐴𝐵 sin ∠𝑃𝐵𝐶 sin ∠𝑃𝐶𝐴
Sol.: We refer to the binary sequences counted sin 20 sin 𝑥 sin 40 4 sin 𝑥 sin 40𝑐𝑜𝑠 10
= sin 10 sin(80−𝑥) sin 30 = sin(80−𝑥)
.
by (𝑎𝑛 ) and (𝑏𝑛 ) as “type A” and “type B”,
respectively. For each binary sequence The identity 2 sin 𝑎 . cos 𝑏 = sin(𝑎 − 𝑏) +
(𝑥1 , 𝑥2 , … , 𝑥𝑛 ) there is a corresponding binary sin(𝑎 + 𝑏)now yields
sequence (𝑦0 , 𝑦1 , … , 𝑦𝑛 ) obtained by setting
2 sin 𝑥(sin 30+sin 50) sin 𝑥(1+2 cos 40)
𝑦0 = 0 𝑎𝑛𝑑 𝑦1 = 𝑥1 + 𝑥2 + ⋯ + 𝑥𝑖 mod 2, i = 1= = ,
sin(80−𝑥) sin(80−𝑥)
1, 2, …, n. (2)
So,
(Addition mod 2 is defined as follows: 0 +0 = 1
+1 = 0 and 0 + 1 = 1 +0 = 1.) Then 2 sin 𝑥 cos 40 = sin(80 − 𝑥) − sin 𝑥 =
2 sin(40 − 𝑥) cos 40.
𝑥𝑖 = 𝑦𝑖 + 𝑦𝑖−1 𝑚𝑜𝑑 2, 𝑖 = 1, 2, … , 𝑛,
This gives x = 40 –x and thus x = 20. It
And it is easily seen that (1) provides a one-to – follows that ∠ACB = 50 = ∠BAC, so triangle
one correspondence between the set of all ABC is isosceles.
binary sequences of length n and the set of
binary sequences of length n +1 in which the 105. Solve the system of equations:
𝟏
first term is 0. Moreover, the binary sequence √𝟑𝒙 (𝟏 + 𝒙+𝒚) = 𝟐
(𝑥1 , 𝑥2 , … , 𝑥𝑛 ) has three consecutive terms 𝟏
√𝟕𝒚 (𝟏 − 𝒙+𝒚) = 𝟒√𝟐
equal 0, 1, 0 in that order if and only if the
corresponding sequence (𝑦0 , 𝑦1 , … , 𝑦𝑛 ) has four
Sol.: Let u =√𝑥, 𝑦 = √𝑦, so the system
consecutive terms equal to 0, 0, 1, 1 or 1, 1, 0, 0
in that order, so the first is of type A if and only becomes
if the second is of type B. The set of Type B 𝑢 2
sequences of length n +1 in which the first term 𝑢+ =
𝑢2 + 𝑣 2 √3
is 0 is exactly half the total number of such
sequences, as can be seen by means of the 𝑣 4√2
𝑣− = .
mapping in which 0’s and 1’s are interchanged. 𝑢2 +𝑣 2
√7
155
Challenging Mathematical Problems
=
1
+
2√2
𝑖 ±(
2
+ 3, 4 occur in a block. Thus the number of
√21)
√3 √7 √21
permutations of the desired from is n! -24 (n -
From which we deduce 2)! + 24(n -3)!
156
Challenging Mathematical Problems
157
Challenging Mathematical Problems
Finally, for those who have only seen Schur’s this the right inequality follows. For the left, we
inequality in three variables, note that in need only show (𝑚 + 𝑥)(𝑚 + 1 − 𝑥) ≥ 2𝑥 for
general any inequality involving 𝑠1 , … , 𝑠𝑘 which x ≤n; this rearranges to (𝑚 − 𝑥)(𝑚 + 1 + 𝑥) ≥
holds for n ≥ k variables also holds for n +1 0, which holds because 𝑚 ≥ 𝑛 ≥ 𝑥.
variables, by replacing the variables 𝑥1 , … , 𝑥𝑛+1
by the roots of the derivative of the polynomial 113. Let 𝑷𝟏 , 𝑷𝟐 , 𝑷𝟑 , 𝑷𝟒 be four points on a
(𝑥 − 𝑥1 ) … (𝑥 − 𝑥𝑛−1 ). circle, and let 𝑰𝟏 be the incenter of the
triangle 𝑷𝟐 𝑷𝟑 𝑷𝟒 , 𝑰𝟐 be the incenter of the
111. Let ABCD be a quadrilateral with AB = triangle 𝑷𝟏 𝑷𝟑 𝑷𝟒 , 𝑰𝟑 be the incenter of the
BC = CD = DA. Let MN and PQ be two triangle 𝑷𝟏 𝑷𝟐 𝑷𝟒 𝒂𝒏𝒅 𝑰𝟒 be the incenter of
segments perpendicular to the diagonal BD the triangle P1P2P3. Prove that I1, I2, I3 and
and such that the distance between them I4 are the vertices of a rectangle.
𝑩𝑫
is d > , with M ∈ AD, N ∈ DC, P ∈AB, and
𝟐 Sol.: Without loss of generally, assume
Q ∈ BC. Show that the perimeter of the 𝑃1 , 𝑃2 , 𝑃3 , 𝑃4 occur on the circle in the order. Let
hexagon AMNCQP does not depend on the 𝑀12 , 𝑀23 , 𝑀34 , 𝑀41 be the midpoints of arcs
positions of MN and PQ so long as the 𝑃1 𝑃2 , 𝑃2 𝑃3 , 𝑃3 𝑃4 , 𝑃4 𝑃1 , respectively.
distance between them remains constant.
Then the line 𝑃3 𝑀1 is the angle bisector of
Solution: The lengths of AM, MN, NC are all ∠𝑃2 𝑃3 𝑃1 and so passes through 𝐼4 . Moreover,
linear in the distance between the segments the triangle 𝑀12 , 𝑃2 𝐼4 is isosceles because
MN and AC; if this distance is h, extrapolating ∠𝐼4 𝑀12 𝑃2 = ∠𝑃1 𝑃2 𝑃3
from the extremes MN = AC and M = N = D
gives that = 𝜋 − 2∠𝑃1 𝑃2 𝐼4 − 2∠𝑀12 𝑃2 𝑃1
2𝐴𝐵 − 𝐴𝐶 = 𝜋 − 2∠𝑀12 𝑃2 𝐼4
𝐴𝑀 + 𝑀𝑁 + 𝑁𝐶 = 𝐴𝐶 +
𝐵𝐷/2
Hence the circle centered at M passing through
In particular, if the segments MN and PQ 𝑃1 and 𝑃2 also passes through 𝐼4 , 𝑎𝑛𝑑 likewise
maintain constant total distance from AC, as through 𝐼3 .
they do if their distance remains constant, the
From this we determine that the angle bisector
total perimeter of the hexagon is constant.
of ∠𝑃3 𝑀12 𝑃4 is the perpendicular bisector of
112. Let m and n be positive integers such 𝐼3 𝐼4. On the other hand, this angle bisector
that n ≤ m. Prove that passes through 𝑀34 , so it is simply the line
(𝒎 + 𝒏)! 𝑀12 𝑀34; by symmetry, it is also the
𝟐𝒏 𝒏! ≤ ≤ (𝒎𝟐 + 𝒎)𝒏 . perpendicular bisector of 𝐼1 𝐼2. We conclude
(𝒎 − 𝒏)!
that 𝐼1 𝐼2 𝐼3 𝐼4 is a parallelogram.
Sol.: The quantity in the middle is (𝑚 + 𝑛)(𝑚 +
𝑛 − 1) … (𝑚 − 𝑛 + 1). If we pair off terms of To show that 𝐼1 𝐼2 𝐼3 𝐼4 is actually a rectangle, it
the form (𝑚 + 𝑥)𝑎𝑛𝑑 (𝑚 + 1 − 𝑥), we get now suffices to show that 𝑀12 𝑀34 ⊥ 𝑀23 𝑀41.
products which do not exceed 𝑚(𝑚 + 1), since To see this, simply note that the angle between
the function 𝑓(𝑥) = (𝑚 + 𝑥)(𝑚 + 1 − 𝑥) is a these lines is half the sum of measure of the
1
concave parabola with maximum at 𝑥 = 2. From arcs 𝑀12 𝑀23 𝑎𝑛𝑑 𝑀34 𝑀41, but these arcs
clearly comprise half of the circle.
158
Challenging Mathematical Problems
159
Challenging Mathematical Problems
Sol.: The conditions imply that A and D are 120. A convex polyhedron P and a sphere S
opposite vertices of a square APDQ such that B, are situated in space such that S intercepts
C, E, F lie on AP, PD, DQ, QA, respectively and on each edge AB of P a segment XY with
that all six sides of the hexagon are tangent to 𝟏
AX = XY = YB = 𝟑 𝑨𝑩. Prove that there
the inscribed circle of the square. The diagonals
exists a sphere T tangent to all edges of P.
BE and CF meet at the center O of the square.
Let T, U, V be the feet of perpendiculars from O Sol.: Let AB and BC be two edges of the
to AB, BC, CD; then ∠TOB = ∠BOU by reflection polyhedron, so that the sphere meets AB in a
across OB, and similarly ∠UOC = ∠COV. segment XY with AX = XY = YB and meets BC in a
𝜋
Therefore 2 = 2∠𝐵𝑂𝐶, proving the claim. segment ZW with BZ = ZW = WC. In the plane
ABC the points X, Y, Z, W lie on the cross-section
118. The polynomials 𝑷𝒏 (𝒙) are defined by of the sphere, which is a circle. Therefore BY. BX
𝑷𝟎 (𝒙) = 𝟎, 𝑷𝟏 (𝒙) = 𝒙 and 𝑷𝒏 (𝒙) = = BZ. BW by power –of-a point; this clearly
𝒙𝑷𝒏−𝟏 (𝒙) + (𝟏 − 𝒙)𝑷𝒏−𝟐 (𝒙) 𝒏 ≥ 𝟐. implies AB = BC, and so the center of S is
For every natural number n ≥ 1, find equidistant from AB and BC. We conclude that
all real numbers x satisfying the any two edges of P are equidistant from S and
equation 𝑷𝒏 (𝒙) = 𝟎. so there is a sphere concentric with S tangent to
all edges.
Sol.: One shows by induction that
𝑥
𝑃𝑛 (𝑥) = [(𝑥 − 1)𝑛 − 1]
𝑥−2
121. Natural numbers k, n are given such
Hence 𝑃𝑛 (𝑥) = 0 if and only if x = 0 or 𝑥 = 1 + that 1 < k < n. Solve the system of n
𝑒 2𝜋𝑖𝑘/𝑛 for some k ∈ {1, …, n -1}. equations.
𝒙𝒊 𝟑 (𝒙𝒊 𝟐 + ⋯ + 𝒙𝒊+𝒌−𝟏 𝟐 ) = 𝒙𝒊−𝟏 𝟐 𝟏 ≤
𝒊≤𝒏
in n real unknowns 𝒙𝟏 , … , 𝒙𝒏 . (Note:
119. The real numbers x, y, z, t satisfy the
𝒙𝟎 = 𝒙𝒏 , 𝒙𝟏 = 𝒙𝒏+𝟏 , 𝒆𝒕𝒄.)
equalities 𝒙 + 𝒚 + 𝒛 + 𝒕 = 𝟎 𝒂𝒏𝒅 𝒙𝟐 +
𝒚𝟐 + 𝒛𝟐 + 𝒕𝟐 = 𝟏. Prove that Sol.: The only solution is 𝑥1 = ⋯ 𝑥𝑛 = 𝑘 −1/3 .
−𝟏 ≤ 𝒙𝒚 + 𝒚𝒛 + 𝒛𝒕 + 𝒕𝒙 ≤ 𝟎. Let L and M be the smallest and largest of the
𝑥𝑖 , respectively. If M = 𝑥𝑖 , then
Sol.: The inner expression is (𝑥 + 𝑧)(𝑦 + 𝑡) =
−(𝑥 + 𝑧)2 , so the second inequality is obvious. 𝑘𝑀3 𝐿2 ≤ 𝑥𝑖 3 (𝑥𝑖 2 + ⋯ + 𝑥𝑖+𝑘−1 2 ) = 𝑥𝑖−1 2
As for the ≤ 𝑀2
first, note that 1
And so M ≤ (𝑘𝐿2 ). Similarly, if L = 𝑥𝑗 , then
1
1 = (𝑥 2 + 𝑧 2 ) + (𝑦 2 + 𝑡 2 ) ≥ 2 [(𝑥 + 𝑧)2 +
𝑘𝐿3 𝑀3 ≥ 𝑥1 3 (𝑥𝑖 2 + ⋯ + 𝑥𝑖−𝑘+1 2 ) = 𝑥𝑖−1 2
(𝑦 + 𝑡)2 ] ≥ [(𝑥 + 𝑧)(𝑦 + 𝑡)] ≥ 𝐿2
160
Challenging Mathematical Problems
If k +1 is not prime, it has a prime divisor Assuming no two bricks are parallel, the 90
greater than 3, but this prime divides k! and not smallest bricks have total volume 891. The 7
k! +48. Hence k +1 is prime, and by Wilson’s other bricks each have volume at least 18,
theorem k! +1 is a multiple of k +1. Since k! +48 giving a total volume of at least 1017, a
is as well, we find k +1 = 47, and we need only contradiction.
46!
check that is not a power of 47. We check
48+1 We have not determined the optimal constant
46!
that = 29(𝑚𝑜𝑑 53) (by cancelling as many (one can improve the above bound to 96
48+1
terms as possible in 46! Before multiplying), but easily), but we note that an arrangement with
that 47 has order 13 modulo 53 and that none 73 nonparallel bricks is possible.
of its powers is congruent to 29 modulo 53.
161
Challenging Mathematical Problems
Hence 𝑅 2 − 𝑂𝐺 2 =
(𝑎 2 +𝑏2 +𝑐 2 )
. On the other doublings, and 2𝑛 − 1 can be obtained in one
9
more step.
hand, by the standard area formula K = 𝑟𝑠 =
𝑎𝑏𝑐 𝑎𝑏𝑐
4𝑅
, we have 2𝑟𝑅 = (𝑎+𝑏+𝑐). We now note that Hence we can obtain 22 − 1, 24 − 1, … , 2256 −
1 𝑖𝑛 (1 + 1) + (2 + 1) + ⋯ + (128 + 1) =
(𝑎2 + 𝑏 2 + 𝑐 2 )(𝑎 + 𝑏 + 𝑐) ≥ 9𝑎𝑏𝑐 263 steps. In 243 steps, we turn 2256 −
1 𝑖𝑛𝑡𝑜 2499 − 2243 . Now notice that the
By two applications of the AM-GM inequality,
numbers 2243 − 2115 , 2115 − 251 , 251 −
so 2𝑟𝑅 ≤ 𝑅 2 − 𝑂𝐺 2 , proving the claim.
219 , 219 − 23 , 23 − 21 , 21 − 1 have all be
125. Let ABCDE be a convex pentagon, and written down; in 6 steps, we now obtain 2499 −
let M, N, P, Q, R be the midpoints of sides 1. We make this into 2998 − 1 in 500 steps, and
AB, BC, CD, DE, EA, respectively. If the make 21996 − 1 in 999 steps. Adding 1 for the
segments AP, BQ, CR, DM have a common initial 1, we count
point, show that this point also lies on EN.
1 + 263 + 243 + 6 + 500 + 999 = 2012
Sol.: Let T be the common point, which we take
Numbers written down, as desired.
as the origin of a vector system. Then 𝐴 × 𝑃 =
0, or equivalently 𝐴 × (𝐶 + 𝐷) = 0, which we 127. Let ℤ+ denote the set of nonzero
may write 𝐴 × 𝐶 = 𝐷 × 𝐴. integers. Show that an integer p > 3 is
prime if an only if for any a, b ∈ ℤ+ , exactly
Similarly, we have 𝐵 × 𝐷 = 𝐸 × 𝐵, 𝐶 × 𝐸 =
one of the numbers
𝐴 × 𝐶, 𝐷 × 𝐴 = 𝐵 × 𝐷. Putting these equalities
𝒑−𝟏
together gives 𝐸 × 𝐵 = 𝐶 × 𝐸, 𝑜𝑟 𝐸 × 𝑵𝟏 = 𝒂 + 𝒃 − 𝟔𝒂𝒃 + ,
𝟔
(𝐵 + 𝐶) = 0, which means the line EN also 𝒑+𝟏
𝑵𝟐 = 𝒂 + 𝒃 + 𝟔𝒂𝒃 +
passes through the origin T. 𝟔
belongs to ℤ+ .
Sol.: We state the problem a bit differently: we 𝑝 = (6𝑐 + 1)(6𝑑 + 1)𝑜𝑟(6𝑐 − 1)(6𝑑
want to write down at most 2012 numbers, − 1)𝑜𝑟(6𝑐 + 1)(6𝑑 − 1).
starting with 1 and ending with 21996 − 1, such
In the first case, 𝑁2 is not an integer and 𝑁1 = 0
that every number written is the sum of two
for 𝑎 = −𝑐, 𝑏 = −𝑑.
numbers previously written. If 2𝑛 − 1 has been
written, then 2𝑛 (2𝑛−1 ) can be obtained by n
162
Challenging Mathematical Problems
In the second case, 𝑁2 is not an integer and 130. Two sets of intervals A, B on a line are
𝑁1 = 0 for 𝑎 = 𝑐, 𝑏 = 𝑑. given. The set A contains 𝟐𝒎 − 𝟏 intervals,
every two of which have a common
In the third case, 𝑁1 is not an integer and 𝑁2 = 0 interior point. Moreover, each interval in A
for 𝑎 = 𝑐, 𝑏 = −𝑑. contains at least disjoint intervals of B.
128. Let M be a nonempty set and *a binary Show that there exists an interval in B
operation on M. That is, to each pair (a, b) which belongs to at least m intervals from
∈𝑴 × 𝑴 one assigns an element a * b. A.
Suppose further that for any a, b ∈M,
Sol.: Let 𝛼1 = [𝑎𝑖 , 𝑏𝑖 ](𝑖 = 1, … , 2𝑚 − 1) be the
(a* b)*b = a and a*(a* b) = b.
intervals, indexed so that 𝑎1 ≤ 𝑎2 ≤ ⋯ ≤
(a) Show that a *b = b *a for all a, b
𝑎2𝑚−1 . Choose k ∈{𝑚, … , 2𝑚 − 1} to minimize
∈M.
𝑏𝑘 . By assumption, the interval 𝛼𝑘 contains two
(b) For which finite sets M does such a
disjoint intervals from B, say 𝛽1 =
binary operation exist?
[𝑐1 , 𝑑1 ] 𝑎𝑛𝑑 𝛽2 = [𝑐2 , 𝑑2 ]. Without loss of
Sol.: generality, assume
(a) First note that [a* (a* b)] *(a* b) = a by 𝑎𝑘 ≤ 𝑐1 < 𝑑1 < 𝑐2 < 𝑑2 ≤ 𝑏𝑘 .
the first rule. By the second rule, we
If 𝑑1 ≤ 𝑏𝑖 for i = 1, 2, …, m, then 𝛽1 ⊂ 𝛼1 for i =
may rewrite the left side as b* (a *b), so
1, 2, …, m, so 𝛽1 satisfies the desired property.
b* (a *b) = a and so b *a = b*[b* (a* b)].
Otherwise, 𝑑1 > 𝑏𝑥 for some s ∈ {1, 2, …, m}. By
b* a = b*[b*(a* b)]. By the second rule
assumption, 𝑐2 > 𝑑1 > 𝑏8 . Since no two of the
this equals a *b, so a* b = b* a.
𝛼 are disjoint, we have 𝑏8 ≥ 𝑎𝑖 for all i, so 𝑐2 >
(b) Such sets exist for all finite sets M.
𝑎𝑖 . On the other hand, by the choice of 𝑘, 𝑏𝑘 <
Identify M with {1, …, n} and define
𝑏1 for i = m, …, 2𝑚1 . Therefore 𝑎𝑖 < 𝑐2 < 𝑑2 ≤
a* b = c ⟺a + b+ c = 0 (mod n).
𝑏𝑘 ≤ 𝑏𝑖 for each 𝑖 ∈ {𝑚, 𝑚 + 1, … , 2𝑚 −
It is immediate that the axioms are
1}, 𝑎𝑛𝑑 𝑠𝑜 𝛽2 has the desired property.
satisfied.
131. The points E and D lie in the interior of
129. Determine whether there exist a sides AC and BC respectively, of a triangle
function f: ℤ⟶ℤ such that for each k = 0, 1, ABC. Let F be the intersection of the lines
…, 1996 and for each m ∈ℤ the equation AD and BE. Show that the area of triangles
𝒇(𝒙) + 𝒃𝒙 = 𝒎 has at least one solution x ABC and ABF satisfies.
∈ℤ.
𝑺𝑨𝑩𝑪 │𝑨𝑪│ │𝑩𝑪│
= + −𝟏
Sol.: Each integer y can written uniquely as 𝑺𝑨𝑩𝑭 │𝑩𝑬│ │𝑩𝑫│
1997m +k with m ∈ℤ and k ∈ {0, …, 1996}.
Sol.: Let the line parallel to BC through F meet
Define the function f by 𝑓(𝑦) = 𝑚 − 𝑘𝑦; then
AB at K and AC at N, Let the line parallel to CA
𝑓(𝑥) + 𝑘𝑥 = 𝑚 has the solution 𝑥 = 1997𝑚 +
through F meet BC at ME and AB at P; let the
𝑘, so the condition satisfied.
line parallel to AB through F meet BC at L and
CA at O. Let 𝑣𝐶 𝑎𝑛𝑑 𝑣𝐹 be the distances of C
and F, respectively, to the line AB. Then
163
Challenging Mathematical Problems
164
Challenging Mathematical Problems
of a side of one of the short indeed be 50, 51, …, 99. Clearly we maximize
triangles. the number of chosen numbers by taking them
(ii) No counter may retrace a two apart, and the list has odd length, so taking
segment already painted red, 99, 98 …, 50 is the only draw a graph with {1, …,
through it can stop on a red 99} as vertices, where two numbers are
vertex, even if another counter adjacent if they sum to 99 or 100.
is already there.
Show that for all integers n >0 136. Let M be the intersection of the
it is possible to paint all of the diagonals of the trapezoid ABCD. A point P
segments red in this fashion. such that ∠APM = ∠DPM is chosen on the
base BC. Prove that the distance from C to
Sol.: The cases n = 1, 2, are trivial; we use them the line AP is equal to the distance from B
as the base cases for an inductive proof. We to the line DP.
describe the moves for A, understanding that
the moves for B and C are the same moves Sol.: Since M lies on the internal angle bisector
2𝜋 4𝜋 of angle ∠APD, it lies at the same distance from
rotated by 𝑎𝑛𝑑 , respectively. To fix
3 3 the lines AP and DP. The ratio of this distance to
directions, imagine the triangle is oriented with 𝐴𝑀
the distance from C to AP is 𝐴𝐶
, while the ratio
one side parallel to the horizontal and the third
vertex above it, and suppose A starts at the of this distance to the distance from B to DP is
𝐵𝑀 𝐴𝑀 𝐵𝑀
bottom left. We first move A right for n –1 𝑀𝐷
. 𝐵𝑢𝑡 𝑀𝐶 = 𝑀𝐷 by similar triangles, so the
steps. We then alternate moving it up to the left latter two distances are indeed the same.
and down to the left for a total of 2n –5 steps.
137. In a group of several people, some are
We then trace a path through the inner triangle acquainted with each other and some are
of side n –2 using the induction hypothesis, not. Every evening, one person invites all
ending at another corner. Finally, we follow the of his acquaintances to a party and
unused edges from that corner, ending three introduces them to each other. Suppose
steps later. that after each person has arranged at
least one party, some two people are still
135. Fifty numbers are chosen from the set unacquainted. Prove that they will not be
{1, …, 99}, no two which sum to 99 or 100.
introduced at the next party.
Prove that the chosen numbers must be
50, 51, …, 99. Sol.: We claim that two people unacquainted
after each person has held at least one party lie
Sol.: In the sequence in different connected components of the
99, 1, 98, 2, 97, 3,… 51, 49, 50, any two adjacent original (and final) graph of acquaintance. If two
numbers sum to 99 or 100, so both cannot people are connected by a path of length n,
occur. Grouping the numbers into 49 pairs plus they will be connected by a path of length n -1
one extra, we see at most 50 numbers can after one person along the path (including
occur, and 50 must be one of them. Since we either of the two people at the ends) holds a
must step at least two terms along the list to party, by a path of length n -2 after two of them
make the next choice, the numbers must hold a party, and so on. After each person holds
165
Challenging Mathematical Problems
a party, the two people on the ends will be 𝑠𝑛 = (3 + 5)𝑠𝑛−1 − 3.5. 𝑠𝑛−2
acquainted.
So 𝑠𝑛−1 must also divide 3.5. 𝑠𝑛−2 . If n >1, then
138. There are n parking spaces along a one 𝑠𝑛−1 is coprime to 3 and 5, so 𝑠𝑛−1 must divide
–way road down which n drivers are 𝑠𝑛−2, which is impossible since 𝑠𝑛−1 > 𝑠𝑛−2 .
travelling. Each driver goes to his favorite
140. Let M be the midpoint of side BC of
parking space and parks there if it is free;
triangle ABC, and let 𝒓𝟏 𝒂𝒏𝒅 𝒓𝟐 be the
otherwise, he parks at the nearest free
radii of the incircles of triangles ABM and
place down the road. If there is no free
ACM. Prove that 𝒓𝟏 < 2𝒓𝟐 .
space after his favorite, he drives away.
How many lists 𝒂𝟏 , … , 𝒂𝒏 of favorite Sol.: Recall that the area of a triangle equals its
parking spaces are there which permit all in radius times half its perimeter. Since ABM
of the drivers to park? and ACM have equal area, we have
Sol.: There are (𝑛 + 1)𝑛−1 such lists. To each 𝑟1 𝐴𝐶 + 𝐴𝑀 + 𝐶𝑀
list of preferences (𝑎1 , … , 𝑎𝑛 ) which allows all =
𝑟2 𝐴𝐵 + 𝐴𝑀 + 𝐵𝑀
drivers to park, associate the list (𝑏2 , … , 𝑏𝑛 ),
where 𝑏𝑖 is the difference mod n+ 1 between And it suffices to show AC + AM + CM <2AB +
the numbers of the space driver i wants and the 2AM +2BM;
space the previous driver took. Clearly any two
Since BM = CM, this simplifies to AC < 2AB + AM
lists give rise to different sequences of 𝑏𝑖 .
+CM.
We now argue that any list of 𝑏𝑖 comes from a
In fact, by the triangle inequality, AC < AM +CM,
list of preferences. Imagine that the n parking
so we are done.
spaces are arranged in a circle with an extra
phantom space put in at the end. Put the first 141. Several positive integers are written on
driver in any space, then for i = 2, …, n, put a blackboard. One can erase any two
driver i in the first available space after the distinct integers and write their greatest
space 𝑏𝑖 away from the space taken by driver i- common divisor and least common
1; this gives a list of preferences if and only if multiple instead. Prove that eventually the
the one space not taken at the end is the numbers will stop changing.
phantom space. However, by shifting the
position of the first driver, we can always Sol.: If a, b are erased and c < d are written
ensure that the phantom space is the space not instead, we have c ≤ min (a, b) and d ≥ max (a,
taken. Thus the sequences of 𝑏𝑖 are equal in b); moreover, ab = cd. From this we may
number to the lists of preferences, so there are conclude 𝑎 + 𝑏 ≤ 𝑐 + 𝑑 by writing 𝑎𝑏 + 𝑎2 =
(𝑛 + 1)𝑛−1 of each. 𝑐𝑑 + 𝑎2 ≤ 𝑎𝑐 + 𝑎𝑑 (the latter since (𝑑 −
𝑎)(𝑐 − 𝑎) ≤ 0) and dividing both sides by a.
139. Find all positive integers n such that Thus the sum of the numbers never decreases,
𝟑𝒏−𝟏 + 𝟓𝒏−𝟏 𝒅𝒊𝒗𝒊𝒅𝒆𝒔 𝟑𝒏 + 𝟓𝒏 . and it is obviously bounded (e.g. by n times the
product of the numbers, where n is the number
Sol.: This only occurs for n = 1. Let 𝑠𝑛 = 3𝑛 + 5𝑛
of numbers on the board); hence it eventually
and note that
166
Challenging Mathematical Problems
stops changing, at which time the numbers an even integer, so Sergey can find at least one
never change. more zero.
142. No three diagonals of a convex 1996 – 145. Find all quadruples of polynomials
gon meet in a point. Prove that the 𝑷𝟏 (𝒙), 𝑷𝟐 (𝒙), 𝑷𝟑 (𝒙), 𝑷𝟒 (𝒙) with real
number of triangles lying in the interior of coefficients such that for each quadruple
the 1996-gon and having sides on its of integers x, y, z, t such that 𝒙𝒚 − 𝒛𝒕 = 𝟏,
diagonal is divisible by 11. one has
𝑷𝟏 (𝒙)𝑷𝟐 (𝒚) − 𝑷𝟑 (𝒛)𝑷𝟒 (𝒕𝟎 = 𝟏.
Sol.: There is exactly one such triangle for each
choice of six vertices of the 1996-gon: if A, B, C, Sol.: If 𝑃1 (1) = 0, then 𝑃3 (𝑧)𝑃4 (𝑡) = −1 for
D, E, F are the six vertices in order, the each pair of integers z, t, and so 𝑃3 𝑎𝑛𝑑 𝑃4 are
corresponding triangle is formed by the lines constant functions; moreover, 𝑃1 (𝑥)𝑃2 (𝑦) = 0,
AD, BE, CF. Hence the number of triangle is so one of 𝑃1 𝑎𝑛𝑑 𝑃2 is identically zero. Ignoring
(1996
6
) ; since 1991 is a multiple of 11, so is the such cases, which are easily enumerated, we
number of triangles. assume 𝑃𝑖 (1) ≠ 0 for all i.
143. Prove that for every polynomial 𝒙𝟐 + We first note that 𝑃1 (𝑥)𝑃2 (1) = 𝑃1 (1)𝑃2 (𝑥)
𝒑𝒙 + 𝒒 with integer coefficients, there for all nonzero integers x, so that 𝑃1 𝑎𝑛𝑑 𝑃2 are
exists a polynomial 𝟐𝒙𝟐 + 𝒓𝒙 + 𝒔 with equal up to a scalar factor; similarly, 𝑃3 𝑎𝑛𝑑 𝑃4
integer coefficients such that the sets of are equal up to a scalar factor. Now note that
values of the two polynomials on the 𝑃1 (𝑥)𝑃2 (𝑎𝑦) = 𝑃1 (𝑎𝑥)𝑃2 (𝑦) for all nonzero a,
integers are disjoint. x, y, so that the difference between the two
sides is identically zero as a polynomial in a. In
Sol.: If p is odd, then 𝑥 2 + 𝑝𝑥 + 𝑞 has the same particular, that means no term in 𝑃1 (𝑥)𝑃2 (𝑦)
parity as q for all integers x, and it suffices to has unequal exponent in x and y, and the same
choose r even and s of the opposite parity as q. is true of 𝑃1 (𝑥)𝑃1 (𝑦) on the other hand, if
If p = 2m is even, then 𝑥 2 + 𝑝𝑥 + 𝑞 = 𝑃1 (𝑥) has term of more than one degree, then
(𝑥 + 𝑚)2 + (𝑞 − 𝑚)2 which is congruent to 𝑃1 (𝑥)𝑃1 (𝑦) contains a term with different
𝑞 − 𝑚2 𝑜𝑟 𝑞 − 𝑚2 + 1 modulo 4. Now it degrees in x and y. Hence 𝑃1 (𝑥) = 𝑐𝑥 𝑘 for
suffices to choose r even and s congruent to some integer k and some constant c, and
𝑞 − 𝑚2 + 2 modulo 4. similarly 𝑃2 (𝑥) = 𝑑𝑥 𝑘 , 𝑃3 (𝑥) = 𝑒𝑥 𝑚 , 𝑃4 (𝑥) =
𝑓𝑥 𝑚 .
144. Sergey found 11 different solutions to
𝟗𝟔
the equation 𝒇 (𝟏𝟗𝒙 − ) = 𝟎. Prove that Thus we must determine when 𝑐𝑑𝑥 𝑘 𝑦 𝑘 −
𝒙
if he had tried harder, he could have found 𝑒𝑓𝑧 𝑚 𝑡 𝑚 = 1 whenever 𝑥𝑦 − 𝑧𝑡 = 1 in
at least one more solution. integers. Clearly k = m since otherwise one of
the two terms on the left dominates the other,
96 and cd =1 by setting x = y = 1 and z = t = 0, and
Sol.: The equations 19𝑥 − 𝑥
= 𝑡 can be
rewritten 19𝑥 2 − 𝑡𝑥 − 96 = 0; 𝑠𝑖𝑛𝑐𝑒 𝑡 2 + similarly ef = 1. Now note that (𝑥𝑦)𝑘 − (𝑧𝑡)𝑘 =
19.96 > 0, it always has two real roots. 1 can only happen in general for k = 1, since for
Therefore the number of zeroes of f (if finite) is k >1, there are no consecutive perfect k-th
powers. We conclude 𝑃1 (𝑥) = 𝑐𝑥, 𝑃2 (𝑥) =
167
Challenging Mathematical Problems
𝑥 𝑥
, 𝑃3 (𝑥) = 𝑒𝑥, 𝑃4 (𝑥) = for some nonzero real impossible. Without loss of generality, assume
𝑐 𝑒
numbers c, e. that b –k ≠ 1 is not the final corner. The first
player now marks (a, b). If the second player
146. Two players play the following game covers (a, b) and (a, b+1), the position is again
on a 𝟏𝟎𝟎 × 𝟏𝟎𝟎 board. The first player stable. Otherwise, the first player marks (𝑎 +
marks a free square, then the second 1, 𝑏 − 1) and the second player is forced to
player puts a 𝟏 × 𝟐 domino down covering cover it and (𝑎 + 2, 𝑏 − 1). Then the first player
two free squares, one of which is marked. marks (𝑎 + 2, 𝑏 − 2) and the second player is
This continuous until one player is unable forced to cover it and (𝑎 + 3, 𝑏 − 2), and so on.
to move. The first player wins if the entire After (𝑎 + 𝑘, 𝑏 − 𝑘) is marked, the result is a
board is covered, otherwise the second stable position.
player wins. Which player has a winning
strategy? (Note that the assumption b –k ≠ 1 ensures
that the moves described do not cross the edge
Sol.: The first player has a winning strategy. Let of the board.) To finish the proof, we need to
us say a position is stable if every square below show that such a chain of corners must exist.
or to the right of a free square is free. Then we Write the labels (𝑎1 , 𝑏1 ), … , (𝑎𝑘 , 𝑏𝑘 ) in a row,
claim the first player can always ensure that on and join two adjacent labels by a segment if
his turn, either the position is stable or there is they are of the form (𝑎, 𝑏), (𝑎 + 1, 𝑏 − 1). If
a free square with exactly one free neighbor (or two adjacent labels (𝑎, 𝑏), (𝑎 + 𝑖, 𝑏 − 𝑗) are not
both). joined by a segment, then either i = 1 or j = 1
but not both. If i = 1, draw an arrow between
Let us label the square in the i-th row and j-th
the labels pointing towards (𝑎 + 𝑖, 𝑏 − 𝑗);
column as (i, j), with (1, 1) in the top left. We
otherwise draw the arrow the other way. Also
call a free square a corner if is not below or to
draw arrows pointing to (𝑎1 , 𝑏1 )𝑎𝑛𝑑 (𝑎𝑘 , 𝑏𝑘 ).
the right of another free square. Let
There is now one more chain of corners (joined
(𝑎1 , 𝑏1 ), (𝑎2 , 𝑏2 ), … , (𝑎𝑘 , 𝑏𝑘 ) be the corners
by segments) than arrows, so some chain has
from top to bottom.
two arrows pointing to it. That chain satisfies
First notice that if (a, b) is a corner such that the condition above, so the first player can use
both (𝑎 + 1, 𝑏 − 1)𝑎𝑛𝑑 (𝑎 − 1, 𝑏 + 1) are it to create another stable position.
nonfree (or off the board), then the first player Consequently, the first player can ensure
may mark (a, b), and however the second player victory.
moves, the result will be a stable position. More
147. Let BD be the bisector of angle B in
generally, if (𝑎, 𝑏), (𝑎 + 1, 𝑏 − 1), … , (𝑎 +
triangle ABC. The circumcircle of triangle
𝑘, 𝑏 − 𝑘) are corners and (𝑎 − 1, 𝑏 +
BDC meets AB at E, while the circumcircle
1)𝑎𝑛𝑑 (𝑎 + 𝑘 + 1, 𝑏 − 𝑘 − 1) are both nonfree
of triangle ABD meets BC at F. Prove that
or off the board, the first player can be sure to
AE = CF.
return to a stable position.
Sol.: By power-of-a-point. AE .AB = AD. AC and
To show this, first note that we cannot have 𝐴𝐸 𝐴𝐷 𝐵𝐶
both a = 1 and b –k = 1, or else the number of CF. CB = CD. CA, so 𝐶𝐹 = (𝐶𝐷 ) (𝐴𝐵). However,
non-free squares would be odd, which is
168
Challenging Mathematical Problems
169
Challenging Mathematical Problems
∞
1 1 4 9 Consider that box, now we have four colours
= 1+ 5∑ 𝑖 =1+ 5 = (white, black, red, yellow) and hence there
2 2 −1 2 8
𝑖=0 33
must be at least ( 4 )+1 = 9 balls of the same
Thus the fraction of good assignments is at least
5 8 5 1
colour.
(8) (9) = 9 > 2.
There can be at most 4 different sizes
151. Find all real numbers satisfying available for these 9 balls of the same
𝟔𝒙 + 𝟐𝟐𝒙 + 𝟐𝟒𝒙 − 𝟑𝟔𝒙 − 𝟏𝟔𝒙 = 𝟏. colour, For if there were 5 (or
more)different sizes, then collection of 5
Ans:- Rewrite the given relation as:
balls, all of different sizes, would not satisfy
6𝑥 + 4𝑥 − 36𝑥 + 24𝑥 − 16𝑥 = 1 the given property.
170
Challenging Mathematical Problems
√[𝒙]+√{𝒙}
154. Let x ≥ 𝟏, 𝒇(𝒙) = , 3 2 3
√𝒙 ∴ x= (±√ ) = ±√ ∴ Max (z)=
2 15 10
where [.] denotes G.I.F. and { }
denotes fractional part. Determine the 3 2 5
√ + √15 = .
10 √30
smallest number k ∋ f(x)≤ 𝒌 for each
x≥𝟏 157. For any positive a, b prove that
𝟏 𝟏
Ans:- Let x = a+ b where a= [x], b= {x} (𝒂 + 𝒂)𝟐 + (𝒃 + 𝒃)𝟐 ≥ 𝟖.
√𝑎+√𝑏 Ans: AM ≥ GM
f(x)=
√𝑎+𝑏
1 1
𝑎+𝑏+2√𝑎𝑏 2√𝑎𝑏 (𝑎 + 𝑎)2 + (𝑎 + 𝑎)2 ≥
2
(f(𝑥)) = =1+
𝑎+𝑏 𝑎+𝑏 1 1
2√(𝑎 + 𝑎)2 + (𝑏 + 𝑏)2
Using AM≥ 𝐺𝑀, ≤ 1 + 1 ⇒ 𝑓(𝑥) ≤ √2.
1 𝑎 𝑏
≥ 2(ab+𝑎𝑏 + 𝑏 + 𝑎)
155. Solve the equation (√𝟐 + √𝟐)𝒙 +
(√𝟐 − √𝟐)𝒙 = 𝟐𝒙 1
≥ 2(2+2) [∵ ab + 𝑎𝑏 ≥ 2]
√2 𝜋 𝜋
Ans:- 1+ = 1 + 𝑐𝑜𝑠 4 = 2 cos2 8
2 158. Find the following limit:
𝟏 𝟏 𝟏
2+ √2 𝑥 2− √2 𝑥 𝐥𝐢𝐦 ( + + ⋯+ )
( 4 ) ⁄2 + ( 4 ) ⁄2 𝒙→∞ √𝒏𝟐 +𝟏 √𝒏𝟐 +𝟐 √𝒏𝟐 +𝒏
𝑛
𝜋 𝜋 Ans:- Let 𝑢𝑛 = √𝑛2
= (𝑐𝑜𝑠 8 )𝑥 + (𝑠𝑖𝑛 8 )𝑥 +𝑛
𝑛 1
⇒ x= 2 ∴ lim 𝑢𝑛 = lim = lim = 1.
𝑥→∞ 𝑥→∞ √𝑛2 +𝑛 𝑥→∞ √1+ 1
𝑛
171
Challenging Mathematical Problems
1 1 1 1 5 1 3 6
So, lim (√𝑛2 + √𝑛2 + ⋯ + √𝑛2 )= 1. Ans:- P(1)= 1+22 = 4 < 2 − 2 = 2 = 4.
𝑥→∞ +1 +2 +𝑛
159. For any real number x and for any The statement is true for n= 2.
positive integer n show that
Let , the statement is true for n= m.
𝟏 𝟐 𝒏−𝟏
[x]+[x+𝒏] + [𝒙 + 𝒏] + ⋯ + [𝒙 + ]= 1 1 1 1
𝒏 ∴P(m)= 1+22 + 32 + ⋯ + 𝑚2 < 2 − 𝑚.
[𝒏𝒙]
Now, we need to show that the statement is
Ans:- Let x= [x]+y, where 0 ≤ y < 1,
also true for n=m+1.
Let p be an integer such that P-1 ≤ 𝑛𝑦 < 𝑃 1 1 1 1
P(m+1)= 1+22 + 32 + ⋯ + 𝑚2 < 2 − (𝑚+1)2
𝑘 𝑘
Now, x+𝑛 = [𝑥] + 𝑦 + 𝑛
1 1 1
< 2− + . [∵ <
𝑚 𝑚(𝑚+1) (𝑚+1)2
𝑃+𝑘−1 𝑘 𝑃+𝑘
Also, <𝑦+𝑛< 1
∀ 𝑚 > 1]
𝑛 𝑛
𝑚(𝑚+1)
𝑃−1+𝑘
So, long as < 1 , i.e. , k < n-(P-1) 1
𝑛 < 2-𝑚+1.
𝑘
So, 𝑦 + 𝑛 < 1 and consequently ∴ The statement is true for n= m+1
𝑘
[x+𝑛] = [x]for k= 0, 1, 2, …, n-P. So, for all n ∊ ℕ the statement is true.
𝑘 Hence proved.
But [x+𝑛] = [x]+1 for k= n-P+1, …..n-1.
𝝅
161. In a 𝛥PQR, ⦟R = 𝟐 . If
1 𝑛−1
∴[x]+ [x+𝑛]+…+[x+ ] 𝑷 𝑸
𝑛 𝒕𝒂𝒏 𝟐 𝒂𝒏𝒅 𝒕𝒂𝒏 𝟐 are the roots of
([𝑥] + [𝑥] + ⋯ + [𝑥]) +
=⏟ equation a𝒙𝟐 + 𝒃𝒙 + 𝒄= 0 (a≠0), then
(([𝑥] + 1) + ([𝑥] + 1) + ⋯ + ([𝑥 + 1))] show that a + b = c.
⏟
𝑝 𝑄
Ans:- (a) tan + tan =
= n[x]+(P-1)……………………(1) 2 2
𝑏 𝑃 𝑄 𝑐 𝑃 𝑄 𝜋
− 𝑎 , tan 2 tan 2 = 2 ∴2+2=
Also, [nx]=[n[x]+ny]= n[x]+(P-1) 4
𝑝 𝑄
Since P-1 ≤ ny < P……(2) tan + tan 𝜋
2 2
∴ 𝑝 𝑄 = tan = 1
1 − tan tan 4
2 2
From equation (1) & (2),
𝑏
1 𝑛−1 −
[x]+[x+𝑛]+…+[x+ ] = [nx]. ⟹ 𝑎
𝑐 =1 ⟹𝑏 =𝑐−𝑎 ⟹𝑎+
𝑛 1−
𝑎
172
Challenging Mathematical Problems
162. If A and B are real orthogonal 164. If 0 < u < 1 and 𝒖𝒏+𝟏 = 𝟏 −
matrices of the same order and |B|+|A|= 0. √𝟏 − 𝒖𝒏 ∀ 𝒏 > 1,
Prove that |A+ B|= 0 Prove that (i) {𝒖𝒏 } converges to zero
Ans:- |A|+|B|=0 𝒖𝒏+𝟏 𝟏
(ii) 𝐥𝐢𝐦 =𝟐
𝒏⟶∞ 𝒖𝒏
⇒ |A|= −|B|
Ans:- (i) 0 < 𝑢1 < 1
|A|.|B|=−1 [∵ |B|=|𝐵 −1 |as they are
orthogonal] ⇒ 0 < √1 − 𝑢1 < 1
1
Ans:- Take x= 0, then f(0) – (f(0))2 ≥ 4 < 0 as 0 < √1 − 𝑢𝑛 < 1
1 1
⇒ (f(0))2 + (2)2 − 2. 2 . f(0) ≤ 0 ∴ 𝑢𝑛+1 < 𝑢𝑛 as 0 < √1 − 𝑢𝑛 <1
1 1 𝑢𝑛+1
⇒ f(0) − 2 = 0 ⇒ f(0) = 2 (ii) Let lim 𝑢𝑛 = 𝑙, then lim
𝑛⟶∞ 𝑛⟶∞ 𝑢𝑛
1
Also, taking x=1 we have f(1) - 2 = 0 ∴ ∴ lim
1− √1−𝑙
= lim
𝑙
= 1+
1 1
=2
1 𝑛⟶∞ 𝑙 𝑛⟶∞ 𝑙(1+ √1−𝑙 ) √1−0
f(0) = f(1)= 2
; Since 𝑢𝑛 converges to zero.
∴ This is not one -to –one function.
173
Challenging Mathematical Problems
𝑥−1
Clearly F (x) satisfies the conditions of
Again putting x= Rolle’s Theorem as F (0) = 0 and
2
𝑥−1 𝑥−7 𝑎
g( )= g ( ) and so on F (1) =∑𝑛𝑘=𝑜 𝑘+1
𝑘
=0
2 2
Generally we have, g(
𝑥−1
)= g (
𝑥−(2𝑛 −1)
) Hence ∃ a ‘c’ ∊ (0, 1) ∋ F′(c) =0 ⇒ F (c) =0
2 2𝑛
𝟏 𝟏 𝟏 𝟏
𝑥 1 168. Show that 1+ 𝟐 + 𝟑 + 𝟒 + ⋯ . + can
∴ g (x)= g (2𝑛 − 1 + 2𝑛 ) 𝒏
never be an integer value.
∴ lim 𝑔 (𝑥) = 𝑔 (−1) 1 1 1
𝑛⟶∞ Ans:- We are to show:- 1+ 2 + 3 + 4 +
1 𝑎𝑛 𝑜𝑑𝑑 𝑛𝑢𝑚𝑏𝑒𝑟
⇒ g (x)= g (-1)= constant ∀ x. ⋯.+ = 𝑎𝑛 𝑒𝑣𝑒𝑛 𝑛𝑢𝑚𝑏𝑒𝑟 ∀ 𝑛 > 1
𝑛
Ans:- f (x)= 𝑥 3 − 3𝑥 2 + 2𝑥 + 1 1
When n=2, LHS= 1+ 2 = 2 = 𝑎𝑛 𝑒𝑣𝑒𝑛 𝑛𝑢𝑚𝑏𝑒𝑟
3 𝑎𝑛 𝑜𝑑𝑑 𝑛𝑢𝑚𝑏𝑒𝑟
𝑘 1
∴ f(x) is concave. Now, P (m+1) = + 𝑚+1
𝑟
174
Challenging Mathematical Problems
=0 1
∴ (cos 𝛼1 . cos 𝛼2 … . cos 𝛼𝑛 )2 ≤ 2𝑛
The last equality holds since f(x) = f(𝑥𝑛 ) =0
holds ∀ 𝑛 ∊ 𝑁. 1 1
∴cos 𝛼1 . cos 𝛼2 … . cos 𝛼𝑛 ≤ √2𝑛 ≤ 𝑛
22
170. The four digit number aabb is a [∵cos 𝛼𝑖 ≥ 0]
square. Find the number.
172. Let f(x, y) = 0 is a circle such that f(0,
2
Ans:- aabb = 𝑛 𝜆) = 0 and f(𝜆,0) = 0 have equal roots and
f(1,1)= - 2 then the radius of the circle is
Then 𝑛2 = 1100a + 11b
(a) 4 (b) 8
= 11(100a + b)
(c) 2 (d) 1
175
Challenging Mathematical Problems
176
Challenging Mathematical Problems
2
From (1) and (2), then minimum value of k is 100(100 + 1) 100.101
⟹{ } = 𝑘2 ⟹ 𝑘 =
5 such that P(n) is divisible by 9. 2 2
= 5050
177
Challenging Mathematical Problems
𝟏𝟑 𝟏𝟑 +𝟐𝟑 1 2𝑛
6. Sum till n terms of the series 𝟏
+ 𝟏+𝟑
+ ∴ 𝑃(𝑛) = 2 (1 − )=
𝑛+1 𝑛+1
𝟏𝟑 +𝟐𝟑 +𝟑𝟑
𝟏+𝟑+𝟓
= 𝝅 𝐬𝐢𝐧 𝟐𝒏𝒙
8. For all n∊ N, ∫𝟎 𝒅𝒙 =
𝐬𝐢𝐧 𝒙
𝟒𝒏𝟐 +𝟑 𝒏(𝒏+𝟏)𝟑 𝒏(𝟐𝒏𝟐 +𝟗𝒏+𝟏𝟑)
(a) 𝟓
, (b) 𝟔
, (c) 𝟐𝟒
, (d) 𝝅
(a) – 𝝅, (b) 0, (c) 𝟐 (d) none
none
𝜋 sin 2𝑛𝑥
13 13 +23 13 +23 +33 Sol.: Let 𝑃(𝑛) = ∫0 𝑑𝑥
Sol.: Let 𝑃(𝑛) = 1
+ 1+3
+ 1+3+5
+ sin 𝑥
6} 𝑃(𝑚+1) − 𝑃(𝑚)
𝜋
1 sin[2(𝑘 + 1)𝑥] − sin 2𝑘𝑥
∴ 𝑃(𝑛) : 24 𝑛(2𝑛2 + 9𝑛 + 13) = ∫ 𝑑𝑥
0 sin 𝑥
𝟏 𝟏 𝟏
7. 𝟏
+ 𝟏+𝟐 + 𝟏+𝟐+𝟑 + ⋯ up to(n+1) terms is 𝑃(𝑚+1) + 𝑃(𝑚)
𝜋
equal to 2 cos[(2𝑘 + 1)𝑥] − sin 𝑥
= ∫ 𝑑𝑥
0 sin 𝑥
𝟐𝒏 𝒏 𝟐𝒏
(a) 𝒏+𝟏
, (b) 𝒏+𝟏, (c) 𝒏+𝟐 (d) none
sin(2𝑘 + 1) 𝑥 𝜋
1 1 1
⟹ 𝑃(𝑚+1) − 𝑃(𝑚) = [ ]
Sol.: Let 𝑃(𝑛) : + + + ⋯ (𝑛 + 2𝑘 + 1 0
1 1+2 1+2+3
1)𝑡𝑒𝑟𝑚𝑠 2
⟹ 𝑃(𝑚+1) − 𝑃(𝑚) = (0 − 0) = 0
1 2 2𝑘 + 1
𝑡𝑛 = = ⟹ 𝑡𝑛
1 + 2 + 3 + ⋯ + 𝑛 𝑛(𝑛 + 1) ∴ 𝑃(𝑚+1) = 𝑃(𝑚)
1 1
= 2[ − ]
𝑛 𝑛+1 Since the proposition is true for n = m+1,
thus it was also true for n = m. Hence the
1 1 1
⟹ 𝑃(𝑛) : {(1 − ) + ( − ) + ⋯ proposition is true ∀ n ∊ N.
2 2 3
1 1
+( − )}
𝑛 𝑛+1
178
Challenging Mathematical Problems
1
(SUBJECTIVE TYPE) (∵ 𝑦 = tan−1 𝑥) =
𝑠𝑒𝑐 2 𝑦
= 𝑐𝑜𝑠 2 𝑦 … … … … … (2)
1) Given 𝒂𝒏+𝟏 = 𝟑𝒂𝒏 − 𝟐𝒂𝒏−𝟏 𝒂𝒏𝒅 𝒂𝟎 =
𝟐, 𝒂𝟏 = 𝟑, show that 𝒂𝒏 = 𝟐𝒏 + 𝟏 ∀ n∊ N 𝜋 𝜋
= sin ( − 𝑦) sin ( − 𝑦)
2 2
Sol.: Step I. For n = 1 𝑎1 = 21 + 1 = 3 = (−1)1−1 (1 − 1)!
𝑛=2 𝜋 𝜋
sin ( − 𝑦) sin ( + 𝑦) = 𝑅. 𝐻. 𝑆 𝑜𝑓 (1)
2 2
179
Challenging Mathematical Problems
𝑑𝑘 𝑦 𝜋 𝜋
𝑃(𝑘) : 𝑑𝑥 𝑘
= (−1)𝑘−1 (𝑘 − 1)! sin {𝑘 2 − = (−1)𝑘−1 𝑘! 𝑠𝑖𝑛𝑘+1 ( 2 + 𝑦) sin {(𝑘 −
𝜋 𝜋
𝑦} 𝑠𝑖𝑛𝑘 { 2 + 𝑦} 1) − (𝑘 + 1)𝑦}
2
𝑑 𝑘+1 𝑦 𝑑 𝑑𝑘 𝑦 𝑑 𝜋
𝑑𝑥 𝑘+1
= 𝑑𝑥 [𝑑𝑥 𝑘 ] = 𝑑𝑥 {(−1)𝑘−1 (𝑘 − = (−1)𝑘−1 𝑘! 𝑠𝑖𝑛𝑘+1 ( 2 + 𝑦) sin {(𝑘 +
𝜋 𝜋 𝜋
1)! sin {𝑘 − 𝑦} 𝑠𝑖𝑛𝑘 { + 𝑦}} 1) ( 2 − 𝑦)}
2 2
𝜋 Step I: For n = 1, 𝐴1 = 𝑎1 , 𝐵1 = 𝑏1 , 𝐶1 = 𝑐1
= (−1)𝑘−1 (𝑘 − 1)! 𝑠𝑖𝑛𝑘−1 ( 2 +
𝜋
𝑦) [sin {𝑘 ( 2 − 𝑦)} cos ( 2 + 𝑦) −
𝜋 𝑆𝑖𝑛𝑐𝑒, 𝑎1 2 = 𝑏1 2 +
𝜋 𝜋 𝜋 𝑐1 2 𝑡ℎ𝑒𝑟𝑒𝑓𝑜𝑟𝑒, 𝐴1 2 ≥ 𝐵1 2 + 𝐶1 2
sin ( 2 + 𝑦) cos {𝑘 ( 2 − 𝑦)}] 𝑠𝑖𝑛2 ( 2 + 𝑦) So the result is true for n = 1
180
Challenging Mathematical Problems
181
Challenging Mathematical Problems
This show that the result is true for all ⟹ 2√𝑎 < 1 + √(4𝑎 + 1) ⟹ 4𝑎 <
non-negative integers.
1 + 4𝑎 + 1 + 2√(4𝑎 + 1)
Again if p is negative integer 𝑖. 𝑒. 𝑝 = −𝑚. ⟹ 2√(4𝑎 + 1) + 2 > 0 which is true
Where m is positive integer. Then we can 𝑇ℎ𝑒𝑟𝑒𝑓𝑜𝑟𝑒, 𝑃(1) 𝑖𝑠 𝑡𝑟𝑢𝑒
write Step II: Assume it is true for n = k,
then
𝑓(0) = 𝑓(𝑝+𝑚) ⟹ 𝑓(𝑝) 𝑓(𝑚) =
1 (𝑏𝑦 𝑎𝑠𝑠𝑢𝑚𝑝𝑡𝑖𝑜𝑛 𝑠𝑡𝑒𝑝)
𝑃(𝑘) : √𝑎 + √𝑎 + √𝑎 + ⋯ + √𝑎 <
⟹ 𝑓(𝑝) 𝑘 𝑚 = 1 1+√(4𝑎+1)
2
⟹ 𝑓(𝑝) = 𝑘 −𝑚 ⟹ 𝑓(𝑝) = 𝑘 𝑝 it is true for K-radical signs
negative integer Step III: For n = k+1
𝑝
Finally let 𝑛 = 𝑞 when p, q> 0 and then
𝑃(𝑘+1) : √𝑎 + √𝑎 + √𝑎 + ⋯ + √𝑎 <
𝑓(𝑝) = 𝑓(𝑛𝑞) = 𝑓(𝑛 + 𝑛 + ⋯ 𝑞 𝑡𝑖𝑚𝑒𝑠) 1+√(4𝑎+1)
2
2 (𝑘 + 1)𝑟𝑎𝑑𝑖𝑐𝑎𝑙 𝑠𝑖𝑔𝑛𝑠
= 𝑓(𝑛) 𝑓(𝑛) 𝑓(𝑛) … . . 𝑞 𝑓𝑎𝑐𝑡𝑜𝑟𝑠 = [𝑓(𝑛) ]
For assumption step
𝑞
∴ 𝑓(𝑝) = [𝑓(𝑛) ] (𝑏𝑦 𝑎𝑠𝑠𝑢𝑚𝑝𝑡𝑖𝑜𝑛 𝑠𝑡𝑒𝑝)
√𝑎 + √𝑎 + √𝑎 + ⋯ + √𝑎 < 1+√(4𝑎+1)
𝑝 2
⟹ 𝑓(𝑛) = 𝑘 = 𝑘 𝑛 .
𝑞
k-radical signs
Hence the result is true for all rational ⟹
number.
√𝑎 + √𝑎 + √𝑎 + ⋯ + √𝑎
6) Show that for all n ∊ N,
√𝒂 + √𝒂 + √𝒂 + ⋯ + √𝒂 ≤ 𝟏+√𝟒𝒂+𝟏 1 + √(4𝑎 + 1)
𝟐 <𝑎+
2
where ‘a’ is fixed positive number and n
radical signs are taken on L.H.S.
182
Challenging Mathematical Problems
= ∑𝑘+1
𝑟=0 𝑘 + 1𝐶𝑟 𝑋𝑘+1−𝑟 𝑌𝑟 .
183
Challenging Mathematical Problems
𝜋
1st term in the 1st group = 1 1 2
= ∫ (1 + cos 2𝑥) 𝑑𝑥
1st term in the 2nd group = (1+2)+1= 4 2 0
𝜋
1 sin 2𝑥
= 2 (𝑥 + 2
) 02
1st term in the 3rd group =
1 𝜋 𝜋
(1+2+3+4)+1= 11 = ( )= 2
2 2 2
………………………………………………
Therefore, the result is true for n -1
………………………………………………
Step II: For n = k+1
1st term in the (k+1) th group = 𝜋
2
(1+2+3+…+2k)+1 𝐼𝑘+1 = ∫ 𝑐𝑜𝑠 𝑘+1 𝑥 cos(𝑘 + 1)𝑥 𝑑𝑥
0
=k(1+2k)+1 = (2𝑘 2+k+1) 𝜋
2
= ∫ 𝑐𝑜𝑠 𝑘 𝑥 (cos(𝑘
The number of terms in the (k+1)th 0
group = (2k +1) + 1)𝑥 𝑑𝑥)𝑑𝑥
𝜋
∴ The sum of the terms in the (k+1) the 2
𝐼𝑘+1 − 𝐼𝑘 = ∫ 𝑐𝑜𝑠 𝑘 𝑥 {cos(𝑘 + 1)𝑥 cos 𝑥
group 0
− cos 𝑘𝑥}𝑑𝑥
(2𝑘 + 1)
= [2 (2𝑘 2 + 𝑘 + 1) + 2𝑘] 𝜋
2
= ∫02 𝑐𝑜𝑠 𝑘 𝑥 {cos(𝑘 + 1)𝑥 cos 𝑥 −
= (2𝑘 + 1)(2𝑘 2 + 2𝑘 + 1) cos(𝑘 + 1)𝑥 cos 𝑥 − sin(𝑘 +
1)𝑥 sin 𝑥}𝑑𝑥
= 4𝑘 3 + 6𝑘 2 + 4𝑘 +
{∵ cos 𝑘𝑥 = cos[(𝑘 + 1)𝑥 − 𝑥]
1 … … … … … … … . (1)
= cos(𝑘 + 1) 𝑥 cos 𝑥
Sum of the terms in the first (k+1) groups + sin(𝑘 + 1)𝑥 sin 𝑥}
= (sum of first k group) + (sum of terms = sin(𝑘 + 1) 𝑥. 𝑐𝑜𝑠 𝑘 𝑥(− sin 𝑥)𝑑𝑥
in the (k+1)th group) = 𝑘 4 + 4𝑘 3 +
Integrating by parts
6𝑘 2 + 4𝑘 + 1 by assumption step and (1)
= (𝑘 + 1)4 . 𝜋
𝑐𝑜𝑠 𝑘+1 𝑥 2
𝐼𝑘+1 − 𝐼𝑘 = [sin(𝑘 + 1) 𝑥 ]
This show that the result is true for n = (𝑘 + 1) 0
𝜋
k+1. Hence, by the principal of 2
− ∫ (𝑘 + 1)
mathematical induction, the result is true 0
for all n ∊N. 𝑐𝑜𝑠 𝑘+1
cos(𝑘 + 1)𝑥 𝑑𝑥 = 0 − 𝐼𝑘+1
𝝅 (𝑘 + 1)
𝝅
9) Show that ∫𝟎𝟐 𝒄𝒐𝒔𝒏 𝒙 𝐜𝐨𝐬 𝒏𝒙 𝒅𝒙 = 𝟐𝒏+𝟏 1 1 𝜋 𝜋
𝐼𝑘+1 = 𝐼𝑘 = . 𝑘+1 . 𝐼𝑘+1 = 𝑘+2
2 2 2 2
𝜋
𝜋
Sol.: Let 𝐼𝑛 = ∫02 𝑐𝑜𝑠 𝑛 𝑥 cos 𝑛𝑥 𝑑𝑥 = This shows that the result is true for n = k+1.
2𝑛+1
184
Challenging Mathematical Problems
𝑚
10) Show that ∑𝒏𝒌=𝟎 𝒌𝟐 𝒏𝑪𝒌 = 𝒏(𝒏 +
= 𝑃(𝑚) + ∑(𝑘 + 1)2 𝑚𝐶𝑘 (𝑅𝑒𝑝𝑙𝑎𝑐𝑒 𝑘 𝑏𝑦 𝑘
𝟏)𝟐𝒏−𝟐 𝒇𝒐𝒓 𝒏 ≥ 𝟏.
𝑘=0
1 = 𝑃(𝑚) + ∑ 𝑘 2 𝑚𝐶𝑘
𝑘=0
𝑃(1) = ∑ 𝑘 2 1𝐶𝑘 = 0 + 1 1𝐶1 = 1 𝑚 𝑚
𝑘=0
+ 2 ∑ 𝑘 𝑚𝐶𝑘 + ∑ 𝑚𝐶𝑘
= 1(1 + 1) 21−2
𝑘=0 𝑘=0
Step III: For n = m+1 This show that the result is true for n = m+1.
Hence, by the principal of mathematical
𝑚+1 𝑚+1
induction, the result is true for all n ∊ N.
𝑃(𝑚+1) = ∑ 𝑘 2 𝑚 + 1𝐶𝑘 = 0 + ∑ 𝐾 2
𝑘=0 𝑘=1
𝑚
11) Show that the sequence {𝒂𝒏 }, where 𝒂𝒏 =
𝑚 + 1𝐶 𝑘 = ∑ 𝑘 2 𝑚 + 1𝐶 𝑘 𝟏.𝟑.𝟓….(𝟐𝒏−𝟏)
𝑘=1 𝟐.𝟒.𝟔…..𝟐𝒏
√(𝟐𝒏 + 𝟏) is a monotonic
2
+ (𝑚 + 1) 𝑚 + 1𝐶𝑚+1 decreasing sequence.
𝑚
1.3.5……(2𝑛−1)
= ∑ 𝑘2 Sol.: ∴𝑎𝑛 = √(2𝑛 + 1)
2.4.6…..2𝑛
𝑘=1
√3
(𝑚𝐶𝑘 + 𝑚𝐶𝑘−1 ) + (𝑚 + 1)2 Step I. For n= 1, 2 𝑎1 = , 𝑎2 =
2
𝑚 1.3 3√5
2 √(4 + 1) =
= ∑ 𝑘 𝑚𝐶 𝑘 2.4 8
𝑘=1
𝑚 𝐻𝑒𝑟𝑒 𝑎1 > 𝑎2 (∴ 2 > 1)
2 2
+ [∑ 𝑘 𝑚𝐶𝑘−1 + (𝑚 + 1) ]
It is monotonic decreasing function which is
𝑘=1
𝑚+1 true for n = 1, 2
= 𝑃(𝑚) + ∑ 𝑘 2 𝑚𝐶𝑘−1
Step II. Assume it is true for n = k, then
𝑘=1
𝑎𝑘−1 > 𝑎𝑘 (∵ 𝑘 > 𝑘 − 1)
185
Challenging Mathematical Problems
𝑎𝑘+1 1 + 𝑎 + 𝑎2 + ⋯ + 𝑎𝑘 + 𝑎𝑘+1
1.3.5 … . . (2𝑘 − 1)(2𝑘 + 1) 𝑃(𝑘+1) :
𝑎 + 𝑎2 + ⋯ + 𝑎𝑘−1 + 𝑎𝑘
= . √(2𝑘 + 3) 𝑘+2
2.4.6 … . .2𝑘(2𝑘 + 2)
≥
𝑎𝑘 (2𝑘 + 1) − √(2𝑘 + 3) 𝑘
= . 𝑜𝑟 1 + 𝑎 + 𝑎2 + ⋯ + 𝑎𝑘 + 𝑎𝑘+1
√(2𝑘 + 1) (2𝑘 + 2)
𝑘+2
≥
⟹ 𝑎𝑘+1 𝑘
(𝑎 + 𝑎2 + ⋯ + 𝑎𝑘 ) … … … … … . (1)
𝑎𝑘 √(2𝑘 + 1)(2𝑘 + 3)
= … … … … … (1) 𝐵𝑦 𝑎𝑠𝑠𝑢𝑚𝑝𝑡𝑖𝑜𝑛 𝑠𝑡𝑒𝑝, 𝑤𝑒 𝑔𝑒𝑡
(2𝑘 + 2)
1 + 𝑎 + 𝑎2 + ⋯ + 𝑎𝑘
𝑘+1
√(2𝑘 + 1)(2𝑘 + 3) ≥( ) (𝑎 + 𝑎2
𝐴𝑠𝑠𝑢𝑚𝑒 𝑘−1
(2𝑘 + 2)
+ ⋯ + 𝑎𝑘−1 )
< 1 … … … . (2)
𝐴𝑑𝑑𝑖𝑛𝑔 𝑎𝑘+1 to both sides, we get
𝑡ℎ𝑒𝑛 (2𝑘 + 1)(2𝑘 + 3) < (2𝑘 + 2)2 1 + 𝑎 + 𝑎2 + ⋯ + 𝑎𝑘 + 𝑎𝑘+1
𝑘+1
⟹ 4𝑘 2 + 8𝑘 + 3 < 4𝑘 2 + 8𝑘 + 4 ⟹ 0 < ≥( ) (𝑎 + 𝑎2
𝑘−1
1 which is true. + ⋯ + 𝑎𝑘−1 )
+ 𝑎𝑘+1 … … … (2)
From (1) and (2), we get 𝑎𝑘+1 < 𝑎𝑘 (∵ 𝑘 +
𝑘+1
1 > 𝑘) 𝑎𝑠𝑠𝑢𝑚𝑒 ( ) (𝑎 + 𝑎2 + ⋯ + 𝑎𝑘−1 )
𝑘−1
This show that the result is true for n = k+1. 𝑘+2
+ 𝑎𝑘+1 ≥ ( )
Hence, by the principal of mathematical 𝑘
induction, the result is true for all n ∊ N. (𝑎 + 𝑎2 + ⋯ + 𝑎𝑘 ) … … … … … . (3)
𝑘 + 1 (𝑘 + 2)
𝑡ℎ𝑒𝑛 ( − ) (𝑎 + 𝑎2 + ⋯
𝑘−1 𝑘
+ 𝑎𝑘−1 ) + 𝑎𝑘+1
12) If ‘a’ fixed real number ≥ 𝟐, then show
𝟏+𝒂+𝒂𝟐 +⋯+𝒂𝒏 𝒏+𝟏
𝑘+2 𝑘
that 𝒂+𝒂𝟐+⋯+𝒂𝒏−𝟏 ≥ 𝒏−𝟏 , 𝒏 ∈ 𝑵, 𝒏 > 1. ≥( )𝑎
𝑘
2
1+𝑎+𝑎2 +⋯+𝑎 𝑛 𝑛+1 ⟹ (𝑎 + 𝑎2 + ⋯ + 𝑎𝑘−1 )
Sol.: let 𝑃(𝑛) : 𝑎+𝑎2 +⋯+𝑎𝑛−1 ≥ 𝑘(𝑘 − 1)
𝑛−1
(𝑘 + 2) 2
1+𝑎+𝑎 2 3
− 𝑎 ≥0
Step I. For n = 2 𝑃(2) : ≥ 1:1 + 𝑎 + 𝑘
𝑎
⟹ 2(𝑎 + 𝑎2 + ⋯ + 𝑎𝑘−1 )
𝑎2 ≥ 3𝑎
+ 𝑘(𝑘 − 1)𝑎𝑘+1
: 𝑎2 − 2𝑎 + 1 ≥ 0 ∶ (𝑎 − 1)2 ≥ 0 𝑘+2 𝑘
− 𝑎 ≥0
𝑤ℎ𝑖𝑐ℎ 𝑖𝑠 𝑡𝑟𝑢𝑒 𝑓𝑜𝑟 𝑛 = 2 𝑘
⟹ 2(𝑎 + 𝑎2 + ⋯ + 𝑎𝑘−1 )
Step II. Assume it is true for n = k, then + 𝑎𝑘 {(𝑘 − 1)𝑘𝑎
− (𝑘 + 2)} ≥ 0
1 + 𝑎 + 𝑎2 + ⋯ + 𝑎𝑘 𝑘 + 1
𝑃(𝑘) : ≥ ⟹ 2(𝑎 + 𝑎 + ⋯ + 𝑎𝑘−1 )
2
𝑎 + 𝑎2 + ⋯ + 𝑎𝑘−1 𝑘−1
+ 𝑎𝑘 (𝑘 − 1)(𝑘(𝑎 − 1)
Step III. For n = k+1, we have to show that − 2) ≥ 0
186
Challenging Mathematical Problems
Which is true 1 1
It remains to be observed that 21 ! + 22 ! +
∵ 𝑎 ≥ 2 𝑎𝑛𝑑 𝑘 ≥ 2 1
{ ⋯ + 2𝑘 ! < 1.
∴ 𝑘 − 1 ≥ 1, 𝑘(𝑎 − 1) − 2 ≥ 0
187
Challenging Mathematical Problems
2) If the unit’s digit in (𝟑𝟏𝟐𝟕)𝟏𝟕𝟑 is (6) The last two digits in 𝟏𝟗𝟏𝟗𝟗𝟏 is
(a) 1; (b) 3; (c) 7; (d) none (a) 17; (b)18; (c) 19; (d) none
3) 𝟒𝟔𝟏 + 𝟒𝟔𝟐 + 𝟒𝟔𝟑 + 𝟒𝟔𝟒 is divisible by (7) The sum 𝟏𝟏𝟗𝟗𝟗 + 𝟐𝟏𝟗𝟗𝟗 + ⋯ + 𝟐𝟎𝟎𝟎𝟏𝟗 is
(a) 3; (b) 10; (c) 11; (d) none multiple of
(a) 1998; (b) 1999; (c) 2000; (d)
Sol.: 461 (1 + 4 + 42 + 43 ) = 461 × 85 = none
460 × 340 which is clearly divisible by 10.
Sol.:
4) A number when divided successively by 4
and 5 leaves remainders 1 and 4 Because ∑2000
1001 𝑖
1999
= ∑1000
1 (𝑗 +
1999 1000
respectively. When it is successively 1000) = ∑1 (𝑗 − 1001)1999 =
divided by 5 and 4, then the respectively ∑1000
1 (−𝑘)1999
remainder will be
(a) 2, 3; (b) 3, 2; (c) 4, 1; (d) none =
∑1000
1 (𝑖)1999 (𝑚𝑜𝑑 2001), it follows that
Sol.:
2000 1000 2000
1999 1999
∴ y = (5 × 1 + 4) = 9 ∑𝑖 = ∑𝑖 + ∑ 𝑖 1999
1 1 1
∴x = (4x+1)=4 × 9+1= 37 1000 1000
= ∑ 𝑖 + ∑ (−𝑖)1999
Now, 37 when divided successively by 1 1
5 and 4. = 0 (𝑚𝑜𝑑 2001)
188
Challenging Mathematical Problems
24 − 1 33 − 1 52 − 1 15 26 24
= . . = . .
2−1 3−1 5−1 1 2 4 (Subjective Type)
= 1170
𝟏𝟑
This includes 1 and the number 360. Thus 1) Prove that 𝟐𝟕𝟎 + 𝟑𝟕𝟎
sum of all the divisors of 360 excluding 1 and
Sol.: Observe that 270 + 370 = 435 + 935 , and
itself is 1170 − 361 = 809.
that 35 is odd. Now 𝑎𝑛 + 𝑏 𝑛 is divisible by
(12) The sum of all cubes of three 𝑎 + 𝑏 when n is odd.
consecutive integers is divisible by
From this is follows that 435 + 935 is divisible
(a) 9; (b) 12; (c) 18; (d) none
by 13.
Sol.: Try by trial method
2) Find the number of positive integer n less
𝟔
than 1991 for which 𝒏𝟐 + 𝟑𝒏 + 𝟐.
189
Challenging Mathematical Problems
190
Challenging Mathematical Problems
(1, 20), (2, 13), showing that √2 is not rational), that 𝜙 is not
rational; that is, it does not equal the ratio of
(3, 10), (7, 6), (10, 5), (17, 4)𝑎𝑛𝑑 (52, 3)
two non-zero integers. This has the following
𝑜𝑟 sixteen pairs in all. implication. If c and d are integers such that
c𝜙 +d = 0, then c and are both 0. For if c𝜙 +d
6) N is a 50 digit number (in base -10). All 𝑑
= 0 and c ≠ 0 then we have 𝜙 = − 𝑐 , a
digits except the 26th (from the left) are 1.
rationed number. Since this can’t be, we must
Given that N is divisible by 13, find its 26th
have c = 0; but this forces d to be 0 too.
digit.
By definition, substituting 0 for x in 𝑥 2 − 𝑥 −
Sol.: Let x be the 26th digit of N. In base −10.
1 yields 0, Since 𝑎𝑥 17 + 𝑏𝑥 16 + 1 is a multiple
N= of 𝑥 2 − 𝑥 + 1, substituting 𝜙 for x in 𝑎𝑥 17 +
11111111 … … 1111 𝑥 ⏟
⏟ 11111111 … … 1111 𝑏𝑥 16 + 1 must yield 0; that is 𝑎 𝜙17 + 𝑏𝜙16 +
25 𝑜𝑛𝑒𝑠 24 𝑜𝑛𝑒𝑠 1 = 0. Since the result of substituting 𝜙 in
𝑥 2 − 𝑥 − 1yields 0, we must have 𝜙 2 = 𝜙 +
We now apply the ‘alternating 3-digit sum’
1. This relation allows us to express all
testing divisibility by 13. Recalling that the
powers of 𝜙 in the form c𝜙 +d where c and d
sum has to be computed starting from the
are integers. For example, 𝜙 3 = 𝜙 + 𝜙 2 =
right. We computer the All sum as 𝐴 + ̅11 ̅̅̅𝑥 −
𝜙 × (𝜙 + 1) = 𝜙 2 + 𝜙 = 2𝜙 + 1, the
𝐵 + 11, where
relation, 𝜙 = 𝜙+ I being used repeatedly.
𝐴= ⏟
111 − 111 + ⋯ − 111 𝐵 Similarly, 𝜙 4 = 𝜙 × 𝜙 3 = 𝜙 × (2𝜙 + 1) =
8 𝑏𝑙𝑜𝑐𝑘𝑠 2𝜙 2 + 𝜙 = 3𝜙 + 2. In general, if we have
= ⏟
111 − 111 + ⋯ + 111 expressed 𝜙 𝑛−1 in the form c𝜙+d, then, we
7 𝑏𝑙𝑜𝑐𝑘𝑠
have 𝜙 𝑛 = 𝜙 × 𝜙 𝑛−1 = 𝜙 × (𝑐𝜙 + 𝑑) =
Clearly A = 0 and B = 111. So, the All sum is 𝑐𝜙 2 + 𝑑𝜙 = 𝑐(𝜙 + 1) + 𝑑𝜙 = (𝑐 + 𝑑)𝜙 + 𝑐
̅̅̅̅𝑥 − 100 𝑜𝑟 1̅𝑥, which obviously is divisible
11
by 13 only for x = 3. Since we have already expressed 𝜙 4 in the
stated form, We may now do the same for any
Hence, the 26th digit of N is 3. higher power of 𝜙 . We display below some of
the results of these computations,
7) Find integers a, b such that 𝒙𝟐 − 𝒙 − 𝟏 is a
divisor of the polynomial 𝒂𝒙𝟏𝟕 + 𝒃𝒙𝟏𝟔 + 𝟏 𝜙 5 = 50 + 3, 𝜙 6 = 8𝜙 + 5,
191
Challenging Mathematical Problems
Sol.: We first note that the choice p =2 does In this equation we must have 𝑐 > 1. (𝐼𝑓 𝑐 =
not work as 22 + 11 = 15 = 3 × 5 has 4 and 1, 𝑡ℎ𝑒 𝑙𝑒𝑓𝑡 𝑠𝑖𝑑𝑒 𝑤𝑜𝑢𝑙𝑑 𝑒𝑥𝑐𝑒𝑒𝑑1. ) If 𝑐 ≥ 3,
1 1 1 1 1 1
not 6 divisors. So P must be an odd prime, then a>b>3, and 𝑎 + 𝑏 + 𝑐 < 3 + 3 + 3 = 1;
implying that 𝑃2 + 11 is even and therefore the above equation could never hold good so
contains the prime 2 as a factor. we must have c>3, which implies that c = 2
1 1 1
We know now use the formula for d(n), the and therefore that a>b>2 and + = Next,
𝑏 2 𝑎
1 1
divisor function. Since 6 = 3 × 2, there are suppose that 𝑏 ≥ 4. Then a >4 and 𝑎 + 𝑏 <
precisely two categories of number with 6 1 1 1 1 1 1
+ 4 = 2; the equation 𝑎 + 𝑏 = 2 could never
divisors, those of the kind 𝑞 5 (with q prime) 4
hold good so, b<4, which means that b= 3.
and those of the kind 𝑞 2 𝑟(with q, r unequal 1 1 1
primes). So, if 𝑃2 + 11 has 6 divisors, then Since + = . We get a = 6. So the values of
𝑎 𝑏 2
𝑃2 + 11 = 𝑞 5 𝑜𝑟 𝑞 2 𝑟 where q, r are primes, q a, b, c are now all known using these values
𝑛 𝑛 𝑛
≠ r. The 1st case is quickly ruled out, our we see that 𝑑13 = 6 , 𝑑14 = 3 𝑎𝑛𝑑 𝑑15 = 2 .
earlier observation tells us that q = 2, but this Three deductions now follows:
does not work, as there is no prime P with
2 3
𝑃2 + 11 = 25 . (a) n is divisible by 6, so , ;
𝑛 𝑛
192
Challenging Mathematical Problems
not, the first five divisors of n are either1, 2, 3, where p is a prime number greater than 5.
4, 5 or 1, 2, 3, 5, 6. So either 𝑑5 = 5 𝑜𝑟 𝑑5 = 6. Each of these cases must now be considered.
𝑛
The first possibility leads to 2 + 1 = In each case, the first four divisors of n are 1,
(5 + 1)3 = 216 𝑜𝑟 𝑛 = 430. while the second 2, 3, 6. We quickly ascertain that p = 7 does
𝑛 not yield a solution because none of the
leasts to 2 + 1 = (6 + 1)3 = 343 𝑜𝑟 𝑛 = 684.
numbers 12 × 7 + 1 = 85, 27 × 7 + 1 =
But 430 is not divisible by 6 (whearas, it is
6
190, 3 × 73 + 1 = 1030 is a cube. Therefore,
already know that 𝑛) and 684 is not divisible p> 7, which in facts means that 𝑝 ≥ 11. If n =
by 5. This contradictory state of affairs tells 24p then the next divisor after 6 is 𝑑5 = 8,
𝑛
us that n does not have 5 as a prime factor. yielding 2 + 1 = (8 + 1)3 = 729, 𝑜𝑟 𝑛 =
Since 16 has following five factorization; 16 1456, but this cannot be, as 24 is not even a
=8 × 2 = 4 × 4 = 4 × 2 × 2 = 2 × 2 × 2 × 2, divisor of 1456.
there are five classes of numbers with 16
divisors; those of the kinds 𝑝15 , 𝑝7 × 𝑞, 𝑝3 × If n = 54p then the next divisor after 6 is
𝑛
𝑞 3 , 𝑝3 × 𝑞 × 𝑟 𝑎𝑛𝑑 𝑝 × 𝑞 × 𝑟 × 𝑠 where p, q, r, 𝑑5 9, 𝑦𝑒𝑖𝑙𝑑𝑖𝑛𝑔 + 1 = (9 + 1)3 =
2
s are distinct primes, each greater than 5. We 1000, 𝑜𝑟 𝑛 = 1998 = 54 × 37. Therefore p =
shall now consider each of these cases in turn. 37. Does this fit the given condition the 16
divisors of 1998 are 1, 2, 3, 6, 9, 18, 27, 37, 54,
We already know that n is divisible by the
74, 111, 222, 333, 666, 999 and 1998,
primes 2 and 3, so the possibility n = 𝑝15 is
therefore 𝑑5 = 9, 𝑑13 = 333, 𝑑14 = 999. The
ruled out.
conditions do indeed hold good!
If 𝑛 = 𝑝7 × 𝑞. then we must have 𝑛 = 27 ×
Finally, of 𝑛 = 6𝑝3 , 𝑤𝑖𝑡ℎ 𝑝 ≥ 11, 𝑡ℎ𝑒𝑛 𝑑5 = 𝑝.
3 = 384 𝑜𝑟 𝑛 = 37 × 2 = 4374; 𝑤ℎ𝑖𝑙𝑒 𝑖𝑓 𝑛 =
So we obtain the equation 3𝑝3 +
𝑝3 × 𝑞 3 , 𝑡ℎ𝑒𝑛 𝑤𝑒 𝑚𝑢𝑠𝑡 ℎ𝑎𝑣𝑒 𝑛 = 23 × 33 =
𝑛 1(𝑝 + 1)3 , 𝑜𝑟 3𝑝3 = 𝑝3 + 3𝑝2 + 3𝑝. This
216. In one of these three cases is 2 + 1 a 3
implies that , an absurdity. So there is no
cube. So, this possibility too is ruled out. 𝑝
solution to be found here.
If 𝑝 × 𝑞 × 𝑟 × 𝑠, then, as earlier, 2, 3 ∊ {p, q, r,
s}. So, n is of the form 6pq where p, q are Thus there is just one number which fits the
distinct primes, say with 5 < 𝑝𝑞. The 16 given conditions: 𝑛 = 1998 = 2 × 33 × 37.
divisors of n are in this case 1, 2, 3, 6,
10) Let n be the positive integer with at least
𝑝(= 𝑑5 ), … , 𝑝𝑞(= 𝑑13 ), 2𝑝𝑞(𝑑14 , 3𝑝𝑞 =
𝑛 4 divisors and let its divisors is be
𝑑15 ), 6𝑝𝑞. The relation 2 + 1 = (𝑑5 + 1)3 𝒅𝟏 , 𝒅𝟐 , 𝒅𝟑 , 𝒅𝟒 , … ., where 𝒅𝟏 < 𝒅𝟐 < 𝒅𝟑 <
reduces to 3𝑝𝑞 + 1 = (𝑝 + 𝒅𝟒 < ⋯ , 𝑤𝑖𝑡ℎ 𝒅𝟏 = 𝟏. find all possible
1)3 , 𝑤ℎ𝑖𝑐ℎ 𝑦𝑒𝑖𝑙𝑑𝑠 3𝑝 = 𝑝2 + 3𝑝 + 3. Since 3 values of n it is known that n = 𝒅𝟏 𝟐 +
is a divisor of each of the quantities 3q, 3p 𝒅𝟐 𝟐 + 𝒅𝟑 𝟐 + 𝒅𝟒 𝟐
3
and 3, we must have 𝑝2 , which is absurd as p
Sol.: Obviously 𝑑1 = 1. If n were odd, then all
is a prime number greater than 5. So this
its divisors would be old and 1+𝑑2 2 + 𝑑3 2 +
possibilities is also ruled out.
𝑑4 2 would be a sum of four old numbers,
If 𝑛 = 𝑝3 × 𝑞 × 𝑟, 𝑡ℎ𝑒𝑛 2, 3 ∈ {𝑝, 𝑞, 𝑟}, so n is therefore even; a contradiction. So n is even,
one of the following forms: 24𝑝, 54𝑝, 6𝑝3 , which means that 𝑑2 = 2. Let 𝑑3 𝑎𝑛𝑑 𝑑4 be
193
Challenging Mathematical Problems
194
Challenging Mathematical Problems
195
Challenging Mathematical Problems
Sol.: Let 2𝑥 + 1 = 𝑥 2 𝑎𝑛𝑑 3𝑥 + 1 = 𝑦 2 where 14) Let T be the set of all triplets (a, b, c) of
x and y are n for which these equations holds integers such that 𝟏 ≤ 𝒂 ≤ 𝒃 ≤ 𝒄 ≤ 𝟔. For
are shown in the following table: each triplet (a, b, c) in T, take the number
a× 𝒃 × 𝒄 and odd all these nmubers
x 1 9 89 881 corresponding to all the triplets in T. Show
y 1 11 109 1079 …….. that this sum is divisible by 7.
196
Challenging Mathematical Problems
𝟏 𝒎
Now adding up Eq. (1) from k = 2 to k = (n 16) Prove that 𝒏 = 𝟖 {(𝟏𝟕 + 𝟏𝟐√𝟐) +
+1), we have 𝒎
(𝟏𝟕 − 𝟏𝟐√𝟐) } + 𝟔 is an integer for all n
4𝑎2 − 2𝑎1 = −𝑎1 ∊ N and hence, show that both
6𝑎3 − 2𝑎2 = −𝑎2
(𝒏 − 𝟏)(𝟐𝒏 − 𝟏) are perfect squares for
8𝑎4 − 6𝑎3 = −𝑎3 … … … … . (2)
⋮ all n ∊ N.
⋮ }
Sol.: As is problem 73, the terms containing
2𝑛𝑎𝑛 − 2(𝑛 − 1)𝑎𝑛−1 √2 vanishes in the expansion of (17 +
= −𝑎𝑛−1 , 2(𝑛 𝑚 𝑚
12√2) + (17 − 12√2) and integral terms
+ 1)𝑎𝑛+1 − 2𝑛𝑎𝑛
are all multiplies of 8 and hence, n is an
1
= −𝑎𝑛 𝑠𝑢𝑚𝑚𝑖𝑛𝑔, 𝐸𝑞. (2), 𝑤𝑒 𝑔𝑒𝑡, 2(𝑛 integers, (prove it) 𝑛 − 1 = × [(17 +
8
+ 1)𝑎𝑛+1 − 2𝑎1 𝑚 𝑚 1
12√2) + (17 − 12√2) + 6 − 8] = 8 ×
𝑛 𝑛 𝑚 𝑚
[(17 + 12√2) + (17 − 12√2) − 2]
= − ∑ 𝑎𝑘 ⟹ ∑ 𝑎𝑘
𝑘=1 𝑘=1 comparing the above expansion from the
= 2𝑎1 − 2(𝑛 + 1)𝑎𝑛+1 result of problem 71. We get,
= 1 − 2(𝑛 + 1)𝑎𝑛+1
17 + 12√2 = (3 + 2√2)2 , 17 − 12√2
1 3 = (3 − 2√2)2 𝑎𝑔𝑎𝑖𝑛 𝑏𝑜𝑡ℎ (17
𝑎1 = , 𝑎𝑛 = (1 − ) 𝑎𝑛−1
2 2𝑛
+ 12√2)(17 − 12√2)𝑎𝑛𝑑 (3
3 1 1 1 1 + 2√2)(3 − 2√2)𝑎𝑟𝑒 𝑒𝑞𝑢𝑎𝑙 𝑡𝑜 1.
⟹ 𝑎2 = (1 − ) , = × =
4 2 2 4 8
3 1 1 𝑚
⟹ 𝑎3 = (1 − ) 𝑆𝑜, × [(17 + 12√2)
6 8 8
𝑚
1 1 1 3 + (17 − 12√2) − 2]
= × = 𝑎𝑛 : (1 − ) 𝑎𝑛−1 is
2 8 16 2𝑛
3 1 𝑚−2
positive as (1 − 2𝑛) 𝑓𝑜𝑟 𝑎𝑙𝑙 𝑛 ≥ 2 is = × {[(3 + 2√2) ]+
8
positive and 𝑎1 , 𝑎2 , 𝑎3 , … … …. are all [(3 − 2√2)𝑚−2 ] − 2 × (3 + 2√2)(3 −
positive. Since each
2√2)}
𝑎𝑖 𝑖𝑠 𝑎 𝑝𝑟𝑜𝑑𝑢𝑐𝑡 𝑜𝑓 (1 −
3 𝑚2 𝑚2
2𝑖
) 𝑎𝑖−1 𝑎𝑛𝑑 𝑎𝑖 > 0 implies that 𝑎2 > 1 (3 + 2√2) + (3 − 2√2)
= ×[ ]
0 … … 𝑎𝑖−1 > 0 and hence, 8 2√2
𝑛
1 𝑚
∑ 𝑎𝑘 = 1 − 2(𝑛 + 1)𝑎𝑛+1 < 1. [ 𝑎𝑛𝑑 2𝑛 − 1 = 4 × [(17 + 12√2) +
𝑘=1 𝑚
(17 − 12√2) + 6 − 4]
∵ 2(𝑛 + 1)𝑎𝑛+1 > 0
1 𝑚
= 4 × [(17 + 12√2) + (17 −
𝑚
12√2) + 2]
197
Challenging Mathematical Problems
𝑚 𝑚 𝑛 𝑛 2
(3+2√2) +(3−2√2)
=[ ] and hence the {(1 + √2) − (1 − √2) }
2
𝑚 4
result show that (3 + 2√2) −
𝑚 𝑚
𝑚 (3+2√2) +(3−2√2) Are consecutive integers. For any n,
(3 − 2√2) and 2
𝑛 𝑛 𝑛 𝑛 𝑛 𝑛
(1+√2) +(1−√2) (1 + √2) − (1 − √2) (1 + √2) + (1 − √2)
are and so 2√2
is also an 𝑎𝑛𝑑
2 2
integer and hence, their sum is also an
integer. Thus, Are integers (prove) and hence
1 𝑚 𝑚 𝑛 𝑛 2
32
[(17 + 12√2) + (17 − 12√2) − (1 + √2) − (1 − √2)
{ }
2] is a square integer. To show that 2
1 𝑛 𝑛 2
Exp. (1) can be written as 2 𝑚(𝑚 + 1) (1 + √2) − (1 − √2)
1
={ }
consider the Exp. (2) × 4
32
𝑚 𝑚
[(17 + 12√2) + (17 − 12√2) − 2] 𝑛 𝑛 2
(1 + √2) − (1 − √2)
𝑎𝑛𝑑 { }
𝑛 𝑛 2 𝑛 𝑛 2 2
(1 + √2) + (1 − √2) (1 + √2) − (1 − √2)
={ } { } 𝑛 𝑛 2
2 2√2 (1 + √2) − (1 − √2)
={ }
4
𝑛 𝑛 2
1 {(1+√2) −(1−√2) }
= 2
[ 4
]. 𝑎𝑟𝑒 𝑖𝑛𝑡𝑒𝑔𝑒𝑟𝑠. 𝑁𝑜𝑤
𝑛 𝑛 2
(1 + √2) − (1 − √2)
𝑛 𝑛 2 ={ }
{(1 + √2) + (1 − √2) } 4
[ ]
4 2𝑛 2𝑛
(1 + √2) + (1 − √2) −2
=
For all n, we shall show that 4
𝑛 𝑛
𝑛
{(1+√2) −(1−√2) }
𝑛 2 𝑛
{(1+√2) +(1−√2) }
𝑛 2 (3 + 2√2) + (3 − 2√2) − 2
= … … … . . (3)
4 4 4
are consecutive integers clearly, for n 𝑛 𝑛 2
= 1, we get (1 + √2) − (1 − √2)
𝑎𝑛𝑑 𝑠𝑖𝑚𝑖𝑙𝑎𝑟𝑙𝑦, { }
4
𝑛 𝑛 2
{(1 + √2) − (1 − √2) } 8 𝑛 𝑛
= (3 + 2√2) + (3 − 2√2) − 2
4 4 = … … … . (4)
𝑛 𝑛 2 4
{(1 + √2) + (1 − √2) }
= 2 𝑎𝑛𝑑 ∴ From Exp (3) and (4), we find that
4
4
= = 1 𝑎𝑛𝑑 ℎ𝑒𝑛𝑐𝑒 𝑓𝑜𝑟 𝑛 = 1 𝑛 𝑛
4 {(1 + √2) + (1 − √2) }
𝑎𝑛𝑑
𝑛 𝑛 2 4
{(1 + √2) + (1 − √2) }
𝑎𝑛𝑑
4
198
Challenging Mathematical Problems
𝑛 𝑛 1 2𝑚+1
{(1 + √2) − (1 − √2) } 𝑓𝑜𝑟 𝑚 = 1, × [(3 + 2√2)
8
4 2𝑚+1
+ (3 − 2√2) − 6]
2𝑘−2 2𝑘+2 1
𝑎𝑟𝑒 𝑖𝑛𝑡𝑒𝑔𝑒𝑟𝑠 𝑜𝑓 𝑡ℎ𝑒 𝑓𝑜𝑟𝑚 𝑎𝑛𝑑 𝑜𝑟 (𝑘 −
4 4 2
1 1
1)𝑎𝑛𝑑 2 (𝑘 + 1) and hence, they differ = × [2 × 3𝐶0 × 33 + 2 × 3𝐶2 × 31
8
1 1 2
by 2 (𝑘 + 1) − 2 (𝑘 − 1) = 1. × (2√2) − 6]
1 𝑛 1 1
𝑆𝑜 × {(17 + 12√2) = × [54 + 144 − 6] = × [192] =
32 8 8
𝑛 24 and hence, is an integer. For any
− (17 − 12√2) − 2}
m>1 let us prove that the expression,
𝑛 𝑛 2 2𝑚+1
1 (1 + √2) − (1 − √2) 1
= ×{ } 𝑓(𝑚 + 1) = [(3 + 2√2) +
8
2 4 2𝑚+1
(3 − 2√2) − 6] is an integer.
𝑛 𝑛 2
(1 + √2) + (1 − √2) Expanding and cancelling the terms,
×{ }
4 we get
=2×
1 (𝑘−1) (𝑘+1)
× 2 𝑜𝑟
1
= 2 (𝑚 − 1 2𝑚+1
2 𝑓(𝑚 + 1) = × [(3 + 2√2)
𝑚(𝑚+1) 8
1)𝑚 𝑜𝑟 𝑒𝑞𝑢𝑖𝑣𝑎𝑙𝑒𝑛𝑡𝑙𝑦 and 2𝑚+1
2 + (3 − 2√2) − 6]
hence, the result.
1
1 𝑛 = 4 × [32𝑚+1 + 2𝑚 +
Note: This [(17 + 12√2) +
32 2
𝑛 1𝐶2 . 32𝑚−1 (2√2) + 2𝑚 +
(17 − 12√2) − 2] gives you an
4
infinite family of square and 1𝐶4 . 32𝑚−3 . (2√2) + ⋯ +
2𝑚−1
triangular numbers. 2𝑚 + 1𝐶2 . 3(2√2) − 3]
𝟏 2
17) Show that for 𝒇(𝒎) = 𝟖 {(𝟑 + 1
= 4 × [2𝑚 + 1𝐶2 . 32𝑚+1 . (2√2) +
𝟐𝒎+𝟏 𝟐𝒎+𝟏
𝟐√𝟐) + (𝟑 − 𝟐√𝟐) − 4
2𝑚 + 1𝐶4 . 32𝑚−3 . (2√2) + ⋯ +
𝟔} 𝒃𝒐𝒕𝒉 𝒇(𝒎) + 𝟏 𝒂𝒏𝒅 𝟐𝒇(𝒎) + 𝟏 are 2𝑚
2𝑚 + 1𝐶2𝑚 . 3(2√2) + 32𝑚+1 − 3]
perfect squares for all n∊ N by showing
that 𝒇(𝒎) is an integer. All the terms in the above expression
except 32𝑚+1 − 3 are multiplies of 4,
Sol.: First let us show that the expression
as the even power of (2√2) is a
1 2𝑚+1 multiple of 4. 32𝑚+1 − 3 = 3[9𝑚 − 1]
𝑓(𝑚) = [(3 + 2√2)
8 is also multiple of 4.
2𝑚+1
+ (3 − 2√2) − 6] 1
Now, 𝑓(𝑚) + 1 = 8 × [(3 +
2𝑚+1 2𝑚+1
2√2) + (3 − 2√2) − 6] + 1
199
Challenging Mathematical Problems
2𝑚+1 2𝑚+1
𝑆𝑖𝑛𝑐𝑒 (1 + √2) (1 − √2)
18) Suppose f is a function on the positive
2𝑚+1 integers, which takes integers (i.e. f: N→Z)
= [(1 + √2)(1 − √2)]
with the following properties
= (−1)2𝑚+1 = −1 (a) 𝒇(𝟐) = 𝟐, (b) 𝒇(𝒎,𝒏) = 𝒇(𝒎) . 𝒇(𝒏) ,
(c) 𝒇(𝒎) > 𝒇(𝒏) if m > n. Find
So the given expression is equal to
f(1983).
2𝑚+1 2𝑚+1 2
(1 + √2) − (1 − √2) Sol.: 𝑓(2) = 2, 𝑓(4) = 𝑓(2.2) = 𝑓(2) . 𝑓(2) = 2.2 =
{ }
2√2 4,
200
Challenging Mathematical Problems
Now, we should prove that 𝑓(𝑚) = 𝑚. If m is 20) Show that 𝑭(𝑷𝟏 𝒙𝟏 × 𝑷𝟐 𝒙𝟐 ) = 𝑭(𝑷𝟏 𝒙𝟏 ) ×
𝑭(𝑷𝟐 𝒙𝟐 )
an even integer, then 𝑓(𝑚) = 2𝑘, 𝑎𝑛𝑑 𝑘 < 𝑚.
Sol.: Any divisors of 𝑃1 𝛼1 is 𝑃1 𝑟1 , 𝑤ℎ𝑒𝑟𝑒 0 ≤
So, 𝑓(𝑚) = 𝑓(2𝑘) = 𝑓(2) × 𝑓(𝑘) = 2 × 𝑘 = 2𝑘 =
𝑟 ≤ 𝛼1
𝑚. So all even m, 𝑓(𝑚) = 𝑚. If m is an odd
𝛼1 𝛼1
integer, let m = 2k +1, and 𝑓(2𝑘) < 𝑓(2𝑘+1) < 𝐹(𝑃1 𝛼1 ) = ∑𝑟=0 𝑇3 (𝑃1 𝑟 ) = ∑𝑟=0(𝑟 + 1)3 =
𝑓(2𝑘+2) , 2𝑘 < 𝑓(2𝑘+1) < 𝑓(2𝑘+2) sum of the cubes of the first 𝛼1 + 1 natural
numbers,
(Because the function 𝑓(𝑛) = 𝑛 is true for all
2
even integer n). But only integer lying (𝛼1 + 1)(𝛼1 + 2)
=[ ]
between 2k and 2𝑘 + 2 𝑖𝑠 2𝑘 + 1,(since the 2
range of f is integer)
2
(𝛼1 + 1)(𝛼2 + 2)
Thus, 𝑓(𝑘+1) = 2𝑘 + 1, 𝑖. 𝑒. 𝑓(𝑚) = 𝑚, in the 𝑠𝑖𝑚𝑖𝑙𝑎𝑟𝑙𝑦, 𝐹(𝑃2 𝛼2 ) = [ ]
2
case of odd m also. Thus, 𝑓(𝑛) = 𝑛, for all n∊
N. 𝐹(𝑃1 𝛼1 . 𝑃2 𝛼2 ) = ∑ 𝑇3 (𝑃1 𝑟 . 𝑃2 𝑠 )
0≤𝑟≤𝛼1
∴𝑓(1983) = 1983. 0≤𝑟≤𝛼2
201
Challenging Mathematical Problems
𝛼1 𝛼2
The sum of the divisors of 2𝑛 𝑎𝑏 other than
3 (𝑠 3
= ∑ ∑(𝑟 + 1) + 1) 2𝑛 𝑎. 𝑏 is
𝑟=0 𝑠=0
𝛼1
9. 22𝑛−1 (2+1 − 1) − 2𝑛 (9. 22𝑛−1 − 9. 2𝑛−1 +
= ∑(𝑟 1)
𝑟=0
𝛼2 = 9. 23𝑛 − 9. 22𝑛−1 − 9. 23𝑛−1 + 9. 22𝑛−1 −
+ 1)3 (∑(𝑠 + 1)3 ) 2𝑛 = 9. 23𝑛−1 − 2𝑛
𝑠=0
= 2𝑛 (9. 22𝑛−1 − 1) = 2𝑛 . 𝑐.
𝛼1 2
(𝛼2 + 1)(𝛼2 + 2)
= ∑(𝑟 + 1) [ 3
] Thus the sum of the divisors of 2𝑛 . 𝑎𝑏 other
2 than itself is 2𝑛 . 𝑐.
𝑟=0
𝛼1
Now, sum of the divisors of 2𝑛 𝑐 other than
𝛼2 3
𝐹(𝑃2 ). ∑(𝑟 + 1) 2𝑛+1 −1 𝑐 2 −1
itself is × − 2𝑛 . 𝑐 = (2𝑛+1 −
𝑟=0 2−1 𝑐+1
3 𝑛
(𝛼1 + 1)(𝛼2 + 2) 1)(𝑐 + 1) − 2 . 𝑐
𝛼2
= 𝐹(𝑃2 )[ ]
2
= (2𝑛+1 − 1)9. 22𝑛−1 − 2𝑛 (9. 22𝑛−1 − 1)
= 𝐹(𝑃2 𝛼2 ) 𝐹(𝑃1 𝛼2 ) . Hence proved. = 9. 23𝑛 − 9. 22𝑛−1 + 2
= 2𝑛 [9. 22𝑛−1 − 9. 2𝑛−1 + 1]
21) If 𝒏𝟏 𝒂𝒏𝒅 𝒏𝟐 are two numbers, such that = 2𝑛 𝑎𝑏
the sum of all the divisors of 𝒏𝟏 other than
𝒏𝟏 is equal to sum of all the divisors of 𝒏𝟐 i.e. the sum of the divisors of 2𝑛 𝑐 other than
other than 𝒏𝟐 , then the pair (𝒏𝟏 , 𝒏𝟐 ) is 2𝑛 𝑐 equal to 2𝑛 𝑎𝑏.
called an anticable number pair. Given 𝒂 =
𝟑. 𝟐𝒏 − 𝟏, primes numbers, then shows
that (𝟐𝒏 𝒂𝒃, 𝟐𝒏 𝒄) is an anticable pair. 22) If n = 𝑷𝟏 , 𝑷𝟐 , P3 and P1, P2 and P3 are
distinct prime numbers. If ∑𝒅 𝒅 =
Sol.: If N = 𝑃1 𝛼1 , 𝑃2 𝛼2 , … , 𝑃𝑛 𝛼𝑛 , then sum of 𝒏
the divisors N is given by the formula. 𝟑𝑵 𝒐𝒓 𝑶𝒄(𝑵) = 𝟑𝑵, then show that
𝟏
∑𝑵
𝒊=𝟏 = 𝟑.
𝛼1 +1 𝛼2 +1 𝒅𝒊
𝑃1 − 1 𝑃2 −1
∑ 𝑑(𝑁) = ×
𝑃1 − 1 𝑃2 − 1 Sol.: The divisors of N are
𝑃𝑛 𝛼𝑛 +1 − 1 1, 𝑃1 , 𝑃2 , 𝑃3 , 𝑃1 𝑃2 , 𝑃1 𝑃3 , 𝑃2 𝑃3 , 𝑃1 𝑃2 𝑃3. It is given
×…..
𝑃𝑛 − 1 that
202
Challenging Mathematical Problems
𝑃1 𝑃2 𝑃3 + 𝑃2 𝑃3 + 𝑃1 𝑃3 + 𝑃1 𝑃2 + 𝑃3 + 𝑃2 + 𝑃1 + 1
=
𝑃1 𝑃2 𝑃3
24) All two digit numbers from 10 to 99 are
But the numerator is the sum of the divisors written consecutively, that is N= 101112
of N. ….. 99.
𝟑𝟐
Show that . From which other two digit
𝑖. 𝑒. ∑ 𝑑 = 3𝑁 = 3𝑃1 𝑃2 𝑃3 𝑎𝑛𝑑 ℎ𝑒𝑛𝑐𝑒, 𝑵
𝑑
number you should start so that N is
𝑛 divisible by (i) 3 and (ii) 𝟑𝟐 .
𝑁
1 3𝑃1 𝑃2 𝑃3 Sol.: N is divisible by 9, if the digit sum is
∑ = =3
𝑑𝑖 𝑃1 𝑃2 𝑃3 divisible by 9. The digit sum of N:
𝑖=1
23) Determine with proof all the arithmetic The number of 1𝑠 occurring in the digits from
program with integer terms with the 10 to 19 = 11 and from 20 to 99 = 8
property that for each positive integer n, So, that of ones is 11+8= 19. Similarly,
the sum of the first n terms is a perfect
square. No of 2𝑠 , 3𝑠 , … .9 are all equal to 19.
Sol.: When n = 1, the first term itself is a So, sum of all digits = 19(1 + 2 + 3 + ⋯ +
perfect square 19×9×10
9) = = 19 × 5 × 9 = 855 and hence,
2
203
Challenging Mathematical Problems
So, the number of digit used to write numbers If the difference between two perfect square
from 1 to 99 is 189. is 4, then one of them is 4 and the other is
zero. Therefore, 1991 − 𝑎 = ±2, (𝑏 − 𝑐)2 = 0.
Total number of digit used in writing up to n
is 1998. ⟹ 𝑎 = 1991 + 2 = 1993 𝑎𝑛𝑑 𝑏 − 𝑐
= 0 𝑜𝑟 𝑎 = 1991 − 2
The total no. of all three digits numbers = = 1989 𝑎𝑛𝑑 𝑏
(999 × 99) × 3 = 2700 > 1998. So, n should = 𝑐. 𝐵𝑢𝑡 𝑏 + 𝑐 = 2𝑏
be less than 999. = 1991 + 𝑎
No. of digits used to write the three digit = 1991
numbers up to N is 1998 − 189 = 1809. + 1993 𝑜𝑟 1991
+ 1989 ⟹ 𝑏 = 𝑐
In each 3 digit number, we use three digits. = 1992 𝑜𝑟 1990.
So, the number of three digits number in N = So, the only 2 values of a are 1993 and 1989
1809
= 603
3
So, therefore N = 703 − 1 = 702 27) Find the last two digit in (𝟓𝟔𝟕𝟖𝟗)𝟒𝟏
Since up to 702, there are 603, three digit Sol.: 56789 ≡ 89 (mod 100) =
numbers 90 two digit numbers and 9 one −11 (𝑚𝑜𝑑 100)
digit numbers.
∴ (56789)49 ≡ (−11)41 (𝑚𝑜𝑑 100)
≡ (−11)40
26) Find all integers values of a such that × (−11)(𝑚𝑜𝑑 100)
quadratic expressions (𝒙 + 𝒂)(𝒙 + ≡ (11)40 × (−11)(𝑚𝑜𝑑 100)
𝟏𝟗𝟗𝟏) + 𝟏 can be factored as (𝒙 + 𝒃)(𝒙 +
112 ≡ 21 (𝑚𝑜𝑑 100), 114 ≡ 21 ×
𝒄) where b and c are integers.
4 (𝑚𝑜𝑑 100), 116 ≡ 21 × 41 =
Sol.: (𝑥 + 𝑎)(𝑥 + 1991)+= (𝑥 + 𝑏)(𝑥 + 𝑐) 61(𝑚𝑜𝑑 100), 1110 ≡ 41 × 61 ≡
01(𝑚𝑜𝑑 100), 114 ≡ (01)40 ≡
⟹ 1991 + 𝑎 = 𝑏 + 𝑐 𝑎𝑛𝑑 1991𝑎 + 1 1(𝑚𝑜𝑑 100), (−11)41 ≡ 1140 ×
= 𝑏𝑐 (−11)(𝑚𝑜𝑑 100) ≡ 1 × (−11)(𝑚𝑜𝑑 100) ≡
−11 ≡ 89 (𝑚𝑜𝑑 100).
∴ (𝑏 − 𝑐)2 = (𝑏 + 𝑐)2 − 4𝑏𝑐
= (1991 + 9)2 That is the last two digits of (56789)41 are 8
− 4(1991𝑎 + 1) and 9 in that order.
= (1991
⏟ + 𝑎)2 − 4 × 1991𝑎 − 4
= (1991 − 𝑎)2 − 4
28) Prove that [𝒙] + [𝟐𝒙] + [𝟒𝒙] + [𝟖𝒙] +
2 2
𝑜𝑟 (1991 − 𝑎) − (𝑏 − 𝑐) = 4 [𝟏𝟔𝒙] + [𝟑𝟐𝒙] = 𝟏𝟐𝟑𝟒𝟓 has no solution.
204
Challenging Mathematical Problems
∴𝑥≥
12345
= 195
20
, when x = 196, the L H S = (33 )3 (1 + 32 )3 , 𝑝𝑟𝑜𝑣𝑖𝑑𝑒𝑑 3𝑛−9 −
63 21
of the given equation becomes 12348. 35 = 0 = (270)3 , Provided 3𝑛−9 =35
205
Challenging Mathematical Problems
Hence the highest power of 10 contained in Sol.: Since n is odd, we have n = 2m +1 for
1994! = 495. some positive integer m. Out of the integer 1,
2, …, n, there are m+ 1 odd ones namely 1, 3,
5, ……, (2m+1) and m even ones, namely 2, 4,
206
Challenging Mathematical Problems
6, ….2m. Consider the pairs Thus the possible values of n are 2, 3, 8, 24.
(𝑚1 , 1), (𝑚2 , 2), … , (𝑚𝑛 , 𝑛). By actual verification we find all these values
of n satisfy the given conditions.
Since there are m +1 odd integers among
𝑚1 , 𝑚2 , … , 𝑚𝑛 and only m even integers Thus n = 2, 3, 8, 24
among 1, 2, …, n therefore at least one of the
odd 𝑚𝑖 ′𝑠 must be paired with an odd i, 37) Determine the largest 3-digit prime factor
consequently, for some positive integer i, of the integer 𝟐𝟎𝟎𝟎𝑪𝟏𝟎𝟎𝟎 .
𝑚𝑖 − 𝑖 must be even, and therefore the 2000.1999….1001
product (𝑚1 − 1), (𝑚2 − 2), … , (𝑚𝑛 − 𝑛) Sol.: 2000𝐶1000 = 1.2.3….1000
must be even.
Every three digit prime is a factor of 1, 2, 3,
…., 1000. Also 2000𝐶1000 is an integer. So
every three digit prime occurs in the prime
36) Determine, with proof, all the positive factors of the denominator at least once. The
integers n for which (i) n is not the square greatest three digit prime factor of 2000𝐶1000
𝟑
of any integer and (ii) [√𝒏] 𝒅𝒊𝒗𝒊𝒅𝒆𝒔 𝒏𝟐 . is the one which occurs once in the
denominator and at least twice in the
Sol.: Suppose [√𝑛] = 𝑡. Since n is not the numerator.
square of any integer, therefore √𝑛 must lie
1
strictly between t and t +1 i.e. 𝑡 < √𝑛 < 𝑡 + Therefore it must be less than × 2000. i.e.
3
1. 𝑆𝑜 𝑡ℎ𝑎𝑡 𝑡 2 < 𝑛 < (𝑡 + 1)2 . less than 666 and as close to it as possible.
(Because then only will it occur twice in the
By hypothesis (ii), 𝑡 3 𝑑𝑖𝑣𝑖𝑑𝑒𝑠 𝑛2 . This implies
numerator and once in the denominator).
that 𝑡 2 must divide 𝑛2 , and consequently t Checking the numbers 666, 665, … for
must divide n. Now 𝑡 2 + 𝑡 𝑎𝑛𝑑 𝑡 2 + 2𝑡 are primality we find that 661 is the first prime in
the only positive integers lying between this sequences, which is the desired answer.
𝑡 2 𝑎𝑛𝑑 (𝑡 + 1)2 which are multiples of t.
Therefore we must have either 𝑛 = 𝑡 2 +
𝑡 𝑜𝑟 𝑛 = 𝑡 2 + 2𝑡.
38) Prove that any number N written in base 7
2 3 |𝑛 3 3 |(𝑡 2 2 will be even or odd according as the sum
Case I: 𝑛 = 𝑡 + 1, 𝑡 ⟹𝑡 + 𝑡) ⟹
𝑡 of its digits is even or odd.
(𝑡+1)2
Case II: 𝑛 = 𝑡 2 + 2𝑡, 𝑡 3 |𝑛2 ⟹ 𝑡 3 |(𝑡 3 + satisfy the given condition. Next, let p be a
𝑡 2𝑝−1 −1
2𝑡)2 ⟹ (𝑡+2)2 prime of the form 4k +1. Suppose ( 𝑝
) =
𝑚2 , for some odd integer m. It is obvious that
𝑡 m cannot be even. Then 24𝑘 − 1 =
⟹ ⟹ 𝑡 = 1, 2 𝑜𝑟 4 ⟹ 𝑛
4 (4𝑘 + 1)𝑚2 .
= 3, 8 𝑜𝑟 24.
Since every perfect square leaves a remainder
1 when divided by 4, therefore R. H. S. will
207
Challenging Mathematical Problems
leave a remainder 1 when divided by 4. But 39) Each of the positive integers 𝒂𝟏 , … , 𝒂𝒏 is
the L. H. S will leave a remainder 3 when less than 1951. The least common multiple
divided by 3. This is not possible. Therefore p of any two of these is greater than 1951.
cannot be of the form 4k +1. Let now p be of 𝟏 𝟏
Show that 𝒂 + ⋯ + 𝒂 < 2.
𝟏 𝒏
the form 4k +3. First consider the case p = 3
2𝑝−1 −1 23−1 −1
(i.e. k = 0)in this case ( )= ( )= Sol.: The numbers of integers from 1 to m,
𝑝 3
𝑚
1 which are multiple of b is [ 𝑏 ]. From the
assumption, we know that none of the
Which is a perfect square. Therefore p = 3 is integers 1, …, 1951 is simultaneously divisible
one of the primes t that we are looking for. by two of the numbers 𝑎1 , … , 𝑎𝑛 . Hence the
Let Now 𝑃 = 4𝑘 + 3, 𝑤𝑖𝑡ℎ 𝑘 > 0, 2𝑝−1 − 1 = number of integers 1, …., 1951 divisible by
19511951
24𝑘+2 − 1(22𝑘+1 − 1)(22𝑘+1 + 1). one of 𝑎1 , … , 𝑎𝑛 𝑖𝑠 [ ]+ ⋯+ [
]. This
𝑎1 𝑎𝑛
1951
number does not exceed 1951. Hence 𝑎 −
Since 22𝑘+1 − 1 𝑎𝑛𝑑 22𝑘+1 + 1 are relatively 1
1951 1951 1951
prime, therefore if their product is of from 1 + ⋯ + 𝑎 − 1 < 1951 𝑎 + ⋯ + 𝑎 <
𝑛 1 𝑛
𝑃𝑚2 , one of them must be 𝑃𝑢2 and the other
𝑛 + 1951 < 2.1951.
must be 𝑣 2 , where u and v are relatively
prime, Since 22𝑘+1 − 1 is of the form 4𝑠 + 1
+ ⋯+
1
< 2. This problem was used at
𝑎1 𝑎𝑛
3 𝑎𝑛𝑑 𝑣 2 must be of the from 4𝑠 + 1,
the MMO 1951. It is due to paul Erdos. The 2
therefore it follows that 22𝑘+1 − 1 cannot be 6
can be replaced by , but even this is not the
of the form 𝑣 2 . Therefore we must have 5
22𝑘+1 − 1 = 𝑃𝑢2 , 22𝑘+1 + 1 = 𝑣 2 . best possible bound.
208
Challenging Mathematical Problems
209
Challenging Mathematical Problems
210
Challenging Mathematical Problems
Since all (2𝑎𝑖 + 1) are odd, this clearly mplies Now write 𝑛 = ℎ𝑥 + 𝐾 𝑤𝑖𝑡ℎ 0 ≤ 𝑘 ≤ 𝑥.
that k must odd. We know that conversely,
given any odd k, can find 𝑎𝑖 . 𝑇ℎ𝑒𝑛 − 1 = (𝑝 − 1)𝑛 = (−1)𝑛 (𝑝 −
1)𝑘 (𝑚𝑜𝑑 𝑞). h cannot be even. Because then
We use a form of induction on k. First, it is true (𝑝 − 1)𝑘 = −1(𝑚𝑜𝑑 𝑞), contradicting the
for k = 1 (take n = 1). Second, we show that if it minimality of x. so h is odd and hence
is true for k, then it is true for 2𝑚 𝑘 − 1. That is (𝑝 − 1)𝑘 = 1 (𝑚𝑜𝑑 𝑞) with 0 ≤ 𝑘 < 𝑥 < 𝑦.
sufficient, since any odd number has the form This contradicts the minimality of y unless k = 0,
2𝑚 𝑘 − 1 for some smaller odd number k. Take so n = ℎ𝑥.
𝑎𝑖 = 2𝑖 [(2𝑚 − 1)𝑘 − 1]𝑓𝑜𝑟 𝑖 = 0, 1, … , 𝑚 − 1.
But x <q, so x = 1, so (𝑝 − 1) = −1 (mod q) p
𝑖+1 (2𝑚
𝑇ℎ𝑒𝑛 2𝑎𝑖 + 1 = 2 − 1)𝐾 − and q are primes so q = p as claimed.
(2𝑖+1 − 1)𝑎𝑛𝑑
So p is the smallest prime divisors of n. We are
𝑎𝑖 + 1 = 2𝑖 (2𝑚 − 1)𝐾 − (2𝑖 − 1). also given that 𝑥 < 2𝑝. so either p = n, or p = 2,
n = 4. The latter dues not work, so we have
So the product of the (2𝑎1 + 1)′𝑆 divided by shown that n = p Evidently n = p = 2 and n = p =
the product of the (𝑎𝑖 + 1)′𝑆 is 2𝑚 (2𝑚 − 3 work. Assume now that p > 3 we show that
1)𝐾 − (2𝑚 − 1)𝑑𝑖𝑣𝑖𝑑𝑒𝑑 𝑏𝑦 (2𝑚 − there are no solutions of this type.
(2𝑚 𝑘−1)
1)𝐾, 𝑜𝑟 .
𝑘 Expand (𝑝 − 1)𝑝 + 1 by the binomial theorem,
Thus if we take these 𝑎𝑖 𝑠 together with those to get (since (−1)𝑝 = −1):
giving k, we get 2𝑚 𝑘 − 1 which completes the 1
induction. 1 + −1 + 𝑝2 − 𝑝(𝑝
2
𝑝(𝑝 − 1)𝑝 − 2
− 1)𝑝2 .
6𝑝3
45) Find all pairs (n, p) of positive integers,
such that :P is prime; 𝒏 ≤ 𝟐𝒑; 𝒂𝒏𝒅 (𝒑 − 𝑇ℎ𝑒 𝑡𝑒𝑟𝑚𝑠 𝑜𝑓 𝑡ℎ𝑒 𝑓𝑟𝑜𝑚 𝑝𝑖 𝑤𝑖𝑡ℎ 𝑖 ≥ 3 are
𝟏)𝒏 + 𝟏 is divisible by 𝒏𝒑−𝟏 obviously divisible by 𝑝3 .
Sol.: Evidently (1, P) is a solution for every prime Since the binomial co-efficient by are integral.
p. Assume n >1 and take q to be the smallest Hence the sum is 𝑃2 + (𝑎 𝑚𝑢𝑙𝑡𝑖𝑝𝑙𝑒 𝑜𝑓 𝑃3 ). So
prime divisor of n. we first show that q = p. Let x the sum is not divisible by 𝑃3 . But for 𝑝 >
be the smallest positive integer for which 3, 𝑃𝑝−1 is divisible by 𝑝3 . So it cannot divide
(𝑝 − 1)𝑥 = 1 (𝑚𝑜𝑑 𝑞). Certainly y exists and (𝑝 − 1)𝑝 + 1, and there are no more solutions.
indeed y < q.
211
Challenging Mathematical Problems
212
Challenging Mathematical Problems
factors is divisible by 2𝑥−1 but not by 2𝑥 . So 𝑦 = the solution (m, n) = (2k, 1). So assume n > 1.
2𝑥+1 𝑚+𝑒 , 𝑚 𝑜𝑑𝑑, 𝑒 = ±1. Plugging this into 𝑚2
Put 𝑛 = 2𝑚𝑛2 −𝑛3 +1.
the original equation we obtain.
Then we have a quadratic equation for m,
2𝑥 (1 + 2𝑥+1 ) = (2𝑥−1 𝑚 + 𝑒)2 − 1 =
namely 𝑚2 − 2ℎ𝑛2 𝑚 + (𝑛3 − 1)ℎ = 0. This
22𝑥−2 𝑚2 + 2𝑥 me or equivalently, 1 + 2𝑥+1 =
has solutions ℎ𝑛2 ± 𝑁, where N is the positive
2𝑥−2 𝑚2 + 𝑚𝑒
square root of 𝑛2 𝑥 4 − ℎ𝑥 3 + ℎ. Since n >1, 𝑛 ≥
Therefore, 1 − 𝑒𝑚 = 2𝑥−2 (𝑚2 − 1, n is certainly real. But the sum and product of
8) … … … … . (𝑖𝑖)𝑓𝑜𝑟 𝑒 = 1 this yields 𝑚2 − 8 ≤ the roots are both positive, so both roots must
0 i.e. m = 1, which fails to satisfy (ii) For e = -1 be positive. The sum is an integer, so, if one
equation (ii) gives us. root is a positive integer, than so is the other.
1 + 𝑚 = 2𝑥−2 (𝑚2 − 8) ≥ 2 (𝑚2 − The larger root ℎ𝑛2 + 𝑁 is greater than ℎ𝑛2 , so
8)𝑖𝑚𝑝𝑙𝑦𝑖𝑛𝑔 ℎ(𝑛3 −1)
the smaller root < < 𝑛. But note that if
ℎ𝑛2
2𝑚2 − 𝑚 − 17 ≤ 0 ℎ𝑒𝑛𝑐𝑒 𝑚 ≤ 3; on the other 2𝑚 − 𝑛 > 0, then since n > 0, we must have
hand m cannot be 1 by (ii). Because m is odd . the denominator (2𝑚 − 𝑛)𝑛2 + 1 smallest than
these values indeed satisfy the given equation. the numerator and hence 𝑚 − 𝑛. So for the
Recall that then 𝑦 = −23 is also good. Thus we smallest root we cannot have 2𝑚 − 𝑛 > 0. But
have the complete list of solutions 2𝑚 − 𝑛 = 0 for the smaller root. Hence ℎ𝑛2 −
𝑛
(𝑥, 𝑦): (0, 2), (0, −2), (4, 23), (4, −23) 𝑁 = 2.
ℎ 2
Now 𝑁 2 = (ℎ𝑛2 − ) = ℎ2 𝑛4 − ℎ𝑛3 +
2
49) Let n be a positive integer and 𝑛2
ℎ, 𝑠𝑜 ℎ = 4
Thus n must be even, put n = 2k
𝑷𝟏 , 𝑷𝟐 , … , 𝑷𝒏 be n prime numbers all
and get the solutions (𝑚, 𝑛) =
larger than 5. Such that 𝑷𝟏 𝟐 + 𝑷𝟐 𝟐 + ⋯ + (𝑘, 2𝑘)𝑎𝑛𝑑 (8𝑘 4 − 𝑘, 2𝑘). We have shown that
𝑷𝒏 𝟐 is divisible by 6. Prove that 6 divides n. any solutions must be of one of the three forms
given, out it is trivial to check that they are all
Sol.: Through possible remainders when divided
indeed solutions.
by 6 are 0, 1, 2, 3, 4, 5, 𝑝𝑖 being prime 𝑝𝑖 2 will
have to leave only remainder 1 or 5. i.e. 𝑝𝑖 2 is of
the from 6𝑚𝑟 ± 1: so should be 𝑝𝑖, Hence 𝑝𝑖 2
is of the form 6𝑚𝑟 + 1: 𝑠𝑜 𝑝𝑖 2 + ⋯ + 𝑝𝑛 2 is QUADRATIC EQUATIONS
divisible by 6 only when n is divisible by 6. AND EXPRESSIONS
50) Find all pairs (m, n) of (+) integers such that (Objective Type)
𝒎𝟐
𝒊𝒔 (+) 𝒗𝒆 𝒊𝒏𝒕𝒆𝒈𝒆𝒓. 𝝅
𝟐𝒎𝒏𝟐 −𝒏𝟑 +𝟏 1) If 𝟎 < 𝛼 < 𝛽 < 𝛾 < 𝟐 , then equation
𝟏 𝟏 𝟏
Sol.: The denominator is 2𝑚𝑛2 − 𝑛3 + 1 = 𝒙−𝐬𝐢𝐧 𝜶
+
𝒙−𝐬𝐢𝐧 𝜷
+
𝒙−𝐬𝐢𝐧 𝜸
= 𝟎 has
𝑛2 (2𝑚 − 𝑛) + 1, 𝑠𝑜 2𝑚 ≥ 𝑛 > 0. 𝐼𝑓 𝑛 = 1, (a) Imaginary root, (b) real and equal
then m must be even, in other words, we have roots, (c) real and unequal root,
(d) rational roots.
213
Challenging Mathematical Problems
𝜋 1 1−3𝑡 2 1
Sol.: Since, 0 < 𝛼 < 𝛽 < 𝛾 < (𝑔𝑖𝑣𝑒𝑛) Case I: For 𝑦 = 3 ⟹ = 3 ⟹ 3 − 9𝑡 2 =
2 3−𝑡 2
3 − 𝑡2
⟹ sin 𝛼 < sin 𝛽 < sin 𝛾
⟹𝑡=0
Now the given equation is
1−3𝑡 2
(𝜒 − sin 𝛽)(𝜒 − sin 𝛾) Case II: For = 3 ⟹ 3−𝑡 2
= 3, ⟹ 1 − 3𝑡 2 =
+ (𝜒 − sin 𝛼)(𝜒 − sin 𝛾) 9 − 3𝑡 2
+ (𝜒 − sin 𝛼)(𝜒 − sin 𝛽) = 0
{not possible}
𝐿𝑒𝑡 𝑓(𝑥) = (𝜒 − sin 𝛽)(𝜒 − sin 𝛾)
∑𝒌−𝟏
𝒓=𝟎 𝒙
𝟐𝒓
+ (𝜒 − sin 𝛼)(𝜒 3) If is a polynomial in x for two
∑𝒌−𝟏
𝒓=𝟎 𝒙
𝒓
− sin 𝛾)
values p and q of k, then roots of equation,
+ (𝜒 − sin 𝛼)(𝜒
𝒙𝟐 + 𝒑𝒙 + 𝒒 = 𝟎 cannot be
− sin 𝛽) = 0
(a) Real; (b) positive (c) rational; (d)
⟹ 𝑓(sin 𝛼) = (sin 𝛼 − sin 𝛽)(sin 𝛼 − irrational
sin 𝛾) > 0
∑𝑘−1
𝑟=0 𝑥
2𝑟
Sol.: ∑𝑘−1 𝑟
is a polynomial in x. i.e.
⟹ 𝑓(sin 𝛽) = (sin 𝛽 − sin 𝛼)(sin 𝛽 − 𝑟=0 𝑥
sin 𝛾) < 0
{1 + 𝑥 2 + 𝑥 4 + ⋯ + 𝑥 2(𝑘−1) } is
⟹ 𝑓(sin 𝛾) = (sin 𝛾 − sin 𝛼)(sin 𝛾 − divisible by {1 + 𝑥 + 𝑥 2 + ⋯ + 𝑥 𝑘−1 }
sin 𝛽) > 0
1−𝑥2𝑘
( ) 1+𝑥 𝑘
1−𝑥2
Hence equation f(x) = 0 has once root between ⟹ 1−𝑥𝑘
= (𝑥 ≠ 1) is a polynomial in x if
( ) 1+𝑥
1−𝑥
sin 𝛼 𝑎𝑛𝑑 sin 𝛽 and other between
sin 𝛽 𝑎𝑛𝑑 sin 𝛾. and only if: (1 + 𝑥 𝑘 ) is divisible by (1 +
𝑥)𝑖𝑓 𝑥 = −1
𝟏
2) If 𝒚 = 𝐜𝐨𝐭 𝒙 𝐭𝐚𝐧 𝟑𝒙 , 𝒕𝒉𝒆𝒏
But it is not possible, because the equation is
𝟏 𝟏
(a) 𝒚 < 𝟑 𝒐𝒓 𝒚 > 3; (b) 𝟑 ≤ 𝒚 ≤ 𝟑; not defined for 𝑥 = −1.
𝟏 𝟏
(c) 𝒚 ≤ 𝟑 𝒐𝒓 𝒚 ≥ 𝟑; (iv) 𝟑 ≤ 𝒚 ≤ 𝟏
Hence, there are no real values of x for which
tan 𝑥 1−3𝑡𝑎𝑛2 𝑥 1−3𝑡 2
the equation is defined.
Sol.: 𝑦 = tan 3𝑥 = 3−𝑡𝑎𝑛2 𝑥
⟹𝑦= 3−𝑡 2
Thus, it can be easily said that the roots of the
2
∴ (𝑦 − 3)𝑡 + 1 − 3𝑦 = equation cannot be Rational.
0 𝑠𝑖𝑛𝑐𝑒 𝑡 𝑖𝑠 𝑟𝑒𝑎𝑙 ⟹ ∆≥ 0
214
Challenging Mathematical Problems
215
Challenging Mathematical Problems
𝜋
⟹ 𝑎𝑘 2 + 𝑏𝑘 + 𝑐 = 0 {∵ Sol.: Given ∫02 {𝑎2 (
cos 3𝑥 3
+ 4 cos 𝑥) + 𝑎 sin 𝑥 −
(1 + 𝑐𝑜𝑠 8 𝑥) ≠ 0} 4
𝑎2 sin 3𝑥 3
20 cos 𝑥} 𝑑𝑥 ≤ 3
⟹ {𝑎2 ( 4
+ 4 sin 𝑥) −
2
⟹ 𝑘 𝑖𝑠 𝑎 𝑟𝑜𝑜𝑡 𝑜𝑓 𝑎𝑥 + 𝑏𝑥 + 𝑐 = 𝜋
𝑎2 1
0 𝑖𝑛 (1, 2) 𝑎 cos 𝑥 − 20 sin 𝑥} | 2
0
≤ − ⟹ 𝑎2 (− +
3 12
3 𝑎2
7) If𝟐𝒂 + 𝟑𝒃 + 𝟔𝒄 = 𝟎 (𝒂, 𝒃, 𝒄 ∊ 𝑹) then the 4
)− 0 − 20 + 𝑎 ≤ 3
quadratic equation 𝒂𝒙𝟐 + 𝒃𝒙 + 𝒄 = 𝟎 has 2 𝑎2
(a) At least one in [0, 1]; (b) at least ⟹ 3 𝑎2 + 𝑎 − 20 + 3
≤ 0 ⟹ 𝑎2 +
one root in [2, 3]; (c) at least one 𝑎 − 20 ≤ 0
root in [4, 5]; (d) none
⟹ (𝑎 + 5)(𝑎 − 4) ≤ 0
𝑎 𝑏
Sol.: (A) Let 𝑓(𝑥) = 3 𝑥 3 + 2 𝑥 2 + 𝑐𝑥. We have Thus, a = 1, 2, 3, 4 {∵ a∊ I}
𝑎 𝑏 2𝑎+3𝑏+6𝑐
𝑓(0) = 0 𝑎𝑛𝑑 𝑓(1) = 3 + 2 + 𝑐 = 6
=
10) If (𝒙 − 𝒄) is a factor of order m of the
0
polynomial f(x) of degree n (𝟏 < 𝑚 < 𝑛),
(∵ 2𝑎 + 3𝑏 + 6𝑐 = 0). then x = c is a root of the polynomial
(a) 𝒇𝒎 (𝒙); (b) 𝒇𝒎−𝟏 (𝒙); (c) 𝒇𝒏 (𝒙);
Thus, 0 and 1 are two roots of f(x) = 0. So, (d) none
𝑓 ′ (𝑥) = 0 𝑖. 𝑒. 𝑎𝑥 2 + 𝑏𝑥 + 𝑐 = 0 has at least
one real root between 0 and 1. Sol.: since (𝑥 − 𝑐) is a factor of order m of the
polynomial 𝑓(𝑥)
8) If α, 𝛽 be the roots of the equation 𝟔𝒙𝟐 −
𝟏 ∴ 𝑓(𝑥) = (𝑥 − 𝑐)𝑚 𝜙(𝑥)
𝟔𝒙 + 𝟏 = 𝟎, 𝒕𝒉𝒆𝒏 (𝒂 + 𝒃𝒙 + 𝒄𝒙𝟐 +
𝟐
𝟏
𝒅𝒙𝟑 ) + 𝟐 (𝒂 + 𝒃𝜷 + 𝒄𝜷𝟐 + 𝒅𝜷𝟐 ) = Where 𝜙(x) is a polynomial of degree (𝑛 − 𝑚)
𝒅 𝒄 𝒃 𝒂
(a) 𝟏
+ 𝟐 + 𝟑 + 𝟏𝟐; (b) 𝟏𝟐𝒂 + 𝟔𝒃 + ⟹ 𝑓(𝑥), 𝑓 ′ (𝑥), … . , 𝑓 (𝑚−1) (𝑥) are all
𝒂 𝒃 𝒄 𝒅
𝟒𝒄 + 𝟗𝒅; (c) 𝟏 + 𝟐 + 𝟑 + 𝟒; (d) zero for x = c but 𝑓 𝑚 (𝑥) ≠ 0 𝑎𝑡 𝑥 = 𝑐.
none 11) The value of a for which one root of
1 (𝒂𝟐 − 𝟓𝒂 + 𝟑)𝒙𝟐 + 𝒙(𝟑𝒂 − 𝟏)𝒂 + 𝟐 = 𝟎
Sol.: Here 𝛼 + 𝛽 = 1, 𝛼𝛽 = 6. Thus the given
1
is twice as large as the other is
expression 2 {(𝑎 + 𝑏) + 𝑏(𝛼 + 𝛽) + 𝟏 𝟏 𝟐
(a) 𝟑; (b) − 𝟑; (c) 𝟑, (d) none
𝑎 𝑏 𝑐 𝑑
𝑐(𝛼 2 + 𝛽 2 ) + 𝑑(𝛼 3 + 𝛽 3 )} = + 1 2
+3 + 4
after calculation of values. Sol.: Let α and 2α be the two roots of the given
equation. So,
9) If a is a positive integer, the number of
3𝑎−1
values of a satisfying 𝛼 + 2𝛼 = − 𝑎2 −5𝑎+3 𝑎𝑛𝑑
𝝅
𝐜𝐨𝐬 𝟑𝒙 𝟑
∫𝟎𝟐 {𝒂𝟐 ( + 𝟒 𝐜𝐨𝐬 𝒙) + 𝒂 𝐬𝐢𝐧 𝒙 − 2
𝟒 2𝛼 2 = 𝑎2 −5𝑎+3 , ⟹ 𝛼 =
𝒂𝟐
𝟐𝟎 𝐜𝐨𝐬 𝒙} 𝒅𝒙 ≤ − , 3𝑎−1
𝟑 − 3(𝑎2 −5𝑎+3) 𝑎𝑛𝑑
(a) Only one; (b) two; (c) three; (d)
none
216
Challenging Mathematical Problems
⟹ ∆ = 𝑐 2 (3𝑎2 − 𝑏 2 + 4𝑎𝑏)2
13) If 𝒙 = (𝜷 − 𝜸)(𝜶 − 𝜹), 𝒚 = (𝜸 − 𝜶)(𝜷 − 16) If the equation 𝒙𝟐 − 𝟑𝒙𝒌 + 𝟐𝒆𝟐 𝐥𝐨𝐠 𝒌 − 𝟏 =
𝜹) 𝒛 = (𝜶 − 𝜹)(𝜸 − 𝜹) then the value of 𝟎 has real roots such that the product of
𝒙𝟑 + 𝒚𝟑 + 𝒛𝟑 − 𝟑𝒙𝒚𝒛 is roots is 7, then the value of k is
(a) 0; (b) 𝜶𝟔 + 𝜷𝟔 + 𝜸𝟔 + 𝜹𝟔 ; (c) (a) ±𝟏; (b) ±𝟐; (c) ±𝟑; (d) none
𝜶𝟔 𝜷𝟔 𝜸𝟔 𝜹𝟔 ; (d) none 2
Sol.: Since, 𝑒 2 log 𝑘 𝑒 log 𝑘 = 𝑘 2
217
Challenging Mathematical Problems
∴ The given equation is 𝑥 2 − 3𝑘𝑥 + 19) Both the root of the equation
(2𝑘 2 − 1) = 0 (𝒙 − 𝒃)(𝒙 − 𝒄) + (𝒙 − 𝒄)(𝒙 −
𝒂) + (𝒙 − 𝒂)(𝒙 + 𝒃) = 𝟎 are
Now, product of roots = 7 (given) always:
⟹2𝑘 2 − 1 = 7 ∴ 𝑘 = ±2 (a) Positive; (b) negative; (c) real; (d)
none
17) Product of real root of 𝒕𝟐 𝒙𝟐 + |𝒙| + 𝟗 = 𝟎,
(a) Is always positive; (b) is always Sol.: The given equation can be written as
negative; (c) does not exist; (d) 3𝑥 2 − 2𝑥(𝑎 + 𝑏 + 𝑐) + 𝑎𝑏 + 𝑏𝑐 + 𝑐𝑎 = 0.
none. 𝑁𝑜𝑤, ∆ 4(𝑎 + 𝑏 + 𝑐)2 − 12(𝑎𝑏 + 𝑏𝑐 + 𝑐𝑎)
Sol.: Since the equation 𝑡 2 𝑥 2 + |𝑥| + 9 = 0 is ⟹ ∆= 2 [(𝑏 − 𝑐)2 + (𝑐 − 𝑎)2 +
always positive for all x ∊ R. (𝑎 − 𝑏)2 ] ⟹ ∆≥ 0. Hence the roots are
real.
∴ The equation does not possess real root.
20) If x denotes the set of real number p for
18) If a, b, c are real and 𝒙𝟑 − 𝟑𝒃𝟐 𝒙 + 𝟐𝒄𝟑 is
which 𝒙𝟐 = 𝒑(𝒙 + 𝒑) has a its roots
divisible by (𝒙 − 𝒂)𝒂𝒏𝒅 (𝒙 − 𝒃), then
greater than P, then x is equal to
(a) 𝒂 = −𝒃 = −𝒄; (b) 𝒂 = 𝟐𝒃 = 𝟐𝒄;
𝟏 𝟏 𝟏
(c) 𝒂 = 𝒃 = 𝒄; 𝒐𝒓 𝒂 = −𝟐𝒃 = (a) (−𝟐, − ), (b) (− , ), (c) null
𝟐 𝟐 𝟒
−𝟐𝒄; (d) none set, (d) (−∞, 𝟎)
Sol.: Since, 𝑓(𝑥) = 𝑥 3 3𝑏 2 𝑥 + 2𝑐 3 is divisible by Sol.: Since the roots are greater than p, i.e. p
𝑥 − 𝑎 𝑎𝑛𝑑 𝑥 − 𝑏 ∴ 𝑓(𝑎) = 0 lies outside both the roots, such that 𝑎𝑓(𝑝) >
0
⟹ 𝑎3 − 3𝑏 2 𝑎 + 2𝑐 3 =
0 … … … . . (1)𝑎𝑛𝑑 𝑓(𝑏) = 0 ⟹ 1(𝑝2 − 2𝑝2 ) > 0 ⟹ 𝑝2 > 0 which
is impossible.
⟹ 𝑏 3 − 3𝑏 3 + 2𝑐 3 =
0 … … (2)𝑎𝑛𝑑 ⟹ −2𝑏3 + 2𝑐 3 = 0 𝐻𝑒𝑛𝑐𝑒, 𝑥 = {𝑝|𝑝 ∊ 𝜙}
∴ 𝑏 = 𝑐. 𝑃𝑢𝑡𝑡𝑖𝑛𝑔, 𝑏 = 𝑐 𝑖𝑛 (1), 𝑤𝑒 𝑔𝑒𝑡. 21) If a and b are rational and 𝛼, 𝛽 be the roots
of 𝒙𝟐 + 𝟐𝒂𝒙 + 𝒃 = 𝟎, then the equation
𝑎3 − 3𝑎𝑏 2 + 2𝑏 3 = 0 ⟹ (𝑎 − 𝑏)(𝑥 2 +
with rational coefficients are one of whose
𝑎𝑏 − 2𝑏 2 ) = 0
roots in 𝜶 + 𝜷 + √𝜶𝟐 + 𝜷𝟐 is
2 2
⟹ 𝑎 − 𝑏 𝑜𝑟 𝑎 + 𝑎𝑏 + 2𝑏 . 𝑇ℎ𝑢𝑠, 𝑎 = (a) 𝒙𝟐 + 𝟒𝒂𝒙 + 𝟐𝒃 = 𝟎; (b) 𝒙𝟐 +
𝑏=𝑐 𝟒𝒂𝒙 − 𝟐𝒃 = 𝟎; (c) 𝒙𝟐 − 𝟒𝒂𝒙 +
𝟐𝒃 = 𝟎; (d) 𝒙𝟐 − 𝟒𝒂𝒙 − 𝟐𝒃 = 𝟎
𝑜𝑟 𝑎2 + 𝑎𝑏 = 2𝑏 2 ⟹ 𝑎 = 𝑏 = 𝑐 𝑎𝑛𝑑
218
Challenging Mathematical Problems
219
Challenging Mathematical Problems
220
Challenging Mathematical Problems
𝑎 −𝑏 +𝑐
1
4) If 𝜷 + 𝒄𝒐𝒔𝟐 𝜶, 𝜷 +
(∏3𝑖=1 ( 𝑖 𝑎 𝑖 𝑖))
2
𝛽𝛾 + 𝛾𝛼 + 𝛼𝛽𝛾 = 𝒔𝒊𝒏𝟐 𝜶 𝒂𝒓𝒆 𝒕𝒉𝒆 𝒓𝒐𝒐𝒕𝒔 𝒐𝒇 𝒙𝟐 + 𝟐𝒃𝒙 + 𝒄 =
𝑖
(1 − 𝑝2 + 𝑝4 ) − 𝑖(𝑝3 − 𝑝1 ) = ⟹ 𝑏 2 − 𝐵2 = 𝑐 − 𝐶 𝑃𝑟𝑜𝑣𝑒𝑑
(−𝑖 − 𝑎)(−𝑖 − 𝑏)(−𝑖 − 𝑐)(−𝑖 −
𝑑) … … … . . (3)
221
Challenging Mathematical Problems
6) Suppose that 𝒂𝟏 > 𝒂𝟐 > 𝒂𝟑 > 𝒂𝟒 > 𝒂𝟓 > [𝑛(𝑛 − 1)(𝑛 − 2) … … … .3]𝑥 2 − [(𝑛 − 1)(𝑛 −
𝒂𝟔 and 𝒑 = 𝒂𝟏 + 𝒂𝟐 + 𝒂𝟑 + 𝒂𝟒 + 𝒂𝟓 + 2) … .2]
𝒂𝟔 , 𝒒 = 𝒂𝟏 𝒂𝟑 + 𝒂𝟑 𝒂𝟓 + 𝒂𝟓 𝒂𝟏 + 𝒂𝟐 𝒂𝟒 + 𝑎1 𝑥 + [(𝑛 − 2) … 1]𝑎𝑛 = 0
𝒂𝟒 𝒂𝟔 + 𝒂𝟔 𝒂𝟐 𝒂𝒏𝒅 𝒓 = 𝒂𝟏 𝒂𝟑 𝒂𝟓 + 𝒂𝟐 𝒂𝟒 𝒂𝟔 .
Then show that all the roots of the ⟹ 𝑛(𝑛 − 1)𝑥 2 − 2(𝑛 − 1)
equation 𝟐𝒙𝟑 − 𝒑𝒙𝟐 + 𝒒𝒙 − 𝒓 = 𝟎
𝑎1 𝑥 + 2𝑎2 = 0
are real
222
Challenging Mathematical Problems
= 𝑥⏟𝑛 + ⏟
𝑝1 𝑒𝑣𝑒𝑛 + ⋯ + 𝑝𝑛−1 𝑒𝑣𝑒𝑛 +
2𝑛𝑎2
(𝑎 2 − ) 𝑜𝑑𝑑 𝑒𝑣𝑒𝑛
1 √ 1 (𝑛 − 1) 𝑝1 + ⋯ + 𝑝𝑛−1
⏟
⟹ 𝑎1 − (𝑛 − 1) ≤𝛼
𝑛 (𝑛 − 1)2 𝑒𝑣𝑒𝑛
⎣ ⎦
⟹ 𝑥 = (2𝑚 + 1) cannot be a root of the
equation 𝑓(𝑥) = 0
2𝑛𝑎2
(𝑎 2 − )
1 √ 1 (𝑛 − 1)
≤ 𝑎1 + (𝑛 − 1) Therefore equation cannot have an integral
𝑛 (𝑛 − 1)2
root.
⎣ ⎦
223
Challenging Mathematical Problems
𝑏2 + 𝑑2 2 𝑏2 − 𝑑2 𝑖. 𝑒. , 𝑎2 𝑙𝑥 2 − 4𝑎𝑐𝑙𝑥 + 2𝑐 2 𝑙 + 𝑚𝑎2 = 0
𝑠𝑜, 𝑝2 = (𝑝 ≠ 2 𝑎𝑠 𝑝2
𝑏2 − 𝑑2 𝑏 − 𝑑2
𝑏2
It show one of the equation (i) is (positive)
𝑎2
≠ 1)
𝑏2 4𝑐 𝑏2 −4𝑎𝑐
Now, 𝑦 + 𝑎2 = 𝑎
⟹ 𝑦 = −( 𝑎2
) <0
𝑐−𝑎
𝐹𝑟𝑜𝑚 𝑒𝑞𝑢𝑎𝑡𝑖𝑜𝑛 (1)𝛼 =
2𝑎𝑝2 − 2𝑏 Hence roots are real and opposite in sign.
2𝑑
⟹𝛼= 11) If 𝐭𝐚𝐧 𝒙 − 𝐭𝐚𝐧 𝒚 = 𝒂 𝒂𝒏𝒅 𝒙 + 𝒚 = 𝟐𝒃,
(𝑏 2 + 𝑑2 )
2(𝑏 − 𝑑) 2 − 2ℎ prove that 𝐭𝐚𝐧 𝒛 𝒂𝒏𝒅 𝐭𝐚𝐧 𝒚 are the roots
𝑏 − 𝑑2
𝑏+𝑑 𝑐 of the equation 𝒙𝟐 − (𝟏 − 𝒂) 𝐭𝐚𝐧 𝟐𝒃 . 𝒙 +
= −( )= −
𝑏−𝑑 𝑎 𝒂=𝟎
Let second root of equation (2) be 𝛽 Sol.: Let tan 𝑥 , tan 𝑦 are the roots of the
equation 𝑥 2 − (1 − 𝑎) tan 2𝑏 . 𝑥 + 𝑎 = 0
224
Challenging Mathematical Problems
𝑥𝑝𝑛 (𝑥) = 𝑥 + 2𝑥 2 + ⋯ + 𝑛𝑥 4 +
(𝑛 + 1)𝑥 𝑛+1
⟹ (1 − 𝑥)𝑝𝑛 (𝑥) = 1 + 𝑥 + 𝑥 2 + 𝑥 3 +
⋯ + 𝑥 4 − (𝑛 + 1)𝑥 𝑛+1
1(1 − 𝑥 𝑛+1 ) If equation (2) not has two real roots, then
= − (𝑛 + 1)𝑥 𝑛+1
1−𝑥 equation (1) not has n (all) real roots.
1−(𝑛+2)𝑥 𝑛+1 +(𝑛+1)𝑥 𝑛+2
⟹ 𝑝𝑛 (𝑥) = (1−𝑥)2
for Equation (2) not has two real root if 𝐵2 − 4𝐴𝑐
negative values of x, 𝑝𝑛 (𝑥)will vanish (discriminant of (2) <0)
when ever
225
Challenging Mathematical Problems
7
= 2(𝑎 − 𝑑) + (𝑏 − 𝑑)2 + (𝑐 − For one root lie in (1, 3) , 𝑤𝑒 ℎ𝑎𝑠 1 <
𝑑)3 𝑏𝑒𝑐𝑜𝑚𝑒𝑠 𝑎≤3⟹5<𝑎≤
7 19
.
3
−2𝐷 + 𝑥𝐷 2 + (2𝐷)3 = −6𝐷 + 4𝐷 2 −
16) Let 𝒇(𝒙) be a polynomial leaving
𝐷3
remainder, 𝑨𝟏 , when divided by (𝒙 − 𝒂𝟏 ).
⟹ 9𝐷 2 + (𝑥 − 4)𝐷 + 4 = The remainder 𝑨𝟐 when divided by (𝒙 −
0 𝑠𝑖𝑛𝑐𝑒 𝐷 𝑖𝑠 𝑟𝑒𝑎𝑙 𝒂𝟐 )………. And finally
𝑨𝒎 , 𝒊𝒇 𝒅𝒊𝒗𝒊𝒅𝒆 𝒃𝒚 (𝒙 − 𝒂𝒎 ). Find the
⟹ (𝑥 − 4)2 − 4.9.4 ≥ 0 ⟹ 𝑥 2 − remainder left by the polynomial, when
8𝑥 − 128 ≥ 0
divided by (𝒙 − 𝒂𝟏 )(𝒙 − 𝒂𝟐 ) … . . (𝒙 − 𝒂𝒎 )
⟹ (𝑥 − 16)(𝑥 + 8) ≥ 0 ∴ 𝑥 ≥
Sol.: 𝑓(𝑎1 ) = 𝐴1 , 𝑓(𝑎2 ) = 𝐴2 , … … … . . , 𝑓(𝑎𝑚) =
16 𝑜𝑟 𝑥 ≤ −8 𝑝𝑟𝑜𝑣𝑒𝑑.
𝐴𝑚
15) Find the value of ‘a’ for which the equation
𝑓(𝑥) = (𝑥 − 𝑎1 )(𝑥 − 𝑎2 ) … . (𝑥 −
(𝒙𝟐 + 𝒙 + 𝟐)𝟐 − (𝒂 − 𝟑)(𝒙𝟐 + 𝒙 +
𝑎𝑚 )𝑄(𝑥) + 𝑅(𝑥) … … … . (𝑦)
𝟐)(𝒙𝟐 + 𝒙 + 𝟏) + (𝒂 − 𝟒)(𝒙𝟐 + 𝒙 + 𝟏)𝟐 =
0, has at least one real root.
226
Challenging Mathematical Problems
Here 𝑅(𝑥) will be a polynomial of degree 17) Given that 𝛼, 𝛾 are the roots of the
(𝑚 − 1) equation 𝑨𝒙𝟐 − 𝟒𝒙 + 𝟏 = 𝟎 and 𝛽, 𝛿 the
roots of the equation 𝑩𝒙𝟐 − 𝟔𝒙 + 𝟏 = 𝟎
𝑅(𝑎1 ) = 𝐴1 , 𝑅(𝑎2 ) = 𝐴2 , … , 𝑅(𝑎𝑚) =
find values of A and B such that 𝛼, 𝛽, 𝛾 and
𝐴𝑚
𝛿 are in H.P.
𝑅(𝑥) = 𝑝𝑚−1 − 𝑥 𝑚−1 + 𝑝𝑚−2 −
Sol.: Given equation are 𝐴𝑥 2 − 4𝑥 + 1 =
𝑥 𝑚−2 + ⋯ + 𝑝0 𝑎𝑛𝑑 , 0 … … … . (1)
𝑅(𝑥) = 𝐵1 (𝑥 − 𝑎2 )(𝑥 − 𝑎3 ) … (𝑥 −
𝐵𝑥 2 − 6𝑥 + 1 = 0 … … … … . . (2)
𝑎𝑚 ) + 𝐵2 (𝑥 − 𝑎1 )(𝑥 − 𝑎3 )(𝑥 − 𝑎𝑚 ) +
𝐵3 (𝑥 − 𝑎1 )(𝑥 − 𝑎2 )(𝑥 − 𝑎4 ) … + ⋯ + 4
𝛼+𝛾 = … … … (𝑖) 𝛽 + 𝛿
𝐵𝑚 (𝑥 − 𝑎1 )(𝑥 − 𝑎2 ) … (𝑥 − 𝐴
𝑎𝑚−1 ) … … . . (1) 6
= … . . (𝑖𝑖) 𝛼𝛾
𝐵
Putting x = 𝑎1 on both sides, we get 1
= … … … . . (𝑖𝑖𝑖) 𝛽𝛿
𝐴
𝑅(𝑎1 ) 1
= … … … (𝑖𝑣)
𝐴1 𝐵
=
(𝑎1 − 𝑎2 )(𝑎1 − 𝑎3 ) … (𝑎2 − 𝑎𝑚 )
Given 𝛼, 𝛽, 𝛾, 𝛿 are in H. p.
= 𝐵1
2𝛼𝛾 1 2𝛽𝛿 1
… … … … … …. ∴𝛽= = 𝑎𝑛𝑑 𝛾 = =
𝛼+𝛾 2 𝛽+𝛿 3
…………………
Since 𝛽 is a root of equation (2)
𝑆𝑖𝑚𝑖𝑙𝑎𝑟𝑙𝑦 𝐵2
∴ 𝐵𝛽 2 − 6𝛽 + 1 = 0
𝐴2
= 1 1
(𝑎2 − 𝑎1 )(𝑎2 − 𝑎3 ) … . (𝑎2 − 𝑎𝑚 ) ⟹ 𝐵 × − 6. + 1
4 2
=0 ⟹𝐵=8
……………………
Since 𝛾 is a root of equation (1) ∴ 𝐴𝛾 2 −
… … … … … … … .. 4 4
4𝛾 + 1 = 0, ⟹ 9 − 3 + 1 = 0 ⟹ 𝐴 = 3
𝐴𝑚
𝐵𝑚 =
(𝑎𝑚 − 𝑎1 )(𝑎𝑚 − 𝑎2 ) … . . (𝑎𝑚 − 𝑎𝑚−1 ) 18) If 𝛼, 𝛽 are the roots of the equations 𝒙𝟐 −
𝒑𝒙 + 𝒒 = 𝟎 then find the quadratic
∴ 𝑅(𝑥)
equation whose roots are
𝐴1 . (𝑥 − 𝑎2 )(𝑥 − 𝑎3 ) … (𝑥 − 𝑎𝑚 )
= (a) (𝜶𝟐 − 𝜷𝟐 )(𝜶𝟑 − 𝜷𝟑 )𝒂𝒏𝒅 𝜶𝟑 𝜷𝟐 +
(𝑎1 − 𝑎2 ) … . (𝑎1 − 𝑎𝑚 )
𝜶𝟐 𝜷𝟐
𝐴2 . − 𝑎1 )(𝑥 − 𝑎3 ) … . . (𝑥 − 𝑎𝑚 )
(𝑥
+ (b) (𝜶𝟐 + 𝜷𝟐 )(𝜶𝟑 + 𝜷𝟑 )𝒂𝒏𝒅 𝜶𝟓 𝜷𝟑 +
(𝑎2 − 𝑎1 )(𝑎2 − 𝑎3 ) … … (𝑎2 − 𝑎𝑚 )
+⋯ 𝜶𝟑 𝜷𝟓 − 𝟐𝜶𝟒 𝜷𝟒
Sol.:
𝑚 𝑚
𝑥 − 𝑎𝑘
⟹ 𝑅(𝑥) = ∑ 𝐴𝑖 (∏ ( )) (a) 𝛼, 𝛽 are roots of 𝑥 2 − 𝑝𝑥 + 𝑞 = 0
𝑎𝑖 − 𝑎𝑘
𝑖=1 𝑘=2
𝑘≠1
227
Challenging Mathematical Problems
228
Challenging Mathematical Problems
𝑁𝑜𝑤, 2𝛼 2 − 5𝛼 + 2 = (𝑎 + 2)(2𝛼 −
Denote a +b by c. We have shown that every
1) < 0 𝑓𝑜𝑟
integer fixed point of Q other that a and b is a
root of the polynomial. 𝐹(𝑥) = 𝑐 − 𝑥 − 𝑝(𝑥) . 1
2
< 𝛼 < 2. We have already seen that
This is of course true for a and b as well. And
1 < 𝛼 < 2.
since p has degree n >1, the polynomial F has
the same degree. S, it cannot have more than 𝐻𝑒𝑛𝑐𝑒 𝛼 6 < 4.
n roots. Hence the result.
𝐴𝑔𝑎𝑖𝑛 𝛼 4 − 𝛼 2 + 2𝛼 − 3 ≥ 0 is
𝟓 𝟑
20) If 𝛼 areal root of 𝒙 − 𝒙 + 𝒙 − 𝟐 = 𝟎, equivalent to
then show that [𝜶𝟔 ] = 𝟑 (for any real
number we denote by [x] the greatest 𝛼 5 − 𝛼 3 + 2𝛼 2 − 3𝛼 ≥ 0 is equivalent
integer not exceeding x) to
229
Challenging Mathematical Problems
Take a prime p, that divides both Let us suppose 𝐷2 < 0 and 𝐷3 < 0. In this
𝜆𝑛−1 𝑎𝑛𝑑 𝜆𝑛 . Then from A, we find that p case both the equation (2) and (3) have only
divides 𝜆𝑛−2 also. non –real roots but equation (1) has only real
roots. Hence, the common roots 𝛼 must be
Thus 𝜙 is a factor of gcd(𝜆𝑛−2 , 𝜆𝑛−1 ) a between (2) and (3).
contradiction, so gcd(𝜆𝑛−2 , 𝜆𝑛 ) is equal to 1.
But then the conjugate 𝛼 of 𝛼 is the other root
Hence we have gcd(𝜆, 𝜆𝑛+1 ) = 1 ∀ 𝑛 ≥ 0. of both (2) and (3).
22) If p, q, r be positive real numbers, but not Hence it follows that (2) and (3) have same
all equal such that two of the equations. set of roots
𝒑𝒙𝟐 + 𝟐𝒒𝒙 + 𝒓 = 𝟎, 𝒒𝒙𝟐 + 𝟐𝒓𝒙 +
𝒑 = 𝟎, 𝒓𝒙𝟐 + 𝟐𝒑𝒙 + 𝒒 = 𝟎 have a
common root say 𝛼. Show that
230
Challenging Mathematical Problems
𝑞 𝑟 𝑝 1 1
⟹
𝑟
=
𝑝
= . This p= q= r contradicting the
𝑞
𝑆𝑖𝑚𝑖𝑙𝑎𝑟𝑙𝑦 16
< 𝑏 2 (1 − 𝑐)(1 − 𝑎)𝑎𝑛𝑑 16 <
given condition. Hence 𝐷2 𝑎𝑛𝑑 𝐷3 cannot be 𝑐 2 (1 − 𝑎)(1 − 𝑏) Multiplying these we get,
negative we may assume 𝐷2 ≥ 0. 1
𝑎2 𝑏 2 𝑐 2 (1 − 𝑎)2 (1 − 𝑏)2 (1 − 𝑐)2 > 163
231
Challenging Mathematical Problems
25) If m, n integers ≥ 𝟎 𝒂𝒏𝒅 𝒇(𝒙) , 𝒈(𝒙) are Roots of this equation are the reciprocals of
polynomial such that (𝒙 − 𝒂)𝒎 𝒇(𝒙) = the roots of (2), ∴ Roots of (3) are
(𝒙 − 𝜶)𝒏 𝒈(𝒙) 𝒘𝒊𝒕𝒉 𝒇(𝒙) ≠ 𝟎, 𝒈(𝒙) ≠ 𝟎.
1 1 1
Show that 𝒎 = 𝒏 𝒂𝒏𝒅 𝒇(𝒙) = 𝒈(𝒙) . , ,
𝛼+𝛽 𝛽+𝛾 𝛾+𝛼
Sol.: Given (𝑥 − 𝑎)𝑚 𝑓(𝑥) = (𝑥 − 1 1 1
Now, 𝛼+𝛽 + 𝛽+𝛾 + 𝛾+𝛼= sum of roots of (3)
𝛼)𝑛 𝑔(𝑥) , 𝑓(𝛼) ≠ 0, 𝑔(𝛼) ≠ 0. We want to prove
that m=n and 𝑓(𝑥) = 𝑔(𝑥) . If possible let m≠n, −𝑎2 + 𝑏 𝑎2 + 𝑏
without loss of any generality. Let m>n. = =
𝑐 − 𝑎𝑏 𝑎𝑏 − 𝑐
∴ 𝑛 − 𝑚 is a +ve integer, so that 27) If 𝛼, 𝛽, 𝛾 are the roots of the cubic equation
𝒙𝟑 + 𝟑𝒙 + 𝟐 = 𝟎 from an equation whose
(𝑥 − 𝛼)𝑚 𝑓(𝑥) = (𝑥 − 𝛼)𝑛−𝑚 𝑔(𝑥)
roots are (𝜷 − 𝜸)𝟐 , (𝜸 − 𝜶)𝟐 , (𝜶 − 𝜷)𝟐 and
(𝑥 − 𝛼) hence show that 𝒙𝟑 + 𝟑𝒙 + 𝟐 = 𝟎 has
𝑖. 𝑒. ⟹ 𝛼 𝑖𝑠 𝑎 𝑟𝑜𝑜𝑡 𝑜𝑓 𝑓(𝑥) = 0 imaginary roots.
𝑓(𝑥)
𝑜𝑟, 𝑦 3 − 2𝑎𝑦 2 + (𝑎2 + 𝑏)𝑦 + (𝑐 − 𝑎𝑏) = Subtracting (2) from (1) (3 + 𝑦)𝑥 − 6 = 0
0 … … … . (2)
6
∴ 𝑥 = 3+𝑦. Putting this value of x in (1), we
Which is required equation. Its roots are 𝛼 +
1 get
𝛽, 𝛽 + 𝛾, 𝛾 + 𝛼. Changing into 𝑦 and
Multiplying by 𝑦 3 , we get (𝑐 − 𝑎𝑏)𝑦 3 + 6 3 6
( ) + 3. +2=0
(𝑎2 + 𝑏)𝑦 2 − 2𝑎𝑦 + 1 = 0 … … . . (3). 3+𝑦 3+𝑦
232
Challenging Mathematical Problems
216 + 18(3 + 𝑦)2 + 2(3 + 𝑦)2 = 0. 𝑦 3 + 29) If a, b, c ∊ R, a ≠ 0, then solve the system of
18𝑦 2 + 82𝑦 + 216 = 0. Which is the required equation: 𝒂𝒙𝟏 𝟐 + 𝒃𝒙𝟏 + 𝒄 = 𝒙𝟐 ; 𝒂 𝒙𝟐 𝟐 +
equation product of all its roots = −216 𝒃𝒙𝟐 + 𝒄 = 𝒙𝟑 ; … … 𝒂𝒙𝒏−𝟏 𝟐 + 𝒃𝒙𝒏−𝟏 + 𝒄 =
𝒙𝒏 𝒂𝒏𝒅 𝒂𝒙𝒏 𝟐 + 𝒃𝒙𝒏 + 𝒄 =
∴ (𝛼 − 𝛽)2 (𝛽 − 𝛾)2 (𝛾 − 𝛼)2 = −216
𝒙𝟏 𝒊𝒔 𝒏 𝒖𝒏𝒌𝒏𝒐𝒘𝒏𝒔 𝒙𝟏 , 𝒙𝟐 , 𝒙𝒏 𝒕𝒉𝒆𝒏
R H S being – 𝑣𝑒, one of the factors. (i) (𝒃 − 𝟏)𝟐 < 4𝒂𝒄;
(ii) (𝒃 − 𝟏)𝟐 = 𝟒𝒂𝒄;
∴ that will make all the three roots imaginary (iii) (𝒃 − 𝟏)𝟐 > 4𝒂𝒄
which is not possible. Every odd integer
equation with real coefficients has at least Sol.: Given system of equation can be written
one real root on the L H S (𝛼 − 𝛽)2 𝑖𝑠 − 𝑣𝑒. as
233
Challenging Mathematical Problems
−(𝑏 − 1) ± 0 𝑠𝑜 𝑥1 = 𝑥2 = ⋯ = 𝑥𝑛
𝑥𝑖 =
2𝑎 (1 − 𝑏) ± √(𝑏 − 1)2 − 4𝑎𝑐
1−𝑏 =
= [∵ (𝑏 − 1)2 2𝑎
2𝑎
= 4𝑎𝑐] 𝑤ℎ𝑒𝑛 𝑎 > 0 𝑤𝑒 𝑔𝑒𝑡
1−𝑏 𝑥1 = 𝑥2 = ⋯ = 𝑥𝑛
𝐻𝑒𝑛𝑐𝑒 𝑥1 = 𝑥2 = ⋯ = 𝑥𝑛 =
2𝑎 (1 − 𝑏) ± √(𝑏 − 1)2 − 4𝑎𝑐
=
Case 3: When (𝑏 − 1)2 > 4𝑎𝑐. Roots of 𝑎𝑥𝑖 2 + 2𝑎
(𝑏 − 1)𝑥𝑖 + 𝑐 = 0, are real and unequal. Let 𝛼
30) Let 𝑷(𝒙) = 𝟎 be a fifth degree polynomial
and 𝛽 be roots.
equation with integer coefficients that has
If 𝑎 < 0 ∀𝑥1 ∊ [𝛼, 𝛽] 𝑎 𝑥1 2 + (𝑏 − 1)𝑥1 + 𝑐 ≥ at least one integer root. If 𝑷(𝟐) =
0 𝟏𝟑 𝒂𝒏𝒅 𝑷(𝟏𝟎) = 𝟓. Compute a value of x
that must satisfy 𝑷(𝒙) = 𝟎.
𝑖. 𝑒. 𝑓(𝑥1 ) ≥ 0
Sol.: Let 𝑃(𝑥) ≡ (𝑥 − 2) 𝑞(𝑥) + 𝑃(2) , 𝑞(𝑥) would
𝑆𝑖𝑚𝑖𝑙𝑎𝑟𝑙𝑦 𝑓𝑜𝑟 𝑎𝑙𝑙 𝑥𝑖 ∊ [𝛼, 𝛽],
have integer coefficients.
(𝑖 = 1, 2, 3 … 𝑛)𝑖. 𝑒. 𝑓(𝑥1 ) ≥ 0
Let r be an integer such that 𝑃(𝑟) = 0
𝐵𝑢𝑡 𝑓(𝑥1 ) + 𝑓(𝑥2 ) + ⋯ + 𝑓(𝑥𝑛 ) = 0, 2
Then 𝑃(𝑟) = (𝑟 − 2). 𝑞(𝑟) + 13 = 0, 𝑆𝑜 𝑟 −
13
𝑓(𝑥1 ) = 𝑓(𝑥2 ) = ⋯ = 𝑓(𝑥𝑛 ) = 0
𝑇ℎ𝑢𝑠, 𝑟 − 2 can only equal ± 𝑜𝑟 ± 13. Leading
∴ 𝑥1 = 𝑥2 = ⋯ = 𝑥𝑛 to r = 3, 1, 15, or −11.
𝑡ℎ𝑒𝑛 𝑒𝑎𝑐ℎ 𝑎𝑥𝑖 2 + (𝑏 − 1)𝑥𝑖 + 𝑐 = 0 𝐿𝑒𝑡 𝑃(𝑥) ≡ (𝑥 = 10), 𝐹(𝑥) + 𝑃(10) . Leads to 𝑟 −
10
.
𝑆𝑜, 𝑥1 = 𝑥2 = ⋯ = 𝑥𝑛 5
= −(𝑏 − 1)
So r can only be 11, 9, 15 or 5. Thus, r = 15.
√(𝑏 − 1)2 − 4𝑎𝑐
±
2𝑎 31) If 𝒙𝟏 , 𝒙𝟐 , 𝒙𝟑 are the roots of 𝒙𝟑 − 𝒙𝟐 + 𝟒 =
𝟎, from the equation whose roots are 𝒙𝟏 +
𝐴𝑙𝑠𝑜, ∀ 𝑥1 ∉ (𝛼, 𝛽)(𝑖 = 1, 2, 3, … 𝑛)𝑖. 𝑒. 𝑓(𝑥𝑖 )
𝒙𝟐 𝟐 + 𝒙𝟑 𝟐 ; 𝒙𝟐 + 𝒙𝟑 𝟐 + 𝒙𝟏 𝟐 ; 𝒙𝟑 + 𝒙𝟏 𝟐 +
≤0
𝒙𝟐 𝟐
𝑏𝑢𝑡 𝑓(𝑥1 ) + 𝑓(𝑥2 ) + ⋯ + 𝑓(𝑥𝑛 ) = 0,
Sol.: 𝑥1 , 𝑥2 , 𝑥3 are the roots of equation
𝑠𝑜 𝑓(𝑥1 ) = 𝑓(𝑥2 ) = ⋯ = 𝑓(𝑥𝑛 ) = 0
𝑥 3 − 𝑥 2 + 4 = 0 … … … … (1)
∴ 𝑥1 + 𝑥2 + 𝑥3 = 1; 𝑥1 𝑥2 + 𝑥2 𝑥3 +
𝑥3 𝑥1 = 0;
234
Challenging Mathematical Problems
2𝑥1 𝑥2 𝑥3 6 3 6
= 𝑥1 + (1 − 𝑥1 )2 − ∴ [− ] + 3 [− ]+2=0
𝑥1 𝑧+6 𝑧+6
8
= 𝑥1 + (1 − 𝑥1 )2 + (𝑧 + 6)3 − 9(𝑧 + 6)2 − 108
𝑥1
= 0, 𝑧 3 + 9𝑧 2 − 216 = 0
8
∴ 𝑦 = 𝑥 + (1 − 𝑥)2 + 𝑙𝑒𝑡 𝑧1 , 𝑧2 , 𝑧3 be the roots of above equation
𝑥
8 then;
= 𝑥2 − 𝑥 + 1 +
𝑥
𝑧1 , 𝑧2 , 𝑧3 = (𝛼 − 𝛽)(𝛼 − 𝛾) (𝛽 − 𝛾)(𝛽 −
3 2 𝛼) (𝛾 − 𝑎)(𝛾 − 𝛽) = 216
𝑜𝑟 𝑥 − 𝑥 + 𝑥 − 𝑥𝑦 + 8 = 0 … … (2)
Putting this value of x in (1), we get Hence, one of the factors in R H S must be
64 16 −𝑣𝑒 say (𝛼 − 𝛽)2 𝑖𝑠 − 𝑣𝑒 𝑖. 𝑒. 𝛼 − 𝛽 =pure
3
− +4=0 imaginary showing that 𝛼 and 𝛽 are conjugate
(𝑦 − 1) (𝑦 − 1)2
complex. Hence, the given equation has two
⟹ (𝑦 − 1)3 − 4(𝑦 − 1) + 16 = 0 imaginary roots.
235
Challenging Mathematical Problems
4 4 (2𝑎 + 𝑏)(𝑥 − 1) + (𝑎 + 𝑏 + 𝑐) = 2𝑥 +
∴𝛾−1= 𝑜𝑟 𝛼 =
𝛼 𝛾−1 1
236
Challenging Mathematical Problems
(𝑤𝑦 + 𝑦)𝑛 − (𝑤𝑦)𝑛 − 𝑦 𝑛 = 𝑦 𝑛 [(𝑤 + 1)𝑛 − 38) Find out at what values of p and q where
𝑤 𝑛 − 1] 𝒙𝟒 + 𝟏 is divisible by 𝒑𝟐 + 𝒑𝒙 + 𝒒?
237
Challenging Mathematical Problems
39) Show that if 𝒂𝟏 , 𝒂𝟐 , … , 𝒂𝒏 are all distinct, ∵ 𝑓(𝑥) . 𝑔(𝑥) 𝑖𝑠 𝑎 𝑑𝑒𝑔𝑟𝑒𝑒 𝑜𝑓 2𝑛, 𝑝(𝑥) . 𝑞(𝑥) must
then the polynomial (𝒙 − 𝒂𝟏 )𝟐 (𝒙 − be both constants. Suppose 𝑝(𝑥) = 𝑎, 𝑞(𝑥) = 𝑏
𝒂𝟐 )𝟐 … . (𝒙 − 𝒂𝒏 )𝟐 + 𝟏 can never be
𝑇ℎ𝑒𝑛 𝑓(𝑥) = 𝑎(𝑥 − 𝑎1 )(𝑥 − 𝑎2 ) … . (𝑥 − 𝑎𝑛 ) +
written as the product of two polynomials
1
with integer coefficients.
𝑔(𝑥) = 𝑏(𝑥 − 𝑎1 )(𝑥 − 𝑎2 ) … . (𝑥 − 𝑎𝑛 ) + 1
Sol.: Suppose that there exists polynomial
𝑓(𝑥) . 𝑔(𝑥) with integer coefficients such that (substituting these conditions these
conditions imply 𝑎2 = −1, 𝑏 2 = −1)
𝑓(𝑥) . 𝑔(𝑥) = (𝑥 − 𝑎1 )2 (𝑥 −
𝑎2 )2 … . (𝑥 − 𝑎𝑛 )2 + 1 … … … . (1) ∴ There is a contradiction and given
polynomial cannot be expressed as the
∴ R H S is always +ve.
product of two polynomials with integer
∴ 𝑓(𝑥) can never vanish. coefficients.
∴𝑓(𝑥) , 𝑔(𝑥) are always +ve, so 𝑓(𝑥) 𝑎𝑛𝑑 𝑔(𝑥) are Sol.: Let A = 𝛼 + 𝛼 3 + 𝛼 4 + 𝛼 −4 + 𝛼 −3 + 𝛼 −1
both always +ve.
= 𝛼 + 𝛼 3 + 𝛼 4 + 𝛼 9 + 𝛼 10 +
Substituting 𝑥 = 𝑎1 , 𝑎2 , … , 𝑎𝑛 in (1) we get 𝛼 12 (∵ 𝛼 13 = 1)
238
Challenging Mathematical Problems
𝐴 × 𝐵 = (𝛼 + 𝛼 3 + 𝛼 4 + 𝛼 9 + 𝛼 10 + 𝛼 12 ) × = 𝑎 + 𝑏 + 𝑐 + 𝑑 = (𝑎 + 𝑏 + 𝑐 + 𝑑) −
(𝛼 2 + 𝛼 5 + 𝛼 6 + 𝛼 7 + 𝛼 8 + 𝛼 11 ) 2𝑑 = 𝑝(1) − 2𝑝(0) ≤ 1 + 2 = 3 < 7
239
Challenging Mathematical Problems
44) Find all polynomials 𝒇(𝒙) with real 𝑓(2𝑢−1) 𝑣 − (2𝑢 − 1)2𝑛 𝑣 2𝑛 + (2𝑢 −
coefficients which satisfy the equality 1)2𝑛−2 𝑣 2𝑛−2 … ….
𝒇(𝒂−𝒃) + 𝒇(𝒃−𝒄) + 𝒇(𝒄−𝒂) = 𝒇(𝒂 + 𝒃𝒄)∀
real numbers a, b, c such that 𝒂𝒃 + 𝒃𝒄 + 𝑓(1−𝑢2 ) 𝑣 = (1 − 𝑢2 )2𝑛 𝑣 2𝑛 + (1 −
𝒄𝒂 = 𝟎 𝑢2 )2𝑛−2 𝑣 2𝑛−2 … …
Sol.: Let 𝑓(𝑥) be a polynomial which satisfy the 𝑓(𝑢2 −2𝑢) 𝑣 = (𝑢2 − 2𝑢)2𝑛 + 𝑣 2𝑛 + ⋯
equation if a = b = 0
𝑓(𝑢2 −𝑢+1) 𝑣 = (𝑢2 − 𝑢 + 1)2𝑛 𝑣 2𝑛 + ⋯
Then 𝑎𝑏 + 𝑏𝑐 + 𝑐𝑎 = 0 𝑓𝑜𝑟 𝑒𝑎𝑐ℎ 𝑐 ∊
𝑅 𝑠𝑜 𝑤𝑒 𝑔𝑒𝑡 𝑓(0−0) + 𝑓(0−𝑐) + 𝑓(𝑐−0) = How equality the leading coefficients of both
27(0 + 0 + 𝑐), ∀ 𝑐 ∊ 𝑅 sides of (1), we get (2𝑢 − 1)2𝑛 +
(1 − 𝑢2 )2𝑛 + (𝑢2 − 2𝑢)2𝑛 = 2(𝑢2 − 𝑢 +
⟹ 𝑓(0) + 𝑓(−𝑐) + 𝑓(𝑐) = 27(𝑐) , ∀ 𝐶 ∊ 𝑅 1)2𝑛 ∀ 𝑢 ∈ 𝑅
⟹ 𝑓(0) + 𝑓(−𝑐) = 𝑓(𝑐) ∀ 𝐶 ∊ 𝑅 𝐿𝑒𝑡 𝑐 = 𝐿𝑒𝑡 𝑢 = −2, 𝑡ℎ𝑒𝑛 (−5)2𝑛 + (−3)2𝑛 + (8)2𝑛 =
0 𝑤𝑒 𝑔𝑒𝑡 𝑓(0) = 0 2(7)2𝑛
Hence, f is even which must be of the from Now above result is true only for x = 1 and x
=2
𝑓(𝑥) = 𝑎𝑛 𝑥 2𝑛 + 𝑎𝑛−1 𝑥 2𝑛−2 + ⋯ +
𝑎1 𝑥 2 𝑤𝑖𝑡ℎ 𝑎1 , 𝑎2 , 𝑎3 , … … . , 𝑎𝑛 ∊ 𝑅 ∴𝑓(𝑥) is either 𝛼𝑛2 𝑓𝑜𝑟 𝑛 = 1. 𝑜𝑟 𝛽𝑥 2 𝑓𝑜𝑟 𝑛 =
2
For any real number u and v the triplet (a, b,
c) will satisfy 𝑎𝑏 + 𝑏𝑐 + 𝑐𝑎. Hence 𝑓(𝑥) can be written as a linear
combination i.e. 𝑓(𝑥) = 𝛼𝑥 2 + 𝛽𝑥 4
𝐿𝑒𝑡 𝑎 = 𝑢𝑣, 𝑏 = (1 − 𝑢)𝑣, 𝑐 = (𝑢2 − 𝑢)𝑣
Hence, 𝑓(𝑥) = 𝛼𝑥 2 + 𝛽𝑥 4 is a polynomial
∴ 𝑎𝑏 + 𝑏𝑐 + 𝑐𝑎 𝑤𝑖𝑙𝑙 𝑏𝑒𝑐𝑚𝑒 (𝑎 + 𝑏)𝑐 + 𝑎𝑏
which satisfies the given equation for 𝛼, 𝛽 ∊ R
= 𝑣 + (𝑢2 − 𝑢)𝑣 + 𝑢𝑣(1 − 𝑢)𝑣 = 𝑣 2 +
45) If 𝛼+ 𝛽+ 𝛾= 0, then show that𝜶𝒏+𝟑 +
(𝑢2 − 𝑢) + 𝑣 2 (𝑢 − 𝑢2 ) = 𝑣 2 + (𝑢2 − 𝑢) +
𝜷𝒏+𝟑 + 𝜸𝒏+𝟑 = 𝜶𝜷𝜸(𝜶𝒏 + 𝜷𝒏 + 𝜸𝒏 ) +
𝑣 2 (𝑢2 − 𝑢) = 0 𝟏
𝟐
(𝜶𝟐 + 𝜷𝟐 + 𝜸𝟐 )(𝜶𝒏+𝟏 + 𝜷𝒏+𝟏 + 𝜸𝒏+𝟏 )
∴The given equation results
Sol.: 𝛼+ 𝛽+ 𝛾 = 0 i.e., Let 𝛼, 𝛽, 𝛾 denote the
𝑓[(2𝑢 − 1)𝑣] + 𝑓[(1 − 𝑢2 )𝑣] + 𝑓[(𝑢2 − 2𝑢)𝑣] roots of equation 𝑥 3 + 𝑞𝑦 + 𝑟 = 0 𝑖. 𝑒. 𝑥 =
= 27(𝑣 2 − 𝑢 + 1)𝑢∀ 𝑢, 𝑣 ∊ 𝑅 … … … . . (1) 𝛼 𝑜𝑟 𝛽 𝑜𝑟 𝛾
240
Challenging Mathematical Problems
0 … … … … (1),
The given equation becomes |𝑎| + |𝑏| =
𝛽 𝑛+3 + 𝑞𝛽 𝑛+1 + 𝑟𝛽 𝑛 =
|𝑎 + 𝑏|.
0 … … … … (2),
But his equality holds if 𝑎𝑏 ≥ 0
𝛾 𝑛+3 + 𝑞𝛾 𝑛+1 + 𝑟𝛾 𝑛 =
0 … … … … (3) 𝑥2
∴ (𝑥−1) ≥ 0 critical points are 0, 1
Adding (1), (2) and (3)
∴ 𝑥 ∊ {0} ∪ (1, ∞)
𝛼 𝑛+3 + 𝛽 𝑛+3 + 𝛾 𝑛+3
+ 𝑞(𝛼 𝑛+1 + 𝛽 𝑛+1 + 𝛾 𝑛+1 ) 2) The number of solutions of the equations
+ 𝑟(𝛼 𝑛 + 𝛽 𝑛 + 𝛾 𝑛 ) 𝟏! + 𝟐! + 𝟑! + ⋯ + (𝒙 − 𝟏)! + 𝒙! =
= 0 … … . (4) 𝒌𝟐 𝒂𝒏𝒅 𝒌 ∊ 𝑰are
(a) 2; (b) 3; (c) 4; (d) none
But by 𝑥 3 + 𝑞𝑥 + 𝑟 = 0, we have𝛼𝛽 + 𝛽𝛾 +
𝛾𝛼 = 𝑞, 𝛼𝛽𝛾 = −𝑟. Sol.: The given equation is 1! +2! +3!
+…+(𝑥 − 1)! 𝑥! = 𝑘 2
𝑖. 𝑒., ∑ 𝛼𝛽 = 𝛼𝛽 + 𝛽𝛾 + 𝛾𝛼
We can readily check that for x < 4 the given
1 equation has the only solutions = 1, 𝑘 =
= (2𝛼𝛽 + 2𝛽𝛾 + 2𝛾𝛼)
2 ±1 𝑎𝑛𝑑 𝑥 = 3, 𝑘 ± 3.
1
= [(𝛼 + 𝛽 + 𝛾)2 − (𝛼 2 + 𝛽 2 + 𝛾 2 )] Now let us prove that there are no solutions
2
1 for 𝑥 ≥ 4.
= − (𝛼 2 + 𝛽 2 + 𝛾 2 )[∵ 𝛼 + 𝛽
2 1! + 2! + 3! + 4! = 33
+ 𝛾 = 0] 1! + 2! + 3! … + 5! = 153
The expressions }
1! + 2! + ⋯ + 6! = 873
∴ (4)𝑔𝑖𝑣𝑒𝑠 𝛼 𝑛+3 + 𝛽 𝑛+3 + 𝛾 𝑛+3 1! + 2! + ⋯ + 7! = 5913
= −𝑞(𝛼 𝑛+1 + 𝛽 𝑛+1 + 𝛾 𝑛+1 )
− 𝑟(𝛼 𝑛 + 𝛽 𝑛 + 𝛾 𝑛 ) End with the digit 3.
241
Challenging Mathematical Problems
5) The range of values of a for which all the 𝑐𝑙𝑒𝑎𝑟𝑙𝑦, 0 < 𝑚(𝑏) ≤ 1. [∵ 𝑏 2
roots of the equation ≥ 0 𝑚𝑎𝑥. 𝑣𝑎𝑙𝑢𝑒 𝑜𝑓 𝑚(𝑏) = 1]
𝟐 𝟐
(𝒂 − 𝟏)(𝟏 + 𝒙 + 𝒙 ) = (𝒂 + 𝟏)(𝟏 +
𝒙𝟐 + 𝒙𝟒 ) are imaginary is
(a) (𝟐, ∞) ; (b) (−∞, −𝟐]; (c) −𝟐 < 7) If 𝒚 = 𝟐[𝒙] + 𝟑 = 𝟑[𝒙 − 𝟐] +
𝑎 < 2; (d) none 𝟓, 𝒕𝒉𝒆𝒏 [𝒚 + 𝒚] 𝒊𝒔 [𝒙] denotes the integral
part of x
Sol.: (1 + 𝑥 + 𝑥 2 )[(𝑎 − 1)(1 + 𝑥 + 𝑥 2 ) −
(a) 10; (b) 11; (c) 12; (d)none
(𝑎 + 1)(1 − 𝑥 + 𝑥 2 )] = 0
Sol.: ∵ 𝑦 = 2[𝑥] + 3 = 3[𝑥 − 2] + 5 … … … . (1)
(1 + 𝑥 + 𝑥 2 ) = 0 has imaginary roots
𝑜𝑟, 2[𝑥] + 3 = 3[𝑥 − 2] + 5 ⟹ 2[𝑥] +
⟹ −2(1 + 𝑥 2 ) + 2𝑎𝑥 = 0, must have 3 = 3{[𝑥] − 2} + 5
imaginary roots
⟹ 2[𝑥]3 = 3[𝑥] − 6 + 5 ⟹ [𝑥] =
⟹ 𝑥 2 − 𝑎𝑥 + 1 = 0, must have 4 𝑓𝑟𝑜𝑚 (1)
imaginary root.
𝑦 = 2, 4 + 3, 𝑦 = 11
⟹ 𝑎2 − 4 < 0, ⟹ −2 < 𝑎 < 2.
∴4≤𝑥<5
242
Challenging Mathematical Problems
∴ 𝑓(1) . 𝑓(3) < 0, (6 + 2𝑎)(4𝑎 + 48) < 0, Sol.: 5{𝑥} = 𝑥 + [𝑥] … … … . . (1)
1
(𝑎 + 3)(𝑎 + 12) < 0 [𝑥] − {𝑥} = … … … . (2)
2
243
Challenging Mathematical Problems
∴ 𝑥 = [𝑥] + {𝑥} … … … . . (3) from (1) & (3) 14) Number of solutions of 𝟑|𝒙| = |𝟐 − |𝒙|| is
we get (a) 0; (b) 2; (c) 4; (d) none
3𝑥 = 2 − 𝑥, 0 ≤ 𝑥 ≤ 2 = 𝑥 − 2,
13) Let F(x) be a function defined by 𝑭(𝒙) =
𝒙 − [𝒙], 𝑹 where [x] is the greatest integer 2 ≤ 𝑥 < ∞ 𝑎𝑡 𝑥 = 2, 3𝑥 − 𝑥 + 2 = 9
less than or equal to x. Then the number of
𝟏 (as 3𝑥 − 𝑥 + 2 is an increasing function for
solutions of F(𝒙) + 𝑭 ( 𝒙) = 𝟏 is
x>2)
(a) 0; (b) 1; (c) 2; (d) none
1 1
⟹ 𝑥 − [𝑥] + −[ ]= 1
𝑥 𝑥
𝑥+1 1
⟹( ) − ([𝑥] + [ ]) = 1 … … . . (1)
𝑥 𝑥
1 1
⟹𝑥+ = [𝑥] + [ ] ≠ 1
𝑥 𝑥
244
Challenging Mathematical Problems
16) The number of integral roots of the ∴ [𝛼] + [𝛽] + [𝛾] = −3 − 1 + 1 = −3.
equation 18) If S be the solution of the equation (𝒙)𝟐 +
[𝒙]𝟐 = (𝒙 − 𝟏)𝟐 [𝒙 + 𝟏], where (x) = least
√(𝒙 + 𝟑) − 𝟒√𝒙 − 𝟏 + integer, [x] = greatest integer, R= real
numbers, Z = integer, N = natural
√(𝒙 + 𝟖) − 𝟔√𝒙 − 𝟏 = 𝟏 is numbers, then
(a) 𝑺 = 𝑹; (b) 𝑺 = 𝑹 = −𝒁; (c) 𝑺 = 𝑹 =
(a) 1; (b) 2; (c) 3; (d) none −𝑵; (d) none
Sol.: Taking √𝑥 − 1 = 𝑡(𝑡 ≥ 0) the equation Sol.: Here (x)= least integer ≥ 𝑥 𝑎𝑛𝑑 [𝑥] =
reduces to √𝑡 2 + 4 − 4𝑡 + √𝑡 2 − 6𝑡 + 9 = 1 greatest integer ≤ 𝑥, so (𝑥) − [𝑥] = 1, if x is
not integer and [x] = (x) if x ∊z.
⟹ |𝑡 − 2| + |𝑡 − 3| = 1
Now, (𝑥 − 1) = (𝑥) − 1, [𝑥 + 1] = [𝑥] + 1, 𝑠𝑜,
It is necessary for t to satisfy 2 ≤ 𝑡 ≤ 3.
(𝑥)2 + [𝑥]2 = (𝑥 − 1)2 (𝑥 + 1)2 ,
∴ 2 ≤ −√𝑥 − 1 ≤ 3 ⟹ 4 ≤ (𝑥 − 1) ≤ 9 ⟹
5 ≤ 𝑥 ≤ 10 ⟹ (𝑥)2 + [𝑥]2 = (𝑥)2 − 2(𝑥) + 1 + [𝑥]2 +
2[𝑥] + 1
245
Challenging Mathematical Problems
(i) All of x, y and z are even which is 𝐻𝑒𝑛𝑐𝑒, 𝑥 = −(𝑎 + 𝑏 + 𝑐), 𝑦 = 𝑎𝑏 + 𝑎𝑐 + 𝑏𝑐,
not possible since in that case 16
𝑧 = −𝑎𝑏𝑐 is solution of our system.
divides each term of (and hence
the whole of) the left hand side
while 16 does not divide 24.
(ii) Two of x, y and z are odd and one 2) Solve: 𝒙𝟏 + 𝒙𝟐 = 𝒂𝟏 ; 𝒙𝟐 + 𝒙𝟑 = 𝒂𝟐 ; 𝒙𝟑 +
of them is even say, x is even. 𝒙𝟒 = 𝒂𝟑 ; … . 𝒙𝒏−𝟏 + 𝒙𝒏 = 𝒂𝒏−𝟏 𝒙𝒏 +
𝒙𝟏 = 𝒂𝒏
We have that: 𝑥 4 + 𝑦 4 + 𝑧 4 − 2𝑥 2 𝑦 2 −
2𝑦 2 𝑧 2 − 2𝑧 2 𝑥 2 = 𝑥 4 − 2𝑥 2 (𝑦 2 + 𝑧 2 ) + Sol.: we have 𝑥2 = 𝑎1 − 𝑥1 , 𝑥3 = 𝑎2 − 𝑥2 =
(𝑧 2 − 𝑦 2 )2 𝑎2 − 𝑎1 + 𝑥1 𝑥4 = 𝑎3 − 𝑥3 = 𝑎3 − 𝑎2 +
𝑎1 = 𝑥1 … ..
= 𝑥 4 − 2𝑥 2 (𝑦 2 + 𝑧 2 ) +
(𝑧 − 𝑦)2 (𝑧 + 𝑦)2 𝑥𝑛 = 𝑎𝑛−1 − 𝑎𝑛−2 + ⋯ ± 𝑎2 ± 𝑎1 ± 𝑥1
Here again 16 divides each term since y and z It should be noted that in the last quality the
are odd. upper signs will occur when n is odd and let
the lower signs when n is even.
Thus in either case the equation has no
solutions in integers. Consider the two cases separately.
246
Challenging Mathematical Problems
247
Challenging Mathematical Problems
248
Challenging Mathematical Problems
249
Challenging Mathematical Problems
5
𝑆𝑢𝑏𝑠𝑡𝑖𝑡𝑢𝑡𝑒 𝑥 = 𝑦 + 𝑖𝑛(1), we get √𝒙 + 𝟐√𝒙 + 𝟐√𝒙 + ⋯ + 𝟐√𝒙 + 𝟐√𝟑𝒙
24
3 1 1 3
(𝑦 + ) (𝑦 + ) (𝑦 − ) (𝑦 − ) =𝒙
24 24 24 24
5
= Sol.: The given equation
12.6.4.3
2
1 2 2
3 2 5 √𝑥 + 2√𝑥 + 2√𝑥 + ⋯ + 2√𝑥 + 2√𝑥 + 2 =
(𝑦 − ( ) ) (𝑦 − ( ) ) =
24 24 12.6.4.3
49 7 7 𝑥 … . . (1)
𝑆𝑜, 𝑦 2 = 𝑖. 𝑒. 𝑦1 = 𝑎𝑛𝑑 𝑦 2
= − ,
242 24 24
On replacing the last letter x on the L H S of
1 1 equation (1) by the value of x, expressed by
𝑐𝑜𝑟𝑟𝑒𝑠𝑝𝑜𝑛𝑑𝑖𝑛𝑔 𝑟𝑜𝑜𝑡𝑠 𝑎𝑟𝑒 − 𝑎𝑛𝑑 .
12 2 (1)we obtain
250
Challenging Mathematical Problems
251
Challenging Mathematical Problems
assumes the from 𝑦 − 3 − 3𝑦 − 2𝑦 2 − 4𝑦 − Thus we have to find all values of a for which
2 − 1 ≥ 0 𝑖. 𝑒. −2𝑦 2 − 6𝑦 − 6 > = 0. system (3) has no less than four different
integer solutions.
This inequality has no solutions. Substituting
1
the value y = 0 into the equation of given Let us compare the numbers – 𝑎 𝑎𝑛𝑑 − 𝑎2, we
system, we obtain 1 1
find their difference, − 𝑎2 − (−𝑎) = − − 𝑎2 +
𝑦=0 𝑎 3 −1
⟹{ 2 𝑎= 𝑎2
𝑥 − 8𝑥 + 12 = 0
𝑦=0
⟹{ (𝑎−1)𝑎2 +𝑎+1
(𝑥 − 2)(𝑥 − 6) = 0 = 𝑎2
𝑠𝑖𝑛𝑐𝑒 𝑎2 + 𝑎 + 1 > 0 for any a,
252
Challenging Mathematical Problems
√3
⟹ <𝑎<0 16) Find out whether the system of equations
3
𝒙 + 𝒚 = 𝟎 𝒙𝟐 + 𝒚𝟐 = 𝟎 𝒂𝒏𝒅 𝐬𝐢𝐧(𝒙 + 𝒚) =
−√3
𝑖𝑓 ≤ 𝑎 < 0, then the equation has no less 𝟎, 𝒙𝟐 + 𝒚𝟐 = 𝒃 are equivalent for (i) b = 2
3
than four integer solutions. (e) if a = -1, then and (ii) b = 5.
the interval [-1, 1] contains only three Sol.: It is clear that both for b = 2 and for b =
integers i.e. conditions of the problem are not 5 the second system is a consequence of the
satisfied. (f) if a <-1, then −1 < −𝑎−2 < 0, first. Since the equation sin(𝑥 + 𝑦) = 0 is a
and for the interval [−𝑎−2 , −𝑎] to contain no consequence of the equation 𝑥 + 𝑦 = 0
less then four, it is necessary that the
inequality −𝑎 ≥ 3 hold true, i.e. the inequality The first system has solutions
𝑎 ≤ −3 be valid. Thus, for 𝑎 ≤ −3, the given (1, −1), (−1, 1)𝑤ℎ𝑒𝑛 𝑏 =
equation has no less than four integer 5 5 5 5
2 𝑎𝑛𝑑 [√ , −√ ] , [√ , √ ] 𝑤ℎ𝑒𝑛 𝑏 = 5
solutions. 2 2 2 2
Combining all the results, we get the set of Let us find the set of solutions of the second
required values of the number a namely the system.
√3 1
interval (−∞, −3)𝑎𝑛𝑑 𝑡ℎ𝑒 𝑖𝑛𝑡𝑒𝑟𝑣𝑎𝑙 [− , ].
2 2 From its first equation we have 𝑥 + 𝑦 =
𝑛𝜋(𝑛 ∊ 𝐼)𝑎𝑛𝑑, consequently, it is equivalent
to the collection of system
15) Solve: 𝐥𝐨𝐠 (𝟐−𝒙) (𝟐 − 𝒚) > 0 𝑥 + 𝑦 = 𝑛𝜋
{ 2 … … … … (1)
𝑎𝑛𝑑 𝐥𝐨𝐠 (𝟒−𝒚)(𝟐𝒙 − 𝟐) > 0 𝑥 + 𝑦 2 = 𝑏, 𝑛 ∊ 𝐼
253
Challenging Mathematical Problems
1
𝑡 2 + 𝑛𝜋𝑡 + 2 (𝑛2 𝜋 2 − 𝑏) = 0, 𝑛 ∊ 𝐼. ⟹ |𝑥| = log12(1 + √1 − 𝑎) ⟹ 𝑥 =
Discriminant 𝐷 ≥ 0 𝑖. 𝑒. , 𝑛2 𝜋 2 − ± log12(1 + √1 − 𝑎)
2𝑏
2(𝑛2 𝜋 2 − 𝑏) ≥ 0 i.e. when 𝑥 2 ≤ .
𝜋2 Whenever a < 1.
It follows that for b = 2 the collection of 18) Solve: 𝒙𝟐 𝟐𝒙+𝟏 + 𝟐|𝒙−𝟑|+𝟐 = 𝒙𝟐 . 𝟐|𝒙−𝟑|+𝟒 +
system (2) has a solutions only for n = 0 and 𝟐𝒙−𝟏
for b = 5 it has a solution for n = -1, n = 0, n
= 1. Thus, for b = 2 these systems are Sol.: We consider two cases according as 𝑥 ≥
equivalent. 3 𝑜𝑟 𝑥 < 3. As we know, if 𝑥 ≥ 3, 𝑡ℎ𝑒𝑛 [𝑥 −
3] = 𝑥 − 3
Comparing the sets of solutions for b = 5, we
find that the initial system are not equivalent. Case I : 𝐿𝑒𝑡 𝑥 ≥ 3. 𝑇ℎ𝑒𝑛 |𝑥 − 3| = (𝑥 − 3).
17) Solve the equation (𝟏𝟒𝟒)|𝒙| − 𝟐(𝟏𝟐)|𝒙| + Hence the given equations becomes
𝒂 = 𝟎 for every value of the parameter a.
𝑥 2 . 2𝑥+1 + 2𝑥−3+2 = 𝑥 2 . 2𝑥−3+4 + 2𝑥−1
Sol.: The given equation can be written as
⟹ 𝑥 2 . 2𝑥+1 + 2𝑥−1 = 𝑥 2 . 2𝑥+1 + 2𝑥−1 which
(12)2|𝑥| − 2(12)|𝑥| + 𝑎 = 0. Let us write 𝑦 =
is satisfied for every x.
(12)2|𝑥| .
Hence the given equations is satisfied for
Then the above equations becomes 𝑦 2 − 2𝑦 +
every 𝑥 ≥ 3
𝑎=0
Case II : 𝐿𝑒𝑡 𝑥 < 3, 𝑡ℎ𝑒𝑛 |𝑥 − 3| = −(𝑥 − 3).
2 ± √4 − 4𝑎
⟹𝑦= ⟹𝑦
2 Hence the given equations becomes,
= 1 ± √1 − 𝑎 … … . (1)
𝑥 2 . 2𝑥+1 + 2−(𝑥−3)+2 = 𝑥 2 . 2−(𝑥−3)+4 + 2𝑥−1
The equation (1) is valid if 1 − 𝑎 > 0 𝑖. 𝑒. 𝑎 <
1. ⟹ 𝑥 2 . 2𝑥+1 + 25−𝑥 = 𝑥 2 . 27−𝑥 + 2𝑥−1
But the equation (3) is unacceptable, since 𝑎𝑛𝑑 2𝑥−1 − 25−𝑥 = 0 … … … … . (2)
12|𝑥| > 1.
1 1
(1) ⟹ 4𝑥 2 = 1 ⟹ 𝑥 2 = ⟹ 𝑥 = 1
4 2
Hence, we have 12|𝑥| = 1 + √1 − 𝑎
(2) ⟹ 2𝑥−1 = 25−𝑥 ⟹ 𝑥 − 1 = 5 − 𝑥 ⟹
2𝑥 = 6 ⟹ 𝑥 = 3
254
Challenging Mathematical Problems
Sol.: As we know, if the log function log 𝑎 𝑥 is The first inequality is reduced to 0 < 25 −
meaning ful then its base a should be > 0 and 𝑥 2 𝑎𝑛𝑑 25 − 𝑥 2 < 16 𝑖. 𝑒. 9 < 𝑥 2 < 25. But
a ≠ 1.Also x > 0 therefore we shall discuss 9 < 𝑥 2 ⟹ 𝑥 2 − 9 > 0 ⟹ (𝑥 − 3)(𝑥 + 3) > 0
25−𝑥 2
two cases according as the 0 < 16
<
⟹ 𝑥 < −3 𝑜𝑟 𝑥 > 3 𝑎𝑛𝑑 𝑥 2 < 25 ⟹
25−𝑥 2
1 𝑎𝑛𝑑 > 1. 𝑥 2 − 25 < 0 ⟹ (𝑥 − 5)(𝑥 + 5) < 0
16
⟹ (𝑥 + 17)(𝑥 − 1) < 0 ⟹ −17 < 𝑥 < Hence combining the two cases we have the
1 … … … (2) solution of the original inequality which
consists of two intervals : −3 < 𝑥 <
∴ Taking (1) and (2) together we find that 1 𝑎𝑛𝑑 3 < 𝑥 < 4.
−3 < 𝑥 < 1 ………….(3)
255
Challenging Mathematical Problems
√ 𝟐 𝟑 𝑛+1 4 1 4
𝟏 𝐱 −𝟐𝐱 +𝟏 𝟏 𝟏−𝐱 Sol.: we have ( ) = (1 + ) ≤
20) Solve : (𝟐) < (𝟐) 𝑛 𝑛
1 4 3 4
(1 + 2) = (2) < 10
Sol.: Since the base of the exponential
1
inequality (i.e. 2) is less than 1, hence the [∵ 𝑛 ≥ 2] ⟹ (𝑛 + 1)4 ≤ 𝑛4 . 10 <
original inequality is equivalent to the 10𝑛 . 10 = 10𝑛+1
inequality. √𝑥 6 − 2𝑥 3 + 1 > 1 − 𝑥
INEQUALITIES 1 1 1 1 1 1
⟹ + ≥ + = >
𝑎 + 𝑏 𝑎 + 𝑐 2𝑎 2𝑎 𝑑 𝑏 + 𝑐
(OBJECTIVE TYPE)
[∵ 𝑎 < 𝑏 < 𝑐] ∴ it represents a triangle.
1) If 𝒏𝟒 < 𝟏𝟎𝒏 for a fixed positive integer
3) The product of three positive reals is 1 and
𝒏 ≥ 𝟐, 𝒕𝒉𝒆𝒏
their sum is greater then sum of their
(a) (𝒏 + 𝟏) < 𝟏𝟎𝒏+𝟏 ; (b) (𝒏 + 𝟏)𝒏 ≥ reciprocals. Exactly one of them is greater
𝟏𝟎𝒏+𝟏 ; (c) 𝒏𝟒 + 𝟏 < 𝟏𝟎𝒏+𝟏 ; (d) than
none (a) -1; (b) 0; (c) 1; (d) none
1
Sol.: Let three positive reals be a, b and ,
𝑎𝑏
256
Challenging Mathematical Problems
1 1 1 (d) 𝒏𝒐𝒏𝒆
We are given 𝑎 + 𝑏 + 𝑎𝑏 > 𝑎 + 𝑏 +
𝑎𝑏 … … . . (1) Sol.: We have, 𝑎2 + 𝑏 2 + 𝑐 2 − 𝑏𝑐 − 𝑐𝑎 − 𝑎𝑏 =
1
1 2
𝑁𝑜𝑤 (𝑎 − 1)(𝑏 − 1) ( − 1)
𝑎𝑏
1 [(𝑏 2 + 𝑐 2 − 2𝑏𝑐) + (𝑐 2 + 𝑎2 − 2𝑐𝑎) +
= 1 + (𝑎 + 𝑏 + ) 1
𝑎𝑏 (𝑎2 + 𝑏 2 − 2𝑎𝑏) = [(𝑏 − 𝑐)2 +
2
1 1
− (𝑎𝑏 + + )−1 (𝑐 − 𝑎)2 + (𝑎 − 𝑏)2 ] ≥ 0
𝑎 𝑏
1
= (𝑎 + 𝑏 + ) 𝐴𝑙𝑠𝑜, 𝑎2 + 𝑏 2 + 𝑐 2 − 𝑏𝑐 − 𝑐𝑎 − 𝑎𝑏
𝑎𝑏 3
1 1 − (𝑏 − 𝑐)2
− (𝑎𝑏 + + ) > 0 4
𝑎 𝑏
1
Using (1)⟹ either all these 𝑎 − 1, 𝑏 − = [4𝑎2 + 4𝑏 2 + 4𝑐 2 − 4𝑏𝑐 − 4𝑐𝑎 − 4𝑎𝑏
1
4
1 𝑎𝑛𝑑 𝑎𝑏 − 1 are positive or exactly one of − 3(𝑏 2 + 𝑐 2 − 2𝑏𝑐)]
them is positive. 1
= [4𝑎2 + 𝑏 2 + 𝑐 2 + 2𝑏𝑐
4
1
But 𝑎 > 1, 𝑏 > 1 𝑎𝑛𝑑 𝑎𝑏 > 1. Thus exactly one − 4𝑎(𝑐 + 𝑏)]
1
of 𝑎, 𝑏, 𝑎𝑏 exceed 1. 1
= 4 [4𝑎2 + (𝑏 + 𝑐)2 − 4𝑎(𝑏 + 𝑐)] =
1
[2𝑎 − (𝑏 + 𝑐)]2 ≥ 0
4
4) If 𝒂 + 𝒃 + 𝒄 = 𝟔, 𝒕𝒉𝒆𝒏 √𝟒𝒂 + 𝟏 +
𝑆𝑖𝑚𝑖𝑙𝑎𝑟𝑙𝑦, 𝑎2 + 𝑏 2 + 𝑐 2 − 𝑏𝑐 − 𝑐𝑎 − 𝑎𝑏
√𝟒𝒃 + 𝟏 + √𝟒𝒄 + 𝟏 is
√3
(a) ≤ 𝟗; (b) > 9; (c) < 9; (d) none ≥ |𝑐 − 𝑎|𝑎𝑛𝑑 𝑎2 + 𝑏 2 + 𝑐 2
2
Sol.: By the Cauchy Schwarz inequality, √3
− 𝑏𝑐 − 𝑐𝑎 − 𝑎𝑏 ≥ |𝑎 − 𝑏|
2
2
(√4𝑎 + 1 + √4𝑏 + 1 + √4𝑐 + 1) ≤
⟹ 𝑎2 + 𝑏 2 + 𝑐 2 − 𝑎𝑏 − 𝑏𝑐 − 𝑐𝑎
(1 + 1 + 1)
√3
≥ max{|𝑏 − 𝑐|, |𝑐
(4𝑎 + 1 + 4𝑏 + 1 + 4𝑐 + 1) = 2
3[4(𝑎 + 𝑏 + 𝑐) + 3] − 𝑎|, |𝑎 − 𝑏|}
257
Challenging Mathematical Problems
𝑥 −1 + 𝑦 −1 + 𝑧 −1
𝐴𝑙𝑠𝑜, ( )
3
𝒙𝟐 𝑥 + 𝑦 + 𝑧 −1
7) If x ∊ R and y = then ≥ ( )
(𝟏+𝒙𝟒 )
𝟏 3
(a) 𝟎 ≤ 𝒚 ≤ 𝟐; (b) 𝟎 ≤ 𝒚 ≤ 𝟏; (c) 𝟎 ≤
𝒚 ≤ 𝟐; (d) none 2 −1
⟹ 𝑥 −1 + 𝑦 −1 + 𝑧 −1 ≥ 3. ( )
3
Sol.: 𝑦 + 𝑦𝑥 4 = 𝑥 2 , 𝑤ℎ𝑒𝑟𝑒 𝑦 ≥ 0, 𝑦𝑥 4 − 𝑥 2 +
9
𝑦=0 ⟹ 𝑥 −1 + 𝑦 −1 + 𝑧 −1 ≥
2
1±√1−4𝑦 2
𝑥2 = 2
, for this exists. 1 − 4𝑦 2 = 0
(𝑥 + 𝑦) ≥ 2√𝑥𝑦.
258
Challenging Mathematical Problems
(𝑝 − 𝑎) + (𝑝 − 𝑏) + (𝑝 − 𝑐) tan 𝛽 tan 𝛼
𝐴𝑙𝑠𝑜, ⟹ > 𝑜𝑟 𝛼 tan 𝛽 > 𝛽 tan 𝛼.
3 𝛽 𝛼
≥ [(𝑝 − 𝑎)(𝑝 − 𝑏)(𝑝
1
− 𝑐)]3
13) If x, y, z are real, distinct and 𝒖 = 𝒙𝟐 +
3𝑝 − (𝑎 + 𝑏 + 𝑐)
⟹ 𝟒𝒚𝟐 + 𝟗𝒛𝟐 − 𝟔𝒚𝒛 − 𝟑𝒛𝒙 − 𝟐𝒙𝒚, 𝒕𝒉𝒆𝒏 𝒖 is
3
(a) Zero; (b) non-negative; (c) non-
≥ [(𝑝 − 𝑎)(𝑝 − 𝑏)(𝑝
1 positive; (d) none
− 𝑐)]3
1
Sol.: 𝑢 = 2 {2𝑥 2 + 8𝑦 2 + 18𝑧 2 − 12𝑦𝑧 −
2𝑝 1
⟹ ≥ [(𝑝 − 𝑎)(𝑝 − 𝑏)(𝑝 − 𝑐)]3 1
6𝑧𝑥 − 4𝑥𝑦} = {(𝑥 − 2𝑦)2 + (2𝑦 − 3𝑧)2 +
3 2
(3𝑧 − 𝑥)2 }
8𝑝3
⟹ (𝑝 − 𝑎)(𝑝 − 𝑏)(𝑝 − 𝑐) ≤
27 ⟹𝑢≥0
⟹ (𝑐)𝑑𝑜𝑒𝑠 𝑛𝑜𝑡 ℎ𝑜𝑙𝑑𝑠
1 𝑏𝑐 𝑐𝑎 𝑏𝑐 𝑐𝑎
𝐴𝑔𝑎𝑖𝑛, 𝑠𝑖𝑛𝑐𝑒 ( + ) ≥ √( . ) 𝑒𝑡𝑐 14) The minimum value of p = 𝒃𝒄𝒙 + 𝒄𝒂𝒚 +
2 𝑎 𝑏 𝑎 𝑏
𝒂𝒃𝒛 when 𝒙𝒚𝒛 = 𝒂𝒃𝒄, 𝒊𝒔
(a) 𝒂𝒃𝒄; (b) 𝟒𝒂𝒃𝒄; (c) 𝟓 𝒂𝒃𝒄; (d)none
∴Adding the inequalities, we get
𝑏𝑐𝑥+𝑐𝑎𝑦+𝑎𝑏𝑧
𝑏𝑐 𝑐𝑎 𝑎𝑏 Sol.: 𝐴. 𝑀 ≥ 𝐺. 𝑀 ⟹ 3
≥
+ + ≥ 𝑎+𝑏+𝑐 = 𝑝 1
𝑎 𝑏 𝑐 (𝑎2 𝑏 2 𝑐 2 , 𝑥𝑦𝑧)
3
tan 𝑥
= 𝑖𝑠 𝑖𝑛𝑐𝑟𝑒𝑎𝑠𝑖𝑛𝑔 2 1
𝑥 ⟹ ≥ 𝑀2 , ⟹ 𝑀 ≤ 1.
2
∴ 𝑓(𝛽) > 𝑓(𝛼) 𝑎𝑠 𝛼 < 𝛽.
As a, b, c, d > 0. So, M = (𝑎 + 𝑏) × (𝑐 + 𝑑) > 0
𝑖. 𝑒. , 0 ≤ 𝑀 ≤ 1.
259
Challenging Mathematical Problems
𝑛−1
𝑥𝑖+1 − 𝑥𝑖 𝑥𝑛 − 𝑥1
√2 ∑ sin ≤ (𝑛 − 1)√2 sin
(SUBJECTIVE TYPE) 2 2(𝑛 − 1)
𝑖=1
≤ √2(𝑛 − 1)
1) If−𝟏 ≤ 𝒂𝟏 ≤ 𝒂𝟐 ≤ ⋯ ≤ 𝒂𝒏 ≤ 𝟏, prove
that 𝜋
sin . 𝑆𝑖𝑛𝑐𝑒 𝑥𝑛 − 𝑥1 ∈ (0, 𝜋).
2(𝑛 − 1)
𝒏−𝟏
∑ √𝟏 − 𝒂𝒊 𝒂𝒊+𝟏 − √(𝟏 − 𝒂𝒊 𝟐 )(𝟏 − 𝒂𝒊+𝟏 𝟐 ) Using the fact that sin 𝑥 < 𝑥 𝑓𝑜𝑟 𝑥 > 0 𝑦𝑒𝑖𝑙𝑑𝑠
𝒊=𝟏
𝝅√𝟐 sin 𝜋 √2𝜋
≤ √2(𝑛 − 1) ≤
𝟐 (2(𝑛 − 1)) 2
𝑛−1
Sol.: Since 𝑥𝑖 ′𝑠 are positive and add up to
∑ √1 − cos 𝑥𝑖 cos 𝑥𝑖+1 − sin 𝑥𝑖 sin 𝑥𝑖+1 1, we can make the substitution 𝑥0 + 𝑥1 +
𝑖=1
⋯ + 𝑥𝑘 = sin 𝑎𝑘 , with 𝑎0 < 𝑎1 < ⋯ <
𝜋
𝑛−1 𝑎𝑛 = 2 , 𝑘 = 0, 1, … . , 𝑛. The inequality
= ∑ √1 − cos(𝑥𝑖+1 − 𝑥𝑖 ) sin 𝑎𝑘 −sin 𝑎𝑘−1 𝜋
becomes ∑𝑛𝑘=1 < ,
𝑖=1 √1+sin 𝑎𝑘−1 √1−sin 𝑎𝑘−1 2
𝑛−1
𝑥𝑖+1 − 𝑥𝑖
= √2 ∑ sin Which can be written as
2
𝑖=1
𝑛 𝑎𝑘 − 𝑎𝑘−1 𝑎 + 𝑎𝑘 − 1
2 sin cos 𝑘
Here we used a subtraction and a double- ∑ 2 2
angle formula. The sine function is concave cos 𝑎𝑘−1
𝑘=1
down on [0, 𝜋]; hence we can Jensen’s 𝜋
inequality to obtain For 0 < 𝑥 < 2 , cos 𝑥 is a decreasing
function and sin 𝑥 < 𝑥. Hence the left side
𝑛−1
1 𝑥𝑖+1 − 𝑥𝑖 of the inequality is strictly less than
∑ sin
𝑛−1 2 𝑎𝑘 −𝑎𝑘−1
𝑖=1 2 cos 𝑎𝑘−1
𝑛−1 ∑𝑛𝑘=1 2
= ∑𝑛𝑘=1(𝑎𝑘 −
1 𝑥𝑖+1 − 𝑥𝑖 cos 𝑎𝑘−1
≤ sin ( ∑ ) 𝜋
𝑛−1 2 𝑎𝑘−1 ) = and problem is solved.
𝑖=1 2
Hence,
260
Challenging Mathematical Problems
Sol.: By using the addition formula for the Sol.: Let us fix 𝑥2 , 𝑥3 … … 𝑥𝑛 and then
cosine we obtain consider the function f: [0, 1] →R.
f(x)= 𝑥1 + 𝑥2 + ⋯ + 𝑥𝑛 − 𝑥𝑥2 … … … . 𝑥𝑛 .
𝑛 𝑛
The function is linear in x, hence attains
∑ ∑ 𝑖𝑗 cos(𝑎𝑖 − 𝑎𝑗 ) its maximum the left side of the inequality
𝑖=1 𝑗=1
𝑛 𝑛 one must choose 𝑥1 𝑡𝑜 be 0 or 1, and by
= ∑ ∑(𝑖𝑗 cos 𝑎𝑖 cos 𝑎𝑗 symmetry, the same is true for the other
𝑖=1 𝑗=1
variables of occurs, if all 𝑥𝑖 are equal to 1,
+ 𝑖𝑗 sin 𝑎𝑖 + sin 𝑎𝑗 ) then we have equality. If at least one of
them is 0, then their product is also zero,
𝑛 𝑛
and the sum of the other 𝑛 − 1 terms is at
= ∑ 𝑖 cos 𝑎𝑖 ∑ 𝑗 cos 𝑎𝑗 most 𝑛 − 1. Which proves the inequality.
𝑖=1 𝑗=1
𝑛 𝑛
+ ∑ 𝑖 sin 𝑎𝑖 ∑ 𝑗 sin 𝑎𝑗
𝑖=1 𝑗=1 7) Find the maximum value of the sum 𝑺𝒏 =
𝒂𝟏 (𝟏 − 𝒂𝟐 ) + 𝒂𝟐 + (𝟏 − 𝒂𝟑 ) + ⋯ +
𝑛 2 𝑛 2
𝟏
𝒂𝒏 (𝟏 − 𝒂𝟏 ), where 𝟐 < 𝒂𝒋 ≤
= (∑ 𝑖 cos 𝑎𝑖 ) + (∑ 𝑖 sin 𝑎𝑖 ) ≥ 0
𝑖=1 𝑖=1 𝟏 𝒇𝒐𝒓 𝒆𝒗𝒆𝒓𝒚 𝒊 = 𝟏, 𝟐, … . , 𝒏.
261
Challenging Mathematical Problems
Sol.: The expression is linear in each of (0, 1, 0, 1, ……0, 1). For n odd. Equality
the variables. So, as in the solutions to the holds when all pairs (𝑥𝑖 , 𝑥𝑖+1 ), 𝑖 =
previous problems, the maximum is 1, 2, … . , 𝑛. consist of a zero and a one,
1 except for one pair which consist of two
attained for 𝑎𝑘 = 2 𝑜𝑟 1, 𝑘 =
1 ones (with the convention 𝑥𝑛+1 = 𝑥𝑖 ).
1,2, … … , 𝑛. 𝐼𝑓 𝑎𝑘 = 𝑓𝑜𝑟 𝑎𝑙𝑙 𝑘, 𝑡ℎ𝑒𝑛 𝑆𝑛 =
2
𝑛
4
, Let us show that the value of 𝑆𝑛 cannot
9) Prove that for numbers a, b, c in the
exceed this number. If exactly m of the 𝒂 𝒃 𝒄
𝑎𝑘′ 𝑠 are equal to 1, then m terms of the interval [0, 1] 𝒃+𝒄+𝟏 + 𝒄+𝒂+𝟏 + 𝒂+𝒃+𝟏 +
sum are zero. Also, at most m trams are (𝟏 − 𝒂)(𝟏 − 𝒃)(𝟏 − 𝒄) ≤ 𝟏
1
equal to 2 , 𝑛𝑎𝑚𝑒𝑙𝑦 those of the from
Sol.: For any non-negative numbers 𝛼, 𝛽,
1 𝛼
𝑎𝑘 (1 − 𝑎𝑘+1 )𝑤𝑖𝑡ℎ 𝑎𝑘 = 1 𝑎𝑛𝑑 𝑎𝑘+1 = 2. the function 𝑥 → is convex for 𝑥 ≥ 0.
𝑥+𝛽
Each of the remaining terms has both Viewed as a function in any of the three
1 1
factors equal to 2, and hence is equal to 4. variables, the given expression is a sum of
Thus the value of the sum is at most two convex functions and two linear
𝑚 (𝑛−2𝑚) 𝑛 functions so it is convex. Thus when two
𝑚. 0 + + = . Which shows that
2 4 4
𝑛 of the variables are fixed, the maximum is
the maximum is 4 .
attained when the third is at one of the
end points of the interval, so the values of
the expression are always less than the
8) If 𝒏 ≥ 𝟐 𝒂𝒏𝒅 𝟎 < 𝒙𝒊≤𝟏 𝒇𝒐𝒓 𝒂𝒍𝒍 𝒊 = 1, 2, …., largest value obtained by choosing a, b, c
n show that (𝒙𝟏 + 𝒙𝟐 + ⋯ + 𝒙𝒏 ) − ∊[0, 1]. An easy check of the eight possible
(𝒙𝟏 𝒙𝟐 + 𝒙𝟐 𝒙𝟑 + ⋯ + 𝒙𝒏 𝒙𝟏 ) ≤ [ ] and
𝒏 cases shows that the value of the
𝟐 expression cannot exceed 1.
determine when there is equality [.]
denote the greatest integer function. 10) If a, b, c, d, e ∊[p, q] with p > 0, prove that
𝟏 𝟏 𝟏 𝟏 𝟏
(1+b+c+d+e) ( + + + + ) ≤ 𝟐𝟓 +
Sol.: Denote the left side of the inequality 𝒂 𝒃 𝒄 𝒅 𝒆
𝟐
by 𝑆(𝑥1 , 𝑥2 … 𝑥𝑛 ) This expression is linear 𝒑 𝒒
𝟔 (√𝒒 − √𝒑 )
in each of the variables 𝑥𝑖 As before, it
follows that it is enough to prove the
inequality when the 𝑥𝑖′ 𝑠 are equal to 0 or Sol.: If we fix four of the numbers and
1. If exactly k of the 𝑥𝑖 ′𝑠 are equal to 0, regard the fifth as a variable x, then the
and the others are equal to 1, then left side becomes a function of the form
𝛽
𝑆(𝑥1 , 𝑥2 , … , 𝑥𝑛 ) ≤ 𝑛 − 𝑘, and since the 𝛼𝑥 + 𝑥+𝑦, with 𝛼, 𝛽, y positive and x
sum 𝑥1 𝑥2 + 𝑥2 𝑥3 + ⋯ + 𝑥𝑛 𝑥1 is at least ranging over the interval [p, q]. This
𝑛 − 2𝑘,S(𝑥1 , 𝑥2 , … , 𝑥𝑛 ) is less than or function is convex on the interval [p, q]
equal to 𝑛 − 𝑘 − (𝑛 − 2𝑘) = 𝑘. So the being the sun of a linear and a convex
maximum of S is less than or equal to function, so it attains its maximum at one
𝑛
min(𝑘, 𝑛 − 𝑘) . 𝑊ℎ𝑖𝑐ℎ is at most [2 ]. If (or possibly both) of the end points of the
follows that for n even. Equality holds interval of definition. As the value of the
when (𝑥1 , 𝑥2 , …, ) =(1, 0, 1, 0, ……, 1, 0) or expression, it is enough to let a, b, c, d, e
take the values p and q.
262
Challenging Mathematical Problems
263
Challenging Mathematical Problems
1 2 2 2 2 𝑛 𝑛
= (10003 + 9993 − 13 − 03 ) 𝑛(𝑛 − 1)(𝑛 − 2) … (𝑛 − 𝑘 + 1) 1 1
4 =∑ ∑
𝑛. 𝑛. 𝑛 … … … 𝑛 𝑘! 𝑘!
1 𝑘=0 𝑘=0
< (100 + 100 − 1)
4 1 2 𝑘−1
< 50. (1 − ) (1 − ) … . (1 − ).
𝑛 𝑛 𝑛
𝟏
14) Show that ∑∞
𝒏=𝟏 (𝒏+𝟏) < 2. In a similar manner,
√𝒏
𝑛+1
Sol.: It is natural to transform the terms of the 1 𝑛+1 1 1
sum as (1 + ) = ∑ (1 − ).
𝑛+1 𝑘! 𝑛+1
𝑘=0
1 1/𝑛 √𝑛 √𝑛
= = − . 2 𝑘−1
√𝑛(𝑛 + 1) 𝑛(𝑛 + 1) 𝑛 𝑛+1 (1 − … (1 − )
𝑛 + 1) 𝑛+1
This allow us to rewrite the sum as 1 + 1 𝑛+1
= ( )
√ 𝑛−√𝑛−1 𝑛+1
∑∞
𝑛=2 . 𝑛
𝑛 1
+ ∑ (1 − )
The sum does not telescope, but it is bounded 𝑛+1
𝑘=0
from above by
2 𝑘−1
∞ (1 − ) … . (1 − ).
√𝑛 − √𝑛 − 1 𝑛+1 𝑛+1
1+∑
𝑛=2
√𝑛√𝑛 − 1 The inequality is now obvious. Since
∞ 1
1 1 comparing the coefficients of : in these
=1+∑( − ) 𝑘
√𝑛 − 1 √𝑛 expressions. We see that for each k, k = 0, 1,
𝑛=2
2, ….., n.
With telescope to 2. This proves the
inequality. 1 2 𝑘−1
(1 − ) (1 − ) … … … (1 − )
𝑛 𝑛 𝑛
1
< (1 − ) (1
𝑛+1
15) For each positive integer n. Show that 2 𝑘−1
− ) … … … (1 − )
𝑛+1 𝑛+1
𝟏 𝒏 𝟏 𝒏+𝟏
(𝟏 + ) < (𝟏 + ) It is worth nothing that
𝒏 𝒏+𝟏
264
Challenging Mathematical Problems
𝑛 + 𝑆𝑛 𝑛 2
> (𝑛 + 1)1/𝑛
𝑛 (∑ 𝑎𝑖 ) = [(𝑎1 + 𝑎2 ) − 𝑎3 + ⋯ + 𝑎𝑛 ]2
𝑖=1
Which has vaguely the look of an arithmetic ≤ (1 + ⋯ 1 )((𝑎1 + 𝑎2 )2
mean geometric mean inequality. We can + 𝑎3 2 … . +𝑎𝑛 2 )
make the idea work in the following way: 𝑛
= (𝑛 − 1) [∑ 𝑎2 + 2𝑎1 𝑎2 ]
1 1
𝑛 + 𝑆𝑛 𝑛 + (1 + 2 + ⋯ + 𝑛) 𝑖=1
=
𝑛 𝑛 This, together with the given inequality,
1 1 implies that
(1 + 1) + (1 + ) + ⋯ + (1 + )
= 2 𝑛
𝑛 𝑛 2
𝑛 1
𝐴 < − (∑ 𝑎𝑖 2 ) + (∑ 𝑎)
3 4 (𝑛 + 1) 𝑛−1
2 + 2 + 3 + ⋯+ 𝑛
𝑖=1 𝑖=1
𝑛
=
𝑛 < − (∑ 𝑎𝑖 2 )
𝑖=1
3 4 𝑛 + 1 1/𝑛
(2. . … … . . ) . 1
2 3 𝑛
+ [(𝑛
𝑛+1
For the right most inequality. We need to
𝑛−𝑆𝑛 𝑛
show that 𝑛−1
> 𝑛−1/(𝑛−1). Again, using the
− 1) [(∑ 𝑎2 − 2𝑎1 𝑎2 )]]
arithmetic mean geometric mean inequality.
𝑖=1
We have
= 2𝑎1 𝑎2 .
1 1 1
𝑛 − 𝑆𝑛 𝑛 − (1 + 2 + 3 + ⋯ + 𝑛) In a similar manner, 𝐴 <
= 2𝑎1 𝑎2 𝑓𝑜𝑟 1 ≤ 𝑖 ≤ 𝑗 ≤ 𝑛.
𝑛−1 𝑛−1
1 1 18) If positive numbers p, q, r such that 𝟐𝒑 =
(1 − 1) + (1 − ) + ⋯ + (1 − )
= 2 𝑛 𝒑𝒒−𝒓
𝑛−1 𝒒 + 𝒓, 𝒒 ≠ 𝒓 show that 𝒒𝟐𝒓𝟐 < 1.
𝑑 sin 𝑥 1 sin 𝑥1
𝐺 ′ (𝑥) = [𝑥 log 𝑥 + (1 − 𝑥) log(1 − 𝑥)] log ( ) ≥ (log +⋯
𝑑𝑥 𝑥 𝑛 𝑥1
= (log 𝑥 + 1) − 1 sin 𝑥𝑛
𝑥 + log )
𝑥𝑛
− log(1 − 𝑥) = log .
1−𝑥
1
𝑊𝑒 𝑠𝑒𝑒 𝑡ℎ𝑎𝑡 𝐺′(𝑥) = 0 𝑖𝑓 𝑎𝑛𝑑 𝑜𝑛𝑙𝑦 𝑖𝑓 𝑥 = 2.
Furthermore, 𝐺′(𝑥) < 0 on the interval
266
Challenging Mathematical Problems
1 1 1 1
= 𝑎1 ( − ) (𝑎1 + 𝑎2 ) ( − )+
√𝑏1 √𝑏2 √𝑏2 √𝑏3
1 1 21) Suppose 𝒂𝟏 , 𝒂𝟐 , … , 𝒂𝒏 be non-negative
(𝑎1 + 𝑎2 + 𝑎3 ) ( − ) + ⋯ + (𝑎1 + 𝑎2 + 𝟏
√𝑏3 √𝑏4 numbers such that 𝒂𝟏 𝒂𝟐 − 𝒂𝒌 ≤ (𝟐𝒌)! for
1
⋯ + 𝑎𝑛 ) 𝟏
√𝑏𝑛 all k. Show that 𝒂𝟏 + 𝒂𝟐 + ⋯ 𝒂𝒏 ≥ +
𝒏+𝟏
𝟏 𝟏
The differences in the parenthesis are all 𝒏+𝟐
+ ⋯ + 𝟐𝒏.
positive. Using the hypothesis we obtain that
this expression is less than or equal to Sol.: We have
1
1 1 1 1 𝑎1 + 𝑎2 + ⋯ + 𝑎𝑛 = (1 − 2) (1.2𝑎1 ) +
𝑏1 ( − ) − (𝑏1 + 𝑏2 ) ( − )
1 1 1 1
√𝑏1 √𝑏2 √𝑏2 √𝑏3 ( − ) (3.4𝑎2 ) + ⋯ + ( − ) ((2𝑛 −
3 4 2𝑛−1 2𝑛
1
+ ⋯ + (𝑏1 + 𝑏2 + ⋯ + 𝑏𝑛 ) 1). 2𝑛𝑎𝑛 )
√𝑏𝑛
1 1 1
= √𝑏1 + √𝑏2 + ⋯ + √𝑏𝑛 . = (1 − − + ) (1.2𝑎1 )
2 3 4
1 1 1 1
𝑇ℎ𝑒𝑟𝑒𝑓𝑜𝑟𝑒, +( − − + )
3 4 5 6
𝑎1 𝑎2 𝑎𝑛 (1.2𝑎1 + 3.4𝑎2 ) + ⋯
+ + ⋯+
√𝑏1 √𝑏2 √𝑏𝑛 1 1
+( − ) (1.2𝑎1
≤ √𝑏1 + √𝑏2 + ⋯ + √𝑏𝑛 2𝑛 − 1 2𝑛
+ 3.4𝑎2 ) + ⋯
Using this result and the Cauchy-Schwarz + (2𝑛 − 1). 2𝑛𝑎𝑛
inequality, we obtain. (√𝑎1 + √𝑎2 + ⋯ +
2 Using the AM-GM inequality and the
√𝑎𝑛 ) hypothesis we obtain.
267
Challenging Mathematical Problems
268
Challenging Mathematical Problems
269
Challenging Mathematical Problems
𝑛 2 𝑛 𝑛 𝑛 (𝑚 − 2)𝑡
3
𝑎𝑗 + 𝑎𝑗 − 1 1+ . 𝐻𝑒𝑛𝑐𝑒, 𝑙𝑖𝑚 → ∞ 𝐿𝑘 = 2.
(∑ 𝑎𝑖 ) − ∑ 𝑎𝑖 = 2 ∑ ∑ 𝑎𝑖 1 + 𝑡2
2
𝑖=0 𝑖=0 𝑖=0 𝑗=0
We can proceed similarly in the case 𝑘 = 2𝑚 +
[1 − (𝑎𝑗 − 𝑎𝑗−1 )] we have equality if 𝑎𝑗 − 1.
𝑎𝑗−1 = 1 for j = 1, …, n.
270
Challenging Mathematical Problems
Denote the distance of the circumcentre M. 30) Let n > 2 and 𝒙𝟏 , 𝒙𝟐 , … . , 𝒙𝒏 be non
from a, b, c by x, y, z. Then, for the LHS we get negative reals. Prove that
𝟏
𝟏
sin 2𝛼 + sin 2𝛽 + sin 2𝛾 (𝒙𝟏 𝒙𝟐 , … . , 𝒙𝒏 )𝒏 + ∑𝒊<𝑗 |𝒙𝒊 − 𝒙𝒋 | ≥
𝒏
𝟏
= 2(sin 𝛼 cos 𝛼 + sin 𝛽 cos 𝛽 + sin 𝛾 cos 𝛾) (𝒙𝟏 + 𝒙𝟐 + ⋯ + 𝒙𝒏 )
𝒏
𝑎 cos 𝛼 + 𝑏 cos 𝛽 + 𝑐 cos 𝛾
= 𝑏𝑢𝑡
𝑅
𝑥 𝑦 𝑧
𝑎 cos 𝛼 + 𝑏 cos 𝛽 + 𝑐 cos 𝛾 = 𝑎. + 𝑏. + 𝑐. Sol.: We may assume that 𝑥1 ≥ 𝑥2 ≥ ⋯ ≥ 𝑥𝑛 .
𝑅 𝑅 𝑅
2.4 Then all the points 𝑥1 , … . , 𝑥𝑛 lie on the
= segment [𝑥𝑛 , 𝑥1 ]. Hence |𝑥𝑖 − 𝑥𝑗 | ≤ |𝑥𝑛 − 𝑥1 |.
𝑅
In addition, |𝑥1 − 𝑥𝑘 | + |𝑥𝑘 − 𝑥𝑛 | = 𝑥1 − 𝑥𝑛
sin 𝛼 + sin 𝛽 + sin 𝛾 𝑅 for k = 2, …., n -1. Together with |𝑥1 − 𝑥𝑛 |we
𝐻𝑒𝑛𝑐𝑒, = ≥ 1.
sin 2𝛼 + sin 2𝛽 + sin 2𝛾 2𝑟 get the estimate ∑𝑖>𝑗|𝑥𝑖 − 𝑥𝑗 | ≥ (𝑛 − 1)(𝑥1 −
𝑥𝑛 )
1
𝑆𝑖𝑛𝑐𝑒 (𝑥1 … 𝑥𝑛 )𝑛 ≥ 𝑥𝑛 , it is sufficient to prove
1 𝑥1 +⋯+𝑥𝑛
that 𝑥𝑛 + 𝑛 (𝑛 − 1) (𝑥1 − 𝑥𝑛 ≥ )
29) Prove that for real numbers 𝒙𝟏 ≥ 𝒙𝟐 ≥ 𝑛
⋯ ≥ 𝒙𝒏 > 0.
𝒙𝟏 𝒙𝟐 𝒙𝒏−𝟏 𝒙𝒏 𝑜𝑟, 𝑥𝑛 + (𝑛 − 1)𝑥𝑖 ≥ 𝑥1 + ⋯ + 𝑥𝑛 , which is
+ + ⋯+ + valid. The proof of this weak inequality was
𝒙𝟐 𝒙𝟑 𝒙𝒏 𝒙𝟏
𝒙𝟐 𝒙𝟑 𝒙𝒏 so simple since. We could get by with huge
≤ + +⋯+ over estimations.
𝒙𝟏 𝒙𝟐 𝒙𝒏−𝟏
𝒙𝟏
+ .
𝒙𝒏
SET THEORY
Sol.: Transfer all terms to the left side and (OBJECTIVE TYPE)
𝑥 𝑥
look at all terms with an 𝑥𝑛 : 𝑓(𝑥) = 𝑛−1 + 𝑛 −
𝑥𝑛 𝑥1 1) Which of the following has only one
𝑥𝑛 𝑥1
− . Let us find the minimum of this subset?
𝑥𝑛−1 𝑥𝑛
(a) { }; (b) {4}; (c) {0}; (d) none
function on the interval [𝑥𝑛−1 , ∞]. the
derivation of 𝑓(𝑥𝑛 ) on this interval is positive, Sol.: Subsets of { }i.e. 𝜙 is 𝜙. Subsets of {4}
and hence the minimum is attained at 𝑥𝑛 = are 𝜙, {4} subsets of {4, 5} are 𝜙 {4}, {5},
𝑥𝑛−1 . Inserting 𝑥𝑛 = 𝑥𝑛−1 into the inequality, {4, 5}. Subsets of {0} are 𝜙, {0}
we get same inequality, but for variables
𝑥1 𝑡𝑜 𝑥𝑛−1 . We finish the proof by induction. ∴ cannot answer is (a).
2) If A = {𝒙 ∈ 𝒄; 𝒙𝟐 = 𝟏}𝒂𝒏𝒅 𝑩 = {𝒙 ∈
𝒄; 𝒙𝟒 = 𝟏}, then A ∆ B =
(a) {−𝟏, 𝟏}; (b) {−𝒊, 𝒊}; (c) {−𝟏, 𝟏, 𝒊, −𝒊}
(d) none
271
Challenging Mathematical Problems
Sol.: 𝑥 2 = 1 ⟹ 𝑥 = −1, 1 ∴ 𝟏
6) If 𝑨 = {(𝒙, 𝒚): 𝒚 = 𝒙 , 𝟎 ≠ 𝒙 ∊
𝐴 = {−1, 1}
𝑹} 𝒂𝒏𝒅 𝑩 = {(𝒙, 𝒚): 𝒚 = −𝒙, 𝒙 ∊ 𝑹}, 𝒕𝒉𝒆𝒏
4 2
𝑥 = 1 ⟹ 𝑥 = −1, 1 ⟹ 𝑥 = −𝑖, 𝑖, −1, 1 (a) A ∩B = 𝜙 (b) A ∩B = A; (c) A∩ B =
B; (d) none
∴ 𝐵 = {−𝑖, 𝑖, −1, 1}
Sol.: Here A and B can be shown as;
∴ 𝐴 ∆𝐵 = (𝐴 − 𝐵) ∪ (𝐵 − 𝐴)
= 𝜙 ∪ {−𝑖, 𝑖} = {−𝑖, 𝑖} ⟹A ∩ B = 𝜙
3) If 𝒏(𝑨) = 𝟑, 𝒏(𝑩) = 𝒚, 𝒕𝒉𝒆𝒏 𝒏(𝑨 × 𝑨 × 7) Let x be the universal set for sets A and B.
𝑩) = If 𝒏(𝑨) = 𝟐𝟎𝟎, 𝒏(𝑩) = 𝟑𝟎𝟎 𝒂𝒏𝒅 𝒏(𝑨 ∩
(a) 36; (b) 102; (c) 108; (d) none 𝑩) = 𝟏𝟎𝟎, then 𝒏(𝑨′ ∩ 𝑩′ ) = 𝟑𝟎𝟎
provides n(x)=
Sol.: 𝑛(𝐴 × 𝐴 × 𝐵) = 𝑛(𝐴) × 𝑛(𝐴) ×
(a) 500; (b) 600; (c) 700; (d) none
𝑛(𝐵) = 3 × 3 × 4 = 36
Sol.: We have 𝑛(𝐴 ∩ 𝐵) = 𝑛(𝐴) + 𝑛(𝐵) −
4) If u = R and let A = {𝒙 ∊ 𝑹: 𝟎 < 𝑥 <
𝑛(𝐴 ∩ 𝐵)
2}, 𝑩 = {𝒙 ∊ 𝑹: 𝟏 < 𝑥 ≤ 3} which of the
following is false? ∴ 𝑛(𝐴 ∪ 𝐵) = 200 + 300 − 100 = 400
(a) 𝑨 ∩ 𝑩 = {𝒙 ∊ 𝑹: 𝟏 < 𝑥 < 2}
(b) 𝑨′ = {𝒙 ∊ 𝑹: 𝒙 ≤ 𝟎 𝒐𝒓 𝒙 ≤ 𝟐} 𝐴𝑙𝑠𝑜, 𝑛(𝐴′ ∪ 𝐵′ ) = 𝑛(𝐴 ∪ 𝐵′ ) = 𝑛(𝑥) −
(c) 𝑨 ∪ 𝑩 = {𝒙 ∊ 𝑹: 𝟎 ≤ 𝒙 ≤ 𝟑}; 𝑛(𝐴 ∪ 𝐵)
(d) 𝒏𝒐𝒏𝒆
∴ 300 = 𝑛(𝑥) − 400 𝑜𝑟 𝑛(𝑥) = 700.
′
Sol.: 𝐴 = 𝑅 − 𝐴 = {𝑥 ∊ 𝑅: 𝑥 ≤ 0 𝑜𝑟 𝑥 ≥ 2}
𝐵′ = 𝑅 − 𝐵 = {𝑥 ∊ 𝑅: 𝑥 ≤ 1 𝑜𝑟 𝑥 > 3}
8) If A ∪ B = A ∪ C and A ∩B = A ∩ C, then
𝐴 ∪ 𝐵 = {𝑥 ∊ 𝑅: 𝑥 ∊ 𝐴 𝑎𝑛𝑑 𝑥 ∊ 𝐵} = {𝑥 ∊ (a) B = C only when A ⊆ C
𝑅: 0 < 𝑥 ≤ 3} (b) B = C;
(c) B= C only when A ⊆ B
(d) Done
5) The set (A ∪ B ∩C) ∩(𝑨 ∪ 𝑩′ ∪ 𝑪′)′ 𝒏𝒄′= Sol.: Let x ∊ B ⟹ x ∊ A ∪ B ⟹ x ∊ A ∪ C
(a) A ∩ C; (b) B’ ∩ C’; (c) B ∩ C’ (d)
none Case I: x ∊ A, x ∊ A ∩ B or x ∊ A ∩ C or x ∊
C, B⊆C
Sol.: (A ∪ B ∪C) ∩(𝐴 ∪ 𝐵′ ∩ 𝐶 ′ )′ ∩ 𝐶 ′
Case II: x ∊ C, x∊ B⟹ x ∊ C or B ⊆ C
= (𝐴 ∪ 𝐵 ∪ 𝐶) ∩ (𝐴′ ∪ 𝐵 ∪ 𝐶) ∩ 𝐶 ′
Similarly C⊆ B
= (𝜙 ∪ 𝐵 ∪ 𝐶) ∩ 𝐶 ′ = (𝐵 ∪ 𝐶) ∩ 𝐶 ′ =
(𝐵 ∩ 𝐶 ′ ) ∪ (𝐶 ∩ 𝐶 ′ ) ∴B=C
= (𝐵 ∩ 𝐶 ′ ) ∪ 𝜙 = 𝐵 ∩ 𝐶′
272
Challenging Mathematical Problems
9) If A = {𝜽: 𝟐 𝒄𝒐𝒔𝟐 𝜽 + 𝐬𝐢𝐧 𝜽 ≤ 𝟐} 𝒂𝒏𝒅 11) A set contains n elements, then its power
𝝅 𝟑𝝅 set
𝑩 = {𝜽: ≤ 𝜽 ≤ } 𝒕𝒉𝒆𝒏 𝑨 ∩ 𝑩 =
𝟐 𝟐 (a) n element; (b) 𝟐𝒏 elements; (c) 𝒏𝒏
𝟑𝝅 elements (d) none
(a) {𝜽: 𝝅 ≤ 𝜽 ≤ } ;
𝟐
𝝅 𝟓𝝅 𝟑𝝅
(b) {𝜽: 𝟐≤ 𝜽 ≤ 𝟔 𝒐𝒓 𝝅≤ 𝜽≤ 𝟐
}; Sol.: As power set is set of all subsets, and
𝟓𝝅 we know number of subsets of a set
(c) {𝜽: 𝝅 ≤ 𝜽 ≤ 𝟔 }
containing n element is 2𝑛 .
(d) None
𝜋
∴Power set contains 2𝑛 𝑒𝑙𝑒𝑚𝑒𝑛𝑡𝑠.
2
Sol.: Let 2 𝑐𝑜𝑠 𝜃 + sin 𝜃 ≤ 2 𝑎𝑛𝑑 ≤𝜃<
2
3𝜋
2
𝑩
12) If A and B are sets, then 𝑨 ∩ (𝑨)is
⟹ 2 − 2 𝑠𝑖𝑛2 𝜃 + sin 𝜃 ≤ 2 ⟹ 2 𝑠𝑖𝑛2 𝜃 −
sin 𝜃 ≥ 0 (a) 𝜙; (b) A; (c) B; (d) none
𝜋 𝐵
⟹ sin 𝜃 (2 sin 𝜃 − 1) ≥ 0 ⟹ ≤𝜃 Sol.: Let x ∊ 𝐴 ∩ ( ) ⟹ 𝑥 ∊ 𝐴 𝑎𝑛𝑑 (𝑥 ∊
𝐴
2 𝐵
5𝜋 3𝜋 𝐴
)
≤ 𝑜𝑟 𝜋 ≤ 𝜃 ≤
6 6
⟹ 𝑥 ∊ 𝐴 𝑎𝑛𝑑 (𝑥 ∊ 𝐵 𝑎𝑛𝑑 𝑥 ∉ 𝐴) ⟹ 𝑥 ∊
𝜋 5𝜋 𝜙
∴ 𝐴 ∩ 𝐵 = {𝜃: ≤ 𝜃 ≤ 𝑜𝑟 𝜃 ≤ 𝜋
2 2
3𝜋 𝐵
≤ } ∴ 𝐴 ∩ ( ) ⊂ 𝜙 … … … … . . (1)
2 𝐴
𝐵
𝑆𝑖𝑛𝑐𝑒 𝜙 ⊂ 𝐴 ∩ ( ) … … … … (2)
𝐴
10) If A and B be two sets such that
𝒏(𝑨 × 𝑩) = 𝟔. 𝐵
∴ 𝐹𝑟𝑜𝑚 (1) 𝑎𝑛𝑑 (2) 𝐴 ∩ ( ) = 𝜙
Let three elements of 𝑨 × 𝐴
𝑩 𝒂𝒓𝒆 (𝟑, 𝟐)(𝟕, 𝟓)(𝟖, 𝟓)then
(a) A= {3, 7, 8}; (b) B = {2, 5}; (c) C =
{3, 5}; (d) none 13) Let R be set of points inside a rectangle of
sides a and b (a, b>1) with two sides along
Sol.: Since (3, 2), (7, 5), (8, 5)∊ A × B, we the positive direction of x-axis and y-axis
have 3, 7, 8∊ A and 2, 5 ∊B. and C be the set of points inside a unit
Also 𝑛(𝐴 × 𝐵) = 6 = 3 × 2 circle central at origin, then
(a) 𝑹 = {(𝒙, 𝒚): 𝟎 ≤ 𝒙 ≤ 𝒂, 𝟎 ≤ 𝒚 ≤ 𝒃};
A = {3, 7, 8} and B = {2, 5} (b) 𝑹 = {(𝒙, 𝒚): 𝟎 < 𝑥 < 𝑎, 0 < 𝑦 < 𝑏};
(c) 𝑹 ∪ 𝑪 = 𝑹;
Since the graphs of 𝑥𝑦 = 4 and y = x, x > (d) None
0 intersect ay one point, we have 𝐴 ∩ 𝐵 =
𝜙 and A ∩B is a single set. Sol.: Since, R denotes the set of points
inside the rectangle of sides a and b for
both a and b >1, then 𝑅{(𝑥, 𝑦): 0 < 𝑥 <
273
Challenging Mathematical Problems
𝑎, 0 < 𝑦 < 𝑏}. Also C is the set of points 17) Of the numbers of 3 teams in a college 21
inside the unit circle, centred at origin, are in the cricket team, 26 are in hockey
such that 𝑆 = {(𝑥, 𝑦): 𝑥 2 + 𝑦 2 < 1} team and 29 are in the football team.
Among team, 14 play football and cricket.
14) Which of the following is not correct? Eight play all the three games. The total
(a) 𝑨 ⊆ 𝑨𝒄 if and only if A = 𝜙; number of members in the three teams is
(b) 𝑨 = 𝑩 is equivalent to A ∪ C= B ∩ C (a) 43; (b) 49; (c) 64; (d) none.
and A ∩ C= B ∩ C;
(a) 𝑨𝒄 ⊆ 𝑨 if and only if A = x, where x is a Sol.: (a) Let C, H, F denote the sets
universal set. members who are on the cricket, hokey
(b) 𝒏𝒐𝒏𝒆 and football team respectively.
Sol.: 𝐴𝑐 satisfies (A) and (B) by definition ∴ n(C) = 21, n(H) = 26, n(F) = 29, n(C ∩
(D) also follows trivially. B) = 14, n(C∩ F) = 15, n(F ∩ C) = 12 and
n(C ∩ H ∩ F)= 8.
Assuming A to be any set other than the
empty set also 𝐵 = 𝐴 𝑎𝑛𝑑 𝐶 = ∴n(C ∪ H ∪ F) = n(C) + n(H)+ n(F) –n(C
𝜙, 𝑤𝑒 ℎ𝑎𝑣𝑒 𝐴 ∪ 𝐵 = 𝐴 − 𝐴 ∪ 𝐶 𝐵𝑢𝑡 𝐵 ≠ ∩ H) –n (H∩ F) –n (F ∩) +n(C∩ H ∩ F)
𝐶, 𝑠𝑜 (c) is incorrect.
= (21 + 26 + 29) − (14 + 15 + 12) +
15) Let S is the set of points inside the square. 9 = 43.
T is the set of points inside the triangle and
C is the set of the points inside the circle. If
the triangle and circle intersect each other (SUBJECTIVE TYPE)
and are contained in the square , then
(a) S∩ T ∩ C ≠ 𝜙; (b) S ∪T ∪C =C; (c) S 1) If the collection of all these three elements
∪T =S ∪C; (d) none subsets drawn from the set {1, 2, 3, ….,
300}. Find the number of these subsets for
Sol.: Since, T ∩ C≠ 𝜙 and S∩ T ∩ C= T ∩ C which the sum of the three elements is a
so option (c) is true multiple of 3.
Also T ⊂ S and C ⊂ S, So S ∪ T ∪ C = S Sol.: For 0 ≤ 𝑗 ≤ 2, 𝑙𝑒𝑡 𝐴𝑗 denote the set of
Also, S∪ T = S = S∪ C. all integers between 1 and 300 which
leave remainder j when divided by 3.
Then |𝐴𝑗 | = 100 𝑓𝑜𝑟 0 ≤ 𝑗 ≤ 2. If a, b, c is
a 3-element subset of the given set
16) The set (𝑨 ∩ 𝑩𝒄 )𝒄 ∪ (𝑩 ∩ 𝑪) =
(a) 𝑨𝒄 ∪ 𝑩; (b) 𝑨𝒄 ∪ 𝑩 ∪ 𝑪; (c) 𝑨𝒄 ∪ 𝑠 ′ = 1, 2, … , 300 then 3 divides a+ b+ c if
𝑩𝒄 ; (d) none and only if
Sol.: Let S = (𝐴 ∩ 𝐵𝑐 )𝑐 ∪ (𝐵 ∩ 𝐶) (i) All a, b, c are in𝐴0 or in 𝐴1 𝑜𝑟 𝑖𝑛 𝐴2
(ii) One of the a, b, c is in 𝐴0 , another in
⟹ 𝑆 = ( 𝐴𝑐 ∪ 𝐵) ∪ (𝐵 ∩
𝐴1 , and the third one in 𝐴2 .
𝐶) (𝐷𝑒 𝑀𝑜𝑟𝑔𝑎𝑛′ 𝑠𝐿𝑎𝑤)
⟹ 𝑆 = 𝐴𝑐 ∪ (𝐵 ∪ (𝐵 ∩ 𝐶)) ∴ 𝑆 = 𝐴𝑐 ∪ 𝐵
274
Challenging Mathematical Problems
275
Challenging Mathematical Problems
276
Challenging Mathematical Problems
subsets (a, b, c) with a +b +c <95 and 10) Find the number of ways to choose an
those such that a +b +c > 97. ordered pair (a, b) of numbers from the
set {1, 2, …., 10} such that |𝒂 − 𝒃| ≤ 𝟓.
Hence the number of subsets with a +b
+c < 95 is equal to that with a +b +c Sol.: Let 𝐴1 = [(𝑎, 𝑏)𝑎, 𝑏 ∊
>97. Thus the set of 3-element subsets (a, {1, 2, 3, … . , 10}, |𝑎 − 𝑏| = {𝑖}, 𝑖 =
b, c) with a+ b +c > 95 will contain those (𝑖,𝑖)
0, 1, 2, 3, 4, 5. 𝐴0 = { 𝑖
=
with a +b +c => 97 and a few more.
1, 2, 3, … , 10} 𝑎𝑛𝑑
(𝑖, 𝑖 + 1)
9) For which positive integral values of n can |𝐴0 | = 10, 𝐴1 = { = 1, 2, 3, … , 9}
𝑖
the set {1, 2, 3, …, 4n} be split into n (𝑖, 𝑖 − 1)1
disjoint 4-element subset {a, b, c, d} such ∪{
(𝒃+𝒄+𝒅)
𝑖
that in each of these sets 𝒂 = .
𝟑
= 1, 2, 3, … , 10} 𝑎𝑛𝑑 |𝐴1 |
Sol.: Suppose {a, b, c, d} is a group in
(𝑏+𝑐+𝑑)
= 9 + 9 = 18.
which 𝑎 = 3
. Then a + b+ c +d =
4a. Hence if such an n-exists, then 4 (𝑖, 𝑖 + 2)
𝐴2 = { = 1, 2, 3, … , 8}
divides 1+ 2 +…+ 4n. However this sum 𝑖
is 2𝑛(4𝑛 + 1). (𝑖, 𝑖 − 2)
∪{
𝑖
Thus a necessary condition for existence
of such a set is that n be even. = 3, 4, … . , 10} 𝑎𝑛𝑑 |𝐴2 |
𝑎+2+𝑎+6+𝑎+7 = 6 + 6 = 12.
𝑎+5=
3
277
Challenging Mathematical Problems
278
Challenging Mathematical Problems
Now, S(X)= S(B) –S(B∩ C), S(Y) = S(C) –S = 610 + 1681 × 40 + 101 × 51 = 73001.
(B∩ C) but S(B) = S(C).
279
Challenging Mathematical Problems
280
Challenging Mathematical Problems
1987 = 21 . 993 + 1 where 993= 1 (mod Sol.: There are 34 integers in A.P. 1, 4, 7, …,
4) ∴ 1988 ∈ 𝐵 100. Let us denote the set {1, 4, 7, …., 100} by
s. Let us group them into 17 pairs.
1989 = 22 . 497 + 1 where 497 = 1 (mod
4) ∴ 1989 ∈ 𝐵. (4, 100), (7, 97), (10, 94), …, (49, 55) and (1,
52). The sum of the integers in each of the
2 4
𝑏𝑢𝑡 ∴ 1998 ∊ 𝐵. first sixteen pairs is 104. The last pair
1998 1998 consists of the two integers which cannot be
1997 = 22 . 449 + 1 where 499= 3(mod paired with any other integer in the given A.P.
4) So as to have the sum 104. It is obvious that
the sum of two integers from the given A.P.
∴ 1997 ∊ A can be 104 if and only if two integers both
belong to some one of the first sixteen pairs
16) If A denote the subsets of the set {1, 11,
written above.
21, 31, ….541, 551} having the property
that no two elements of A odd up to 552. Let us try to construct a subset of S which is
Show that A cannot have more than 28 as big as possible, and has the property that
elements. no two numbers of the set add up to 104.
Such a set can have at the most 18 members,
Sol.: Observe that S consists of 56 numbers in
namely the two integers 1 and 100, and
A.P. Hence first term is 1 and common
exactly one out of each of the remaining16
difference is 10. The sum of every pair of
pairs.
numbers equidistant from the beginning and
the end is 552. Also, the sum of no two others The moment we odd one more member of S
elements can be 552. to it, it will have both the one of the sixteen
pairs (4, 100) …. (49, 55) i.e. it will have two
We divide S into 28 pairs: (1, 551), (11, 541),
distinct integers whose sum is 104.
(21, 531) ………….. (271, 282), if A consists of
at the most 28 elements, then it is possible to Therefore in any set of 20 (in fact 19!)
choose these elements in such a way that at distinct integers chosen forms, there will
most one element from a pair is in A. however always be two distinct integers whose sum is
if A contains 29 (or more) elements, then by 104.
the pigeon hole principle, A must contain
both the elements of at least one pair, and
therefore A contain two elements whose sum 18) Find all possible sets of consecutive
is 552. positive integers such that the sum of the
numbers in the set is 795 (e. g. the sets
Since A has the property that no two element {30}, {9, 10, 11}, {4, 5, 6, 7, 8} and {6, 7, 8,
of A add up to 552, therefore A cannot have 9} are the sets of consecutive positive
more than 28 elements. integers with sum 30.)
17) Show that in any set of 20 distinct integers Sol.: Suppose 𝑛 + 1, 𝑛 + 2, … , 𝑛 + 𝑘 is a set of
chosen from the set {1, 4, 7, …, 100} there k consecutive integers whose sum is 795 i.e.
will always be two distinct integers whose (𝑛 + 1) + (𝑛 + 2) + ⋯ + (𝑛 + 𝑘) = 795 ⟹
sum is 104. 1
{𝑘𝑛 + 2 𝑘(𝑘 + 1)} = 795 … … . . (1)
281
Challenging Mathematical Problems
We are required to find all solutions of (1) in 19) If the 7 element set A= {a, b, …, g}, find a
integers. collection T of 3- element subsets of A such
that each pair of element from A occurs
Case I: If k is doubly even, say = 4m, then exactly in one of the subsets of T.
4𝑚𝑛 + 2𝑛(4𝑚 + 1) = 795.
Sol.: If the 3-element subsets in the class are
Here L HS is even and RHS is odd, and pairwise disjoint, then one of the subsets has
consequently no solution is possible. number of elements and so at least one pair
Case II: If k is singly even, say = 4𝑚 + 2, the has one element in common but not two or
(1) becomes (4𝑚 + 2)𝑛 + (2𝑚 + 1)(4𝑚 + more. If just one pair of subsets has common
3) = 795. elements, they have to have two elements in
common. So this is also not possible. Thus the
So that 2m +1 must divide 795, i.e. 2m +1 = problem reduces to finding a class of all 3-
1, 3, 5, 15, 53, 159, 265, 795. Also then 2𝑛 = element subsets with precisely one element
795 common between any two of the subsets.
− (4𝑚 + 3) > 0 i.e. 2m+1 cannot have
2𝑚+1
Clearly, {(a, b, d), (b, c, e), (c, d, f), (d, e, g), (e,
values other than 1, 3, 5, 15.
f, a ), (a, c, g), (b, f, g)}, in one such class. Any
For these values, we have k = 4m +2 = 2, 6, permutation of A will give another class.
10, 30 and the corresponding values of n are
𝑘(𝑘+1) 20) Let S in the set {1, 2, 3, …, 𝟏𝟎𝟔 }. Show that
[795− ]
2
i.e. 396, 129, 74, 11. for any subset A of S with 101 elements.
𝑘
We can find 100 distinct elements 𝒙𝒊 of S,
Thus the sets are {397, 398}, {130, 131, …, Such that the sets 𝒙𝒊 + A are all pair wise
135}{75, 76, …, 84,}{12, …, 13, 41} …. (A) disjoint.
{Note that 𝒙𝒊 + 𝑨 is the set
Case III: If k is odd, say 2m +1, then (1) 𝒙
becomes (2𝑚 + 1)𝑛 + (𝑚 + 1)(2𝑚 + 1) = {𝒂 + 𝒂𝒊 } 𝒊𝒔 𝒊𝒏 𝑨]
795.
Sol.:
So that 795 must be divided by 2m +1. The
Having found 𝑥1 , 𝑥2 , … , 𝑥𝑘 there are k. 101 -
possible values of 2m +1 are 1, 3, 5, 15, 53,
795 100 for –bidden values for 𝑥𝑘+𝑖 of the form
159, 265, 795 Further more 𝑛 = (2𝑚+1) − 𝑥1 + 𝑎𝑚 − 𝑎𝑛 with m and n unequal and
(𝑚 + 1) > 0, which gives n = 794, 263, 157, another k forbidden values with m= n.
45 respectively, when 2m +1 = 1, 3, 5, 15
Since 99.101.100+99=106 − 1.
The other values do not give positive values
of n and therefore must be rejected. The We can successively choose 100 distinct 𝑥𝑖 .
corresponding sets are {795}, {264, 265,
266}, {158, 159, 160, 161, 162}, {46, 47, ….,
60} …… (B) All the possible sets are those
given in (A) and (B).
282
Challenging Mathematical Problems
283
Challenging Mathematical Problems
Last term of third bracket on RHS of (1) is i.e., n = 5, 12, 14, 15, 16 (as n< 17). Thus the
3.n and so on number of solution if five
284
Challenging Mathematical Problems
285
Challenging Mathematical Problems
∴ 𝑓(𝑓(6) ) = 𝑓(13) m n
1 4r 4s+2
286
Challenging Mathematical Problems
287
Challenging Mathematical Problems
Sol.: The number of ways in which 4 𝑁𝑜. 𝑜𝑓 𝑤𝑎𝑦𝑠 𝑖𝑛 𝑤ℎ𝑖𝑐ℎ 𝑎𝑡 𝑙𝑒𝑎𝑠𝑡 𝑜𝑛𝑒 𝑜𝑓 𝑡ℎ𝑒 𝑑𝑖𝑐𝑒
[ ]
soldiers can be selected out of 15 are the 𝑠ℎ𝑜𝑤𝑠 𝑎𝑛 𝑒𝑣𝑒𝑛 𝑛𝑢𝑚𝑏𝑒𝑟
number of ways batches can be formed. [𝑁𝑜. 𝑜𝑓 𝑎𝑙𝑙 𝑝𝑜𝑠𝑠𝑖𝑏𝑙𝑒 𝑜𝑢𝑡𝑐𝑜𝑚𝑒𝑠 𝑜𝑛 𝑛
Now 4 soldiers can be selected out of 15 . 𝑑𝑖𝑐𝑒𝑠][𝑒𝑣𝑒𝑟𝑦 𝑑𝑖𝑐𝑒 𝑠ℎ𝑜𝑤𝑛 𝑜 𝑜𝑑𝑑 𝑛𝑢𝑚𝑏𝑒𝑟]
in 15𝐶4 ways = 1365. Again, if can batch
has two watch for 4 hours, then 1365 ⟹ Required ways = 6𝑛 − 3𝑛 = 189 (𝑔𝑖𝑣𝑒𝑛).
batches will watch for 1365 × 4 = 5460.
𝐵𝑦 𝐻𝑖𝑡 𝑎𝑛𝑑 𝑇𝑟𝑖𝑎𝑙 𝑡ℎ𝑒 𝑒𝑞𝑢𝑎𝑙𝑖𝑡𝑦 𝑖𝑠 𝑡𝑟𝑢𝑒 𝑓𝑜𝑟 𝑛
= 3.
288
Challenging Mathematical Problems
289
Challenging Mathematical Problems
Sol.: In general, we know that, for the 29) In how many ways the letters of the word
distribution equation 𝑥1 + 𝑥2 + 𝑥3 + ⋯ + PERSON can be placed in the squares of
𝑥𝑛 = 𝑛. The number of ways in which n the adjoining fig. So that no row remains
things can be distributed among r in such empty?
a ways end can receive none, one or more (a) 81; (b) 18720; (c) 18721; (d) none
or all of n items are 𝑛 + 𝑟 − 1𝐶𝑟−1 .
Sol.: In PERSON total letters = 6 which
∴ for the distribution equation are to be filled in 8 squares.
⟹ 𝑊 = (𝑛𝐶𝑛−1 + 𝑛𝐶𝑛 ) + 𝑛 + 1𝐶𝑛−1 + ⋯ + Now the three players (not able to throw)
2𝑛 − 1𝐶𝑛−1 the ball in first placed and 1 youngest
player i.e. 4 can arrange themselves in 4!
⟹ 𝑊 = {(𝑛 + 1𝐶𝑛 + 𝑛 − 1𝐶𝑛−1 ) + ⋯ + Ways to thrown the ball.
2𝑛 − 1𝐶𝑛−1 } − 𝑛𝐶𝑛
∴ Required ways = 4𝐶1 4! = 96.
…………………………………………………
31) The total number of ways in which a
…………………………………………………… bigger can be given at least one rupee from
four 25 paise coins three 50 paise coins
⟹ 𝑊 = (2𝑛𝐶𝑛 + 2𝑛 − 1𝐶𝑛−1 ) − 𝑛𝐶𝑛 ⟹ and 2 one rupee coin is
𝑊 = 2𝑛𝐶𝑛 − 𝑛𝐶𝑛 (a) 55; (b) 54; (c) 53; (d) none
290
Challenging Mathematical Problems
The next prize can be given in 20 ways (a) 𝟑𝟏𝟐 ; (b) 𝟑𝟏𝟐 − 𝟏𝟐; (c) 𝟑𝟏𝟐 + 𝟏;
(d) none
The next prize can be given in 20 ways.
Sol.: First stall can be filled in 3 ways, 2nd
The last prize can be given in 20 ways. stall in 3 ways and so on. Similarly, 12th
Total number of ways all the four prizes stall in 3 ways.
can be given is 20 × 20 × 20 × 20 = ∴ Number of ways of loading steamer is
160000.
3 × 3 × 3 … . .× 3(12 𝑡𝑖𝑚𝑒𝑠) = 312 .
33) The number of ways in which 4 particular
persons A, B, C, D and 6 move persons can
stand in a queue. So that A always stand (SUBJECTIVE TYPE)
before B. B stand C and C before D is
𝒏
(a) 10! 4!; (b) 10! -4!; (c)
𝟏𝟎!
; (d) none 1) Evaluate: ∑∞ 𝑪𝒓
𝒓=𝟎 𝟐𝒏−𝟏
𝟒! 𝑪𝒓
Sol.: Total number of arrangements of 10 Sol.: A general method of finding the sum
persons when there is no restriction = of a series ∑∞
𝑟=0 𝑢𝑟 is to express
10! 𝑢𝑟 𝑎𝑛𝑑 𝑣𝑟 − 𝑣𝑟+1 so that 𝑣𝑛 → 0 𝑎𝑠 𝑛 →
∞, 𝑤𝑒 𝑔𝑒𝑡.
Number of ways in which A, B, C, D can be
∞ ∞
arranged among themselves = 4!
Lt ∑ 𝑢𝑟 = Lt ∑(𝑣𝑟 − 𝑣𝑟+1 )
𝑛 →∞ 𝑛 →∞
∴ Number of arrangements of 10 persons 𝑟=0 𝑟=0
when A, B, C, D occurs in a particular
10!
= Lt (𝑣0 − 𝑣𝑟+1 ) = 𝑣0 .
𝑛 →∞
order = 4!
In the present case, it is easily verified
34) A father with 8 children taken 3 at a time that
to Nicco Park, as often as he can without
talking the same children together more 𝑛𝐶𝑟 𝑛𝐶 ∟𝑛 ∟𝑟 ∟2𝑛 − 𝑟
− 𝑟+1 =
than once. How often will be father go? 2𝑛𝐶𝑟 2𝑛𝐶𝑟+1 ∟𝑟 ∟𝑛 − 𝑟 ∟2𝑛
(a) 56; (b) 106 ; (c) 206; (d) none. ∟𝑛 ∟𝑟 + 1
−
∟𝑟 + 1 ∟𝑛 − 𝑟 − 1
Sol.: [The number of times he can select 3
children out of 8]= [The number of visits ∟2𝑛 − 𝑟 − 1
he (the father) can make] ∟2𝑛
∟𝑛 ∟2𝑛 − 𝑟 ∟2𝑛 − 𝑟 − 1
= [ ]
∴ The number of ways of selecting 3 ∟2𝑛 ∟𝑛 − 𝑟 ∟𝑛 − 𝑟 − 1
children out of 8 = 8𝐶3 = 56. ∟𝑛 ∟2𝑛 − 𝑟 − 1
=
∟2𝑛 ∟𝑛 − 𝑟
35) In a steamer there are stalls for 12 animals
[(2𝑛 − 𝑟) − (𝑛 − 𝑟)]
and there are horse cows and calves (not
1 ∟𝑛 ∟2𝑛 − 𝑟 − 1
less than 12 each) ready to be shipped. In =
2 ∟2𝑛 − 1 ∟𝑛 − 𝑟
how many ways can the ship load be
made?
291
Challenging Mathematical Problems
∞
1 𝑛𝐶𝑟 𝑛𝐶𝑟 (2𝑛 + 𝑟 − 1)!
= ∑ ={ } (2𝑛)!
2 2𝑛 − 1𝐶𝑟 2𝑛 − 1𝐶𝑟 (2𝑛)! (𝑟 − 1)!
𝑟=0
∞ = (2𝑛)! 2𝑛 + 𝑟 − 1𝐶2𝑛
𝑛𝐶 𝑛𝐶
= 2 ∑ [ 𝑟 − 𝑟+1 ] = (2𝑛)!
2𝑛𝐶𝑟 2𝑛𝐶𝑟+1
𝑟=0
(A natural number). Hence, P is divisible
𝑛𝐶 by (2n)!
=2 0 =2
2𝑛𝐶0
4) How many 3-digit numbers are of the from
2) Given that the number C is greater than 1, 𝒂𝒃𝒄, with a, c < b and a ≠ 0?
show that one of the two number
√𝒄 + 𝟏 − √𝑪, √𝑪 − √𝑪 − 𝟏 is always Sol.: Since the digit at hundred’s place
greater than the number. cannot be zero
= (−1)2𝑛 𝑟(𝑟 + 1)(𝑟 + 2) … . (𝑟 5) There are two sets of parallel lines, their
+ 2𝑛 − 1) equations 𝒙 𝐜𝐨𝐬 𝜶 + 𝒚 𝐬𝐢𝐧 𝜶 = 𝑷; 𝑷 =
= 𝑟(𝑟 + 1)(𝑟 + 2) … . (𝑟 𝟏, 𝟐, … . , 𝒎 𝒂𝒏𝒅
+ 2𝑛 − 1) 𝒚 𝐜𝐨𝐬 𝜶 − 𝒙 𝐬𝐢𝐧 𝜶 = 𝒒; 𝒒 =
𝟏, 𝟐, … . , 𝒏(𝒏 > 𝑚) where 𝛼 is a given
(𝑟 + 1)! 𝑟(𝑟 + 1)(𝑟 + 2) … . (𝑟 + 2𝑛 − 1) const. Show that the lines from
=
(𝑟 − 1)! 𝟏
𝒎(𝒎 − 𝟏)(𝟑𝒏 − 𝒎 − 𝟏) squares.
(𝑟 + 2𝑛 − 1)! 𝟔
=
(𝑟 − 1)!
292
Challenging Mathematical Problems
Sol.: The equation 𝑥 cos 𝛼 + 𝑦 sin 𝛼 = 𝑝; 𝑝 = Therefore, number of squares whose sides
1, 2, 3, … . , 𝑚 represents m parallel lines such are of length 2 units = (𝑚 − 2)(𝑛 − 2)
that the distance between two consecutive containing in this, manner, we find that the
lines is one unit. Similarly the equation number of squares whose sides are of length
𝑦 cos 𝛼 − 𝑥 sin 𝛼 = 𝑞; 𝑞 = 1, 2, … . , 𝑛 2 units = (𝑚 − (𝑚 − 1))(𝑛 − (𝑚 − 1))
represents n parallel lines such that the
distance between any two consecutive lines is Hence, Total number of squares
one unit.
= (𝑚 − 1)(𝑛 − 1) + (𝑚 − 2)(𝑛 − 2) + ⋯ +
We observe that the slope of each line of first (𝑚 − (𝑚 − 1))(𝑛 − (𝑚 − 1))
set is 𝑚1 = − cot 𝛼 and the slope of each line 𝑚−1
of second set is 𝑚2 = tan 𝛼.
= ∑ (𝑚 − 𝑟)(𝑛 − 𝑟)
Clearly, 𝑚1 𝑚2 = −1 𝑟=1
𝑚−1
Therefore every line of first set is
= ∑ {𝑚𝑛 − 𝑟(𝑚 + 𝑛) + 𝑟 2 }
perpendicular to every line of second set. 𝑟=1
= 𝑚𝑛(𝑚 − 1) − (𝑚 + 𝑛)
We observe that four lines consisting of two
lines of the first set and two lines of the 𝑚−1 𝑚−1
second set will form a square, if the distance ∑ 𝑟 + ∑ 𝑟2
between two parallel lines of first set is same 𝑟=1 𝑟=1
as the distance between two parallel lines of
𝑚(𝑚 − 1)
second set. = 𝑚𝑛(𝑚 − 1) − (𝑚 + 𝑛)
2
(𝑚 − 1)𝑚(2𝑚 − 1)
Since m < n therefore the length of the side of +
the largest square formed by the two sets of 6
lines is (𝑚 − 1) units and the length of the 𝑚(𝑚 − 1)
side of the smallest square is 1 unit. Clearly, = {6𝑛 − 3(𝑚 + 𝑛) + (2𝑚 − 1)}
6
two lines at a unit distance from the set of m
𝑚(𝑚−1)
parallel lines can be chosen in (𝑚 − 1) ways, = {6𝑛 − 3𝑚 − 3𝑛 + 2𝑚 + 1} =
6
namely (1, 2) (2, 3) (3, 4)……., (m-1, m) and 𝑚(𝑚−1)(3𝑛−𝑚−1)
6
.
two lines at a unit distance from the set of n
parallel lines can be chosen in (𝑚 − 1) ways,
namely (1, 2)(2, 3), …., (n-1, n). Therefore
number of squares whose sides are of length 6) There are n straight lines in a plane such
1 unit (m-1)(n-1). that 𝒏𝟏 are parallel in different direction,
𝒏𝟐 are parallel in different direction and so
Similarly two lines at a distance of 2 units on, 𝒏𝒌 𝒂𝒓𝒆 parallel in another direction
from the set of m parallel lines can be chosen such that 𝒏𝟏 + 𝒏𝟐 + ⋯ 𝒏𝟐 = 𝒏. Also no
in (𝑚 − 2) ways, namely (1, 3) (2, 4), …, (m-2, three of the given lines meet a point. Show
m) and two lines at a distance of 2 units from that the total number of inter section is
the set of n parallel lines can be chosen in (n - 𝟏
(𝒏𝟐 − ∑𝒌𝒓=𝟏 𝒏𝒓 𝟐 ).
2) ways, namely (1, 3)(2, 4), ….., (n -2, n) 𝟐
293
Challenging Mathematical Problems
Sol.: If no two of n given lines are parallel Thus, we have the following cases for the
and no three of them meet at a point, then selection of seats.
the total number of points of intersection
is 𝑛𝐶2 . But it is given that there are k sets Case I: When two seats are selected on the
right side sixth seat and one seat on its right
of 𝑛1 , 𝑛2 , 𝑛3 , … , 𝑛𝑘 parallel lines such that
side;
no line in one set is parallel to a lines in
any other set. Also lines of one set do not Since no two adjacent seats are selected, so
intersect with each other. we can select either 1st and 3rd or 2nd and 4th
or 1st and 4th seats. So, there are 3 ways to
Therefore, lines of one set do not provide
select 2 seat on the left side of sixth seat one
any points of intersection. Hence, total
the right side of sixth seat there are 9 seats.
number of points of intersection
Therefore, one seat (excluding 7th seat) on the
= 𝑛𝐶2 − (𝑛1 𝐶 + 𝑛2 𝐶 + ⋯ + 𝑛𝑘 𝐶 +) right side of sixth seat can be chosen in 8
2 2 2
ways.
𝑛(𝑛 − 1) 𝑛1 (𝑛1 − 1) 𝑛2 (𝑛2 − 1)
= −{ + Case II: When two seats are selected on the
2 2 2
right side of sixth seat and one seat on its left
𝑛𝑘 (𝑛𝑘 − 1) side.
+ ⋯+ }
2
In this case, one seats are selected on the
𝑛(𝑛 − 1) 1 right side of sixth seat and one seat on its left
= − {(𝑛1 2 + 𝑛2 2 + ⋯ + 𝑛𝑘 2 )
2 2 side.
− (𝑛1 + 𝑛2 + ⋯ + 𝑛𝑘 )}
In this case, one seat on the left side of sixth
𝑛(𝑛 − 1) 1
= − {(𝑛1 2 + 𝑛2 2 + ⋯ + 𝑛𝑘 2 ) seat can be chosen in 4 ways (any one of the
2 2 first four seats) and the number of ways of
− 𝑛}
selecting two seats on the right side of sixth
𝑛2 1 seat is same the number of non-negative
= − (𝑛 2 + 𝑛2 2 + ⋯ + 𝑛𝑘 2 )
2 2 1 integral solutions of the equation 𝑥1 + 𝑥2 +
𝑘
1 𝑥3 = 7, where 𝑥1 ≥ 1, 𝑥2 ≥ 1 𝑎𝑛𝑑 𝑥3 ≥ 0.
= [𝑛2 − ∑ 𝑛𝑟 2 ]
2 Here, 𝑥1 is the number of vacant seats
𝑟=1
between sixth seat and the first seat selected
on the right side of sixth seat, 𝑥2 𝑖𝑠 the
7) There are 15 seats in as row numbered as number of vacant seats between sixth seat
1 to 15. In how many ways can 4 persons and the first seat selected on the right side of
sit in such a way that seat number 6 is sixth seat, 𝑥2 is the number of vacant seats
always occupied and no two person sit in between first and second seat selected on the
adjacent seats. right side of sixth seat and 𝑥3 is the number of
vacant seats on the right side of the second
Sol.: Since seat number ‘6’ is always occupied selected seat.
and no two persons can occupy adjacent
seats. Therefore , at most two persons can sit Let 𝑦1 = 𝑥1 − 1, 𝑦2 = 𝑥2 − 1 𝑎𝑛𝑑 𝑦3 = 𝑥3 .
on the left side of sixth seat. Then 𝑥1 + 𝑥2 + 𝑥3 = 7
294
Challenging Mathematical Problems
295
Challenging Mathematical Problems
𝑛 (2𝑚+1)
Case II: If 𝑛 = 2𝑚 + 1 𝑆 = ∑𝐾=1 (2𝑘 −
∑ 𝑇𝑘 . (𝑛 − 𝑘)! = 𝑛!
2𝑚 − 1) 𝑥𝑘
𝑘=1
𝑛2
∴ 𝑆𝑚𝑎𝑥 = 1 + 3 + ⋯ + (2𝑚 − 1) = 𝑚2 =
4
296
Challenging Mathematical Problems
∞ 𝟐 9
{∑(−1) 𝑟−1
. 𝑐2𝑟−2 } = ∑ 𝑥 𝑘 {(𝑥 10 )𝑘𝑘𝑘 − 1}
𝑟=1 𝑘=0
2 9
= (𝑐0 − 𝑐2 + 𝑐4 … . ) … … … . (𝑖)𝑎𝑛𝑑
∞ 2 = {(𝑥 10 )𝑘𝑘𝑘 − 1} ∑ 𝑥 𝑘
{∑(−1)𝑟−1 . 𝑐2𝑟−1 } 𝑘=0
9
𝑟=1
= (𝑐1 − 𝑐3 = 𝑀 ∑ 𝑥𝑘,
𝑘=0
+ 𝑐5 … . )2 … … (𝑖𝑖)
⟹ 𝐵 = (𝑀 + 1) ∑9𝑘=0 𝑥 𝑘 , 𝑖. 𝑒. ∑9𝑘=0 𝑥 𝑘𝑘𝑘 is
divisible by ∑9𝑘=0 𝑥 𝑘 .
𝐺𝑖𝑣𝑒𝑛: (1 + 𝑥)𝑛 = 𝑐0 + 𝑐1 𝑥 + 𝑐2 𝑥 2 + ⋯ +
𝑐𝑛 𝑥 4 … … . (𝑖𝑖𝑖)
(𝑖𝑣) × (𝑣) = {(1 + 𝑖)(1 − 𝑖)}𝑛 = Sol.: Let 𝑓(𝑥) = 𝑥 𝑚 , 𝑡ℎ𝑒𝑛 𝑓 𝑛 (𝑥) =
(𝑐0 − 𝑐2 + 𝑐4 … . )2 + (𝑐1 − 𝑐3 + 𝑐5 … . )2 𝑚(𝑚 − 1)(𝑚 − 2) … . (𝑚 + 1 −
𝑛)𝑥 𝑚−𝑛 𝑤ℎ𝑒𝑟𝑒 𝑓 𝑛 (𝑥)𝑖𝑠 𝑛𝑡ℎ derivative of f(x).
⟹ 2𝑛 = (𝑐0 − 𝑐2 + 𝑐4 … . )2 + (𝑐1 − 𝑐3 +
𝑚!
𝑐5 … . )2 , ∴ 𝑓 𝑛 (𝑥) = 𝑥 𝑚−𝑛
(𝑚 − 𝑛)!
⟹ ∑𝑛𝑟=0 𝑐𝑟 = (𝑐0 − 𝑐2 + 𝑐4 … . )2 +
𝐿𝑒𝑡 𝑔(𝑥) = 𝑥 8 + 𝑥 7 , 𝑡ℎ𝑒𝑛 𝑔𝑛 (𝑥) =
(𝑐1 − 𝑐3 + 𝑐5 … . )2 8! 7!
(8−𝑛)!
𝑥 8−𝑛 + (7−𝑛)! 𝑥 7−𝑛 Multiplying both
sides by (1+ x), then
Sol.: Let
297
Challenging Mathematical Problems
After (8 − 𝑛)𝑛𝑡ℎ differentiation of (i) we get The same is true for all k, so ∑ 𝑆(𝑎) =
7! (𝑛+1)!
𝑥 8−𝑛 … … . . (𝑖) if into the from ax +b. ∑𝑛𝑖=1 𝑘𝑖 … … … . (1)
(7−𝑛)! 2
Let h(x) = (x+1) 𝑔𝑛 (𝑥). Second way, if n! is not a divisors of 𝑆(𝑏) − 𝑆(𝑐)
for any b ≠ c, then each 𝑆(𝑎) must have a
8! (8 − 𝑛)!
𝑁𝑜𝑤 ℎ8−𝑛 (𝑥) = . different remainder mod n! Since there are n!
(8 − 𝑛)! 0!
permutations, these remainders must be
7! (9 − 𝑛)!
+ . precisely the numbers 0, 1, 2, …, n! -1. Thus
(7 − 𝑛)! 7!
8! (9 − 𝑛)! (𝑛!−1)𝑛!
+ . 𝑥 ∑ 𝑆(𝑎) = 𝑚𝑜𝑑 𝑛! … … … … . . (2)
(8 − 𝑛)! 2
7!
= 8! (9 − 𝑛)𝑥 + 7! (8 − 𝑛) Combining (1) and (2), we get
+ 8.7!
𝑛
(𝑛 + 1)!
= 8! (9 − 𝑛)𝑥 + 7! (16 − 𝑛)𝑐𝑜𝑚𝑝𝑎𝑟𝑖𝑛𝑔 𝑎 = ∑ 𝑘𝑖
2
8! (9 − 𝑛)𝑥 𝑖=1
(𝑛! − 1)𝑛!
= 7! (72 − 8𝑛)! 𝑏 = 7! (16 − 𝑛) ≡ 𝑚𝑜𝑑 𝑛! … … … … . (3)
2
298
Challenging Mathematical Problems
(𝑥𝑛 , 𝑥𝑛−1 , … , 𝑥1 ), 𝑖𝑓 |𝑆(𝜋0 )| ≤ 𝑆(𝜋𝑖 ) must lie in this interval. For this
(𝑛+1) (𝑛+1) (𝑛+1)
𝑜𝑟 |𝑆(𝜋)| ≤ , then we are done. particular 𝜋𝑖 we have |𝑆(𝜋𝑖 )| ≤ 2
.
2 2
Thus we assume |𝑆(𝜋0 )| > (𝑛 +
(𝑛+1) 16) Find the number of non-degenerate
1)𝑎𝑛𝑑 |𝑆(𝜋)| > 2 triangle whose vertices lie in set of points
(s, t) in the plane such that 𝟎 ≤ 𝑺 ≤ 𝟒, 𝟎 ≤
Note that 𝑆(𝜋0 ) + 𝑆(𝜋) = (𝑥1 + 2𝑥2 + ⋯ +
𝒕 ≤ 𝟒, S and t are integers.
𝑛𝑥𝑛 ) + (𝑥𝑛 + 2𝑥𝑛−1 + ⋯ + 𝑛𝑥1 ) =
(𝑛 + 1)(𝑥1 + 𝑥2 + ⋯ + Sol.: There are 25 points in the given set, we
𝑥𝑛 )𝑎𝑛𝑑 ℎ𝑒𝑛𝑐𝑒 𝑡ℎ𝑎𝑡 |𝑆(𝜋0 ) + 𝑆(𝜋)| = 𝑛 + 1. can choose 3 out of them in (25 ) ways. Let us
(𝑛+1) 3
Since each of 𝑆(𝜋0 )𝑎𝑛𝑑 𝑆(𝜋) 𝑒𝑥𝑐𝑒𝑒𝑑𝑠 2
is count the number of ways in which the 3
absolute value, they must have opposite points chosen will lie on a line. L : The given
signs. Thus, one of 𝑆(𝜋0 )𝑎𝑛𝑑𝑆(𝜋) is greater set S contains 5 horizontal lines 5 points each.
(𝑛+1) (𝑛+1)
than , and the other is less than − .
2 2 We can choose 3 points from any of them in
Now, starting from 𝜋0 , we can obtain any (53) ways.
permutation by successive, transpositions of
neighboring elements. In particular, there Hence the number of ways in which L can be
exists a chain 𝜋0 , 𝜋1 , … . , 𝜋𝑚 of permutations a horizontal line is 5. (53) = 50. Similarly the
𝜋𝑥+1 is obtained from 𝜋𝑖 by interchanging number of ways in which L can be a vertical
two of its neighboring terms. line is 50.
This means that if 𝜋𝑖 = As shown in fig.(ii) S contains 5 lines of slope
𝑦1 , 𝑦2 , … … . . , 𝑦𝑛 𝑎𝑛𝑑 𝜋𝑖+1 = 1; one line contain 5 points, 2 lines contain 4
(𝑧1 , 𝑧2 , … , 𝑧𝑛 ) 𝑡ℎ𝑒𝑛 𝑡ℎ𝑒𝑟𝑒 𝑖𝑠 𝑎𝑛 𝑖𝑛𝑑𝑒𝑥 𝑘, 1 ≤
points each and 2 lines contain 3 points each.
𝑘 ≤ 𝑛 − 1, such that 𝑧𝑘 = 𝑦𝑘+1 , 𝑧𝑘+1 = So the number of ways in which L can be line
𝑦𝑘 𝑎𝑛𝑑 𝑧𝑖 = 𝑦𝑖 , 𝑗 ≠ 𝑘, 𝑗 ≠ 𝑘 + 1.
of slope 1 is (53) + 2(43) + 2(33) = 20.
(𝑛+1)
Because the numbers 𝑥𝑖 do not exceed 2 Similarly, the number of ways in which L can
in absolute value, we have |𝑆(𝜋𝑖+1 ) − be a line of slope −1 𝑖𝑠 20.
𝑆(𝜋𝑖 )| = |𝑘𝑥𝑘 + (𝑘 + 1)𝑧𝑘+1 − 𝑘𝑦𝑘 −
1
(𝑘 + 1)𝑦𝑘+1 | = |𝑦𝑘 − 𝑦𝑘+1 | ≤ |𝑦𝑘 | + |𝑦𝑘+1 | ≤ As shown in fig (ii) there are 3 lines of slope 2
𝑛 + 1. It follows that the difference between each containing 3 points; and there are 3 lines
any two consecutive numbers in the sequence of slope 2, each containing 3 points. So the
𝑆(𝜋0 ), 𝑆(𝜋1 ), … . , 𝑆(𝜋𝑚 ) is at most n +1 in number of ways in which L can have slope 2
1
absolute value. Recall that the numbers
or 2 is 6 (33)= 6 similarly L can have slope
𝑆(𝜋0 ) 𝑎𝑛𝑑 𝑆(𝜋𝑚 ) = 𝑆(∼ 𝜋
) regarded as points
1
on the real line, lie outside of and on opposite − 𝑜𝑟 − 2 𝑖𝑛 6 ways.
2
(𝑛+1) (𝑛+1)
sides of the interval [− , ].
2 2 Since no other line can contain more than two
points of S, the number of ways in which the 3
Because this interval has length n +1, it
points chosen will lie is
follows that at least. One of the numbers
50+50+20+20+6+6= 152
299
Challenging Mathematical Problems
The required number of triangle is therefore Sol.: (i) S consists of single digit numbers, two
(25
3
) − 152 = 2148. digits numbers three digits numbers and four
digit numbers.
17) For non-negative integers n, r the binomial
coefficient (𝒏𝒓) denotes the number of No. of single digit number = 4, No. of two
digit number = 4 × 3 = 12 (since repeatation
combinations of n objects chosen r at a
is not allowed, there are four choices for ten’s
time, with the convention that (𝒏𝟎) =
place and three choices for unit’s place)
𝟏 𝒂𝒏𝒅 (𝒏𝒓) = 𝟎 𝒊𝒇 𝒏 < 𝑟.
No. of three digit number = 4 × 3 × 2 = 24
𝒏−𝒓+𝟏 𝒓−𝟏 𝒏
Prove that ∑∞ 𝒅=𝟏( 𝒅 ) (𝒅−𝟏) = ( 𝒓 ) for all
integers n, r with 𝟏 ≤ 𝒓 ≤ 𝒏. No. of four digit number = 4 × 3 × 2 × 1 =
24
Sol.: We use a combinatorial argument to
establish the obviously equivalent identity ∴ n(s) = 4 +12+ 24+ 24 = 64.
300
Challenging Mathematical Problems
[Since there are 24 digit numbers, each of 1, 19) Find the number of 6 digit natural
24 numbers where each digit appears at least
2, 3, 4 occurs in each of the four digits in 4
=
twice.
6 times]
Sol.: We consider number like 222222 or
So, the sum of all the single digit, two digit,
233200 but not 212222. Since the digit 1
three digit and four digit number =
occurs only once.
10+330+6660+66660= 73, 660.
The set of all such 6 digits can be divided into
(i) There are just four single digits
following classes.
numbers 1, 2, 3, 4.
(ii) There are 4 × 4 = 16 two digits 𝑆1 = the set of all 6 digit numbers where a
numbers, as digit can be repeated. single digit is repeated 6 times.
(iii) There are 4 × 4 × 4 = 64 three
digit numbers. 𝑛(𝑆1 )= 9.
(iv) There are 4 × 4 × 4 × 4 = 256
four digit numbers. Since ‘0’ cannot be a significant number when
all its digits are zero.
So, that total number of numbers up to 4 digit
numbers that could be formed using the 𝐿𝑒𝑡 𝑆2 be the set of all six digit numbers,
digits 1, 2, 3 and 4 is 4 + 16 + 64 + 256 = made up of three distinct digits.
340. Here we should have two cases : 𝑆2 (𝑎) one
Sum of the 4 single digit numbers = with the exclusion of zero as a digit and other
𝑆2 (𝑏) with the inclusion of zero as a digit.
1+2+3+4= 10. To find the sum of 16, two
digit number each of 1, 2, 3, 4 occur in each of 𝑆2 (𝑎) The numbers of ways, three digit could
16
units and ten’s place 4
= 4 𝑡𝑖𝑚𝑒𝑠. So, the be chosen from 1, 2, …, 9 is 9𝐶3 . Each of these
sum of all these 16 numbers is = 10 × three digits occurs twice. So, the number of
4(1 + 2 + 3 + 4) + 4(1 + 2 + 3 + 4) = 400 + six digit number in this case is
40 = 440.
6! 9 × 8 × 7 720
9𝐶3 × = ×
Similarly, the sum of all the 64 three digit 2! × 2! × 2! 1 × 2 × 3 8
numbers 100 ×
64
× (1 + 2 + 3 + 4) + 10 × = 9 × 8 × 7 × 15 = 7560.
4
64 64
× (1 + 2 + 3 + 4) + 1 × × (1 + 2 + 3 + 𝑆2 (𝑏) the three digits used include one zero,
4 4
4) = 16,000 + 1,600 + 160 = 17,760. implying we have to choose the other two
digits from the 9 non zero digits.
Again the sum of al the 256 four digit
9×8
numbers = 1000 ×
256
× (1 + 2 + 3 + 4) = This could be done in 9𝐶2 = 1.2
= 36. Since
4
6,40,000 + 64,000 + 6,400 + 640 = zero can not be in the leading digit. So let us
7,11, 040. fix one of the fixed non-zero number in the
extreme left. Then the other five digits are
Therefore, sum of all the number is 10 + 440 made up of 2 zeros, 2 fixed non zero number
+17,760+ 7,11,040= 7,29,250. and the another non-zero number, one of
which is put in the extreme left.
301
Challenging Mathematical Problems
In this case the number of six digit numbers If a and b are the two non-zero numbers a
5 6!
that could be formed is 2!×2!×2! × 2 (since used twice and b four times, then we get 2!×4!
from either of the pairs of fixed non-zero and when a used four times, b twice, we again
numbers, one can occupy the extreme digit) 6!
get 2!×4!. So, when 2 of the nine non-zero
= 60.
digits are used to from the six digit number in
So, the total number in this case = 36 × 60 = this case, the total numbers got is 9𝐶2 × 2 ×
6!
2160. = 36 × 5 × 6 = 1080.
4!×2!
Now let us take 𝑆4 , the case where the six 20) If x = {1, 2, 3, … 4}, where n ∊N, show that
digit number consists of exactly two digits, the number of r combinations of x which
one of which occurs twice and the other four contain no consecutive integer is given by
times. (𝒏−𝒓+𝟏 ), 𝒘𝒉𝒆𝒓𝒆 𝟎 ≤ 𝒓 ≤ 𝒏 − 𝒓 + 𝟏.
𝒓
302
Challenging Mathematical Problems
303
Challenging Mathematical Problems
304
Challenging Mathematical Problems
𝑚
2
[∑(𝑚𝐶𝑟 ) = 2𝑚𝐶𝑚 ]
𝑟=0
305
Challenging Mathematical Problems
306
Challenging Mathematical Problems
So, total number of isosceles but non- 27) Find the number of integer solutions to the
equilateral triangles are 9 × 21 = 189. But equation 𝒙𝟏 + 𝒙𝟐 + 𝒙𝟑 = 𝟐𝟖 𝒘𝒉𝒆𝒓𝒆 𝟑 ≤
the 7 equilateral triangles are also to be 𝒙𝟏 ≤ 𝟗, 𝟎 ≤ 𝒙𝟐 ≤ 𝒑 𝒂𝒏𝒅 𝟕 ≤ 𝒙𝟑 ≤ 𝟏𝟕
considered as isosceles.
Sol.: considered three numbered boxes whose
∴Total number of isosceles triangle are 196. contents are denoted as 𝑥1 , 𝑥2 , 𝑥3
Note this problem can be generalized to a respectively. The problem now reduces to
polygon having n vertices. Find the number of distributing 28 balls in the three boxes such
acute, obtuse, right, isosceles and equilateral that the first box has at least 3 and not more
triangles. than 9 balls, the second box has at most 8
balls and the third box has at least 7 and at
25) Show that for any set of 10 points chosen most, 17 balls. In first put 3 balls in the first
within a square whose sides are of length box, and 7 balls in the third box. So, now the
3 units, there are two points in the set problem reduces to finding the number of
whose distance most √𝟐. distribution of 18 balls in 3 boxes such that
the first has at most (9- 3) = 6, the second at
Sol.: Divide the square into 9 unit squares as
most 8 and the third at most (17-7)=10. The
given in the figure. Out of the 10 points
number of ways of distributing 18 balls in 3
distributed in the big square, at least one of
boxes with no condition is (18+3−1 ) = (20 )=
the small squares must have at least two 3−1 2
307
Challenging Mathematical Problems
308
Challenging Mathematical Problems
(i) Every two colours appear on (condition 4 satisfied). Now, consider the four
exactly one ball. colour GRYV. No. three of these colours are
(ii) Every two balls have exactly found on a ball. (condition 3 is satisfied).
one colour in common. Thus, the total number of colour is 7 and the
(iii) There are four colours such total number of balls is also 7.
that any three of them appear
on one ball. 31) A mathematical conjection consisted of a
(iv) Each ball has three colours. part I and part II with a combined total of
Find the number of balls and 28 problems. Each contestant solved 7
colour used. problems altogether. For each pair of
problems there were exactly two
Sol.: Let us represent each of the balls by a contestants who solved both of them.
line segment with three points to show the 3 Show that there was a constants who in
colours. Thus, Roy is a ball with three colours part I solved either no problem or at least
red, orange and yellow. We have to have 4 problems.
three more balls such that on yellow. So, next
drawn lines through R, O, Y to meet at a Sol.: We will find the total number of
common point G standing for green colour. contestants. Since for each pair of problems
But the balls with colours RG, OG AND YG there were exactly two contestants let us
must have a third colour in them say indigo assume that an arbitrary problem 𝑃1 was
(i), violet (v) and Blue (B). Thus we have 7 solved by r contestants. Each of these r
balls and 7 colours in all. 7 colours R, O, Y, G, I, constestants solved 6 more problems, solving
V, B and 7 balls. 1. ROY, 2. RIG, 3. RVB, 4. Ova, 6r more problems in all counting
5. YBa, 6. YVI, 7. IBO multiplicants. Since every problem, other
than 𝑃1 , was paired with 𝑃1 and was solved by
exactly two constants, each of the remaining
27 problems (i.e. other than 𝑃1 ) is counted
twice among the problems solved by the r
contestants. i.e. 6r = 2 × 27 𝑜𝑟 𝑟 = 9.
Therefore an arbitrary problem 𝑃1 is solved
by 9 contestants.
9×28
Hence, in all we have 7
= 36 contestants,
as each contestant solves 7 problems.
Clearly any pair of the above 7 balls have From the rest of the proof, let us assume the
exactly one colour in common (satisfying countrary that is every constant solved either
condition 2). Each of the balls contribute 3 1, 2, or 3 problems in part 1.
pairs of colours. In all, we have 21 pairs of Let us assume that there are n problems in
columns in all the 7 balls. Now 7 colours lead part 1 and let x, y, z be the number of
7×6
to 2
= 21 pairs of colours and each pair of contestants who solved 1, 2, and 3 problems
colours is found in exactly one ball satisfying in part 1.
condition 1. Each ball has 3 colours
309
Challenging Mathematical Problems
Since every one of the contestants solve (ii) It first 2 digits are identical.
either 1, 2, or 3 problems in part 1,
Let (i) hold. Then the sequence starts with
we get 𝑥 + 𝑦 + 𝑧 = 36 … … … (1) one of 1, 02, 12, 20, 21. First suppose that it
starts with 01. Now the condition that the
𝑥 + 2𝑦 + 3𝑧 = 9𝑛 … … … . (2) (since each sequence contain “2 consecutive identical
problem was solved by 9 contestants). digits” is symmetric w. r. t. all 3 digits 0, 1, 2.
Since every contestant among y solves of a Hence these are
1
pair of problems in part I and every 𝑎𝑛 𝑒𝑞𝑢𝑎𝑙 𝑎 𝑛𝑢𝑚𝑏𝑒𝑟, 𝑛𝑎𝑚𝑒𝑙𝑦 𝑚 = 3 𝑎𝑛−1 , of
contestants among z solves 3 pairs of sequence of length n -1 and starting with 0, 1,
problems was solved by exactly two or 2. So by appending 0 as first digit to each
contestants, we get the following equations (𝑛 − 1) sequences starting with 1, we get m
sequence of length n which start with 01.
𝑛(𝑛 − 1)
𝑦 + 3𝑧 = 2. 𝑛𝐶2 = 2. Similarly, there are m sequences of length n
2 starting with 02, 01, 12, 20 or 21.
= 𝑛(𝑛
− 1) … … … … (3) Thus there are 6𝑚 = 2𝑎𝑛−1 sequence in this
case. Let (ii) hold. Then the sequences starts
From eq.(1). Eq. (2) and Eq. (3), we get
with 00 or 11 or 22 and its remaining 𝑛 − 2
𝑧 = 𝑛2 − 10𝑛 + 36 𝑎𝑛𝑑 𝑦 = −2𝑛2 + 29𝑛 − digits can form any (𝑛 − 2) ternary sequences
29 2 23 Hence rhere are 3𝑛−2 𝑛 sequence starting
108 = −2 (𝑛 − ) − < 0.
4 8 with 00; and the same holds for 11 and 22.
As y < 0 is not an acceptable result, our Thus there are 3 × 3𝑛−2 = 3𝑛−1 sequences in
assumption is wrong. Hence, there is at least this case.
one contestants who solved either no
problem from part 1 or solved at least 4 Required recurrence relation is 𝑎𝑛 = 2𝑎𝑛−1 +
problems from part 1. 3𝑛−1 with initial conditions𝑎1 = 0, 𝑎2 = 3.
310
Challenging Mathematical Problems
(ii) 𝑃𝑛 has positive coefficients is unique common value to that they approach
therefore an increasing convex 𝑎𝑛 < 𝑡 < 𝑏𝑛 ∀ 𝑛.
function for 𝑥 ≥ 0.
1
(iii) 𝑃𝑛 (0) = 0, 𝑃𝑛 (1) ≥ 1. Number of uniquely satisfies 1 − 𝑛 < 𝑃𝑛 (𝑡) <
(iv) 𝑃𝑛 (𝑥1 ) = 𝑥𝑛 . 1∀ 𝑛.
Since the condition 𝑥𝑛+1 > 𝑥𝑛 is equivalent to 34) Find the number of isosceles triangle with
1
𝑥𝑛 > 1 − integer sides, if no sides a exceeds 1994.
𝑛
We can reformulate the problem as follows Sol.: Let 2 equal sides of an isosceles ∆ be P
show that there is unique positive real units each and let remaining sides be q units.
1
number t such that 1 − < 𝑃𝑛 (𝑡) < 1 for Case I : P > q. q can take values 1, 2, 3, ….., P -1
𝑛
every n. (if 𝑃 − 1 > 0) condition for p, q, q be a sides
of a ∆ is automatically satisfied here, for each
Since 𝑃𝑛 is continuous and increases from 0 to positive integer P > 1, we can have P -1
a value of ≥ 1 for 0 ≤ 𝑥 ≤ 1, there is unique isosceles ∆ is
values 𝑎𝑛 and 𝑏𝑛 such that 𝑎𝑛 < 𝑏𝑛 , 𝑃𝑛 (𝑎𝑛 ) =
1 1994
1 − 𝑛 , 𝑃𝑛 (𝑏𝑛 ) = 1 … … . . (2)
∑ (𝑃 − 1) = 1 + 2 + 3 + ⋯ + 1993
𝑝=2
By definition (1)
(1993 × 1994)
=
1 1 1 1 2
𝑃𝑛+1 (𝑎𝑛 ) = (1 − ) (1 − + ) = 1 − = 1998721.
𝑛 𝑛 𝑛 𝑛
1 case II: p < q in order that p, q, may be sides
𝑃𝑛+1 (𝑎𝑛−1 ) = 1 − . 𝑊𝑒 𝑠𝑒𝑒 𝑡ℎ𝑎𝑡 𝑎𝑛
𝑛+1 of ∆ we must have 2p > q. i.e. 𝑝 < 𝑞 < 2𝑝.
< 𝑎𝑛+1 … … … . (3)
If p is even say 2m, then q can take value 1, 2,
1 …., m -1 if p is odd say 2𝑚 − 1 then q can take
𝐴𝑙𝑠𝑜 𝑠𝑖𝑛𝑐𝑒 𝑃𝑛+1 (𝑏𝑛 ) = 1 + 𝑎𝑛𝑑 𝑃𝑛+1 (𝑏𝑛+1 )
𝑛 𝑝−1
values 1, 2, …., m -1 = ( ). Numbers of
=1 2
1−1 3−1
possible isosceles ∆ is 2 + 2 + ⋯ +
𝑏𝑛 > 𝑏𝑛+1 . Since 𝑃𝑛 is convex, the graph of 1993−1
1 + 1 + 2 + 3 + ⋯ for q = 1994, p +q >p
𝑃𝑛 (𝑥) lies below, the chord 𝑦 = 𝑏𝑛
𝑥 𝑓𝑜𝑟 0 ≤ 2
𝑞
is true. Also, we must have 2 < 𝑝 < 𝑞. If q is
𝑥 ≤ 𝑏𝑛
𝑞−1 𝑞−2
even there are 𝑞 − 2
− 2 = possible
1 𝑎
𝐼𝑛 𝑝𝑎𝑟𝑡𝑖𝑐𝑢𝑙𝑎𝑟 𝑃𝑛 (𝑎𝑛 ) = 1 − 𝑛 ≤ 𝑏𝑛 from this 𝑞−1
𝑛 values for p. If q is odd, there (𝑞 − 1) − 2
−
and the fact than 𝑏𝑛 ≤ 1.we find that 𝑏𝑛 − 𝑞−1
𝑏𝑛 𝑏𝑛 1 2
= possible value for p.
𝑛
≤ 𝑎𝑛 , 𝑏𝑛 − 𝑎𝑛 ≤ 𝑛
≤ for all n.
𝑛
𝑞−2
There are in all isosceles ∑𝑞 𝑒𝑣𝑒𝑛 +
Thus we have2 in finite bounded 2
𝑞−1
sequences{𝑎𝑛 }, {𝑏𝑛 } the first is increasing the ∑𝑞 𝑜𝑑𝑑 ∆𝑠 1 ≤ 𝑞 < 1994
2
second decreasing 𝑎𝑛 > 𝑏𝑛 and the different
between their nth numbers approaches 0 as n 1 ≤ 𝑞 ≤ 1994 1 ≤ 𝑞 ≤ 1994
on creases. We conclude that there is a
311
Challenging Mathematical Problems
312
Challenging Mathematical Problems
𝑓(1 + 1) = 𝑓(1) + 𝑓(1) ⟹ 𝑓(1) = 27(1) Hence, even extension of the function from [0,
4] is 𝑓(−𝑥) = 𝑒 −𝑥 + sin(−𝑥) ⟹ 𝑓(−𝑥) =
𝑥 = 2 𝑦 = 1 𝑓(2 + 1) = 𝑓(2) + 𝑓(1) ⟹
𝑒 −𝑥 − sin 𝑥
𝑓(3) = 2𝑓(1) + 𝑓(1) = 3𝑓(1)
∴ 𝑓(𝑥) = 𝑒 −|𝑥| − sin|𝑥|
𝑥 = 2 𝑦 = 2 𝑓(2 + 2) = 𝑓(2) + 𝑓(2) ⟹
𝑓(4) = 4𝑓(1)
𝜋𝑥 𝜋𝑥 Graphically
Sol.: 𝑓(𝑥) = sin ( 𝑛! ) − cos {((𝑛+1)!)} = 𝑡1 −
𝑡2 𝑝𝑒𝑟𝑖𝑜𝑑 𝑜𝑓
2𝜋
𝑡1 = 𝜋 = 2(𝑛!) = 𝐴 (𝑠𝑎𝑦)
(𝑛!)
⟹ 2𝑏𝑥 + (𝑏 + 𝑐) = 8𝑥 + 3
313
Challenging Mathematical Problems
𝟏
(a) -2; (b) -1; (c) ; (d) none
𝟐
9) If 𝒇(𝒙) be defined for all x > 0 and be
𝑥2 𝑥2 𝒙
Sol.: 𝑓 ( 2 ) + 𝑓(𝑥 2 𝑦 2 ) = cos (log 2 ) + continuous,. Let 𝒇(𝒙) satisfy 𝒇 ( ) =
𝑦 𝑦 𝒚
2 2 )}
cos{log(𝑥 𝑦 = cos(𝑝 − 𝑞) + cos(𝑝 + 𝑞), 𝒇(𝒙) − 𝒇(𝒚) for all x, y and 𝒇(𝒆) =
𝟏 𝒕𝒉𝒆𝒏 𝒇(𝒙) =
𝑤ℎ𝑒𝑟𝑒 𝑝 = log 𝑥 2 .
(a) Bounded; (b) 𝒙𝒇(𝒙) → 𝟏 𝒂𝒔 𝒙 →
𝑞 = log 𝑦 2 = 2 cos 𝑝 . cos 𝑞. 𝟎; (c) 𝐥𝐨𝐠 𝒙 ; (d) none
∴ 𝑅𝑒𝑞𝑑. 𝑣𝑎𝑙𝑢𝑒 = cos 𝑝 . cos 𝑞 − Sol.: If we have 𝑓(𝑥) = log 𝑥 … … … . (𝑖), then
1 𝑥
(cos 𝑝 . cos 𝑞) =0 the conditions 𝑓 (𝑦) = 𝑓(𝑥) − 𝑓(𝑦) 𝑎𝑛𝑑 𝑓(𝑒) = 1
2
are satisfied (i). 𝑓(𝑥) is not bounded as 𝑓(𝑥) is
increasing function.
𝒙𝟏 −𝒙𝟐
7) If 𝒇(𝒙𝟏) − 𝒇(𝒙𝟐) = 𝒇 ( ) 𝒇𝒐𝒓 𝒙𝟏 , 𝒙𝟐 ∈
𝟏−𝒙𝟏 𝒙𝟐
(−𝟏, 𝟏), 𝒕𝒉𝒆𝒏 𝒇(𝒙) =
𝟏−𝒙 𝟏−𝒙 𝟐𝒙 −𝟐−𝒙
(a) 𝐥𝐨𝐠 ; (b) 𝐭𝐚𝐧−𝟏 ; (c) 10) If 𝒇(𝒙) = 𝟐
, 𝒕𝒉𝒆𝒏 𝒇(𝒙+𝒚) 𝒇(𝒙+𝒚) =
𝟏+𝒙 𝟏+𝒙
𝟏 𝟏
𝟏−𝒙 (a) {𝒇(𝟐𝒙) + 𝒇(𝟐𝒚) }; (b) 𝟐 {𝒇(𝟐𝒙) − 𝒇(𝟐𝒚) };
𝐜𝐨𝐭 −𝟏 𝟏+𝒙 ; (d) none 𝟐
𝟏
(c) 𝟐 {𝒇(𝟐𝒚) − 𝒇(𝟐𝒙) }; (d) none
Sol.: For (a) is correct is because LHS =
1−𝑥 1−𝑥 (1−𝑥 )(1+𝑥 )
log 1+𝑥1 − log (1+𝑥2 ) = log (1+𝑥1 )(1−𝑥2 ) 𝑎𝑛𝑑 2𝑥+𝑦 +2−(𝑥+𝑦)
1 2 1 2
Sol.: 𝑓(𝑥 + 𝑦) − 𝑓(𝑥 − 𝑦) = { 2
} ×
2𝑥−𝑦 +2−(𝑥−𝑦) 1
(𝑥 −𝑥 )
{1− 1 2 }
1−𝑥1 𝑥2 (1−𝑥 )(1+𝑥 ) { 2
} = 4 (22𝑥 + 2−2𝑥 + 22𝑦 + 2−2𝑦 )
𝑅𝐻𝑆 = log (𝑥1 −𝑥2 ) = log (1+𝑥1 )(1−𝑥2 ) .
{1+ } 1 2
1−𝑥1 𝑥2
1
= {𝑓(2𝑥) + 𝑓(2𝑦) }
2
𝟏
8) If 2 𝒇(𝒙) − 𝟑𝒇 (𝒙) = 𝒙𝟐 , 𝒙 is not equal to
zero, then 𝒇(𝟐) = 11) If f is even function defined on the interval
(a) -1;
𝟕
(b) − 𝟒; (c) 0; (d)none (−𝟓, 𝟓) then the real values of x satisfying
𝒙+𝟏
the equation 𝒇(𝒙) = 𝒇 ( ) are
𝒙−𝟏
−𝟏±√𝟓 −𝟐±√𝟓
(a) 𝟐
; (b) 𝟐
; (c) 0; (d) none
314
Challenging Mathematical Problems
𝑥+1 𝑥+1
Sol.: 𝑓(𝑥) = 𝑓 (𝑥+2) , ⟹ 𝑥 = 𝑥+2 , ⟹ 𝑥 2 + 2𝑥 =
−1±√5
𝑥 + 1, ⟹ 𝑥 = . Both lie in (−5, 5). (SUBJECTIVE TYPE)
2
12) If 𝒇(𝒙) = 𝒙𝒆𝒙(𝟏−𝒙), then 𝒇(𝒙) is In fact , we will prove a more general
𝟏 property, namely that if f and g are two
(a) Increasing on [− 𝟐 , 𝟏]; (b) decreasing
𝟏 functions defined on the non-negative
on [− 𝟐 , 𝟏]; (c) increasing on R; (d) integers such that 𝑓(𝑛) ≥ 𝑔(𝑛) for all n, and f is
none subjective and bijective, then f = g. The proof
is based on the wall ordering of the set of
Sol.: 𝑓 ′ (𝑥) = 𝑒 𝑥(1−𝑥) . (1 + 𝑥 − 2𝑥 2 ) =
positive integers, namely on the fact that any
−𝑒 𝑥(1−𝑥) . (𝑥 − 1)(2𝑥 + 1) set of positive integers has a smallest
element.
𝑎𝑛𝑑 𝑒 𝑥(1−𝑥) > 0∀ 𝑥, 𝑓(𝑥) 𝑖𝑠 𝑑𝑒𝑐𝑟𝑒𝑎𝑠𝑖𝑛𝑔. ⟹
𝑓 ′ (𝑥) < 0, Assume f ≠ g, and let no be such that 𝑓(𝑛0) >
𝑔(𝑛0) If we let M= 𝑔(𝑛0) , then the set A =
1
⟹ (𝑥 − 1)(2𝑥 + 1) < 0, ⟹ − 2 ≤ 𝑥 ≤ 1. {𝑘, 𝑔(𝑘) ≤ 𝑛} has exactly M +1 elements, since
g is bijective. On the other hand, since, 𝑓 ≥ 𝑔
13) If 𝒇(𝒙) = {𝒙(𝒙 − 𝟑)}𝟐 increase for the and no does not belong to A, the set B =
values of x lying the interval {𝑘, 𝑓(𝑘) ≤ 𝑀} is included in A but has at least
(a) 𝟏 < 𝑥 < 3; (b) 𝟎 < 𝑥 < ∞; (c)
one less element, namely no. Hence the values
−∞ < 𝑥 < 0; (d) none
of f do not exhaust all numbers less than M
𝑑𝑦 +1. Which contradicts the subjectivity of f.
Sol.: 𝑑𝑥 = 2𝑥(𝑥 − 3)(2𝑥 − 3) and for
Therefore, 𝑓(𝑛) = 𝑔(𝑛) = 𝑛 + (−1)𝑛 is the
increasing,
only solution.
𝑑𝑦 3
> 0, ⟹ 0 < 𝑥 < , 3 < 𝑥 < ∞,
𝑑𝑥 2
3
⟹ 𝐼𝑛 𝑃𝑎𝑟𝑡𝑖𝑐𝑢𝑙𝑎𝑟 0 < 𝑥 < 2. 2) Find all function f: N →N with the property
that 𝒇{𝒇(𝒎) + 𝒇(𝒏) } = 𝒎 + 𝒏 for all m and
n.
315
Challenging Mathematical Problems
Thinking of 𝑓(𝑚) 𝑎𝑛𝑑 𝑓(𝑛) as positive integers. 𝑓(𝑛) = 0. A backwards induction shows that,
We have 𝑓(𝑓(𝑓(𝑚) + 𝑓(𝑛) ) + 𝑓(𝑘) ) = 𝑓(𝑚) + 𝑓(𝑛−1) = 𝑓(𝑛−2) = ⋯ = 𝑓(0) = 0
𝑓(𝑛) + 𝑘 on the other hand by using the given
Hence in order for f not be identically zero,
relation for f
there must exist m such that 𝑔(𝑘) = 0 for all
𝑓(𝑚) + 𝑓(𝑛) + 𝑓(𝑘) 𝑤𝑒 𝑜𝑏𝑡𝑎𝑖𝑛 k≥ 𝑚.
𝑓(𝑚 + 𝑛 + 𝑓(𝑘) ) = 𝑓(𝑓 𝑓(𝑚) + 𝑓(𝑛) + Assume m minimal, that is, 𝑔(𝑚−1) ≠ 0, then
𝑓(𝑘) ) = 𝑓(𝑚) + 𝑓(𝑛) + 𝑘 on the one hand, 𝑓(𝑘) = 0 𝑓𝑜𝑟 𝑘 ≥ 𝑚 − 1, and
on the other hand 𝑓(𝑘) = 2𝑘−𝑚 𝑓(𝑚) 𝑓𝑜𝑟 𝑘 >
For m = n = 0 this reduces to 𝑓(𝑓(𝑘) ) = 𝑚, 𝑠𝑜 𝑓𝑜𝑟 𝑘 ≥ 𝑚 the function is strictly
2𝑓(0) + 𝑘. increasing. This together with 𝑓(𝑛) +
𝑓(𝑛+𝑔(𝑛)) = 𝑓(𝑛+1) implies that 𝑛 + 𝑔(𝑛)
Also for k = 0 and m, n arbitrary, we have
cannot exceed m; hence 𝑔(𝑛) > 𝑚 − 𝑛. Thus
𝑓(𝑚 + 𝑛 + 𝑓(0) ) = 𝑓(𝑚) + 𝑓(𝑛) . all other solutions (𝑓. 𝑔)𝑠𝑎𝑡𝑖𝑠𝑓𝑦 𝑓(0) − 𝑓(1) =
⋯ = 𝑓(𝑚−1) = 0,
𝐻𝑒𝑛𝑐𝑒 𝑓 (𝑓(𝑚 + 𝑛 + 𝑓(0) )).
𝑓(𝑘) = 2𝑘−𝑚 𝑎 𝑓𝑜𝑟 𝑘 ≤ 𝑚 and a arbitrary,
The left hand side of this equality is equal to and 𝑔(𝑘) ≤ 𝑚 − 𝑘, 𝑓𝑜𝑟 𝑘 ≤ 𝑚, 𝑔(𝑘) =
2𝑓(0) + 𝑚 + 𝑛 + 𝑓(0) and the right hand side is 0 𝑓𝑜𝑟 𝑘 ≥ 𝑚.
equal to m +n. It follows that 𝑓(0) = 0 and for
all m, n, 𝑓(𝑚+𝑛) = 𝑓(𝑚) + 𝑓(𝑛) , that is, f is
additive. Choose m = 1 and use induction to 4) Let F: N→ be such that 𝒇(𝒏+𝟏) > {𝒇(𝒏) } for
show that 𝑓(𝑛) = 𝑓(1) 𝑛 . From 𝑓(𝑓(𝑚) + 𝑓(𝑛) ) = all n ∊ N. Show that 𝒇(𝒏) = 𝒏 ∀ 𝒏 ∊ 𝑵.
𝑚 + 𝑛 one obtains 𝑓(1)2 (𝑚 + 𝑛) = 𝑚 + 𝑛, for
Sol.: This problem might look easy to people
all m, n. This can happen only if 𝑓(1) = 1, so
familiar with the axiomatic description of the
the only solution to the functional equation is
set of positive integers. The solution uses
the identity function f: N→ N, 𝑓(𝑛) = 𝑛
again the property that every set of natural
3) Find all pairs of functions f, g: N → N numbers has a smallest element.
satisfying 𝒇(𝒏) + 𝒇{𝒏 + 𝒈(𝒏) } = 𝒇(𝒏+𝟏) Let us look at the set
{𝑓(𝑓(1) ), 𝑓(2) , 𝑓(𝑓(2) ), 𝑓(3) , 𝑓(𝑓(3) ), 𝑓(𝑛) , 𝑓(𝑓(𝑛) ), … . }
Sol.: One possibility is that f is identically
equal to 0 and g is arbitrary. Another Note that these are, exactly the numbers that
possibility is that g is identically equal to zero
appear in the inequality 𝑓(𝑓(𝑛) ) < 𝑓(𝑛+1). This
and 𝑓(𝑛) = 2𝑛 𝑓(0) .
set has a smallest element, which cannot be of
Let us find the remaining pairs of functions. the from 𝑓(𝑛+1) because then it, would be
Note that the identically implies 𝑓(𝑛+1) ≥ 𝑓(𝑛) larger than 𝑓(𝑓(𝑛) ). Thus it is of the form
for all n; hence f is increasing. If for a certain 𝑓(𝑓(𝑛) ). The same argument shows that for
n, 𝑔(𝑛) ≥ 1, then 𝑓(𝑛+1) ≤ 𝑓(𝑛+𝑔(𝑛)) ; hence this n, 𝑓(𝑛) =
1. 𝐼𝑓 𝑛 𝑖𝑡𝑠𝑒𝑙𝑓 𝑤𝑒𝑟𝑒 𝑔𝑟𝑒𝑎𝑡𝑒𝑟 𝑡ℎ𝑎𝑛 1, 𝑤𝑒 𝑤𝑜𝑢𝑙𝑑 𝑔𝑒𝑡 1 =
316
Challenging Mathematical Problems
𝑓(𝑛) > 𝑓(𝑓(𝑛−1) ), which is impossible. Hence ℎ𝑒𝑛𝑐𝑒 𝑓(𝑓(𝑛) ) 𝑓(𝑛+2) = 0, which is impossible.
𝑓(1) = 1 𝑎𝑛𝑑 𝑓(𝑛) > 1 𝑓𝑜𝑟 𝑛 > 1. Therefore 𝑓(𝑛) 𝑓𝑜𝑟 𝑛 > 1. we use induction to
show that 𝑓(𝑓(𝑛) ) < 𝑓(𝑛+1) . The inequality is
Considering the restriction𝑓: {𝑛 ≥ 2} → {𝑛 ≥
true for n =1, since 𝑓(2) > 1 =
2}, the same argument applies maintains
mutadis to show that 𝑓(2) = 2 𝑎𝑛𝑑 𝑓(𝑛) > 𝑓(𝑓(1) ). 𝐴𝑙𝑠𝑜 𝑖𝑓 𝑓(𝑛+1) > 𝑓(𝑓(𝑛) ), then 𝑓(𝑛+1) ≥
2 𝑓𝑜𝑟 𝑛 > 2. By induction one shows that 𝑓(𝑓(𝑛) )+1.
𝑓(𝑘) = 𝑘 , 𝑎𝑛𝑑 𝑓(𝑛) 𝑘 𝑓𝑜𝑟 𝑛 > 𝑘 thus the unique
Hence 𝑓(𝑓(𝑛) )𝑓(𝑛+2) + 1 ≥ 𝑓(𝑓(𝑛+1) )𝑓(𝑓(𝑛) ) +
solution to the problem is identify function.
𝑓(𝑓(𝑛+1) ). Since 𝑛 + 1 > 1, 𝑤𝑒 ℎ𝑎𝑣𝑒 𝑓(𝑛+1) >
5) Find all functions f: N →N with the 1, 𝑡ℎ𝑢𝑠 𝑓(𝑓(𝑛+1) ) > 1, which implies that
𝟏
property that for all n ∊ N, + 𝑓(𝑛+2) > 𝑓(𝑓(𝑛+1) ).
𝒇(𝟏) 𝒇(𝟐)
𝟏 𝟏 𝒇{𝒇(𝒏) }
+ ⋯+ 𝒇 = Therefore the function satisfies 𝑓(𝑛+1) >
𝒇(𝟐) 𝒇(𝟑) (𝒏) 𝒇(𝒏+𝟏) 𝒇(𝒏+𝟏)
𝑓(𝑓(𝑛) ) for all N. In view of problem 6, the
Sol.: The equality from the statement reminds only function with this property is the
1 1
us of the well-known identify + + ⋯ + identify function, and we done.
1.2 2.3
1 𝑛
𝑛(𝑛+1)
= 𝑛+1, which shows that the function f:
N → N, 𝑓(𝑛) = 𝑛 is a solutions.
6) Find all function f : No → No satisfying
Let us prove that this is only function with the following two conditions: (ii) For any m , n
required property. ∊𝑵𝟎 , 𝒘𝒊𝒕𝒉 𝒎 ≥ 𝒏, 𝒇(𝒎𝟐) ≥ 𝒇(𝒏𝟐) (i) for
𝟐
(𝑓(𝑛) ) any m, n ∊𝑵𝟎 , 𝟐𝒇(𝒎𝟐 + 𝒏𝟐 ) = {𝒇(𝒎) } +
The ratio 𝑓 𝑓 remainds us of the previous 𝟐
(𝑛+1) {𝒇(𝒏) } ;
problem. In fact, we will reduce the present
problem to the previous one. Sol.: Substituting successively m = 0 and n =
0 in (a) and subtracting the two relations
Plugging in n = 1 into the given relation
yields 𝑓(𝑚)2 − 𝑓(𝑛)2 = 2 (𝑓(𝑚2 ) ) (𝑓(𝑛2 ) )
yields
which together with (b) implies that f is
𝑓(𝑓(1) )𝑓(1) = 1; 𝑙ℎ𝑒𝑛𝑐𝑒 𝑓(1) = 1. increasing i.e. if 𝑚 ≥ 𝑛, then 𝑓(𝑚) ≥ 𝑓(𝑛) .
Plugging m = n = 0 into (b) yields 𝑓(0) =
Replacing the given equality for into the one
0 𝑜𝑟 1.
for n+1 we obtain
Case I : 𝑓(0) = 1, 𝑡ℎ𝑒𝑛 2𝑓(𝑚2 ) = 𝑓(𝑚2 ) + 1, so
𝑓(𝑓(1) ) 1 𝑓(𝑓(𝑛+1) )
+ = 𝑓(1) = 1 𝑝𝑙𝑢𝑔𝑔𝑖𝑛𝑔 𝑚 = 𝑛 = 1 in (a) we get
𝑓(𝑛+1) 𝑓(𝑛+1) 𝑓(𝑛+2) 𝑓(𝑛+2)
1
𝑓(2) = 1. 𝐴𝑙𝑠𝑜 𝑓(2𝑛 ) = 2 (𝑓(22𝑛−1 )2 + 1). This
𝑇ℎ𝑖𝑠 𝑖𝑠 𝑒𝑞𝑢𝑖𝑣𝑎𝑙𝑒𝑛𝑡 𝑡𝑜 𝑓(𝑓(𝑛) )𝑓(𝑛+2) + 1 = implies that 𝑓(2𝑘) = 1 for all non-negative
𝑓(𝑓(𝑛+1) )𝑓(𝑛+1) 𝑁𝑜𝑡𝑒 𝑡ℎ𝑎𝑡 𝑓(𝑛+1) = integers k. By the monotonicity of f, we
1 𝑖𝑚𝑝𝑙𝑖𝑒𝑠 𝑡ℎ𝑎𝑡 𝑓(𝑓(𝑛+1) ) = 1; conclude that 𝑓(𝑛) = 1for all non-negative
integers n.
317
Challenging Mathematical Problems
Case II : 𝑓(0) = 0. 𝑇ℎ𝑒𝑛 2𝑓(𝑚2 ) = z, then 𝒇(𝒎) = 𝒇(𝒏) ; (ii) 𝒇(𝒎𝒏) = 𝒇(𝒎) 𝒇(𝒏)
𝑓(𝑚2 ) 𝑓(𝑚) 2 for all m, n ∊z.
𝑓(𝑚2 ) 𝑜𝑟 = ( ) 𝑠𝑖𝑛𝑐𝑒 𝑓(2) = 𝑓(1)2 ,
2 2
we obtain Sol.: Show first that 𝑓(0) = 0 𝑜𝑟 𝑓(𝑛) = 1
for all n ∊ z consider non constant
2
𝑓(22𝑛 ) 𝑓(22𝑛−1 )2 𝑓(22𝑛−1 ) solution of the given equation. Show that
=( )= ( )
2 2 2 𝑓(𝑘𝑝) = 0 for all integers k. Using Fermat’s
𝑓(2) 2 little theorem, prove that 𝑓(𝑚) = 𝑓(𝑚) 𝑝 for
=⋯= ( )
2 each integer m. This 𝑓(𝑚) = 0 𝑜𝑟 𝑓(𝑚) =
𝑓(1)2𝑛+1 ±1. Choose m = a, a primitive root, with
= .
22𝑛 respect to p, Then 𝑓(𝑎) ≠ 0. Consider the
On the other hand, (a) implies that 𝑓(1) = cases 𝑓(𝑎) = 1 𝑎𝑛𝑑 𝑓(𝑎) = −1 separately.
𝑓(1)2 . So either 𝑓(1) = 0 𝑜𝑟 𝑓(1) = 2 𝑝
0 𝑖𝑓
𝑓(𝑛) ≡ 0, 𝑓(𝑛) ≡ 1, 𝑓(𝑛) ={ 𝑛
If 𝑓(1) = 0, the above chain of equalities 1 𝑖𝑓 𝑝 × 𝑛
implies that 𝑓(22𝑛 ) = 0 𝑓𝑜𝑟 𝑛 ≥ 0.
Monotonicity implies that f is identically 0 𝑖𝑓 𝑝/𝑛
𝑓(𝑛) = {1 𝑖𝑓 𝑝 × 𝑛 , 𝑛 𝑖𝑠 𝑎 𝑠𝑞𝑢𝑎𝑟𝑒
equal to zero.
1 𝑖𝑓 𝑝 × 𝑛, 𝑛 𝑖𝑠 𝑛𝑜𝑡 𝑠𝑞𝑢𝑎𝑟𝑒
𝑓(𝑚2 )
If 𝑓(1) = 2, 𝑡ℎ𝑒𝑛 𝑓(22𝑛 ) = 2. 22𝑛 . 𝑆𝑖𝑛𝑐𝑒 2
= The last function is precisely Legendre’s
𝑓(𝑚) 2 symbol.
( 2
) 𝑓(𝑚) is always even. We
have𝑓(𝑚+1) 2 = 2𝑓(𝑚 + 1)2 ≥ 2𝑓(𝑚2 + 1) =
𝑓(𝑚)2 + 𝑓(1)2 > 𝑓(𝑚)2 . which implies that
8) Find all the function f : z → z which
𝑓(𝑚+1) > 𝑓(𝑚) .
satisfies the equation 𝒇(𝒂𝟑 + 𝒃𝟑 + 𝒄𝟑 ) =
Consequently, 𝑓(𝑚+1) − 𝑓(𝑚) − 2 ≥ 0 {𝒇(𝒂)𝟑 + 𝒇(𝒃)𝟑 + 𝒇(𝒄) }𝟑
2𝑛−1
𝐵𝑢𝑡 ∑2𝑚=0 (𝑓(𝑚+1) − 𝑓(𝑚) − 2) = 𝑓(22𝑛 ) − Sol.: Show that 𝑓(0) = 0 and hence 𝑓(𝑥) =
−𝑓(𝑥) for all x∊ z. Prove that 𝑓(1) = −1, 0 𝑜𝑟 1
𝑓(0) − 2. 22𝑛 =
and hence 𝑓(2) = 2𝑓(1) 𝑓(3) = 3𝑓(1) . 𝐹𝑜𝑟 𝑥 > 3
0 𝑣𝑎𝑟𝑦𝑖𝑛𝑔 𝑛 𝑤𝑒 𝑐𝑜𝑛𝑐𝑙𝑢𝑑𝑒 𝑡ℎ𝑎𝑡 𝑓(𝑚+1) =
prove that 𝑥 3 is a sum of five cubes has
𝑓(𝑚) + 2 for all 𝑚 ≥ 0. Thus 𝑓(𝑛) =
absolute value smaller than x, using the
2𝑛 𝑓𝑜𝑟 𝑎𝑙𝑙 𝑛 ∊ 𝑁0 .
identity. (2𝑘 + 1)3 = (2𝑘 − 1)3 + (𝑘 + 4)3 +
𝐼𝑛 𝑐𝑜𝑚𝑐𝑙𝑢𝑠𝑖𝑜𝑛, 𝑓(𝑛) identically equal to zero. (4 − 𝑘)3 + (−5)3 + (−1)3 using this
𝑓(𝑛) identically equal to 1, or 𝑓(𝑛) = 2𝑛 for all representation, prove that 𝑓(𝑥) = 𝑥𝑓(1) .
n, are the only possible solutions.
𝑓(𝑥) = −𝑥, 𝑓(𝑥) = 0 𝑜𝑟 𝑓(𝑥) = 𝑥
318
Challenging Mathematical Problems
f(1995) = 1996 and (ii) 𝒇(𝒙𝒚) = 𝒇(𝒙) + We start by computing the value of 𝑓(3) . Since
𝒇(𝒚) + 𝒌𝒇{𝐠𝐜𝐝(𝒙, 𝒚)} for all x, y ∊N? the function is increasing, 𝑓(3) 𝑓(5) = 𝑓(15) <
𝑓(18) = 𝑓(2) 𝑓(9) ℎ𝑒𝑛𝑐𝑒 𝑓(3) 𝑓(5) <
Sol.: Using (b), get an expression for 𝑓(𝑥 2 ) 2𝑓(9) 𝑎𝑛𝑑 𝑓(9) < 𝑓(10) = 𝑓(2) 𝑓(5) = 2𝑓(5).
and hence for 𝑔(𝑥 4 ). Using 𝑥 4 = 𝑥. 𝑥 3 , 𝑥 3 =
Combining the two inequalities we get
𝑥. 𝑥 2 , get another expression for 𝑔(𝑥 4 ). Show
𝑓(3) 𝑓(5) < 4𝑓(5) ℎ𝑒𝑛𝑐𝑒 𝑓(3) < 4. We also have
that k = 0 or -1. Using prime decomposition,
that 𝑓(3) > 𝑓(2) = 2; thus 𝑓(3) can be equal
define if f suitably for these value of k.
only to 3.
319
Challenging Mathematical Problems
= (4𝑓(𝑚) + 1)(4𝑓(𝑛) + Write 𝑃(𝑛) (𝑘) for the kth composite of P. That
1) = 𝑔(3𝑚 + 1)𝑔(3𝑛 + 1). is 𝑃(𝑛) (0) = 𝑛 𝑎𝑛𝑑 𝑃(𝑛) (𝑘+1) =
𝑝(𝑛) (𝑘) . 𝐹𝑜𝑟 𝑛 ≥ 12, 𝑙𝑒𝑡 𝑔(𝑛) be the smallest
This g is multiplicative, in the case sense that
integer k such that n is not in the image of
𝑔(𝑥𝑦) = 𝑔(𝑥) 𝑔(𝑦) 𝑓𝑜𝑟 𝑎𝑙𝑙 𝑥, 𝑦 ∊ 3𝑁0 + 1.
𝑃(𝑘) . Such a k exists because a side from 9 and
Conversely, given any multiplicative bijection 11, every integer in the image of
from 3𝑁0 + 1 𝑜𝑛𝑡𝑜 4𝑁0 + 1, we can construct 𝑃(𝑛) (𝑘) 𝑓𝑜𝑟 𝑘 > 0 is greater than or equal to
a function f having the required property by 𝑃(12) (𝑘) , and an easy induction shows that
letting 𝑓(𝑥) = (𝑔(3𝑥 + 1)). 𝑃(𝑚) (𝑘) > 𝑛 + 𝑘 𝑓𝑜𝑟 𝑛 ≥ 12.
It remains only to exhibit such a bijection. Let Let 12= 𝑆1 ≤ 𝑆2 ≤ ⋯ be the integers greater
𝑃1 𝑎𝑛𝑑 𝑃2 denotes the sets of primes of the for than or equal to 12, not in the image of P, in
3n +1 and 3n +2, respectively and let creasing order. Then for every integer 𝑛 ≥ 12,
𝑄1 𝑎𝑛𝑑 𝑄2 denote the sets of primes of the there exists a unique integer h(n) such that
form 4n +1 and 4n +3 respectively. Since
𝑛 = 𝑃(𝑔(𝑛) ) 𝑆(𝑛(𝑛))
each of these sets is infinite, there exists a
bijection h from 𝑃1 ∪ 𝑃2 𝑡𝑜 𝑄1 ∪
𝐹𝑜𝑟 𝑛 ≥ 12, 𝑠𝑒𝑡
𝑄2 𝑡ℎ𝑎𝑡 𝑚𝑎𝑝𝑠 𝑃1 bijectively onto 𝑄1 𝑎𝑛𝑑 𝑄2 .
Define g as following 𝑔(1) = 1 , 𝑎𝑛𝑑 𝑓𝑜𝑟 𝑛 > 𝑃(𝑔(𝑛) )(𝑆 ℎ(𝑛) + 1)ℎ(𝑛) 𝑜𝑑𝑑
1, 𝑛 ∈ 3 𝑁0 + 1, let the prime factorization of 𝑓(𝑛) = {
𝑃(𝑔(𝑛)+1 ) (𝑆 ℎ(𝑛) − 1) ℎ(𝑛) 𝑒𝑣𝑒𝑛
n be n = ∏ 𝑝𝑖 (with possible repititions among
the 𝑝𝑖 ′𝑠), then define 𝑔(𝑛) = ∏ ℎ(𝑝𝑖 ). 𝐹𝑜𝑟 𝑛 ≤ 7, 𝑝𝑢𝑡 𝑓(𝑛) = 𝑓(19−𝑛) . To show that
𝑓(𝑓(𝑛) ) = 𝑃(𝑛)
Note that g is well –defined, because if 𝑛 ∈
3 𝑁0 + 1, then there must be an even number We need only consider 𝑛 ≥ 12, and we may
of 𝑃2 type primes that divide n. Each of these examine two cases. If h(n) is odd, then
primes gets mapped by h to a prime in 𝑄2 , 𝑔(𝑓(𝑛) ) = 𝑔(𝑛) 𝑎𝑛𝑑 ℎ(𝑓(𝑛) ) = ℎ(𝑛) +
and since there are an even number of such
320
Challenging Mathematical Problems
1 𝑖𝑠 𝑒𝑣𝑒𝑛, 𝑠𝑜 𝑓(𝑓(𝑛) ) = 𝑓 (𝑃(𝑔(𝑛) ) (𝑆 ℎ(𝑛) + also happen that 𝑓(𝑥) = 𝑥 + 1 for some real
number x and 𝑓(𝑦) = 𝑦 − 1 for some other
1)) = 𝑃 (𝑔(𝑛)+1) 𝑆 ℎ(𝑛) = 𝑃(𝑛)
real number y. We have to resolve this before
concluding anything. 𝑆𝑢𝑝𝑝𝑜𝑠𝑒
Similarly, if h(n) is even, then 𝑔(𝑓(𝑛) ) =
𝑔(𝑛+1) 𝑎𝑛𝑑 ℎ(𝑓(𝑛) ) = ℎ(𝑛) − 1𝑖𝑠 𝑜𝑑𝑑, 𝑓(𝑥0 ) = 𝑥0 − 1
𝑠𝑜 𝑓(𝑓(𝑛) ) = 𝑓𝑃(𝑔(𝑛) +1) (𝑠 ℎ(𝑛) − 1) = 𝑓𝑜𝑟 𝑠𝑜𝑚𝑒 𝑟𝑒𝑎𝑙 𝑛𝑢𝑚𝑏𝑒𝑟 𝑥0 . 𝑇ℎ𝑒𝑛 𝑢𝑠𝑖𝑛𝑔 (3)
(𝑔𝑛 +1)
𝑃 ( 𝑠 ℎ(𝑛) ) = 𝑃(𝑛)
𝑎𝑛𝑑 (2), 𝑤𝑒 𝑔𝑒𝑡 1 + 𝑥02 = 𝑓(𝑥02 ) = 𝑓(𝑥0 )2 −
2𝑥0 = (𝑥0 − 1)2 − 2𝑥0 = 𝑥0 2 − 4𝑥0 +
1. 𝑇ℎ𝑖𝑠 𝑓𝑎𝑐𝑒𝑠 𝑥0 = 0. but the we obtain 1 =
14) Find all functions f : R→ R which obey the 𝑓(0) = 𝑓(𝑥0 ) = 𝑥0 − 1 = −1, which is absurd,
equation 𝒇{(𝒙 − 𝒚)𝟐 } = {𝒇(𝒙) }𝟐 − we concluded that 𝑓(𝑥) = 𝑥 + 1 for real
𝟐𝒙𝒇(𝒚) + 𝒚𝟐 . number x.
Sol.: One can easily guess that 𝑓(𝑥) = 𝑥 is a If follows that 𝑓(𝑥) = 𝑥 and 𝑓(𝑥) = 𝑥 + 1 and
solution of this functional equation. Are there the only solutions of the given functional
any other solutions which are not obvious but equation.
hidden in the equation? Indeed there is one
more solution, 𝑓(𝑥) = 𝑥 + 1 which is not
apparent from the equation. We see that for 15) If f : R →R be a function such that (i)
the function, 𝑓(𝑥) = 𝑥 + 1 𝑤𝑒 ℎ𝑎𝑣𝑒 𝒇(𝒙+𝒚) = 𝒇(𝒙) + 𝒇(𝒚) ∀ 𝒙, 𝒚 ∊ 𝑹 𝒂𝒏𝒅 (ii)
𝟏 𝒇(𝒙)
𝑓((𝑥−𝑦)2 ) = (𝑥 − 𝑦)2 + 1 = 𝑥 2 − 2𝑥𝑦 + 𝑦 2 + 𝒇 ( 𝒙) = 𝒙𝟐
∀ 𝒙 ≠ 𝟎, 𝒔𝒉𝒐𝒘 𝒕𝒉𝒂𝒕 𝒇(𝒙) =
1, 𝑎𝑛𝑑 𝒄𝒙 ∀ 𝒙 ∈ 𝑹 where c is constant.
𝑓(𝑥)2 − 2𝑥𝑓(𝑦) + 𝑦 2 = (𝑥 + 1)2 − Sol.: It is easy to check that (a) gives 𝑓(0) =
2𝑥(𝑦 + 1) + 𝑦 2 = 𝑥 2 − 2𝑥𝑦 + 𝑦 2 + 1. 0 𝑎𝑛𝑑𝑓 𝑓(−𝑥) = −𝑓(𝑥) for all real x, we know,
1 1
for x ≠ 0 and x ≠ 1, the identity 𝑥−1 − 𝑥 =
How do we compute these two and other if
1
any? Put y = 0 in (1) to obtain 𝑓(𝑥 2 ) = 𝑥(𝑥−1)
𝑓(𝑥)2 − 2𝑥𝑓(0) 𝑎𝑛𝑑 𝑝𝑢𝑡 𝑥 = 0 to get, 𝑓(𝑦2 ) +
𝑓(0)2 + 𝑦 2 . This in conjunction with the property (a)
gives
𝑇𝑎𝑘𝑖𝑛𝑔 𝑦 = 0 𝑖𝑛 (3)𝑤𝑒 𝑠𝑒𝑒 𝑡ℎ𝑎𝑡 𝑓(0)2 =
1 1 1
𝑓(0) 𝑔𝑖𝑣𝑖𝑛𝑔 𝑓(0) = 0 𝑜𝑟 𝑓(0) = 1. 𝑇𝑎𝑘𝑖𝑛𝑔 𝑥 = 𝑦 𝑓( )−𝑓( ) = 𝑓( )
𝑥−1 𝑥 𝑥(𝑥 − 1)
in (1), we obtain 𝑓(0) = 𝑓(𝑥)2 − 2𝑥𝑓(𝑥) + 𝑥 2 =
(𝑓(𝑥) − 𝑥)2 Now an application of (b) yields
𝐼𝑓 𝑓(0) = 0, then the above relation shows 𝑓(𝑥 − 1) 𝑓(𝑥) 𝑓(𝑥(𝑥 − 1))
− 2 = 2
(𝑥 − 1)2 𝑥 𝑥 (𝑥 − 1)2
that 𝑓(𝑥) = 𝑥 for all x ∊R. If 𝑓(0) =
1, 𝑡ℎ𝑒𝑛 𝑓(𝑥) − 𝑥 = ±𝑖 𝑎𝑛𝑑 ℎ𝑒𝑛𝑐𝑒 𝑓(𝑥) = 𝑥 ± 1, 𝑇ℎ𝑖𝑠 𝑠𝑖𝑚𝑝𝑙𝑖𝑓𝑖𝑒𝑠 𝑡𝑜
which sign should we choose here? It may
321
Challenging Mathematical Problems
𝑓(2) +2𝑓(1)
obtain 𝑓(𝑥) = ( 4
) 𝑥,valid for all x ≠ 0 If we replace x by 𝑥 − 1 and y by 1 in (1), we
and x ≠ 1. get
𝑓(𝑥)
The above problems reveal the fact that using … … … . . (4)
𝑓(𝑥−2)
simple manipulations, we can solve some
functional equations on R. We have not A comparison of (3) and (4) shows that
exactly effectively used any structure of R to
arrive at the solution, next few problems tell 𝑓(𝑥−1) +1
𝑓(𝑥) = 𝑓(𝑥−2) {𝑓 } … … … . . (5) putting
(𝑥−1) −1
us how to use the known structure(s) of real
numbers to solve equations. x = 3 in (3), we get
322
Challenging Mathematical Problems
Comparing two expressions for 𝑓(5) , we see that maps non-negative reals to non-negative
that (𝑓(2) − 1)2 = 1 + 2𝑓(2) − 𝑓(2)2 . reals, since f is one-one and 𝑓(0) = 0. We
conclude that 𝑓(𝑥) > 0. Whenever x > 0.
The quadratic equation for 𝑓(2) simplifies to Suppose x > y we consider different cases:
𝑓(2)2 = 2𝑓(2). (a) suppose x > y≥ 0. Here we obtain
This is the most difficult and important step is (𝑐)𝑐𝑜𝑛𝑠𝑖𝑑𝑒𝑟 the case y < x < 0. Then 0 <
getting a solution of our problem. The set −𝑥 < −𝑦 and by (a), we conclude that
follows familiar track. We compute 𝑓(3) = 𝑓(−𝑥) < 𝑓(−𝑦).
3, 𝑓(4) = 4 𝑎𝑛𝑑 𝑓(5) = 5. Suppose 𝑓(𝑘) = 𝑘 for
all natural numbers 𝑘 < −= 𝑛, where n is a Using the fact that f is an odd function, this
natural number. reduces to 𝑓(𝑦) < 𝑓(𝑥) . It follows that f is a
strictly increasing function on R. Since 𝑓(𝑟) =
𝑓(𝑛) +1 𝑟 for all rational number r, we obtain 𝑓(𝑥) = 𝑥
Then (5) shows that 𝑓(𝑛+1) = 𝑓(𝑛−1) { }
𝑓(𝑛) −1
for real number x.
𝑆𝑖𝑛𝑐𝑒 𝑓(𝑛−1) = 𝑛 − 1 𝑎𝑛𝑑 𝑓(𝑛) = 𝑛, we obtain
𝑓(𝑛+1) = 𝑛 + 1. We conclude that 𝑓(𝑛) = 𝑛 for
all natural numbers n. Replacing y by 𝑥𝑧 in 17) Find all f : R →R such that 𝒇(𝒇(𝒙) + 𝒚) =
(1), we get 𝒇(𝒙𝟐 − 𝒚) + 𝟒(𝒙)𝒚 ∀ 𝒙, 𝒚 ∈ 𝑹.
𝑥 + 𝑥𝑧 1+𝑧 1 + 𝑓(𝑧) Sol.: It is easy to check that 𝑓(𝑥) ≡
𝑓( ) = 𝑓( )= ,
𝑥 − 𝑥𝑧 1−𝑧 1 − 𝑓(𝑧) 0 𝑎𝑛𝑑 𝑓(𝑥) = 𝑥 2 are solutions of this problem.
𝑤ℎ𝑒𝑟𝑒 𝑤𝑒 ℎ𝑎𝑣𝑒 𝑢𝑠𝑒𝑑 (1) again, comparing We show that these are the only solutions of
these two expressions and solving for 𝑓(𝑥𝑧), the problem.
we obtain 𝑓(𝑧𝑥) = 𝑓(𝑧) 𝑓(𝑥) . 𝐴 𝑝𝑟𝑖𝑜𝑟𝑖 this is Suppose 𝑓(𝑎) ≠ 𝑎2 for some a. Replacing by y
valid for x ≠ 0 and z ≠ 1. But since 𝑓(0) = (𝑥 2 −𝑓(𝑥) )
in (1) by , 𝑤𝑒 𝑔𝑒𝑡 𝑓(𝑥) (𝑥 2 − 𝑓(𝑥) ) = 0.
0 𝑎𝑛𝑑 𝑓(1) = 1, we see that this multiplicative 2
323
Challenging Mathematical Problems
Thus f is periodic with period 𝑎2 . This implies Since there can be at most one fixed point of
that 𝑓(𝑓(𝑥) ) = 𝑓(𝑓(𝑥) + 𝑎2 ) = 𝑓(𝑥 2 −𝑎2 ) + 𝑓(𝑥) in (-1, 0). We conclude that
4𝑓(𝑥) 𝑎2 . Putting y = 0 in (1) we get another
2𝑢 + 𝑢2 = 𝑢. 𝑇ℎ𝑖𝑠 𝑓𝑜𝑟𝑐𝑒𝑠 𝑢(𝑢 + 1) = 0, we
expression 𝑓(𝑓(𝑥) ) = 𝑓(𝑥 2 ) contradicting the assumption that u ∊(-1, 0).
It follows that there is no fixed point of 𝑓(𝑥) in
Invoking the periodicity of f. we note that
(-1, 0). Similar analysis shows that 𝑓(𝑥) has no
𝑓(𝑥) 𝑎2 = 0
fixed in (0, ∞) as well. Thus 0 is the only
However, we have observed that a ≠0 by our possible fixed point of 𝑓(𝑥) if at all it has any.
choice of a. It follows that if 𝑓(𝑥) ≠ 𝑥 2 , then we However taking x = y in (i), we see that
must have 𝑓(𝑥) ≡ 0. This completes our claim 𝑓(𝑥 + 𝑓(𝑥) + 𝑥𝑓(𝑥) ) = 𝑥 + 𝑓(𝑥) +
and determines all the solutions of the 𝑥𝑓(𝑥) 𝑓𝑜𝑟 𝑎𝑙𝑙 𝑥 ∊ (−1, ∞). Thus each 𝑥 +
problem. 𝑓(𝑥) + 𝑥𝑓(𝑥) , 𝑥 ∊ (−1, ∞) is a fixed point of f.
We conclude that 𝑥 + 𝑓(𝑥) + 𝑥𝑓(𝑥) = 0 for all
x∊ (-1, ∞) we see that
18) Find all f: (−𝟏, ∞) → (−𝟏, ∞) such that the 𝑦 𝑥𝑦 𝑥−𝑦
function 𝑥 + 𝑓(𝑦) + 𝑥𝑓(𝑦) = 𝑥 − − = .
1+𝑦 1+𝑦 1+𝑦
(i) 𝒇{𝒙 + 𝒇(𝒚) + 𝒙𝒇(𝒚) } = 𝒚 + 𝒇(𝒙) +
𝒚𝒇(𝒙) , 𝒇𝒐𝒓 𝒂𝒍𝒍 𝒙, 𝒚 ∊ (−𝟏, ∞) 𝑇ℎ𝑢𝑠 𝑤𝑒 𝑜𝑏𝑡𝑎𝑖𝑛 𝑓(𝑥 + 𝑓(𝑦) + 𝑥𝑓(𝑦) )
𝒇(𝒙) 𝑥−𝑦 𝑦−𝑥
(ii) is strictly increasing on each of = 𝑓( )= .
𝒙 1+𝑦 1+𝑦
the intervals (-1, 0) and (0, ∞)
𝑥
It follows that 𝑓(𝑥) = − ( ) indeed satisfies
Sol.: Let f: (−1, ∞) → (−1, ∞) be a function of 1+𝑥
𝑓(𝑥) (a) we can easily check that is also satisfied
the desired type since is strictly
𝑥 (b).
increasing on the interval (-1, 0) the equation
𝑓(𝑥) = 𝑥 can have at most one solution in (0, The fixed points, can also be used in proving
∞). Moreover x = 0 may be a solution in non-existence of solutions to some functional
𝑓(𝑥) = 𝑥. Thus the equation 𝑓(𝑥) = 𝑥 can have equations. The following problem illustrates
at most three solution in (−1, ∞). In other this point.
words, there are at most three fixed points of
𝑓(𝑥) in the domain (−1, ∞).
19) Find all functions f: R →R such that
Suppose u ∊ (-1, 0) is a fixed point of 𝑓(𝑥). 𝒇{𝒙 − 𝒇(𝒚) } = 𝒇{𝒇(𝒚) } + 𝒙𝒇(𝒚) + 𝒇(𝒙) − 𝟏
Thus we have 𝑓(𝑢) = 𝑢. Taking x = y = u in holds for all x, y ∊R.
(a). we see that 𝑓(2𝑢 + 𝑢2 ) = 2𝑢 + 𝑢2 . This
shows that 2𝑢 + 𝑢2 is also a fixed point 𝑓(𝑥). 𝑥2
Sol.: We easily see that 𝑓(𝑥) = 1 − 2
satisfies
We claim that 2𝑢 + 𝑢2 is also a fixed point is the equation (1), We show that this is the
in the interval (-1, 0). In fact 2𝑢 + 𝑢2 = only function which obey the relation (1). Let
𝑢(2 + 𝑢) < 0. Since u <0 and 2+u>1>0 S denote the range of f. Put c = 𝑓(0) . Taking x
because u>-1. On the other hand 2𝑢 + 𝑢2 >
= y= 0 in (1), we obtain
−1 because 2𝑢 + 𝑢2 + 1 = (𝑢 + 1)2 > 0.
324
Challenging Mathematical Problems
Alternate Solution: As in the first solution, we 20) Find all functions f : [𝟏, ∞) → [𝟏, ∞) which
take c = 𝑓(0) . Putting x = 𝑓(𝑦) in (1), we can satisfy,
𝑐+1−𝑓(𝑦)2 (i) 𝒇(𝒙) ≤ 𝟐(𝟏 + 𝒙)∀𝒙 ∈ [𝟏, ∞);
solve for 𝑓(𝑓(𝑦) ): 𝑓(𝑓(𝑦) ) = 2
… … . . (3)
𝟐
(ii) 𝒙𝒇(𝒙+𝟏) = {𝒇(𝒙) } − 𝟏∀ 𝒙 ∈
𝑥2 [𝟏, ∞).
𝐼𝑛𝑡𝑟𝑜𝑑𝑢𝑐𝑖𝑛𝑔 𝑔(𝑥) = 𝑓(𝑥) + 2
it is easy to
𝑐−1
compute 𝑔(𝑥 − 𝑓(𝑦) ) = 𝑔(𝑥) + 2 Sol.: It is easy to verify that 𝑓(𝑥) = 𝑥 + 1
satisfies both (a) and (b). We show that is the
Note that the given equation has no constant only solutions.
solutions. Thus we may find
325
Challenging Mathematical Problems
We have 𝑓(𝑥)2 = 𝑥𝑓(𝑥 + 1) + 1 ≤ By an easy induction, we see that 𝑓(𝑥0 +𝑘) <
2 𝑥0 − 𝑘 + 1
𝑥(2(𝑥 + 1)) + 1 = 1 + 4𝑥 + 2𝑥 <
2(1 + 2𝑥 + 𝑥 2 ) = 2(1 +
If k is large enough, then 𝑓(𝑥0+𝑘) < 1. This
𝑥)2 . 𝐼𝑡 𝑓𝑜𝑙𝑙𝑜𝑤𝑠 𝑡ℎ𝑎𝑡 𝑓(𝑥) < √2(1 + 𝑥) using
contradiction forces 𝑓(𝑥) = 1 + 𝑥 𝑓𝑜𝑟 𝑎𝑙𝑙 𝑥 ∈
this fresh bound, we obtain
[1, ∞)
𝑓(𝑥)2 = 𝑥𝑓(𝑥 + 1) + 1 < √2𝑥 (2 + 𝑥) + 1
In some cases the functional relation may
= √2𝑥 2 + 2√2𝑥 + 1 reveal some useful information about the
< √2(𝑥 2 + 2𝑥 + 1) function.
= √2(𝑥 + 1)2
𝑓(𝑥0 + 2)2 − 1 (𝑥0 − 1)2 − 1 2) On the two square fields, the area of one is
𝑓(𝑥0 +3) = <
𝑥0 + 2 𝑥0 + 2 1 heal are, while the other one is border by
𝑥0 (𝑥0 − 2) 2%. The different in their areas is
= < 𝑥0 − 2
𝑥+2 (a) 𝟒𝟎𝟎𝒎𝟐 ; (b) 404𝒎𝟐 ; (c) 410 𝒎𝟐
; (d) none
326
Challenging Mathematical Problems
Sol.: Each side of a square field is 5) The interior angles of a hexagon are in the
2
√1 ℎ𝑒𝑐𝑡𝑎𝑟𝑒 √10,000 𝑚 = 100𝑚. ratio 1 :2 :2: 3: 2: 2, then the largest angle
∴ each side of the second square field is is
(100 + 100 × 1%)𝑚 = 102𝑚. (a) 150°; (b) 170°; (c) 180°; (d)
∴ Required area = (10,404 − 10,000)𝑚2 = Sol.: The sum is interior angles of a hexagon =
404𝑚2. (2 × 6 − 4)90° = 720°
3) The diagonal of a square A is (x+y). The
diagonal of a square B with twice the area Now let its angles be x, 2x, 2x, 3x, 2x, 2x
of A is respectively
(a) √(𝒙 + 𝒚); (b) √(𝟐𝒙 + 𝒚); (c)
∴ 𝑥 + 2𝑥 + 2𝑥 + 3𝑥 + 2𝑥 + 2𝑥 = 720.
√(𝒙 + 𝟐𝒚); (d) none
327
Challenging Mathematical Problems
(a) AO : AD; (b) 𝑨𝑶𝟐 : 𝑶𝑫𝟐 ; (c) AO : of length 6 cm and 8 cm, respectively are
𝑎 2 3𝑎2 3𝐴𝐵2
∴ 𝐴𝐷 2 = 𝑎2 − ( ) = =
2 4 4
∴ 3𝐴𝐵2 = 4𝐴𝐷 2
328
Challenging Mathematical Problems
1 1
𝐴𝑙𝑠𝑜, ∠𝐵𝐷𝐶 = ∠𝐶𝑂𝐵 = × 120° = 60°.
2 2
329
Challenging Mathematical Problems
PQ is the chord of contact of the circle PQT , ∠𝐵𝑃𝑇 = 180° − ∠𝑇𝑃𝐴 = 180° − ∠𝐴𝑃𝐼 −
PQ ⊥ AT and the midpoint 1 of PQ lies on AT. ∠𝐼𝑃𝑇 = 180° − 𝛽 − 90° + 𝛼 = 90° + 𝛼 −
𝛽 … … … . (2)
Now to prove that I is the in centre of ∆ABC, it
is enough to prove that BI is the angle But APT is a tangent to circle PQT ∠BPT =
bisector of ∠B. and CI is the angle bisector of ∠PQT = ∠IQT from (1) and (2),
∠C respectively. By symmetry ∠PTI = ∠QTI =
we get 90° + 𝛼 − 𝛽 = 90° − 𝜶 ∴ 2𝛼 = 𝛽
𝛼°
∴ ∠𝐼𝐵𝐷 = 𝛽 − ∠𝑃𝐵𝐼 = 2𝛼 − 𝛼 = 𝛼 ∴
∠𝐼𝐵𝐷 = ∠𝑃𝐵𝐼
∴ PBTI is cyclic
⟹ 𝐴𝐶 × 𝐵𝐶 = 𝑥𝑦 + 𝑟𝐴𝐵 + 𝑟 2 ⟹ 𝑥𝑦
∴ ∠𝐼𝛽𝑇 = ∠𝐼𝐵𝐷 + ∠𝐷𝐵𝑇 = 𝛽 − 𝛼 + 90° −
= 𝐴𝐶 × 𝐵𝐶 − 𝑟𝐴𝐵 − 𝑟 2
𝛽 = 90° − 𝛼
𝑁𝑜𝑤 𝐴𝐼 2 × 𝐵𝐼 2 = (𝑥 2 + 𝑟 2 )(𝑦 2 + 𝑟 2 )
𝑆𝑖𝑛𝑐𝑒 𝑃𝐵𝑇𝐼 𝑖𝑠 𝑐𝑦𝑐𝑙𝑖𝑐, ∠𝐼𝑃𝑇 = ∠𝐼𝐵𝑇 = 90° −
= 𝑥 2 𝑦 2 + 𝑟 2 (𝑥 2 + 𝑦 2 ) + 𝑟 4
𝛼 … … … (1)
330
Challenging Mathematical Problems
= 𝑥 2 𝑦 2 + 𝑟 2 [(𝑥 + 𝑦)2 − 2𝑥𝑦] + 𝑟 4 circle k lies inside L and now D lies on the
= 𝑥 2 𝑦 2 + 𝑟 2 [(𝐴𝐵2 − 2𝑥𝑦)] radius OC.
+ 𝑟4
= 𝑥 2 𝑦 2 + 𝑟 2 𝐴𝐵2 − 𝑟 2 + 2𝑥𝑦 + 𝑟 4
= 𝑥 2 𝐴𝐵2 + (𝑟 2 − 𝑥𝑦)2
= 𝑟 2 𝐴𝐵2 + [𝑟 2 − 𝐴𝐶 × 𝐵𝐶 + 𝑟𝐴𝐵 + 𝑟 2 ]2
= 𝑟 2 𝐴𝐵2
+ [2𝑟 2 − 𝐴𝐶 × 𝐵𝐶 + 𝑟𝐴𝐵]2
331
Challenging Mathematical Problems
1
∠PAT = x = ∠AOB (angle in the alternate
2
1
Let A’P and P’B be any other chords so that ∠PBS = y = 2∠AOB. ∴ x = y since AT is
A’PB’ may be collinear and the extension of tangent to circle 𝑐1 , we get ∠PAT = x =
these chords meet the circle c at C and D. 1
∠𝐴𝑂1 𝑃
2
∴ A 𝑂1 𝑂 are collinear. Similarly B 𝑂2 𝑂 are Further 𝑃𝑂2 ∥ 𝑂𝑂1 𝑎𝑛𝑑 𝑃𝑂1 ∥ 𝑂𝑂2 .
collinear. Let AT, BS be the common tangents
So join 𝑃𝑂1 𝑎𝑛𝑑 𝑃𝑂2 . Through 𝑂2 draw a line
to circles C and 𝑐1 and C and 𝑐2 respectively.
parallel to P𝑂1 to must the circle 𝑐2 𝑖𝑛 𝐵. Now
Let ∠PAT = x and ∠PBS = y since AT is these two parallel lines drawn meet at O. If
tangent to circle c. we drawn a circle with O as centre and radius
OA = OB, then the circle touches 𝑐1 at A and
332
Challenging Mathematical Problems
respectively and the other two sides (or procedure BB‘(BB) is perpendicular to CD
their extensions) go through C and D and equal to CD. Join AB’(or AB”)
respectively.
Draw CP . DS⊥ r to AB’ produced and extend
Sol.: Draw BB’ ⊥ r’ to l and BB’ = CD. Join AB’ PC and SD. Through B draw BR and BQ
and extend it to y. Through C and D draw perpendicular to SR and PQ. PQRS is the
333
Challenging Mathematical Problems
∴ ∠𝐼𝐴𝐶 ′ = ∠𝐼𝐴𝐵′
Draw C𝑃1 , 𝐷𝑆1 ⊥ 𝑟 𝑡𝑜 𝐴𝐵′′ and through B
draw ⊥ r to c𝑝1 𝑎𝑛𝑑 𝐷 𝑆1 produced meeting ∴ ∠𝐼𝐶 ′ 𝐴 = ∠𝐼𝐵′ 𝐴
them at Q and 𝑟1 respectively. 𝑃1 𝑄1 𝑅1 𝑆1 is the
[𝐹𝑟𝑜𝑚 (1)] … … … . (2)
required square. Thus there are two
solutions(The proof is similar to that to the AI is common. So ∆𝑠 𝐴𝐼𝐶 ′ 𝑎𝑛𝑑 𝐴𝐼𝐵′ are
former one). argument from(2)
6) In ∆ ABC, AB ≠ AC. The bisector of ∠ B and ∠BC’I = 180° − ∠𝐼𝐶 ′ 𝐴 = 180° − ∠𝐼𝐵′ 𝐴 =
∠C meet their opposite sides AC and AB at ∠𝐼𝐵′ 𝐶
B’ and C respectively. The Two bisectors
∠𝐶 ′ 𝐼𝐵 = ∠𝐵′𝐼𝐶 (vertically opposite ∠s)
intersects at I. Show that, if IB’ = IC’, then
∠BAC = 𝟔𝟎°. 𝐼𝐶 ′ = 𝐼𝐵′ (𝑔𝑖𝑣𝑒𝑛)
E and D lie on opposite, sides of the bisectors But by hypothesis AB ≠ AC and hence, the
CC’ , where IE and ID are the perpendiculars assumption that E and D are on the same side
drawn from the in centre I to AB and Ac.) In of CC’ is false.
the figure D and E lie on the same side of CC’.
IE = ID (in radii of ∆ABC)
334
Challenging Mathematical Problems
In the figure, we have taken AB < AC, and E ∠CMP = ∠CMN = ∠CND (angle in the
and D lie on opposite sides of CC’. alternate segment)
If AB> AC, then E and D lie on opposite sides ∠CNP = ∠CNM = ∠CMB (angle in the
of BB’ alternate segment)
𝐴𝐶 2 = 𝑎2 + 𝑏 2 − 2𝑎𝑏 cos(180° − 𝜃) = 𝑎2 +
Sol.: Let ‘O’ be the centre of the circle.
𝑏 2 + 2𝑎𝑏 cos 𝜃
OM= ON = OC each being the radius of the
same circle, end AMON is a square.
335
Challenging Mathematical Problems
𝑆𝑖𝑚𝑖𝑙𝑎𝑟𝑙𝑦 𝑖𝑛 ∆𝐶𝐸𝐷, 𝐶𝐸 2 = 𝑐 2 + 𝑑2 −
2𝑐𝑑 cos(90° + 𝜃) = 𝑐 2 + 𝑑 2 + 2𝑐𝑑 sin 𝜃
∴ 𝐴𝑐 2 + 𝐶𝐸 2 = 𝑎2 + 𝑏 2 + 𝑐 2 + 𝑑2 +
2𝑎𝑏 cos 𝜃 + 2𝑐𝑑 sin 𝜃
𝐴𝐶
𝐼𝑛 ∆𝐴𝐶𝐸, = sin 𝜃 ⟹ 𝐴𝐶 = 2 sin 𝜃 1
𝐴𝐸 By hypothesis, this area is equal to 2 𝑎2 =
> 𝑏(∵ 𝐴𝐸 = 2) … … … . (1) 1
𝑎2 sin 𝜃 ⟹ sin 2𝜃 = 2 ⟹ 2𝜃 = 30° 𝑜𝑟 150°
𝐶𝐸
𝑎𝑛𝑑 = cos 𝜃 (𝐴𝐸 = 2) ⟹ 𝜃 = 15° 𝑜𝑟 75°
𝐴𝐸
⇒ 𝐶𝐸 = 2 cos 𝜃 > 𝑐 …….(2) [If the acute angle of the rhombus is 30°, the
other angle which is obtuse is 150°]
(Because in ∆𝑠 ABC and CDE, ∠B and ∠D are
𝐵𝐷 𝐴𝐵
obtain angles and AC is the greatest side of By sine formula, = (𝐼𝑛 ∆𝐴𝐵𝐷)
sin 2𝜃 sin(90−𝛩)
∆ABC and CE is the greatest side of ∆CDE) 𝑎×2 sin 𝜃 cos 𝜃
⟹ 𝐵𝐷 = cos 𝜃
= 2𝑎 sin 𝜃
∴ 𝐴𝐶 2 + 𝐶𝐸 2 = 𝑎2 + 𝑏 2 + 𝑐 2 + 𝑑2 +
𝐴𝐶 𝑎
2𝑎𝑏 cos 𝜃 + 2𝑐𝑑 cos 𝜃 = 4 𝐴𝑔𝑎𝑖𝑛 = (𝐼𝑛 ∆𝐴𝐵𝐶)
sin(180 − 2𝜃) sin 𝜃
9) A rhombus has half the area of the square 𝐴𝐶: 𝐵𝐷 = cos 15° : sin 15° =
with the same side length. Find the ratio of sin 75° : sin 15° = sin(45° + 30°) : sin(45° −
the longer diagonal to that the shortest 30°)
one.
Sol.: If a is the side of the rhombus, then area = sin 45° cos 30°
1 2 + cos 45° sin 30° : sin 45° cos 30°
of the rhombus is 2
𝑎 sin 2𝜃 × 2.
− cos 45° sin 30°
1 1
= (√3 + 1): (√3 − 1)
2 2
= (√3 + 1): (√3 − 1)
336
Challenging Mathematical Problems
𝑋𝐸 𝐴𝐸 𝐸𝑌
𝐴𝐶 √3 + 1 From (3) and (4) 𝐵𝐷 = 𝐴𝐷 = 𝐷𝐶
𝑜𝑟, = : (2 + √3).
𝐵𝐷 √3 − 1
𝑋𝐸 𝐸𝑌 𝐵𝐷 𝑋𝐸
⟹ = ⟹ = .
𝐵𝐷 𝐷𝐶 𝐷𝐶 𝐸𝑌
10) From a point E on the median AD of ∆ ABC, But D is the midpoint of BC and hence BD =
the perpendiculars EF is dropped to the DC ⟹XE = EY i.e., XE = YE ……….(5).
sides BC. From a point M on EF
Now draw XY parallel to BC through E. join
perpendiculars MN and MP are drawn to
AM join the collinear points P, E, N. MPAN is a
the sides AC and AB respectively. If N, E, P
cyclic quadrilateral as ∠MPA+∠MNA = 90° +
are collinear. Show that M lies on the
90° = 180°.
internal bisector of ∠BAC.
Since EF is perpendicular to BC and XY is
Sol.: Before proving the main problem, let us
drawn parallel to BC. ∠XEM = ∠EFB = 90°. In
prove the following: If in ∆ABC, AD is the
the quadrilateral MPXE. ∠MPX+∠MEX =
median xy is a line segment parallel to BC
90°+90° = 180° and hence MPXE is a cyclic
intersecting the median AD at E, then AE is
quadrilateral and in the quadrilateral MENY.
the median of ∆AXY, or in other words XE =
∠MEY = ∠MNY = 90° …………..(6)
YE.
So MENY is a cyclic quadrilateral, since ∠MEY
and ∠MNY are subtended by MY at E and N
and they are equal by (6). In ∆𝑠 MEX and
MEY, XE = YE.
That is AM bisects the vertical angle A of ∆ parallel to CA; 𝑷𝟓 𝑷𝟔 parallel to BC. Here
ABC. That is M lies on the bisector of ∠A. 𝑷𝟐 , 𝑷𝟓 lie on AB; 𝑷𝟑 , 𝑷𝟔 on CA; and 𝑷𝟒 on
BC. Show that 𝑷𝟔 𝑷𝟏 is parallel to AB.
338
Challenging Mathematical Problems
1 𝑏2
𝑇ℎ𝑒𝑟𝑒𝑓𝑜𝑟𝑒 𝑃6 𝑃1 ∥ 𝐴𝐵 Similarly, 𝑟1 = 𝑎 +
2 8𝑎
𝒃)
Sol.: Let O be the centre of the circle which 14) A rigid square plate ABCD of unit side
touches CD and passes through the points A rotates in its own plane about the middle
and B, E the point of tangency and F the point point of CD unit the new position of A
at which OE meets AB. Since OE ⊥ OC, and AB coincide with the old position of B. How
∥DC, therefore OF ∥ AB, Also OF = b- far is the new position of B from the old
1 position of A?
𝑟1 𝑎𝑛𝑑 𝐹𝐵 = 2 𝑎
339
Challenging Mathematical Problems
2
∴ 𝐴𝑃 = , 𝑠𝑜 𝑡ℎ𝑎𝑡
√5
4 4√5
𝐴𝐵′ = 2𝐴𝑃 = =
√5 5
340
Challenging Mathematical Problems
In ∆CD’B, CD’+D’B>CB, therefore if instead of 18) Given any acute−angled ∆ ABC, let points
building the bridge along DE, the bridge along A’B’ C’ be located as follows: A’ is the point
some other line the distance required to be where altitude A on BC meets the onwards
travelled would be greater. –facing semicircle drawn on BC as
diameter points B’. C’ are located similarly.
Show that [𝑩𝑪′ 𝑨]𝟐 + [𝑪𝑨𝑩′ ]𝟐 +
17) ∆ABC is scalene with ∠A having measure [𝑨𝑩𝑪]𝟐 = [𝑨𝑩𝑪′ ]𝟐 ,
greater than 𝟗𝟎°.Determine the set of 𝒘𝒉𝒆𝒓𝒆 |𝑨𝑩𝑪|𝒅𝒆𝒏𝒐𝒕𝒆𝒔 𝒕𝒉𝒆 𝒂𝒓𝒆𝒂 𝒐𝒇 ∆𝑨𝑩𝑪
points D on the extended line BC for which etc.
341
Challenging Mathematical Problems
2 (𝑐 − 𝑎) (𝑎 − 𝑏)
1 1
∴ [𝐵𝐶′𝐴]2 = ( . 𝐵𝐶. 𝐴′ 𝐷) = 𝐵𝐶 2 . 𝐴′𝐷2 𝑆𝑖𝑚𝑖𝑙𝑎𝑟𝑙𝑦, 𝑦 =
(𝑐 + 𝑎)
,𝑧 =
(𝑎 + 𝑏)
2 4
1
= 𝐵𝐶 2 . 𝐴𝐷 2 cot 𝐵 cot 𝐶 𝑏−𝑐 𝑐−𝑎
4 𝑁𝑜𝑤, 𝑥 + 𝑦 + 𝑧 = +
𝑏+𝑐 𝑐+𝑎
= [𝐴𝐵𝐶]2 cot 𝐵 cot 𝐶 … … (1)
𝑎−𝑏
+ ,
𝑆𝑖𝑚𝑖𝑙𝑎𝑟𝑙𝑦, [𝐶𝐴𝐵′ ]2 𝑎+𝑏
𝑝 = (𝑏 + 𝑐)(𝑐 + 𝑎)(𝑎 + 𝑏)
= ∑ 𝑎2 (𝑏 − 𝑐)
∑ 𝑎(𝑏 2 − 𝑐 2 ) = − ∑ 𝑎2 (𝑏 − 𝑐),
19) Given a ∆ABC, define the equalities x, y, z 𝑆𝑜 𝑡ℎ𝑎𝑡 ∑ 𝑏𝑐(𝑏 − 𝑐) = −(𝑏 − 𝑐)(𝑐 −
𝑩−𝑪 𝑨
as follows : x = 𝐭𝐚𝐧 𝟐 . 𝐭𝐚𝐧 𝟐 ; 𝒚 = 𝑎)(𝑎 − 𝑏),
𝑪−𝑨 𝑩
𝐭𝐚𝐧 𝟐
. 𝐭𝐚𝐧 𝟐 ; 𝒛 = ∑ 𝑎(𝑏 2 − 𝑐 2 ) = − ∑ 𝑎2 (𝑏 − 𝑐).
𝑨−𝑩 𝑪
𝐭𝐚𝐧 . 𝐭𝐚𝐧 . 𝑷𝒓𝒐𝒗𝒆 𝒕𝒉𝒂𝒕 𝒙 + 𝒚 + 𝒛 + 1
𝟐 𝟐
𝑆𝑜 𝑡ℎ𝑎𝑡 𝑥 + 𝑦 + 𝑧 = (𝑝) (−(𝑏 −
𝒙𝒚𝒛 = 𝟎.
𝑐)(𝑐 − 𝑎)(𝑎 − 𝑏)) = −𝑥𝑦𝑧
Sol.: By Napier’s analogies,
𝐻𝑒𝑛𝑐𝑒 𝑥 + 𝑦 + 𝑥 + 𝑥𝑦𝑧 = 0.
𝐵−𝐶 (𝑏 − 𝑐) 𝐴
tan [ ]=[ ] cot ( )
2 (𝑏 + 𝑐) 2
(𝑏 − 𝑐)
𝑆𝑜 𝑡ℎ𝑎𝑡 𝑥 =
(𝑏 + 𝑐)
342
Challenging Mathematical Problems
respectively. So that Bx. AB= Since 𝑥𝐼𝑌 𝑖𝑠 𝑎 𝑠𝑡. 𝑙𝑖𝑛𝑒, 𝑡ℎ𝑒𝑟𝑒𝑓𝑜𝑟𝑒 ∠𝑥𝐼𝐵 +
I𝑩𝟐 𝒂𝒏𝒅 𝑪𝒚. 𝑨𝑪 = 𝑰𝑪𝟐 . Given that the ∠𝐵𝐼𝐶 + ∠𝐶𝐼𝑌 = 180° … … . . (5)
points x, I, y lie on a straight line, find the
Adding corresponding sides of relations (2),
possible values the measure of ∠A.
(3) and (4), and using (5),
𝐵𝑥
Sol.: Since Bx. AB= I𝐵2 , 𝑡ℎ𝑒𝑟𝑒𝑓𝑜𝑟𝑒 𝐵𝐼 =
1 1 1
we have 2
∠𝐴 + (90° + 2 ∠𝐴) + 2 ∠𝐴 =
𝐵𝐼
𝐵𝐴
… … … … (1)
180° 𝑆𝑜 𝑡ℎ𝑎𝑡 ∠𝐴 = 60°.
𝐵𝑥 𝐵𝐼
In ∆s B x I and BIA, ∠xBI = ∠ABI, and 𝐵𝐼 = 𝐵𝐴
from (1). Therefore ∆s BxI and BIA are 21) The diagonals AC and BD of a cyclic
similar and hence equiangular. quadrilateral ABCD intersect at P. Let O be
1
the circumcentre of ∆APB and H be the
∴ ∠𝐵𝐼𝑥 = ∠𝐵𝐴𝐼 = ∠𝐴 … … … . . (2)
2 orthocenter of ∆CPD. Show that the points
𝐴𝑙𝑠𝑜, ∠𝐵𝐼𝐶 = 180° − (∠𝐼𝐵𝐶 + ∠𝐼𝐶𝐵) altitude of ∆PCD. This will ensure that the
1 1 orthocenter H of the triangle PCD lies on PM
= 180° − ( ∠𝐵 + ∠)
2 2 and consequently the points O, P, H are
1
= 180° − [90° − ∠𝐴], collinear. To complete the proof join AO, draw
2
OL ⊥AP and consider ∆s PLO and PMC. In
343
Challenging Mathematical Problems
1
these two triangles ∠LOP = 2 ∠𝐴𝑂𝑃 (because therefore, 𝐵𝑥 = 𝑥𝐶 = 𝑏, similarly, 𝐷𝑦 =
𝑦 𝐸 = 𝑑, 𝐹𝑧 = 𝑧𝐴 = 𝑓,
O is the circumcentre of ∆ABP, and OL ⊥
chord AP) = ∠APB (being the angle ∴ zx = f+a+b, xy = b +c +d, yz = d+ e +f,
subtended by the chord AP at a point B of the since ∆xyz is equilateral,
circumcentre of the circle ABP)=
∠PCD(angles in the same segment of a circle). therefore f+ a +b = b +c +d = 𝑑 + 𝑒 + 𝑓 =
1
[(𝑓 + 𝑎 + 𝑏) + (𝑏 + 𝑐 + 𝑑) + (𝑑 + 𝑒 + 𝑓)]
3
Now in ∆s PLO and PMC, ∠LOP = ∠PCM
(proved), and ∠LOP = ∠CPM (vert opp ∠S), 1
= (𝑎 + 𝑏 + 𝑐 + 𝑑 + 𝑒 + 𝑓)
therefore ∠CMP= ∠PLO = 90° and 3
1
consequently PM ⊥ CD, and the proof is + (𝑏 + 𝑑
3
complete. + 𝑓) … … … . . (1)
22) Show that there exist convex hexagon in which a, b, c, d, e, f are numbers 1, 2, 3, 4, 5, 6
1
the plane such that (a) all its interior in some order so, that 3 (𝑎 + 𝑏 + 𝑐 + 𝑑 + 𝑒 +
angles are equal (b) its sides are 1, 2, 3, 4, 𝑓) = 7.
5, 6 in some order.
We shall choose values of b, d, f out of the
Sol.: Suppose there exists a convex hexagon given values in such a manner (to simplify the
ABCDEF such that all its interior angles are working) that b +d +f is a multiple of 3. This
equal, and the lengths of its sides are a, b, c, d, can be done in several ways. One with this
e, f respectively. Produced AB and DC meet at choice, b +d+ f = 15, so that f +a +b= b +c
x, CD and EF to meet at y, and EF and BA to 1
+d = d+ e +f = 7 + 3 . 15 = 12
meet at z. It can easily be seen that ∆xyz is
equilateral. In fact since the hexagon ABCDEF Using the values b = 4, d = 5, f = 6, we have a
us equiangular, each of its interior angles is = 2, c = 3, e = 1. Thus we get a = 2, b = 4, c =
120°, consequently each of its interior angles 3, d = 5, e = 1, f= 6 we therefore have the
is 60°be ∠XBC= ∠xcB = 60°, ∠EDY =∠DEY = following construction:
60°, ∠FAZ= ∠AFZ = 60°.
Construct an equilateral triangle xyz having
It follows that ∠𝐵𝑥𝐶, ∠𝐷𝑦𝐸, ∠𝐹𝑧𝐴 are each each side equal to 12 units. Cut of P 𝑥𝑏 =
equal to 60°. Since ∆BxC is equilateral, 𝑥𝑐 = 4 units, 𝑦𝐷 = 𝑦𝐸 = 5 𝑢𝑛𝑖𝑡𝑠, 𝑧𝐴 = 𝑧𝐹 =
6 𝑢𝑛𝑖𝑡𝑠
344
Challenging Mathematical Problems
Sol. Sol.:
345
Challenging Mathematical Problems
(Since Q, T, R are in a straight line). Adding corresponding sides of (1) and (2),
we have [APD]+[BQC]= [ACP]+[BQC]=
Also PN +QN = QN+NR. Since two sides of a
[AQCP], = [AQP]+[PQC]……(3)
triangle are together greater than the third,
therefore in ∆ QNR, 𝑄𝑛 + 𝑁𝑅 > 𝑄, 𝑖. 𝑒. 𝑃𝑁 + Since Q is the midpoint of AB, therefore ∆s
𝑄𝑁 > 𝑃𝑇 + 𝑄𝑇. AQP and BQP have equal bases AQ and QB,
and a common vertex P. Consequently
Hence the sum of the distances of P and Q
[AQP]=[QBP] …………..(4)
from T is the least.
Again, since P is the midpoint of CD, therefore
∆s PQC and QPD have equal bases CP and PD,
25) ABCD is a quadrilateral and P, Q are mid- and a common vertex Q. Consequently, [PCQ]
points of CD, AB, AP, DQ meet at x and BP, = [QPD] …………..(5)
CQ meet at y. Show that ∆ADx+∆BCy=
Adding corresponding sides of (4) and (5),
area of quadrilateral PxQy
we have [AQP]+[PQC]= [QBP]+[QPD]=
Sol.: As usual we shall denote the area of a [QBPD] …………..(6)
∆ABC by [ABC] and that of a quadrilateral
From (3) and (6), we have [APD]+[BQC]=
ABCD by [ABCD] join AC and PQ. Since P is the
[QBPD] ………..(7)
midpoint of DC, therefore ∆APD and ∆ACP
have equal bases PD and CP, and a common Since [APD]= [AxD] +[DxP] …………….(8)
vertex A.
[BQC] = [BYC]+[BYQ] …………….(9), [QBPD]=
[QBY]+[PxQY]+[DxP] ………..(10)
[AxD]+[DxP]+[ByC]+[ByQ]=[QBy]+[PxQy]+
[DxP] i.e. [AxD]+[ByC]= [PxQy] as described.
(because Q is the mid points of AB), and a touches the side BC at T. The line through
346
Challenging Mathematical Problems
Sol.:
Let Al produced meet BC in H. Since ∆s ABC
and AB’C’ have ∠A in common, therefore in
order to show that ∆s ABC and ∆AB’C’ are
similar, we should compute ∠B’ (or ∠C’) in
terms of ∠A, ∠B and ∠C, and show that ∠B’ is
equal to either ∠B or ∠C.
347
Challenging Mathematical Problems
circumcircle of ∆s PBC, PCA, and PAB are circumcentre of the triangle and the point
congruent. O are collinear.
Let us denote the circles PBC, PCA, PAB by Sol.: Suppose three congruent circles with
𝑐1 , 𝑐2 , 𝑐3. respectively and their centres by L, centres P , Q, R lie inside a ∆ABC, and are such
M, N respectively, Since L and M are the that the circle with centre P touches AB and
centres of two congruent circles 𝑐1 𝑎𝑛𝑑 𝑐2 AC, that with centre Q touches BC, and AB,
respectively. And the points P, C lie on both and that with centre R touches CA and BC. BC
these circles, therefore LC = LP = MC = MP. and AB, and that with centre R touches CA
and BC.
Therefore P, L, C, M are the vertices of a
rhombus. Similarly P, L, B, N are the vertices
of a rhombus, and P, M, A., A, N are also the
vertices of a rhombus.
348
Challenging Mathematical Problems
AB and AC. Therefore P lies on the internal 𝐶𝐴2 + 𝐶𝐵2 = 2𝐶𝐹 2 + 2𝐴𝐹 2
bisector of ∠A.
1 3
So that, 2
𝐴𝐵2 = 𝐴𝐶 2 + 𝐵𝐶 2 − 2 (4 𝐵𝐶 2 ),
Similarly Q and R lie on the internal bisector
𝑖. 𝑒. , 𝐴𝐵2 = 2𝐴𝐶 2 − 𝐵𝐶 2
of ∠B and ∠C respectively.
Since ∠C is an obtuse angle, therefore A𝐵2 >
Therefore AP, BQ, CR when produced meet at
𝐴𝐶 2 + 𝐵𝐶 2 , 𝑠𝑜 𝑡ℎ𝑎𝑡 2𝐴𝐶 2 − 𝐵𝐶 2 > 𝐴𝐶 2 +
the in centre l of ∆ABC. Since QR ∥BC, RP ∥CA,
𝐵𝐶 2 .
PQ∥AB, it follows that l is also the in centre of
𝐴𝐶
∆PQR. 𝑖. 𝑒. , 𝐴𝐶 2 > 2𝐵𝐶 2 𝑜𝑟 𝐵𝐶 > √2
29) Let G be the centriod of ∆ABC in which the Since AGCP is an 118𝑚 , therefore
angle at C is obtuse AD, Fare the medians
from A, C respectively on to the sides BC, ∠𝑃𝐴𝐺 = 180° − ∠𝐶𝐺𝐴 = 180° − ∠𝐷𝐺𝐹 =
Sol.: Since chords BF and DG of the circle ∆ABC are similar, the correspondence G→A,
BDGF meet at A (outside the circle), therefore A→B, P→C being a similarity.
1
BA. FA = DA. GA so that 2 𝐴𝐵2 = 30) Let 𝑨𝟏 , 𝑨𝟐 , 𝑨𝟑 , … . , 𝑨𝒏 is n sided regular
2 3 𝟏 𝟏 𝟏
𝐴𝐷 2 , 𝑖. 𝑒. 𝐴𝐷 2 = 𝐴𝐵2 … … … … . . (𝑖) polygon such that 𝑨 =𝑨 +𝑨 .
3 4 𝟏 𝑨𝟐 𝟏 𝑨𝟑 𝟏 𝑨𝟒
349
Challenging Mathematical Problems
𝜋 2
𝜋 3𝜋 5𝜋
∴ 𝑎2 + 𝐴1 𝐴4 . 𝐴2 𝐴3 = [2 acos (𝑛)] ; 𝑖. 𝑒., Now (2) is satisfied by 𝜃= 7 , , , 𝜋, … …
7 7
𝜋
𝐴1 𝐴4 = 𝑎 (4𝑐𝑜𝑠 2 𝑛
− 1)
Rejecting the factor cos 𝜃 + 1 which
𝜋
⟹ 𝑥 = acos ( ) 𝑖𝑠 𝑎 𝑟𝑜𝑜𝑡 𝑜𝑓 𝑡ℎ𝑒 𝑒𝑞𝑢𝑎𝑡𝑖𝑜𝑛
𝑛
8𝑥 3 − 4𝑥 2 − 4𝑥 + 1 = 0 … … … … (1)
350